241
12/15/09 1. Febrile neutropenia (chemotherapy for tumor patient) is a medical emergency. Admit the patient to the hospital, obtain blood cultures, then start broad-spectrum antibiotics such as IV ceftazidime or cefepime. The most common site of mucositis si the gastrointestinal tract, and the frequently identified organism are G-, Particular P.aeruginosa. 2. In patient with acute ‘mechanical’ back pain (disc herniation) without significant neurologic deficit, conservative approach is preferred for a period of 4-6 weeks. This includes early mobilization, muscle relaxants, and NSAIDs. Best rest and physical therapy has not been shown to be help. If the pain persists after 4-6 weeks of conservative treatment or progressive neurologic deficit evolves, high-resolution diagnosis modalities are usually employed: MRI and CT with or without contrast myelography. 3. Advanced age is probably the single most important risk factor for breast cancer. That is why women of more than 40-50 years of age would need regular annual mammograms. Interestingly, only 10 percent of women diagnosed with breast cancer have a positive family history. 4. When questionable ulcer is visualized by flexible sigmoidoscopy, a biopsy is recommended for two purposes: (1) to rule out possible cancerous lesions, and (2) distinguish ulcerative colitis from Crohn’s disease, since the treatment of each given conditions is different. 5. Mitral valve prolapse (MVP) typically causes a mid-systolic click over the cardiac apex, with a short systolic murmur at the apex after the click if mitral regurgitation is present. Squatting decreases the murmur. MVP is sometimes associated with chest pain, anxiety, palpitations, and/or hayperventilation. 6. The most common cause of overflow incontinence in elderly males is an enlarged prostate, which is usually revealed by rectal examination, and is characterized by a high post-void residual volume. 7. The typical symptoms of aortic stenosis are exertional dyspnea, syncopy, and angina. The physical exam will reveal a systolic ejection murmur radiating to the apex and carotid arteries. 8. A prolonged QRS interval (i.e., typically bundle branch block) suggests a bradyarrhythmia, whereas a prolonged QT interval suggests a tachyarrhythmia. When the heart rate is lower than 30bmp, cerebral perfusion is impaired, even if there is a compensatory increase in stroke volume. A patient with a disease of the sinus node (SSS) or conduction system can have intermittent bradycardia and typically manifest with recurrent syncopy episodes. Ventrical beats, by themselves, are not associated with syncope. The most important differential diagnosis of syncope is generalized seizures. Distinguishing features of a seizure include the presence of sustained clonic-tonic movement, tongue biting, and postictal confusion. 9. Palliate radiation, along with anti-androgen therapy, is the treatment of choice for metastatic prostate cancer. Anti-androgen therapy consists of Leuprolide (LHRH analogue). Flutamide is considered inferior to LHRH analogues. 10. While effective in the treatment of many bacterial infections, aminoglycosides can also cause nephrotoxicity and ototoxicity. Gentamicin sometimes causes severe vestibulotoxicity, resulting in potentially permanent vertigo and ataxia. Benign positional vertigo occurs when calcium crystals within the inner ear shift position. 1

Usmle World Step 2 Notes

Embed Size (px)

Citation preview

12/15/09

1. Febrile neutropenia (chemotherapy for tumor patient) is a medical emergency. Admit the

patient to the hospital, obtain blood cultures, then start broad-spectrum antibiotics such as IV

ceftazidime or cefepime. The most common site of mucositis si the gastrointestinal tract, and

the frequently identified organism are G-, Particular P.aeruginosa.

2. In patient with acute ‘mechanical’ back pain (disc herniation) without significant neurologic

deficit, conservative approach is preferred for a period of 4-6 weeks. This includes early

mobilization, muscle relaxants, and NSAIDs. Best rest and physical therapy has not been

shown to be help. If the pain persists after 4-6 weeks of conservative treatment or progressive

neurologic deficit evolves, high-resolution diagnosis modalities are usually employed: MRI

and CT with or without contrast myelography.

3. Advanced age is probably the single most important risk factor for breast cancer. That is why

women of more than 40-50 years of age would need regular annual mammograms.

Interestingly, only 10 percent of women diagnosed with breast cancer have a positive family

history.

4. When questionable ulcer is visualized by flexible sigmoidoscopy, a biopsy is recommended

for two purposes: (1) to rule out possible cancerous lesions, and (2) distinguish ulcerative

colitis from Crohn’s disease, since the treatment of each given conditions is different.

5. Mitral valve prolapse (MVP) typically causes a mid-systolic click over the cardiac apex, with

a short systolic murmur at the apex after the click if mitral regurgitation is present. Squatting

decreases the murmur. MVP is sometimes associated with chest pain, anxiety, palpitations,

and/or hayperventilation.

6. The most common cause of overflow incontinence in elderly males is an enlarged prostate,

which is usually revealed by rectal examination, and is characterized by a high post-void

residual volume.

7. The typical symptoms of aortic stenosis are exertional dyspnea, syncopy, and angina. The

physical exam will reveal a systolic ejection murmur radiating to the apex and carotid arteries.

8. A prolonged QRS interval (i.e., typically bundle branch block) suggests a bradyarrhythmia,

whereas a prolonged QT interval suggests a tachyarrhythmia.

When the heart rate is lower than 30bmp, cerebral perfusion is impaired, even if there is a

compensatory increase in stroke volume. A patient with a disease of the sinus node (SSS) or

conduction system can have intermittent bradycardia and typically manifest with recurrent

syncopy episodes. Ventrical beats, by themselves, are not associated with syncope. The most

important differential diagnosis of syncope is generalized seizures. Distinguishing features of

a seizure include the presence of sustained clonic-tonic movement, tongue biting, and

postictal confusion.

9. Palliate radiation, along with anti-androgen therapy, is the treatment of choice for metastatic

prostate cancer. Anti-androgen therapy consists of Leuprolide (LHRH analogue). Flutamide is

considered inferior to LHRH analogues.

10. While effective in the treatment of many bacterial infections, aminoglycosides can also cause

nephrotoxicity and ototoxicity. Gentamicin sometimes causes severe vestibulotoxicity,

resulting in potentially permanent vertigo and ataxia.

Benign positional vertigo occurs when calcium crystals within the inner ear shift position.

1

Patients present with recurrent, acute-onset vertigo that is precipitated by head movement.

Nystagmus, nausea and vomiting are common associated findings.

11. Suspect alpha-a antitrypsin (AAT) deficiency in non-smoking adults <45 years of age with

panlobular emphysema on chest x-ray. Patients with AAT deficiency are also at risk for liver

disease (neonatal hepatitis, cirrhosis, and/or liver failure). Measurement of the serum alpha-

antitrypsin level is minimally invasive but can be highly diagnosis.

12. Acute mitral regurgitation can occur due to papillary muscle dysfunction in patients with

acute myocardial infarction. Acute mitral regurgitation characteristically causes a rise in left

atrial pressure without significant changes in left atrial size or left ventricular size or left

ventricular ejection fraction.

13. In patient presenting with significant peripheral vascular disease, calcium channel blockers

are preferred as anti-hypertensive agents.

14. Normally, tachypnea is the mode of respiratory compensation for metabolic acidosis, as it

allows for increased CO2 losses in exhale air. Relative hypoventilation in patients with COPD

can impair this respiratory compensation. (cause lathargy).

Winter’s formula: pCO2=1.5[HCO3]+8

Defective tubular bicarbonate reabsorption is the defect seen in type II renal tubular acidosis

(RTA). Type II RTA is oftentimes inherited, and may be a component of Fanconi syndrome. Type

II RTA can also occur with use of drugs like carbonic anhydrase inhibitors.

Low tubular ammonium production is a component of Type I RTA. The primary defect in type

I RTA is failure to excrete sufficient H+ in the urine. Without sufficient hydrogen ions in the

tubular fluid, ammonium cannot be produced.

15. Atrial flutter is recognized by a sawtooth pattern on EKG. In atrial flutter, the ventricular rate

is determined by AV node conduction. A 2:1, 3:1 or 4:1 heart block is often seen. (The

symptom is not caused by atrial ectopy—P wave).

16. The serum albumin level should always be measured simultaneously with the serum calcium

level in order to calculate the correct total serum calcium value. With every 1 g/dL change in

serum albumin, serum calcium changes by 0.8 mg/dL.

17. Suspect multiple myeloma in an elderly patient with anemia, renal failure, and hypercalcemia.

Dx: Serum immunoelectrophoresis. Sx: back pain—most common

18. Indicators of a severe asthma attack include normal to increase PCO2 values (indicates CO2

retention because of severe airway obstruction (air trapping) and /or respiratory muscle

fatigue.), speech difficulty, diaphoresis, altered sensorium, cyanosis, and ‘silent’ lungs.

Lung hyperinflation and tachycardia are typical findings during an asthma attack and not

indicators of severity.

19. Typical and atypical antipsychotics are good for treating acute agitation in elderly and

demented patients. Atypical antipsychotics like quetiapine and risperidone may also be used.

It is important to know that prolonged use of both typical and atypical antipsychotics can

increase mortality in elderly, but they are safe in the acute setting. (Lewy body dementia is

one exception where typical antipsychotics should not be used, as these patients tend to be

very sensitive to these medicines)

Lorazepam and other benzodiazepines may be used to treat agitation in young patients. They

are also beneficial in treating alcohol withdrawal. However, benzodiazepines are typically

contraindicated in older patients for several reasons. First, older patients tend to metabolize these

2

drugs slowly, making their effects very long-lasting. Plus, in the elderly, benzodiazepines often

worsen confusion. Finally, the elderly are at increase risk for all adverse events associated with

benzodiazepines including withdrawal, dependence, and motor impairment.

Memantine is an agent used to treat moderate to severe Alzheimer’s disease. It works by

blocking glutamate’s actions on the NMDA receptor.

20. Anticoagulation with warfarin places patients at risk for hemorrhage. Retroperitoneal

hematoma may occur even without a supratherapeutic INR. Back pain and signs and

symptoms of hemodynamic compromise should raise suspicious for retroperitoneal

hematoma. (know CT scan)

21. The diagnosis of acute limb ischemia should be considered in patients who present with the

‘five’ P’s: pain, pulselessness, paresthesias, poikilothermia (coldness), and pallor.

Angiography will typically show an abrupt cutoff of arterial blood flow. IV heparin should be

started immediately upon suspicion. The definitive treatment is surgical embolectomy or

intra-arterial fibronolysis/mechanical embolectomy via inteventional radiology. Streotokinase

is a fibronolytic agent. While IV infusion of a fibrinolytic agent is an option for STEMI

patients, patients with acute limb ischemia typically are treated with direct intra-arterial, not

IV, administration of a fibrinolytic agent on the clot using an angiographic catheter.

22. Staphylococcal scalded skin syndrome is a disease of children. It presents with sudden onset

diffuse erythema, skin tenderness, fever, flaccid bullae, facial edema and perioral crusting.

Toxic epidermal necrolysis is more benign than staph scalded skin syndrome. Hot, tender,

erythematous morbilliform or discrete macules and loose patches of skin are characteristic. The

oral mucosa is frequently involved, with blistering and erosions. Patients give a history of taking

sulfonamides, phenytoin, barbiturates, or NSAIDs.

Scarlet fever is a group A beta-hemolytic Strep infection. Exudative tonsillitis precedes the

condition. The rash typically appears as scarlet macules over generalized erythema.

Erysipelas begin as a small erythematous patch that progresses to a red, indurated, tense and shiny

plaque. It usually occurs over the cheek, and there is often a history of trauma or pharyngitis. The

presence of ‘raised, sharply demarcated’ advancing margins is a classic feature. Local signs of

inflammation are universal. Overlying skin streaking and regional lymphadenopathy indicate

lymphatic involvement. Over 80% of the cases are due to Strep, thus penicillin is the drug of

choice.

Impetigo is a contagious infection of the skin that is caused by Staph, or strep or both. The lesions

appear as macules, vesicles, and bullae with honey colored crusts.

23. Acyclovir can cause crystalluria with renal tubular obstruction during high-dose parenteral

therapy, especially in inadequately hydrate patients.

24. Eczema herpeticum is a form of primary herpes simplex virus infection associated with atopic

dermatitis. Numerous vesicles over the area of atopic dermatitis are typical. The infection can

be life-threatening in infants; thus prompt treatment with acyclovir should be initiated.

25. The known risk factors for the development of pancreatic cancer include: family history,

chronic pancreatitis, smoking, DM, obesity, and a diet high in fat. Alcoholism is not a risk

factor pancreatic cancer.

26. 3 clinical findings are characteristic of aortic dissection: abrupt onset ‘tearing’ pain in the

chest or back, variation in pulse or blood pressure between the right and left arms, and

widened mediastinum on CXR. When two or more of these findings are present, the

3

incidence of dissection exceeds 80%.

27. Iron deficiency anemia is one of the most common presentations of celiac sprue (intermittent

abdominal distention, flatulence, greasy stools—malabsorption syndrom). Up to 24% of cases

of celiac disease is associated with dermatitis herpetiformis.

28. OHS (Obesity hypoventilation syndrome) is defined by severe obesity (>150% of ideal body

weight) and alveolar hypoventilation during wakefulness. The ABG will demonstrate

hypercapnia, hypoxemia and respiratory acidosis (pH 7.3, pO2 60mmHg, pCO2 69 mmHg) as

a consequence of decreased lung compliance. Weight loss, ventilator support, oxygen therapy,

and progestins (a respiratory stimulant) are all potential therapies for these patients.

29. The basic pathology in myasthenia gravis lies at the neuromuscular junction (that’s why it’s

called myasthenia) and is mediated by auto-antibodies against the acetylcholine receptor.

30. HIV patients are at high risk for tuberculosis. A positive PPD test (>5mm in HIV) require

prophylaxis with isoniazid (+pyridoxine) for 9 months. Alternatives to the use of isoniazid

include the use of pyrazinamide with rifamipin or rifabutin for 2 months. If a patients if

intolerant ot pyrazinamide, rifampin is given alone for 4 months. A combination of 3 or 4

drugs is used to treat active tuberculosis infection.

31. Situational syncope should be considered in the differential diagnosis of syncopal episodes.

The typical scenario would include a middle age or older male, who loses his consciousness

immediately after urination, or a man who loses his consciousness during coughing fits.

32. Hypertension in patients with thyrotoxicosis is predominantly systolic and caused by

hyperdynamic circulation.

33. False negative

34. Lymphogranuloma venereum (LGV) is a sexually transmitted disease caused by Chlamydia

thrachomatis serotypes L1,2 and 3. The disease starts to 1 to 4 weeks after initial contact and

manifests with generalized malaise, headaches and fever. A papule appears and subsequently

transforms into an ulcer, typically located in the vulvovaginal region. The ulcer is painless and

the disease may go unnoticed until inguinal adenitis develops about a month later. In women,

however, the lymphadenopathy involves more commonly the deep nodes around the rectum

and anus. If untreated at this stage, LGV progresses into a severe and chronic disease causing

ulceration, proctocolitis, rectal stricture, rectovaginal fistulas and elephantiasis.

Granuloma inguinale (GI) is a distinct condition that presents similarly but is due to Donovania

granulomatis. Unlike LGV, the ulcer and lymphadenopathy of granuloma inguinale present at the

same time. Moreover, the ulcer of GI has irregular borders and is characterized by a beefy red

granular base.

35. Once the specific causative bacterium responsible for native valve bacterial endocarditis is

identified, the antibiotic coverage can be narrowed from empiric to specific. Native valve

bacterial endocarditis due to Strep viridans highly susceptible to penicillin is well-treated with

IV penicillin G or IV ceftriaxone.

36. While most live vaccines (BCG, varicella, anthrax, oral typhoid, intranasal influenze, oral

polio, and yellow fever) are C/I in patients with HIV, the MMR vaccine is an exception and

can be given to patients who have CD4 counts greater than 200/mL and not evidence or

history of an AIDS-defining illness.

37. Hyperparathyroidism predisposes to the development of pseudogout. Pseudo tends to present

as an acute onset, painful monoarthritis affecting the knee. Rhomboid shaped, positively

4

birefringent crystals are diagnosic.

Struvite crystals have the shape of coffin lids and are seen in nephrolithiasis causes by chronic

urinary tract infection with urease-producing organisms.

38. The null hypothesis is always the statement of no relationship between the exposure and the

outcome. To state the null hypothesis correctly, you should recognize the study design first.

Cross-section: no association; cohort study: the same

39. The most important causes of thyrotoxicosis with low radioactive iodine uptake include: 1.

subacute painless thyroiditis; 2. subacute granulomatous thyroiditis—De Quervain’s, intense

pain; 3.iodine-induced thyroid toxicosis; 4.levothyroxine overdose; 5. Struma ovarii

40. Leydig cell tumors, the estrogen production can be increased with secondary inhitibition of

LH and FSH. High testosterone, high estrogen

41. Howell-Jolly bodies (single round, blue inclusions on Wright stain) are nuclear remnants

within RBC typically removed by the spleen. Their presence strongly suggests physical or

functional hyposplenism.

42. Hypertrophic cardiomyopathy is inherited in an autosomal dominat pattern.

43. Hypertension can be one of the presentating signs of polycythemia (know the various

presentations).

44. Steroid-induced myopathy is a well described result of long-term corticosteroid use. It is

characterized by painless proximal muscle weakness. Steroid myopathy will improve slowly

once the offending medication is discontinued. Serum Ck and ESR are normal. Insidious,

acute and chronic.

12/16/09

1. Lead-time bias should always be considered while evaluating any screening test. This bias

occurs when there is an incorrect assumption or conclusion of prolonged apparent survival

and better prognosis due to a screening test. What actually happens is that detection of the

disease was made at an earlier point in time, but the disease course itself or the prognosis

didn’t change. So the screened patients appeared to live longer from the time of diagnosis to

the time of death. (USMLE tip: think of lead-time bias when you see ‘a new screening test’

for poor prognosis diseased like lung or pancreatic cancer.)

2. The main causes of metabolic alkalosis include exogenous administration of alkali, removal

of acidic gastric secretions due to vomiting or NG tube aspiration, renal hydrogen ion loss due

to mineralocorticoid excess and contraction alkalosis.

3. Hyperemesis gravidarum typically causes metabolic alkalosis with respiratory compensation

via hypercapnia. PE causes a primary respiratory alkalosis due to hyperventilation.

4. Blood PH 7.43, PaO2 100mmHg, PaCO2 25mmHg, HCO3- 16 mEq/L. Aspirin intoxication

causes a mixed respiratory alkalosis and metabolic acidosis. Respiratory alkalosis results from

increased respiratory drive, while metabolic acidosis results from the accumulation of

salicylate and organic acids.

5. Memorize!Lesion Manifestation posterior limb of the internal capsule (lacunar infact)

unilateral motor hemiparesis of the face, arm and leg without any higher cortical dysfunction and visual field abnormalities.

Middle cerebral artery contralateral hemiplegia, conjugate eye deviation toward the side

5

occlusion of the infarct, hemianesthesia and homonymous hemianopia. *aphasia (dominant hemisphere), *hemineglect (non-dominant hemisphere lesions).

Anterior cerebral artery occlusion

contralateral weakness that predominantly affects the lower extremity. Associated findings include abulia, akinetic mutism, emotional disturbances, deviation of the head and eyes toward the lesion, and sphincter incontinence.

vertebrobasilar system lesion (*supplying the brain stem)

‘alternate’ syndromes, with contralateral hemiplegia and ipsilateral cranial nerve involvement.

6. Toxic adenoma presents as symptoms suggestive of thyroid toxicosis. There is radioactive

iodine uptake in the nodule, and suppression of uptake in the rest of the thyroid gland. Patients

with toxic nodule do not have infiltrative ophalmopathy.

Patients with Hashimoto’s thyroiditis are hypothyroid or euthyroid. The thyroid scan shows

heteogenous pattern.

Grave’s –diffusely increased radioactive iodine uptake + infiltrative ophalmopathy+pretibial

myxoedema.

The radioactive iodine uptake in patients with painless thyroiditis is markedly reduced.

7. Esophagoscopy is indicated when a patient with GERD fails to repond to empiric treatment,

or when a patient had features of complicated disease.

8. Nephritic syndrome is characterized by dependent edema, hypertension, and hematuria

(dysmorphic red blood cells or red blood cell cast). Rash, low-grade fevers and proteinuria

may also be present.

9. Agioedema is characterized by the rapid onset non-inflammatory edema of the face, acral

extremities, genitals, trachea and abdominal organs. It is due to a deficiency in C1 esterase

inhibitor, which results in elevated levels of the edema-producing factors, C2b and

bradykinin.

Low C1q levels are associated with familial systemic lupus erythematous.

10. Fluphenazine is a high potency ‘typical’ antipsychotic medication that occasionally causes

hypothermia by disrupting thermoregulation and the body’s shivering mechanism. Patients

taking antipsychotics should be advised to avoid prolonged exposure to extreme temperatures.

11. Any male adolescent who presents with epistaxis (nosebleeds), a localized mass, and a bony

erosion on the back of the nose had an angiofibroma until proven otherwise. This is typically

found in the back of the nose or upper throat (nasopharynx). Reactive nasal polyps normally

do not cause bony erosions. These are usually associated with chronic infections or allergies,

and the main complaint is obstruction rather than bleeding.

12. Osteoarthritis is a non-inflammatory arthritis presenting with pain that is worse with activity

and improved with rest. X-ray findings include joint space narrowing and osteophytes.

Synovial fluid analysis will reveal fewer than 2000 WBC/ml, no organisms, and no crystals.

Gouty arthritis—punched out erosions with a rim of cortical bone. 2,000-50,000 WBC/ml,

negative birefringent crystals.

RA—periarticular osteopenia and joint margin erosions. 2,000-50,000 WBC/ml

Infectious arthritis—normal joint space with soft tissue swelling, >50,000 cells/ml Gram staining

Psudogout, or calcium pyrophosphate dyhydrate (CPPD) deposition—calsification of

cartilaginous structures (chondrocalcinosis).

6

13. indications for upper endoscopy in the management of GERD: 1-Nausea/vomiting; 2-Weight

loss, anemia or melena/blood in the stool; 3-Long duration of symptoms (>1-2 years), esp in

Caucasian males >45 years old; 4-Failure to respond to proton pump inhibitors.

14. Vitiligo is an autoimmune condition characterized by area of depigmentation lacking

melanocytes. Vitiligo is sometimes associated with other autoimmune conditions such as

pernicious anemia, autoimmune thyroid disease, type I diabetes mellitus, primary adrenal

insufficiency, hypopituitarism, and alopecia areata.

15. hemochromatosis is an autosomal recessive disorder characterized by increased skin

pigmentation, diabetes, cirrhosis and arthralgia in the later stages.

16. CDC vaccination schedules on a yearly basis:

Tetanus and diphtheria (Td): every 10 years after 18. A single tetanus, diphtheria, and acellular

pertussis (Tdap) booster between 19-64.

Influenze: all adults agen 50 years and older. It should also be given to all adults with chronic

cardiovascular, pulmonary, hepatic, renal, or metabolic (e.g.diabetes) disease, immunosupprssion,

or pregnancy. Healthcare workers, close contact of children aged 0-59 months, and those in

nursing homes should also be vaccinated. Healthy, non-pregnant persons aged 5-49 years may

receive the intranasal influenza vaccine.

Pneumococcal vaccine (PCV): all adults aged 65 years and older. It should also be given to all

adults with chronic cardiovascular, pulmonary, hepatic, renal, or metabolic (e.g. diatetes) disease,

or immunosuppression. Persons vaccinated before age 65 need a booster in 5 years.

17. Dubin-Johnson and Rotor syndrome are two familial disorders of hepatic bile secretion that

result in conjugated hyperbilirubinemia. A dark granular pigment is present in the hepatocytes

of patients with Dubin-Johanson, but not Rotor. For diagnosis of Dubin-Johnsom, conjugated

hyperbilirubinemia with a direct bilirubin fraction of at least 50% and an otherwise normal

liver function profile must be present. Confirmation can be obtained by evaluating the urinary

coproporphyrin for usually high levels of coproporphyrin I80% (normal 80% III) .

18. Fresh frozen plasma is the therapeutic agent of choice for the coagulopathy in patients with

liver failure.

19. Primary HIV infection causes a febrile illness that can closely resemble infectious

mononucleosis. The key distinctions: rash (unless antibiotics have been administered),

diarrhea—HIV; a tonsillar exudates –EBV

20. The sudden onset of vertigo, vomiting and occipital headache in a hypertensive patient is

strongly suggestive of cerebellar hemorrhage. Other manifestation are 6th nerve paralysis,

conjugate deviation, blepharospasm and coma.

Heat stroke—failure of body thermoregulation upon exposure to high environmental temperatures.

High core body temperature (>105F), dehydration, confusion, coma, and dry, flushed skin. “heat

exhaustion” less severe.

Vestibular neuronitis—acute onset of vertigo and nystagmus without any other neurological deficit

Meiiere disease—increase pressure of the endolymph, vertigo, tinnitus, hearing problem. Vomiting

is not prominent.

21. V/Q scan is a useful tool in diagnosis pulmonary thromboembolism. Mismatched perfusion

defect is characteristic, but it is found in less than 50% of the patients.

22. Actinic keratoses—sun –exposed, erythemarous papules with a central scale and a ‘sand

paper-like’ texture, squamous cell carcinoma.

7

Atopic dermatitis –light microscopy—spongiosis (edema of the epidermis)

23. The typical VitK deficiency –NPO for a prolonged period of time + broad spectrum

antibiotics. PT is elevated more than PTT.

Similar in newborn who had not received prophylactic vitK (home born)

24. Hypercalcemia and intractable ulceration –MEN syndromes

25. ARDS-acute onset, bilateral patchy airspace disease on CXR, PCWP<18 or no clinical

evidence of increase LVEDP, and PaO2/FiO2 <200.

26. Suspect rosacea in 30- to 60- year old pts with telangiectasia over the cheecks, nose and chin.

Flushing of these areas is typically precipitated by hot drinks, heat, emotion and other causes

of rapid body temperature changes. Topical antibiotic such as metronidazole is the most

frequently prescribed initial therapy. DD: Acne vulgaris, seborrheic dermatitis, carcinoid

syndrome, SLE

27. Infective endocarditis –progressive subacute fevers, chills, malaise, and dyspnea. It is

diagnosed using Duke’s criteria. Complications include vascular (i.e.embolic) and immune

complex phemomena.

28. P.jiroveci is an opportunistic pathogen, and an important cause of pneumonia in

immunocompromised hosts (chemotherapy). Bilateral diffuse interstitial infiltrates beginning

in the perihilar region is a characteristic finding on CXR.

29. Cohort study—incidence. Cross-section—prevalence.

30. TB CXR

31. Viral arthritis (parvovirus, hepatitis, HIV, mumps and rubella) can present with symmetric

small joint inflammatory arthritis. It is distinguished for other causes of symmetric

inflammatory arthritis by the fact that it tends to resolve with two months. Positive

inflammatory markers such as ANA and RF may occur. Tx involves use of NSAIDs for

resolution of symptoms. Antiviral therapy is unnecessary as the symptoms are self-limited.

32. Repeated nocturnal awakenings associated with dyspnea, cough and a chocking sensation, esp

in an overweight or obese pt.—GERD. Central sleep apnea—not classically choking, cough

33. De Quervain tenosynovitis –mother hold baby with thumb outstretched (abducted/extended)

34. Patient with an inferior MI may suffer right ventricle infarction, leading to right sided heat

failure. Hypotension, jugular venous distention and clear lung fields should raise suspicion for

right ventriclar infarction. Tx—IV fluids and avoidance of preload reducing medications such

as nitrates and diuretics.

35. Fatty liver (steatosis), alcoholic hepatitis and early fibrosis of the liver can be reversible with

the cessation of alcohol intake. True cirrhosis (with regenerative nodules) is irreversible,

regardless of alcohol abstinence.

36. Individual who received blood transfusions before 1992 should screen HCV; before 1986-

HBV.

37. Herpetic whitlow is the most common viral infection of the hand. HSV-1 &2, self-limiting.

Increase risk in health care workers.

38. Toxic megacolon—MCC is ulcerative colitis. Tx—IV steroids, nasogastric decompression

and fluid management.

39. A compression fracture of the vertebrae is a common complication of advanced osteoporosis

—acute back pain without an obvious preceding trauma in a predisposed patient. Neurologic

exam will be normal. Note: absent ankle reflex can be seen in elderly patients as part of the

8

normal aging process.

Lumbago is usually related to physical strain and has less dramatic onset. Usually paravertebral

muscle tenderness rather than spinal tenderness is the feature.

40. Rupture of the ventricular free wall is a mechanical complication of MI with peak incidence

3-7 days after anterior wall MI. Acute decompensation and PEA (pulseless electrical activity)

secondary to rapid development of tamponade is typical.

41. Digitalis toxicity –increase ectopy and vagal tone. Specific—Atrial tachycardia with AV block

42. Oral isotretinoin –moderate to severe acne that is predominantly nodulocystic form, and those

developed scars. High yield!!!

oral antibiotics (tetracyclines)—moderate to severe inflammatory acne.

Mild, inflammroty—topically benzyl peroxide, antibiotics (erythromycin, clindamycin,

tetracyclins

Initial, non-inflammatory comedones—topical retinoids

43. Ankylosing spondylitis—HLA B27

44. Primary hyperaldosteronism (Conn’s syndrome)—hypertension, mild hypernatremia,

hypokalemia and metabolic alkalosis. Dx—a low rennin level and an elevated aldosterone

level.

12/17

1. Acute vertigo (sensation of spinning of head) is often due to the dysfunction of the labyrinth.

BPPV (benign paroxysmal positional vertigo)—vertigo related to head positioning. Global

cerebral ischemia results in presyncope (lightheadedness with visual blurring, not spinning)

and syncope.

2. SLE—young woman with joint pain, fatigue, ankle edema and facial rash. Kidney disease—

immune complex-mediated, serum C3 level is decreased. IgE-mediated reaction is important

in allergic interstitial nephritis like methicillin-induced interstitial nephritis. Cytotoxic

antibodies cause Goodpasture’s syndrome. Delayed hypersenditivity reaction may be

important in non-specific chronic glomerulonephritis. Non-immune kidney damage may be

related to toxins, hypertension, diabetes mellitus or other factors.

3. MS usually presents during the 3rd or the 4th decade with recurrent focal neurologic

dysfunction. The attacks are non-predictable and erratic in presentation. The symptoms may

last a few weeks with variable recovery. MS is one of the few conditions that may present

with bilateral trigeminal neuralgia. Another clue is the presentation of optic neuritis.

4. Theophylline toxicity – CNS stimulation (headache, insomnia), GI disturbances (nausea,

vomiting), and cardiac toxicity (arrhythmia). The mechanisms responsible for the toxicity

may include phosphodiesterase inhibition, adenosine antagonism, and stimulation of

epinephrine release. * Remember the interaction with ciprofloxacin and erythromycin.

5. Pressors such as norepimephrine can cause ischemia of the distal fingers and toes sencondary

to vasospasm. The diagnosis is suggested by symmetric duskiness and coolness of all

fingertips. Raynaud’s phenomenon—pallor to cyanosis to erythema. Cholesterol emboli

—“blue toe syndrome”, symmetrical involvement of all digits unusual.

6. AF is the MCC chromic arrhythmia in adults. HR–irregularly irregular, rapid rate. Rhythm

control or rate control plus anticoagulation may be used to decrease the risk of embolic stroke

in patients with atrial fibrillation.

9

7. The first step in Tx fo acute variceal bleeding is to establish vascular access with two large

bore IV needles or a central line.

8. In HIV-infected Pt, diarrhea can be due to a variety of organisms. Therefore, and attempt

should be made to identify the causal organism prior to instituting therapy so that appropriate

antibiotic may be chosen.

9. Pt who have had an episode of rheumatic fever should receive antibiotic prophylaxis with

penicillin to prevent further attacks. The duration of treatment is not firmly established, but

most physicians will treat until the pt reaches the age of 18.

10. The ATP 3 (adult treatment Panel 3) recommends lifestyle and /or medications to decrease

the risk of coronary artery disease or coronary artery disease-related outcomes based upon

risk factors and LDL cholesterol levels. Risk category LDL Goal LDL levels at which

lifestyle modification starts

LDL level at which drug thrapy starts

CAD or CAD risk equivalent

<100 mg/dL ≥100 mg/dL ≥130 mg/dL

2+ risk factors <130 mg/dL ≥130 mg/dL ≥160 mg/dL0-1 risk factors <160 mg/dL ≥160 mg/dL ≥190 mg/dLRisk factors: men ≥45 years, women ≥55 years hypertension,smoking, HDL < 40 mg/dL, and

family history of premature CAD (male <55, femail <65). HDL≥60 negates one risk factor. CAD

risk equivalent: DM, carotid artery disease, abdominal aortic aneurysm, peripheral arterial disease,

or a 10-year risk of CAD of ≥ 20%.

11. The classic triad of RCC (flank pain, hematuria, and a palpable abdominal renal mass) is

uncommon (10%), when present, it strongly suggests advanced/metastatic disease. Hematuria

is seen only in about 40% pt. Scrotal varicocele, (the majority are on left-side), may be

observed in a few Pt (<10%). Variceles typically fail to empty when the pt is recumbent.

Presence of this finding should always raise suspicion for a RCC, which is most commonly

due to obstruction of the gonadal vein where it enters the renal vein. 20% of pt may also have

constitutional symptoms like fever, night sweats, anorexia, weight loss, or an easy

fatigability. Increased erythropoietin by kidney mass can produce polycythemia and

thrombocytosis. CT scan of the abdomen is most sensitive and specific for diagnosis the

RCC.

12. Suspect Alport’s syndrome in pt with recurrent episodes of hematuria, sensorineural deafness

and a family history of renal failure. EM—alternating areas of thinned and thickened

capillary loops with splitting of the glomerular basement membrane.

13. Serum sickness-like reaction may occur 1-2 weeks after administration of certain drugs, such

as penicillin, amoxicillin or cefaclor in the setting of a viral illness. Prominent symptoms

include fever, urticarial rash, polyarthragia and lymphadenopathy. The condition derives its

name from its similarities to true serum sickness, an immune-complex mediated

huypersensitivity reaction to non-human proteins. Serum-sickness like reaction is clinical

diagnosis, and should resolve with withdrawal of the offending agent; it does not represent a

true drug allergy.

14. Light’s criteria may be used to distinguish transudative and exudative pleural effusions.

Exudates have either an effusion to serum total protein ratio greater than 0.5, an effusion to

serum LDH ratio greater than 0.6, or and effusion LDH concentration greater than 2/3 the

10

upper limit of normal for serum LDH. Complicated effusions (PH<7.2, glucose <60 mg/dL,

or gram stain or culture) require chest tube drainage.

15. Vanishing bile duct syndrome is a rare condition characterized by progressive ductopenia, or

loss of intrahepatic bile ducts. The most common cause of ductopenia in adults in primary

biliary cirrhoiss. Other causes include failing liver transplantation, hodgkin’s disease, graft-

versus-host disease, sarcoid, CMV infection, HIV, and medication toxicity. PSC—are

periductal portal tract fibrosis and segmental stenosis of extrahepatic and intrahepatic bile

ducts. Bile duct loss is not classically observed.

16. Isonizid causes idiosyncratic liver injury with histological features similar to those seen in pt

with viral hepatitis.

17. Tinea corporis is a superficial fungal infection characterized by an erythematous, scaly,

pruritic rash with central clearing. It’s treated with topical antifungals—terbinafine.

18. The appropriate blood product to administer for anemia is packed RBC. In pt with

preexisting heart disease, a hemoglobin level of 10g/L or higher

(hematocrit ≥30%) should be maintained. Blood is collected from donors as whole blood,

but hen is separated into packed RBC, platelets, fresh frozen plasma, and cyoprecipitate. FFP

contains all clotting factors and plasma proteins. Thansfusion of FFP is generally used to

reverse a coagulopathy (e.g. to rapidly lower a high INR in a patient taking wafarin).

Cryoprecipitate is rich in factor VIII, fibrinogen deficiency, and VWF disease. It’s very

useful in volume sensitive patients.

19. The MCC of hypernatremia is hypovolemia. Mild cases can be treated with 5% dextrose in

0.45% saline. Severe cases should be initially treated with 0.9% saline.

20. Oral trimethoprim-sulfamethoxazole is effective in preventing PCP infection in transplant pt.

These pts should also be vaccinated against influenza, pneumococcus, and Hepatitis B.

Ganciclovir or vilgancivlovir can be used to prevent CMV infections.

21. Elevated BNP (brain natriuretic peptide) levels and an audible S3 are both signs of increased

cardiac filling pressure. Brain BNP is secreted in response to ventrical stretch and wall

tension when cardiac filling pressures are elevated. Specifically, cardiac myocytes secrete the

BNP precursor, proBNP, which is then cleaved into biologically active c- terminal BNP and

inactive n-terminal proBNP. It is common practice in the emergency department to measure

the BNP level to differential dyspnea of the heart failure from dyspnea of non-cardiac

etiology. The level of circulating BNP correlates both with the severity of left ventricular

filling pressure elevation as well as with mortality. A low BNP level argues against acute

heart failure with a very high negative predictive value.

22. Sarcoidosis is diagnosis in the setting of a compatible history, CXR findings of hilar

adenopathy with or without reticulonodular infiltrates, and biopsy of noncaseating

granulomas, Elevated serum ACE levels also provides support for the diagnosis. For

symptomatic disease, administration of system glusocorticoids is indicated.

23. One of the known complications of ERCP is an iatrogenic biliary enteric fistula characterized

by the presence of air in the biliary tree. Other complications included pancreatitis, biliary

peritonitis, sepsis, hemorrhage, and adverse effects from the contrast, sedative, or

anticholinergic agents.

24. Chronic Myelogenous leukemia (CML) is characterized by leukocytosis, anemia, and

increased number of mature granulocytic forms, such as segmented neutrophils and band

11

forms on peripheral blood film. The disease is mostly seen after the age of 50 and presents

with fatigue, malaise, low-grade fever, anorexia, weight loss and bone pains. Night sweats

and fever associated with increased metabolism due to granulocytic cell tumor turn over may

occur. Examination of a bone marrow biopsy specimen reveals hypercellularity with

prominent granulocytic hyperplasia. The leukocyte alkaline phosphate score is low. The only

other diseases where this may happen are hypophosphatemia and paroxysmal nocturnal

hemoglobinuria (PNH).

Elevated leukocyte alkaline phosphatase is characteristic of Leukemoid Reaction. Presence of

Philadelphia chromosome and the low leukocyte alkaline phosphatase makes the CML more likely

than the leukemoid reaction.

Increased bone marrow iron is seen in hemachromatosis, anemia of chronic disease and

sideroblastosis.

Absent of measurable erythropoietin in Urine is an important diagnostic feature of polycythemia

vera.

Auer rods are seen in AML.

Tartrate resistant acid phosphatase is present in hairy cell leukemia.

25. Indications for hemodialysis: 1-refractory hyperkalemia; 2-vulume overload or pulmonary

edema not responding to diauretics; 3-refratory metabolic acidosis (PH<7.2); 4-Uremic

pericarditis; 5-Uremic encephalopathy or neuropathy; 6-coagulopathy due to renal failure.

26. Hypocalcemia can occur during or immediately after surgery in parients undergoing major

surgery and requiring extensive transfusions. Hperactive deep tendon reflexes may be the

initial manifestation.

27. papilledema usually presents with transient loss of vision lasting a few seconds with changes

in head position and can be confirmed with ophthalmologic examination. Norma individual

have a blind spot in there visual fields at the location of the optic nerve head, but this blind

spot enlarges in pts with papilledema. Papilledema is caused by increased intracranial

pressure, which has its own associated symptoms such as morning headaches or changes in

headache intensity with position.

Optic neuritis is commonly associated with multiple sclerosis, but can be seen with other disorders

as well. It typically presents as unilateral eye pain and visual loss with an associated afferent

papillary defect.

Amaurosis fugax is transient monocular blindness that lasts only a few minutes and is usually

vascular in origin. It would be unlikely in the is pt given young age and low risk of

atherosclerosis.

Uveitis can present with eye pain and redness if it is in the anterior chamber, or painless visual

changes secondary to floaters or reduced visual acuity if the posterior chamber is affected.

28. Vitreous hemorrhage typically presents as a sudden loss of vision and onset of floaters. It is

characterized by a fundus that is hard to visualize, an even if it visualized, details may be

obscured. It’s the MCC in diabetic retinopathy. For pt with underlying medical conditions,

conservative treatment (i.e. upright position during sleep, which enhances settling of the

hemorrhage) is recommended.

Retinal detachment—pt typically complain of photopsia with showers of floaters.

Central retinal vein occlusion presents as a sudden, painless, unilateral loss of vision. It is also

noted in pt with history of hypertension. Opthalmoscopic sighs are disk swelling, venous dilation

12

and tortuosity, retinal hemorrhage, and cotton wool spots.

Diabetic retinopathy pt are usually asymptomatic, even though changes in fundoscopy are seen.

Vitreous hemorrhage frequently occurs in pt with proliferative diabetic retinopathy. Once a

diabetic patients presents with a sudden onset of visual loss with numerous floaters, a vitreous

hemorrhage is most likely to have occurred.

Pt with macular degeneration typically present with painless progressive blurring of central vision,

which can be acute or insidious. It occurs bilaterally.

29. HAV is the most common vaccine-preventable disease among travelers. The HAV vaccine

should be considered for people anticipating travel to developing countries. CDC

recommends—HAV, HBV, typhoid vaccine as well as a polio booster vaccine, HAV—Asia

and African countries. Yellow fever—sub-Saharan African and equatorial South American

sountries. Rabies—outdoors or in rural areas, children.

30. Acute post-strep glomerulonephritis occurs 10-20 days after strep throat or skin infections. It

presents with hematuria, hypertension, red cell casts, and mild proteinuria.

31. CT scan of the head without contrasts is the initial diagnostic test of choice when a patient

presents with impaired consciousness or seizures or focal neurologic signs.

32. Spinal cord compression is characterized by signs and symptoms of upper motor neuron

dysfunction distal to the site of compression, such as weakness, hyperreflexia, decreased

sensation, and bowel/bladder dysfunction (saddle anesthesia and compromised bladder or

bowel control). Cord compression is a medical emergency requiring prompt diagnosis via

spinal MRI.

33. Vancomycin + ceftrixone—community-acquired bacterial meningitis in adults and children

since it covers the 3 most frequent etiologic agents: Strep. Pneumoniae, H. influenzae, and

Neisseria meningitides. Many pneumococcal stains have become resistant ot penicillin and

cephalosporins, thus, vancomycin is needed in addition to ceftriaxone. Vancomycin alone

does not sufficiently penetrate the blood-brain barrier.

Ampicillin is included in the empiric regimen to cover Listeria monocytogenes, which is also an

important cause of meningitis in pt older than 55. Other pt who are at risk include

immunocompromised pt, pt with malignancies (esp lymphoma), and pt taking corticosteroids.

IV cefotaxime + ampicillin – pt less than 3 months of age. Cefotaxime covers the 3 common

community-acquired pathogens and ampicillin covers Listeria. Although ceftrixone can be used in

neonates, cefotaxime is generally preferred because ceftrizxone have been associated with biliary

sludging.

IV ceftazidime+vancomycin –hospitalized pt who develop meningitis, esp after neurosurgery,

These drugs cover pseudomonas and Strap aureus respectively.

34. hypertestrogenism in cirrhosis leads to gynemastia, testicular atrophy, decreased body hair,

spider angiomas and palmar erythema.

35. Herpes zoster ophthalmicus is characterized by dendriform corneal ulcers and a vesicular

rash in the trigeminal distribution.

Herpes simplex keratitis presents with pain, photophobia, and decrease vision. Dendritic ulcer is

the most common presentation. There may also be minute clear vesicles in the corneal epithelium.

The typical vesicular rash of herpes zoster and system symptoms are not seen.

Dacryocystitis is an infection of the lacrimal sac due to obstruction of the nasolacrimal duct. It is

characterized by pain, swelling, tenderness, and redness in the tear sac area. Mucous or pus can be

13

expressed.

Bacterial keratitis is usually seen in contact lens wearers, and following corneal trauma. The

cornea appears hazy with a central ulcer and adjacent stromal abscess. Hypopyon may be present.

The history is the only factor by which a diagnosis of trigeminal neuralgia is made clinically. It is

characterized by a stabbing or shock-like pain over the distribution of the maxillary or mandubular

division of the trigeminal nerve. The pain may be triggered by light tough or vibration. Rash is not

seen.

36. Always suspect parenchymal brain hemorrhage whenever a patient presents with a history of

uncontrolled systemic hypertension and focal neurological signs.

37. Asymptomatic patients with Paget’s disease generally do not require any treatment.

Symptomatic pt are best treated with oral or IV biophosphonates (decrease bone resorption,

persists for months to years after tx is stopped. Superior to clacitonin—inhibits osteoclastic

bone absorption and reduces disease activity, weaker than bisphosphonates.)

Indications for tx: 1—bone pain; 2-Hypercalcemia of immobilization; 3-neurological deficit; 4-

high output cardiac failure; 5—preparatio ofr orthopedic surgery; 6—involvement of weight-

bearing bones (to prevent deformities).

38. HCV RNA is the single most sensitive serological marker used in screening marker used in

screening for HCV infection.

39. Chromic lymphocytic leukemia—LN biopsy confirms the diagnosis.

40. A essential tremor is characterized by a postural tremor (not a resting tremor unlike

Parkinson’s disease) and usually disturbs the performance of fine motor tasks. Sometimes, it

runs in families with AD inheritance. The pathophysiologic basis of the this condition is

unclear. The inhibition of the tremor by a small amount of alcohol is typical. Propranolol, a

non-selective beta-blocker, is effective in reducing the tremor probably by blocking beta-2

receptors.

41. Atracurium is a neuromuscular blocking agent that is metabolized in plasma and hydrolyzed

by serum esterases. Its use is safe in pt with renal and liver dysfunction. Pancuronium and

mivacurium are excreted mostly unchanged in the urine. Therefore, their effect is prolonged

in pt with renal dysfunction. Rocuronium is cleared mostly by liver. Succinylcholine is a

depolarizing agent with ultra-short duration of action. In pt with renal dysfunction,

succinylcholine may cause apnea and hyperkalemia.

42. Poroxysmal supraventricular tychycardia (PSVT) most commonly results from accessory

conduction pathway through the AV node. Vagal maneuvers and medications that decrease

conduction through AV nodes often resolve the PSVT.

43. Campylobacter jejuni is the most frequent precipitant of Guillain-Barre syndrome.

E.Coli (O157:H7) –hemolytic uremic syndrome

Samonella, Shigella, Campylobacter, Yersinia and Chlamydia—reactive arthritis.

44. Confidence interval (CI) of the mean is calculated by the following formula. CI

(mean)=mean ± standard score (z) ’standard error of mean (SE).

12/18/09

1. pt suffering from panic disorder have an increase incidence of depression, agoraphobia,

generalized anxiety and substance abuse. Dx criteria: 1- At least 3 attacks in a 3 week period.

14

2-No clear circumscribed stimulus. 3- Abrupt onset of symptoms which peak within 10

minutes. * extremely important question!!

2. Hyperpigmentation of the skin and mucous membranes is characteristic of primary

adrenocortical deficiency, and is due to the increased levels of ACTH. This clinical feature is

not seen in patients with secondary adrenal insufficiency, which is due to hypothalamo-

pituitary failure.

3. Diabetes insipidus presents as polyuria, polydipsia and excretion of dilute urine in the

presence of elevated serum osmolality. Primary polydipsia is due to excessive water drinking;

both plasm and urine are diluted. SIADH results in hyponatremia, low serum osmolality and

inappropriately high urine osmolality.

4. classic symptoms for carbon monoxide (CO) poisoning—polycythemia, nausea, dizziness,

and headaches, works in enclosed space where he is constantly exposed to CO from

automobile exhaust. Always consider CO poisoning in pt with environmental risk factor who

present with headache, nausea, and dizziness.

5. Ludwig’s angina is infection of the submaxillary and sublingual glands (bilateral cellulites,

typically Strep and anaerobes). The source of infection is most commonly an infected tooth,

usually the second or third mandibular molar. Asphyxiation is the most common cause of

death in this disease.

6. Congenital toxoplasmosis triad—chorioretinitis, hydrocephalus, and intracranial calcificatons.

Congenital rubella syndrome (CRS) triad—deafness, cataracts, and cardiac defects.

HSV—most infections are acquired during passage of the fetus through the birth canal—

encephalitis, chorioretinitis and disseminated disease.

7. Glasgow coma scale (GCS) is a component of the primary survey in all trauma pts.Eye openingSpontaneous 4To verbal command 3To pain 2None 1Verbal responseOriented 5Disoriented/confused 4Inappropriate words 3Incomprehensible sound 2None 1Motor responseObeys 6Localizes 5Withdraws 4Flexion posturing (Decorticate) 3Extension posturing (Decerebrate) 2None 1

Head injuries can be classified according to GCS score as minor (13-14), moderate (9-12) and

severe (<9).

8. Transfusion reactions occur acutely during or immediately following the transfusion of blood

products. Such reactions are immune-mediated; perform host antibody reacts with antigens on

transfused blood products causing the release of inflammatory mediators and activation of

complement. Acute reactions can be hemolytic, a severe reaction that may cause death, or

15

nonhemolytic, a dose-dependent self-limited reaction characterized by fever and rigors.

Malignant hyperthermia is caused by inhaled anesthetics. Patients typically have fever,

tachycardia, acidosis, rhabdomyolysis and are at high risk for cardiac arrest and death.

Drug fever most commonly occurs one to two weeks following initiation of therapy. It is more

commonly associated with the use of anticonvulsants and trimethoprim/sulfamethoxazole, though

the list of possible drugs is lengthy.

9. The abrupt onset of hypoxia with respiratory failure, cardiogenic shock, and DIC should raise

suspicion for amniotic fluid embolism (AFE) in a pregnant female who has undergone

amniocentesis or delivery. Respiratory support is always first step in management.

10. CVS is the best test for detection of fetal chromosomal abnormalities in the first trimester of

pregnancy.

11. The most significant risk factor for distal limb reduction defects associated with CVS

sampling is the gestational age of the fetus. CVS before 10 weeks gestational age is associated

with greater risk.

12. Allergic contact dermatitis presents with erythema, edema, pruritus, tiny vesicles and weepy

or crusted lesions 24-48 hours after contact with the allergen. It is due to cell-mediated (delay,

type IV) hypersensitivity.

13. Symmetric distal sensorimotor polyneuropathy is the most common type of diabetic

neuropathy, and is characterized by the classic ‘stocking glove’ pattern of sensory loss. Aside

from symmetric distal sensorimotor polyneuropathy, diabetes can also cause

mononeuropathies of cranial and peripheral nerves.

14. pulsus paradoxus is defined as a difference in systolic pressure of >12 mmHg during

inspiration. Increase venous return to the right heart during inspiration impedes left

ventricular filling and thus causes a drop in systolic pressure. Causes include cardiac disease

(tamponade, pericardial effusion ), and pulmonary disease (tension pneumothorax, and severe

asthma).

15. Lesch-Nyhan syndrome is secondary to a deficiency in hypocanthine-guanine phosphoribosyl

transferase (HPRT). Symptoms include self-mutilation, neurologic features (e.g. mental

retardation, dystonia, choreoathetosis, spasticity), gouty arthritis, and tophus formation. Gout

is usually seen in pts above 50 years of age, therefore, suspect Lesch-Nyhan syndrome if you

see a boy with gout.

16. Intellectualization is the transformation of an unpleasant event into a purely intellectual

problem with no emotional component.

17. Primary hyperparathyroidism is the most common cause of hypercalcemia in ambulatory

patients. Most patients are asymptomatic, and are identified only through routine chemistry

testing.

Familiar hypocalciuric hypercalcemia is a rare AD disorder. Pt are usually asymptomatic, but have

a positive family history and very low urinary calcium level. The underlying pathology involves

the presence of abnormal calcium-sensing receptors on the parathyroid cells.

18. Every case of leukocoria (a white reflex in the eye) is considered a retinoblastoma, until

proven otherwise; therefore, such cases should be promptly referred to an ophthalmologist. *

Extremely high yield question!!!

19. Child development—USMLE frequently testes:

Language development:

16

Social smile: 2 months

Babbles: 6 months

2 words, obeys 1-step command: 1 year

2-3 word phrases, obey 2-step command: 2 years

Gross motor development:

Holds head: 3 months

Rolls back to front and front to back: 4 months

Sits well unsupported: 6 months

Walk alone: 12 months

Walk up and down stairs without help: 24 months

Fine motor development:

Raking grasp: 6 months

Throw object: 12 months

Builds tower of 2 blocks: 15 months

Builds tower of 6 blocks/turn pages of books: 24 months

Social development:

Recognizes parents: 2 months

Recognizes strangers (stranger anxiety): 6 months

Imitates action/come when called: 12 months

Plays with other children: 18 months

Parallel play: 24 months

20. Iron deficiency anemia is common in infants and toddlers who drink excessive amount of

cow’s milk. Empiric iron therapy is appropriated for children with microcytic, hypochromic

anemia.

21. Tamoxifen acts as an estrogen antagonist on breast tissue, and is used in the tx and prevention

of breast cancer. It acts as an estrogen receptor agonist on the endometrium and increase the

risk of endometrial carcinoma. Tamoxifen has an overall mortality benefit, and also decrease

risk for osteoporosis.

22. When a pt is interested in alternative therapy, the physician should first inquiry as to why.

23. The hyperdynamic type of septic shock is characterized by (1) and elevated cardiac output, (2)

low systemic vascular resistance, right atrial pressure and pulmonary capillary wedge

pressure, and (3) a frequently normal mixed venous oxygen concentraton—results from

hyperdynamic circulation and improper distribution of the cardiac output. It is not indicative

of normal tissue perfusion in this case!

24. Atrial fibrillation (AF) is identified on EKG by an irregularly irregular R-R interval with

absent P waves and narrow QRS complexes. In unstable pts, immediate cardioversion should

be performed. In stable pts who have been in AF < 48 hours, cardioversion is appropriate

(Beta-blocker or CCB). In stable pts in AF> 48 hours, 3-4 weeks of rate control and

anticoagulation should precede cardioversion.

25. A muffled voice should make on consider a diagnosis other than uncomplicated pharyngitis or

17

tonsillitis. A peritonsillar abscess is potential complication of tonsillitis and requires both IV

antibiotic therapy and urgent drainage of the abscess. Deviation of the uvula and unilateral

lymphadenopathy can be helpful in distinguishing a peritonsillar abscess from epiglottitis.

26. Dihydropyridine Ca-channel antagonists can cause peripheral edema and should always be

considered in the differential diagnosis of this condition, along with other causes, such as

heart failure, renal disease and venous insufficiency. These medications can cause significant

edema due to the property to dilate peripheral blood vessels; it is NOT an allergic reaction!

27. Dermaromyositis—classic cutaneous findings accompanied by proximal muscle weakness

(violaceous poikiloderma: periorbital edema— heliotrope sign; on the chest and lateral neck

—shawl sign; elbows and knees—Gottron’s sign). Over 10% of adult pts will develop an

internal malignancy, most commonly an ovarian cancer. Regular age-appropriate cancer

screening is essential in these pts.

Inflammatory diseases associated with aortic aneurysms: Behcet syndrome, Takayasu arteritis,

giant cell arteritis, ankylosing spondylitis, rheumatoid arthritis, psoriatic arthritis, relapsing

polychondritis and Reiter syndrome.

Inflammatory diseases associated with renal failure: SLE, scleroderma, Goodpasture syndrome,

relapsing polychondritis and Wegener granulomatosis.

Inflammatory diseases associated with alveolar hemorrhage: Goodpasture, Wegener

granulomatosis, polyarteritis, Churg-strauss syndrome, Bechet syndrome and antiphospholipid

antibody syndrome, DM causes pulmonary fibrosis.

Inflammatory diseases associated with carpal tunnel syndrome : RA, sarcoidosis and amyloidosis.

28. Perioperative beta-blockers lower the incidence of perioperative myocardial ischemic events

in pts at risk for cardiac complications following noncardiac surgery.

29. Risk factors for the development of diabetic foot ulcers include diabetic neuropathy,

peripheral vascular disease, poor glycemic control, bony abnormalities of the foot, male sex,

chronic diabetes (>10 years), and a history of previous ulcer or amputation. Neuropathy is

found in approximately 80% of diabetics with foot ulcers. High yield!!!

30. Pt with chronic HCV who show persistently normal liver enzymes on multiple occasions have

minimal histological abnormalities, therefore they do not need to be treated with interferon or

antiviral drugs at this stage.

31. Exertional heat stroke occurs in otherwise healthy individuals undergoing conditioning in

extreme heat and humidity as a result of the thermoregulatory center’s inability to adequately

dissipate heat.

32. Generalized anxiety disorder (GAD)—excessive anxiety and concern regarding multiple

events or activities for a minimum of six months. 3 or more of the following symptoms must

also be present: sleep impairment, easy fatigability, restlessness, muscle tension, poor

concentration, or irritability. Bispirone is a first-line drug for treating GAD. Also benzo

(alprazolam) or antidepressants (venlafaxine, escitalopram). Bispirone does not cause the

physical dependence and withdrawal symptoms.

33. Reactive arthritis is a seronegative spondyloarthropathy resulting from enteric or

genitourinary infection. Findings in reactive arthritis may include urethritis, conjunctivitis,

mucocutaneous lesions, enthesitis (Achilles tendon pain) and asymmetric oligoarthritis. Not

all extra-articular manifestations need be present to suspect the Dx. NSAIDs are the first line

therapy.

18

34. Confounding refers to the bias that can result when the exposure-disease relationship is mixed

with the effect of extraneous factores (i.e.confounders). Selection bias results from the manner

in which people are selected for the study, or from the selective losses from follow-up.

Obsever’s bias and measurement bias distort the measure of association by misclassigying

exposed /unexposed and /or disease/non-diseased subjects. Recalled results from the

inaccurate recall of past exposure by people in the study.

35. Hypercalcemia, hyperphosphatemia, and increased parathyroid hormone levels are

characteristic biochemical abnormalities of secondary hyperparathyroidism in chronic renal

failure.

36. Loop diuretics—reversible or permanent hearing loss and /or tinnitus. These ototoxic effects

typically occur in pts taking high doses of loop diuretics, those with coexistent renal failure,

or in pts who are also being treated with other known ototoxic medications: aminoglycosides,

aspirin, chemotherapeutic agents.

37. Brain abscess can present with fever, headache, focal neurologic chagens, seizure, spacticity,

or signs of increased intracranial pressure. Congenital heart disease, heart trauma, infections

of the jaw or mouth, infections of the face or scalp, meningitis, and cranila instrumentation

increase the risk of developing a brain abscess.

38. The classical pathophysiology mechanism responsible for hypercalcemia of malignancy is the

production of PTH related peptide (PTHrP), which is structurally homologous with PTH and

can stimulate PTH receptors. The serum calcium level is increased, and the serum phosphate

level is depressed. PTHrP is present n very low concentrations in normal people, and is

thought to be responsible for calcium excretion in breast milk and transplacental calcium

transport. PTHrP can be measured in the circulation by using a PTHrP specific assay. The

elevated calcium level suppresses PTH release, thereby resulting in low serum PHT levels,

however, ectopic PTH secretion have been describe in a small number of cases.

Hypercalcemia of lung malignancy is most commonly due to the production of PTHrP by

malignant cells.

39. The history of loss of small amounts of urine simultaneously occurring with activities that

increase intraabdominal pressure, along with a physical examination demonstrating pelvic

floor weakness, is diagnosis of stress incontinence. Urine analysis, cystometry and postvoid

residual volume are normal

Detrusor instability, bladder irritation from a neoplasm, and interstitial cystitis result in urge

incontinence, which causes sudden and frequent loss of moderate to large amounts of urine. This

is often accompanied by nocturia and frequency.

Diabetic neuropathy causes overflow incontinence, which is characterized by loss of small amount

of urine from an over distended bladder, and a markedly increased residual volume. Patients

usually have a long history of diabetes that is not well controlled.

40. Pts with recurrent hypercalciuric renal stones should be treated with increased fluid intake,

sodium restriction, and a thiazide diuretic. Calcium restriction is not advised. The Tx of a first

uncomplicated calcium stone is hydration and observation.

Renal stones are predominantly of 4 types: 1-calcium oxalate/phosphate (75%), 2-uric acid (10-

15%), 3-struvite/triple phosphate (10-15%), 4- cysteine (<1%).

41. Serum CK level are used for screening the muscular dystrophies. Muscle biopsy can confirm

the diagnosis in most cases. The gold standard is genetic studies, which is required in atypical

19

cases.

42. Vitiligo is characterized by macular depigmentation what involves that acral and peri-orificial

areas. It is caused by the autoimmune destruction of melanocytes.

43. The most common adverse effect of inhaled corticosteroid therapy is oropharyngeal thrush

(oral candidiasis).

44. All patients who have the myocardial infarction should receive secondary prevention. The

following medications have shown to have a mortality benefit when given as secondary

prevention: 1-Aspirin (75-325 mg/day). 2-Beta-blockers. 3-ACEIs. 4-lipid-lowering statin

drugs. Clopidogrel should be included as secondary prevention following unstable

angina/NSTEMI for at least 12 months. It should also be prescribed for thirty days (bare metal

stents) to one year (drug eluting stents) following PCI, as it has been shown to help prevent

subacute stent thrombosis.

Low molecular weight heparin (LMWH) is usually given post-MI for the first 48 hours or until

angiography is performed, but it is not necessary after the pt has been discharged from hospital.

Wafarin is indicated if the pt has evidence of ventricular thrombus, which is usually seen in

individuals with apical infarcts and dyskinetic wall motion abnormalities.

12/19/09

1. Primary sclerosing cholangitis (PSC) is caused by inflammation and fibrosis of the

intrahepatic and extrahepatic biliary ducts. It is frequently associated with ulcerative coitis.

Microscopic polyangiitis is a pauci-immune, necrotizing vasculitis of small vessels. While it is

often p-ANCA positive, it is not associated with IBD and typically presents with a wide range of

organ-specific manifestations (eg. Skin rash, hemoptysis, chest pain, GI bleeding, arthralgias)

PBC—anti-mitochondrial antibodies are typically positive.

2. Antibiotic therapy is the most accepted and recommended management of for the eradication

of H.pylori in pts with gastric mucosa-associated lymphoid tissue (MALT) lymphoma without

any metastasis.

3. Always suspect malignant hypertension in pt with very high BP (≥200/140 mmHg). Presence

of papilledema on pphothalmoscopy confirms the diagnosis. The pathologic change

responsible for end-organ damage in malignant hypertension is fibrinoid necrosis of small

arterioles. Renal failure with elevated creatinine level and oliguria can develop rapidily with

tx, but is not required to diagnose.

4. So far, there have been no case reports of congenital rubella syndrome in womrn inadvertently

vaccinated during early pregnancy. Mx—reassurance.

5. Torsades de points is a polymorphic ventricular tachycardia which occurs in the setting of a

prolonged QT interval and is seen in pts with familiar long QT syndrome, malnourished pts

predisposed to hypomagnesemia (such as alcoholic), and in pts taking certain drugs (tricyclic

antidepressants), certain antiarrhythmics (amidarone, sotalol), and anti-infective agents

(moxifloxacin, fluconazole). TX—cessation of any offending agents, and initiation of

magnesium sulfate.

Sodium bicarbonate is used in tricyclic antidepressant and aspirin overdose.

6. Severe vomiting produces metabolic alkalosis due to the unbalanced loss of hydrogen ions in

the gastric fluid. Contraction alkalosis also occurs in the is setting due to ECF volume

contraction resulting from vomiting. The decrease in ECV volume activates the rennin-

20

angiotensin-aldosterone system causing water retention at the expense of hydrogen ion losses.

7. Subdural hematomas are serious intracranial hemorrhages that occur due to tearing of

bridging veins. Blunt trauma is the most common cause, and surgical evacuation is usually

require.

Small vessel hyalinosis is believed to contribute to the phathophysiology of lacunar infarction.

Lacunar strokes occur due to occlusion of small penetrating arteries and account for 25% of

ischemic strokes. Hypertension and diabetes are two major risk factors. Lacunar strokes often

affect the internal capsule and result in pure motor dysfunction. Small vessel hyalinosis has no

association with subdural hematoma,

8. Temporal arteritis should be suspected in pts over 50 years of age with new-onset temporal

headache, jaw claudication, vision loss, or symptom of polymyalgia rheumatica (CK, EMG

normal, ESR ↑). Once the diagnosis is suspected, immediate initiation of high dose steroids is

indicated to prevent damage to the retinal artery and other vessels. The ESR is elevated in

temporal arteritis and diagnosis is confirmed by temporal artery biopsy.

9. All pts with cirrhosis of the liver should be screened for esophageal varices by endoscopy.

Primary prophylaxis should include beta-blockersm which will significantly reduce the risk of

bleeding.

10. Amaurosis黑朦 fugax is characterized by visual loss that is usually monocular, transient, and

described as ‘like a curtain falling down’. Ophthalmoscopy reveals zones of whitened,

edematous retina following the distribution of the retinal arterioles. Non-invasive evaluation

of the carotids is useful in providing information regarding the degree of carotid artery

stenosis.

Central retinal artery occlusion (CRAO) –sudden painless loss of vision in one eye; however, the

funduscopic findings differ. Ophthalmoscopy in pts with CRAO reveals pallor of the optic disc,

cherry red fovea, and boxcar segmentation of blood in both the retinal arteries and veins.

Central retinal vein occlusion –sudden , painless, unilateral loss of vision. It is also noted in pts

with a history of hypertension. Ophthalmoscopic signs are disk swelling, venous dilation,

tortuosity, retinal hemorrhages and cotton wool spots.

Vitreous hemorrhage presents with a sudden loss of vision and floaters in the visual field. Diabetic

retinopathy is the most common cause. An important clue to the diagnosis is a fundus that is hard

to visualize with obscure details.

Pts with hypertensive retinopathy usually do not show any symptoms associated with visual

impaiement. There is initially focal spasm of arterioles, followed by progressive sclerosis and

narrowing. Fundoscopy findings may reveal AV nicking, copper wiring or silver wiring, exudates

and hemorrhages (specific finding depends on the grade of retinopathy.)

11. The presence of a systolic-diastolic abdominal bruit in a pt with hypertension and

atherosclerosis is strongly suggestive of renal artery stenosis. Arortic coarctations are

congenital strictures along the aorta. They can cause hypertension and differences in blood

pressures between the upper and lower extremities. Pts are most often diagnosed in infancy or

childhood.

12. In the US, sarcoidosis most commonly affects young- to middle- aged African females,

causing insidious-onset hyspnea and dry cough in the absence of constitutional symptoms.

Sarcoidosis also typically affects the skin (protean manifestations, most commonly erythema

nodosum) and the eye (uveitis).

21

Idiopathic pulmonary fibrosis presents insidiously with progressive dyspnea on exertion and a dry,

nonproductive cough. Auscultation reveals ‘Velcro-like’ inspiratory crackles. Fever and chest pain

are typically absent and uveitis is not a assocated feature.

13. Fat-soluble vitamins are stored in the body for long periods of time, and hence have the

potential to accumulate to toxic levels when supplements are taken. VitD toxicity leads to

hypercalcemia, constipation, abdominal pain, weight loss, polyuria, and polydipsia. In cases

of toxicity, both VitD and calcium should be withheld until symptoms improve and levels

return to normal.

VitA toxicity may cause dry skin, headache, abdominal pain, and blurry vision. Can also cause

psudotumor ceribri.

14. Loss to follow-up in prospective studies creates a potential for selection bias.

15. Ultrasound or other imaging techniques (CT scan) is indicated when pts with pyelonephritis

do not respond after 48-72 hours of the appropriate antibiotic therapy, searching for any

underlying pathologies (eg. obstruction) or complications (eg. Renal, perirenal abscess).

16. Polymyositis and dermatomuyositis are two forms of inflammatory myopathy and present

proximal muscle weakness. A diagnosis of inflammatory myopathy is supported by elevated

ESR and CK, and is confirmed by muscle biopsy. Tx—high dose corticosteroids.

Riluzole is used to slow the progress of amyotrophic lateral sclerosis (ALS). ALS presents with

propressive weakness of the arms, legs and cranial nerves, with a mixture of upper and lower

motor neuron signs.

Amitriptyline--fibromyalgia

17. Monoclonal gammapathy of undermined significance (MGUS) are usually Mx initially iwht

reassurance and regular follow-up. Labs—an M component (IgA, G or M) <3000mg/dL, and

fewer than 10% plasma cells in the BM. 25% risk of development of a serious disease, 1% yr

—multiple myeloma.

18. The combination of arterial/venous thrombosis and thrombocytopenia in pts receiving heparin

therapy is highly suggestive of heparin-induced thrombocytopenia. Antibodies against

heparin-platelet factor-4 complex are responsible for the disease.

19. The BUN levels is elevated in pts with upper GI bleeds bz the bacterial breakdown of

hemoglobin in the GI tract results in the absorption of urea. A BUN level > 40 in the presence

of a normal serum creatinine level may indicate an upper GI bleed, The other common

scenario where you can see elevated BUN without increased creatinine levels is the

administration of steroids. Extremely important!!

20. know how to interpret the strength of association and dose response relationship from a study.

21. “lone atrial fibrillation” is atrial fibrillation in the absence of other risk factors for stroke, such

as previous stroke or TIA, DM, hypertension, heart failure, age over 65 years, or valvular

heart disease, Aspirin is the only antithrombotic therapy required for pts with lone atrial

fibrillation.

Warfarin with a goal INR of 2-3 is the recommendation for pts who have AF and other risk factors

for stroke. When starting warfarin therapy, it takes at least 3-4 days to achieve appropriate levels

of anticoagularion. In fact, warfarin therapy may actually cause hypercoagulability initially. (This

is bz the first VitK dependent factors to be lost are protein C and S, which are both

anticoagulants.) In order to provide anticoagulation during this window of time, pts are often

‘bridged’ with a heparin product.

22

22. Enthesitis is a condition in which inflammation and pain occur at the site of tendon and

ligment attachment to bone and is a common finding in anklosing spondilitis (AS). Typical

sites of enthesitis include the heels, tibial tuberosities and iliac crests.

Proteinuria + arthritis—lupus

Subcutaneous nodules—RA, gout.

23. Torticollis 斜颈 is a common form of focal dystonia involving the sternocleidomastoid

muscle. It can occur idiopathically but is very often medication-related. Discontinuation of

causative agents may improve symptoms.

Akathisia静坐不能 is a sensation of restlessness that causes the pt to move frequently.

Athetosis手足徐动症 refers to slow, writhing movements that typically affect the hands and feet.

Athetoid movements are characteristic of Huntington’s disease. Chorea and athetosis often occur

together.

24. A normal distribution is symmetric and bell-shaped. All its measures of central tendency are

equal: mean=median=mode. Positively skewed (tail on the right), mean>median>mode.

Negatively skewed (tail on the left), mean<median<mode.

25. Acute angle closure glaucoma presents as unilateral eye pain, redness, and a dilated pupil with

poor light response, It’s important to distinguish acute angle closure glaucoma from migraine,

cluster headache, temporal arteritis, and keratoconjunctivitis bz failure to correctly diagnose

this disease can lead to blindness.

26. Know how to interpret the confidence interval. Know the relationship between confidence

interval and ‘p’ value.

27. Electrical alternans is when the amplitudes of the QRS complexes vary from beat to beat. It is

fairly specific for pericardial effusion. Enlargement of the cardiac silhouette can be seen on

CXR as well. Echocardiogram can more definitely demonstrate a pericardial effusion.

New-onset right bundle branch block—PE

28. Hypersensitivity pneumonitis (HP) si inflammation of the lung parenchyma caused by antigen

exposure. Acute episodes present with cough, breathlessness, fever and malaise that occur

within 4-6 hours of antigenic exposure. Chronic exposure may cause weight loss, clubbing,

and honeycombing of the lung. The cornerstone of HP Mx is avoidance of the responsible

antigen.

29. One of the most common causes of hyperkalemia is medications—ACEI, angiotensin receptor

blockers and spironolactone—by blocking aldosterone synthesis or action. Other medications

—trimethoprim and pentamidine. Pt with hyperkalemia—Mx—review all the medication

first.

30. Baker cysts develop as a result of excessive fluid production by an inflamed synovium, as

occurs in cases of RA, osteoarthritis and cartilage tears. The excess fluid accumulates in the

popliteal bursa which expands, creating a tender mass in the popliteal fossa. Baker cysts

occasionally burst and release their contents into the calf, resulting in an appearance similar to

a DVT.

31. Acanthosis nigricans—symmetrical, hyperpigmented, velvety plaques in the axilla, groin, and

neck. It is associated with DM (insulin resistance) in younger patients, and GI malignancy in

older individuals.

32. HIV screening-- ELISA

33. Pseudomonas aeruginosa is a common cause of severe, pulmonary infections in cystic fibrosis

23

pts. Prefered therapy—combination of an aminoglycoside (tobramycin) and and

antipseudomonal penicillin (piperacillin). Ciprofloxacin is C/I in children as it may cause

cartilage destruction and growth retardation.

34. Pulmonary neoplasm in superior sulcus—apical lung tumor cause compression of the

sympathetic trunck (Horner syndrome), the brachial plexus (pancoast syndrome), the right

recurrent laryngeal nerve (hoarse voice), and the superior vena cava (SVC syndrome).

Pancoast syndrome is characterized by shoulder pain radiating into the arm in an ulnar

distribution and is caused by tumor invasion of the 8th cervical and 1st thoracic nerves.

35. Insulin resistance typical for pt with central-type obesity is the key pathogenic factor in the

development of type-2 DM and associated abnormalities (hypertension, dyslipidemia).

Extremely high yield!!

36. Bronchiectasis is marked by bronchial dilation, impaired clearance of secretion and airway

obstruction. High resolution chest CT is used to make the definitive Dx. Massive and

potentially lethal hemoptysis is a feared complication.

37. Adrenal TB—MCC of primary adrenal insufficiency in developing countries; autoimmune

adrenalitis—developed countries.

38. Women produce androgens, such as androstenedione (AS), dehydroepiandrosterone (DHEA),

testosterone (T) and dehydroepiandrosterone sulfate (DHEA-S). AS, DHEA and T are

produced by the ovaries and adrenals, whereas DHEA-S is predominantly produced in the

adrenal glands only. AS, DHEA and DHEA-S are not true androgens cz they do not interact

with the androgen receptor. They can be converted to testosterone, and overproduction of

these hormones can lead to clinical features of androgen excess (adrenal tumor).

39. Pulmoary embolism should be suspected in all pt at risk for DVT who present with acute-

onset tachypnea, dyspnea and chest pain. Dx—helical CT scanning of the chest.

40. Hyperlipidemia (decreased clearance of LDL due to decreased LDL receptors), unexplained

hyponatremia (ISADH) and elevated serum muscle enzymes(myopathy) are indicationa for

thyroid function tests—hypothyroidism.

41. Untreated hyperthyroid pts are at risk for rapid bone loss resulting from increased osteoclastic

activity in the bone cells, also at risk for cardiac tachyarrhythmias, including AF.

42. Chagas disease is a chronic disease that can cause megaesophagus, megacolon, and /or

cardiac dysfunction. The protozoan trypanosome cruzi, endemic to Latin America.

43. Heparin-induced thrombocytopenia is seen in about 5-15 of pts taking therapeutic heparin

with onset between 3-15 days after initiation of heparin and resolution with 4-5 days of

discontinuation of heparin. While the elevated PTT is a therapeutic effect of heparin, the

thrombocytopenia is an adverse effect of the same.

44. In ARDS, mechanical ventilation with low tidal volumes (prevent lung damage) and PEEP

(up to 15cm H2O) can improve oxygenation. Potential complications of PEEP include

barotraumas and tension pneumothorax.

12/20/09

1. Cerebellar dysfunction is common among chronic alcohol abuser. Symptoms include gait

instability, difficulty with rapid movements, and intention tremor

The ‘clasp knife’ phenomenon refers to s sunned reduction in muscle tone and increase in laxity

after initial resistance to passive movement. It is seen in pts with hypertonia and is not

characteristic of cerebellar disease.

24

2. Obstructive sleep apnea is an under-diagnose condition common in the obese population.

Snoring, daytime sleepiness, and apneic episodes during sleep should raise suspicion of OSA.

Affected pts are at risk for hypertension, heart disease, cor pulmonale, and accidents.

Polysomnography is used for diagnosis.

3. Mitral regurgitation can cause a holosystolic murmur that radiates to the apex. Mitral valve

prolapse is the MCC of mitral regurgitation (developed country). Mitral regurgitation can lead

to left atrial dilation and atrial fibrillation.

4. Hydatid cysts in the liver are due to infection with Echinococcus granulosus. ‘Eggshell’

calcification of a hepatic cyst on CT scan is highly suggestive of hydatid cyst. Aspiration of

the cyst is generally not indicated due to the risk of anaphylactic shock sen\condary to spilling

of cyst contents. Tx is generally surgical resection under the cover of albendazole.

Cysticercosis is secondary to infection with tenia solium and generally results in cysts in the brain

or in muscle.

5. Beta-blockade is the most appropriate initial intervention for acute aortic dissection. Type A

dissections involve the ascending aorta and are treated with medical therapy and surgery,

while Type B dissections involve only the descending aorta and are usually treated with

medical therapy alone.

Hydralazien, nifedipine, and vasodilates would increase shear stress on the aorta, should be avoid.

Heparin and fibinolytisc, like streptokinase should never be used if aortic dissection is suspected

—inhibit the body’s efforts to contain the dissection and could greatly exacerbate the condition.

6. The presentation of cryoglobulinemia (mixed essential cryoglobulinemia) includes palpable

purpura, glomerulonephritis, non-specific systemic symptoms, arthralgias,

hepatosplenomeagly, peripheral neuropathy, and hypocomplementemia. Most patients also

have HCV. The demonstration of circulating cryoblobulin is confirmatory. All such pts should

be tested for HCV antibody.

Henoch-Schonlein purpura usually presents in childhood as palpable purpura on the buttocks,

abdominal pain, arthralgias, proteinuria and hematuria with RBC casts on urinalysis. Serum

complement levels are normal. HCV infection is not associated with this diease.

7. A fixed upper airway obstruction will decrease the airflow rate during inspiration, active

expiration and passive expiration. (see pictures in notes)

8. Cerebral emboli are one of the most dreaded consequences of endocarditis. They occur when

pieces of infected valvular vegetations break off and enter the CNS circulation. Tx is with

antibiotics.

Todd’s palsy: a focal neurologic defecit following a seizure. EEG can aid in the dx of an epileptic

seizure.

9. Shy-Dragger syndrome—1.parkinsonism; 2-Autonomic dysfunction (postural hypotension,

abnormal sweating, disturbance of bowel or bladder control, abnormal salivation or

lacrimation, impotence, gastroparesis, etc); 3- widespread neurological sings (cerebellar,

pyramidal or lower motor neuron). Always consider Shy-Dragger syndrome when a patient

with Parkinsonism experiences orthostatic hypotension, impotence, incontinence, or other

autonomic symptoms. Anti- Parkinsonism drugs are generally ineffective, and tx is aimed at

intravascular volume expansion with fludrocortisone, salt supplementation, alpha-adrenergic

agonists, and applications of constrictive garments to the lower body.

Idiopathic orthostatic hypotension is seen secondary to the degeneration of postganglionic

25

sympathetic neurons; however, the pathology is confined to the autonomic nervous system, and

there are no signs Parkinsonism or cerebellar involvement.

Riley-Day syndrome or familiar dysautonomia is an autosomal-recessive disease seen

predominantly in children of Ashkenazi Jewish ancestry. It is characterized by gross dysfunction

of the autonomic nervous system with sever orthostatic hypotension.

10. immobilization of an individual with a high bone turnover results in increased osteoclastic

activation that can lead to hypercalcemia. Bisphosphonate therapy in pts who are immobilized

is helpful in reducing hypercalcemia and preventing osteopenia.

11. GI manifestations of diabetic autonomic neuropathy include postpranial bloating, early satiety,

constipation and diarrhea. Mx: 1-improved glycemic control; 2—Small, frequent meals; 3—A

dopamine antagonist (eg. Metoclopramide, domperidone) before meals; 4—Bethanechol; 5—

erythromycin (interacts with motilin receptors and promote gastric emptying); 6—Cisapride is

effective in diabetic gestroparesis, but it is currently available only through the manufacturer.

Fisk factors—cardiac arrhythmias.

12. Heat stroke is defined as a temperature above 40.5 (105F). Exertional heat stroke occurs in

otherwise healthy individual exercising in extreme hear. Dehydration, hypotension,

tachycardia and tachypnea are common. Systemic effects like seizure, ARDS, DIC, and

hepatic/renal failure may also occur.

13. Local therapy (eg. Resection of metastases, local irradiation) is rarely curative in pts with

metastatic breast cancer, but it can be tried in pts with a respectable solitary metastatic focus

without signs of systemic involvement. Criteria for pulmonary metastasectomy: 1) the

primary tumor is controlled (resected); 2) no other sites of disease exist; 3) the operative risk

is low (no significant co-morbid condition) and 4) complete resection is possible.

14. SLE’s effect on the kidneys can range from minimal mesangial lupus nephritis to advance

sclerosing lupus nephritis and are graded class I-IV. The tx and outcomes are different

depending on the class of lupus nephritis. Therefore, renal biopsy is required in all pts with

new onset lupus nephritis. Immunosuppressive therapy may be given once the lupus nephritis

is classified.

15. American Cancer Association urges physician to offer yearly PAS blood tests and digital

rectal examination (DRE) beginning at age: 1—50 years for men who have at least a 10-year

life expectancy; 2—45 years for men at risk *African Americans *men with a first-degree

relative diagnosed with prostate cancer at an early age (younger than age 65); 3—40 years for

men at even higher risk* men with several first degree relatives who had prostate cancer at an

early age. Depending on the results of this initial test, further testing might not be needed until

age 45.

16. Initial hematuria suggests urethral damage. Terminal hematuria indicates bladder or prostatic

damage, and total hematuria (the entire urinary cycle) reflects damage in the kidney or ureters.

Clots are not usually seen with renal causes of hematuria. Pt has painless terminal hematuria

with clots and should be evaluated for bladder cancer by cystoscopy.

17. The C/I of triptans: 1-familial hemiplegic migraine; 2-uncontrolled hypertension; 3-coronary

artery disease; 4-prinzmetal angina, 5-pregnacy, 6-ischemic stroke, 7-basilar migraine

18. Right mainstem bronchus intubation is a relatively common complication of endotracheal

intubation. It causes asymmetric breath sound immediately after intubation and is fixed by

withdrawal of the tube (ideal position—between the vocal cords and the carina for greatest

26

safety and effect.)

19. Prolonged placement of central lines can lead to subclavian vein thrombosis and result in arm

swelling. Catheters should be removed and duplex ordered to document the thrombus and for

the need of anticoagulation,

20. Obstructive sleep apnea (OSA) is the most common medical cause of excessive daytime

sleepiness in the US. It occurs due to poor oropharyngeal tone and results in daytime

sleepiness, morning headaches, and depression.

Narcolepsy is a sleep disorder characterized by poorly regulated rapid eye movement (REM)

sleep. It, like OSA, is characterized by excessive daytime sleepiness, but pts also suffer from

cataplexy, sleep attacks, hypnagogic 入睡前发生的 /hypnopompic 半醒的 hallucinations, and

sleep paralysis.

21. Ventrical aneurysm is a late complication of MI characterized by persistent ST elevations

(only infarcted leads) on ECG. Pts may have CHF, ventricular arrhythmias, and/or thrombus

formation long with mitral regurgitation (a pansystolic murmur at the apex with radiation to

the axilla). Echocardiography demonstrating dyskinetic wall motion of a portion of te left

ventricle may be used to confirm the Dx.

Interventricular wall rupture—5 days after MI, causes an acute left-to-right shunt with right sided

heart failure and new-onset systolic murmur head best at the left lower sternal border.

Ventricular free wall rupture—5 days after MI, causes acute pericardial tamponade and rapid

decompensation with pulseless electrical activity.

Papillary muscle rupture—3-7 days after MI, causes severe acute mitral regurgitation and

pulmonary edema. It does not typically cause persistent ST elevation on ECG.

22. Stepwise approach to abort acute attacks of migraine: simple analgesics (eg. acetaminophen),

NSAIDs, triptans and ergot derivatives. Serotonin agonist (triptans, Rizatriptan, zolmitriptan,

sumatriptan, frovatriptan and almotriptan) are generally preferred over ergotamines bz it has

fewer side effects and greater efficacy than ergotamine.

Ergotamine derivatives are considered when acute attacks of migraine last for longer than 48

hours and are frequently recurrent. The use of these drugs has been shown to be associated with

rebound headache and vascular occlusion. For this reason, ergotamine derivatives are generally

avoided in pts with coronary artery disease, hypertention, peripheral vascular disease, liver

disease, renal disease and complicated migraine,

Propranolol and verapamil are used ofr prophylaxis of migraine, These drugs are not used to abort

acute attackes.

23. Myasthenia crisis is life-threatening condition hat is characterized by weakness of the

respiratory and pharyngeal muscles. Tx of myasthenia crisis consists of endotracheal

intubation and withdrawal of anticholinesterases for several days. The MCC is an intercurrent

infection and in such cases antibiotics are an important part of Mx. DD—cholinergic crisis.

Physicians used to perform the Tensilon (pyridostimine) test. However, since cholinergic

crisis has become uncommon, and since the tx of both conditions is to withdraw

anticholinesterases, physicians are no linger doing this test.

24. Infectious diarrhea is classified into bloody/inflammatory and non-bloody. Bloody—E. coli

O157:H7, enteroinvasive E. coli, Shigella, Salmonella, Campylobacter, Yersinia, Entamoeba,

and C.difficile. E. coli O157:H7 had been transmitted via the intake of undercooked

hamburger meat from fast food chains,

27

25. Basal cell carcinoma presents as a slow-growing papule with pearly, rolled borders and

overlying telangiectasia. It is the most common malignant tumor of the eyelid.

Keratoacanthoma is a rapid growing ‘vocano-like’ nodule with a central keratotic plug. While

these lesions are classically self-limited, many are treated as well-differentiated squamous cell

carcinomas. Early tx is indicated is the lesion is near an important structure, such as the eye.

Seborrheic keratoses are most often greasy, brown, crust-like lesions with a stuck-on appearance.

26. Suspect dermatitis herpetiformis in a pt with (1)GI symptoms suggestive of malabsorption,

and (2) pruritic papules and vesicles over the extensor surfaces of the elbows, knees, posterior

aspect of neck and scalp. The presence of aniti-endomysial antibodies is characteristic. Pts

also suffer from celiac sprue or gluten sensitive enteropathy. There is an increased risk for

gastro-intestinal lymphomas, which can be reduced by following a gluten-free diet.

Erythema multiforme—a sudden onset of symmetric erythematous skin lesions. Target skin

lesions with a clear center and concentric erythematous rings may be noted. HSV infection,

systemic diseases, drug reactions are often assocated with this condition.

Acantholytic dermatosis is also know as Grover disease. Pts present with pruritis and

erythematous to brown keratotic papules over the anterior chest, upper back, and lower rib cage.

The etiology is unknown.

27. Hypothyroidism should always be considered in pts with an unexplained elevation of serum

CK concentration and myopathy—check serum TSH.

28. Methotrexate is a first line tx for RA and its side-effects include stomatitis, nausea, anemia

and hepatotoxicity—Tx folic acid.

Felty syndrome is an uncommon form of RA accompanied by splenomegaly and granulocytopenia

(neutrophil count <2,000/mm3). Granulocytopenia generally does not occur until symptoms of

arthritis have been present for over 10 years. Splenomegaly is a common finding in pts with RA

without granulocytopenia.

29. Increased extracellular PH levels (eg. Respiratory alkalosis) can cause in the affinity of serum

albumin to calcium, thereby increasing the levels of albumin-bound calcium, and

consequently decreasing the levels of ionized calcium. The level of calcium bound to organic

and inorganic anions remains unaltered. Ionized calcium is the only physiologically active

form—manifestation of hypocalcemia.

30. Small cervical LNs are a commons observation in children and young adults (upper

respiratory infection). Pts with asymptomatic, soft (rubbery) LN can be observed for node

growth or the development of symptoms. A nodal diameter >2.0—biopsy; <1cm, almost

benign.

31. The pH of the pleural fliud <7.2—an empyema—drained, removal of the fluid by

thoracostomy. Glucose <60mg/dL—indication for tube thoracostomy. Infected pleural space

is usually initially drained with a chest tube.

32. In mechanically ventilation pts with respiratory alkalosis in the setting of an appropriate tidal

volume (6ml/kg of idea body weight), the respiratory rate should be lowered. (Reductions in

tidal volume can trigger an increased ventilatory rate, potentially exacerbating the situation.)

33. The association between DM and mucormycosis is frequently tested on the USMLE. The

MCC cause of mucormycosis is Rhizopus. Pseudomonas is a cause fo malignant otitis externa

in DM pts; it may cause black necrotic lesions in the ear.

34. The sign and symptoms of PE are nonspecific and highly variable. It is a diagnosis that should

28

be suspected in any pt who presents with some combination of sudden-onset shortness of

breath, pleuritic chest pain, low-grade fever, and hemoptysis. Tachycardia, and hypoxemia are

common exam findings. Calf swelling and Virchow’s triad (stasis, endothelial injury, and

hypercoagulable state) are not always present in PE.

35. Elderly and critically ill pts with immobility, poor nutrition and sensory impairment are at risk

of developing decubitus褥疮 ulcers. Ulcers form as a result of uninterrupted pressure on the

tissue overlying bony prominences. The elbow, coccyx, hips and heels are sites at expecially

high risk. Pt repositioning every 2 hours and pressure reducing devices (air/foam mattresses)

are important methods of prevention.

36. Epidural abscesses must be considered in pts with fever and back pain, particularly in those

who are immunosuppressed or use injection drugs. The diagnostic test of choice is an MRI of

the spine. Tx consists of antibiotics and surgical decompression.

37. Non-invasive positive pressure ventilation (NIPPV) is an excellent option for pts with COPD

exacerbation. It should be tried before intubation (more complications, includes infections)

and mechanical ventilation in COPD pts with CO2 retention. NIPPV is recommended in a pt

of respiratory distress with a pH <7.35 or PaCO2 >45mmHg or respiratory rate >25 per min.

C/I: septic, hypotensivem or dysrhythmic pts.

38. Always remember that the ABC’s come first in the tx of any pt. The tx of choice for RMSF is

doxycycline for both adults and children.

39. Hypercalcemia is a common finding in a pt with multiple myeloma. Hypercalcemia may

cause severe constipation, anorexia, renal tubular dysfunction, and neurologic symptoms. *

other scenario could be metastatic bone cancers such as lung, breast, renal, thyroid, etc.

Always look for hypercalcemia as a cause of constipation. Extremely high yield!!

40. Corrected Ca++ = 0.8(normal albumin 4.0-measured albumin) + measured Ca++

hypocalcemia caused by hypoalbuminemia is not required to be treated.

41. In hyperkalemia, removal of K+ from the body can be achieved with dialysis, cation exchange

resins (kayexalate) or diuretics.

42. Paget’s disease is caused by osteoclast dysfunction which results in a ‘mosaic’ pattern of

lamellar bone. ↑alkaline phosphatase level, and characteristic x-ray findings like femoral

bowing. Bone and joint pain, skeletal deformities and hearing loss (enlarging cranial bones

leads to entrapment of cranial nerve 8) are common symptoms.

43. Nasal eosinophilia –allergic rhinitis, although this finding is not specific. Nasal eosinophilia is

absent in pts with infectious causes and vasomotor rhinitis. Rhinitis Dx—nasal cytology.

44. Pt who develop serous bleeding (eg. Intracerebral hemorrhage) due to excess anticoagulation

with warfarin should be given FFP for the rapid reversal of anticoagulation. Vit K reverses the

action of warfarin, but takes 8-12 hours to be effective. Rapid reduction of blood pressure in

the presence of an acute intracerebral hemorrhage is not recommended. Giving nitroprusside

to rapidly reduce the BP may lead to cerebral hypoperfusion and make matters worse. The

goal systolic BP in such pts is 140-160mmHg. INR <5, no significant bleeding Omit next warfarin doseINR 5-9, no significant bleeding Stop warfarin temporarilyINR >9 Stop warfarin, give oral vitamin K

12/20/09

29

1. Pt with PID should be treated with appropriate antibiotics, counseled on safe-sex practices,

and encouraged to inform any sexual partners from the past 60 days of their condition so that

they can be treated as well. Pts with PID should also be screened for HIV, syphilis, HBV,

cervical cancer (pap smear), and HCV if they have a history of IV drug use.

2. Hypothyroidism is an important cause of reversible changes in memory and mentation. It will

be accompanied by systemic changes such as weight gain, fatigue, and constipation.

3. Elevated PT/INR levels in a patient with hepatic failure many be due to vitK deficiency or

liver cirrhosis. Regardless of the cause, the first step in this setting is empiric administration

of vit K, since there is usually an underlying vitK deficiency due to several comorbidities.

FFP is indicated if the pt is actively bleeding, or if the pt needs immediate surgery or an

invasive procedure. Cryoprecipitate is required for coagulation factor deficiencies such as

factor VIII deficiency, Platelet transfusion is indicated only if the pt is actively bleeding, and

the platelet count is less than 20,000 to 30,000/mm3. Packed red cells are usually indicated if

the hemoglobin concentration is <8 in asymptomatic pts, or <10 in symptomatic pts.

4. Multiple observational studies have demonstrated that pts with hypertension have

approximately for times the risk of stroke when compared to non-hypertensive subjects. *

Know the concept of multiple risk factors. Know the association between hypertwnsion and

strokes. This is a very common scenario for USMLE 2 & 3.

5. If the outcome of a case-control study is not common in the population, the odds ratio is close

to the relative risk.

6. In pts who present with stable angina and hypertension, a beta-block is the drug of choice.

7. Bupropion is FDA-approved for smoking cessation. It should be used in conjunction with

counseling and nicotine replacement.

8. Rubella—erythematous and maculopapular. It stars on the face and progresses to the trunk

and extremities. Prodomal symptoms include fever, lymphadenopathy, and malaise. Occipital

and posterior cervical lymphadenopathy are suggestive of the Dx. Adult women usually have

associated arthtitis, which is another diagnostic clue, Some patients may have mile coryza and

conjunctivitis.

measles is also erythematous and maculopapular, and similarly progresses from the head to the

trunk and extremities. There is usually a prodrome of fever, cough, coryza, and conjunctivitis. The

presence of Koplik’s spots is suggestive. Arthritis is not commonly seen.

The rash of chicken pox is pruiritic and usually develops after a prodrome of fever and malaise.

The lesions appear in consecutive crops, so lesions of several different stages are often visible on

examinations (ie. Popular, vesicular and crusted lesions).

Posterior cervical lymphadenopathy and a maculopapular rash man be seen in infectious

mononucleosis; however, these pts usually also complain of a sore throat ad pharyngitis would be

seen on exam. Leukocytosis is common. In infectious mononucleosis, rash ofter develops after the

administration of ampicillin.

Rash with generalized nontender lyphadenopathy is also seen in secondary syphilis. This rash is

classically a symmetric maculopapular eruption involving the entire trunk and extremities,

including the palms and soles.

9. Osteophytes are the most common findings on cervical radiography in pts with cervical

spondylosis, but specificity of these findings is low. Cervical spondylosis-- chronic neck pain,

limited neck rotation and lateral bending is due to osteoarthritis and secondary muscle spasm.

30

Typical radiographic findings include bony spurs and sclerotic facet jounts.

10. alpha-1 antitrypsin deficiency is associated with panacinar emphysema and cirrhosis.

11. Metaprolol is a selective beta1-adrenergic antagonist that when administered in high doses,

can block bronchodilatory beta2-adrenergic receptors causing bronchoconstriction in

susceptible individual. (History of eczema—may mild undiagnosed intermittent asthma.

12. The peripheral skeleton is more commonly affected by RA than the axial skeleton. The

cervical spine is most commonly affected in RA, resulting in C1-C2 instability subaxial

subluxation.

13. Organophosphate poisoning (Farmer, suicide)—bradycardia, miosis, rhonchi, muscle

fasciculations, salivation, lacrimation, urimation and defecation, Atropine administration can

reverse these effects. Of equal importance is the removal of any clothes (which may be

contaminated with pesticides) and washing of the skin to prevent further transcutaneous

absorption.

14. Aminoglycosides are antibiotics used to treat serious gram-negative infections (multi-drug

resistant organism). They are potentially nephrotoxic and drugs levels and renal function must

be monitored closely during therapy.

Nafcillin is a common cause of acute renal failure due to acute interstitial nephritis (AIN). This

drug is used to treat infections caused by methicillin-sensitive Staph aureus (MSSA). AIN—

eosinophils and white blood cell casts present in urine.

15. Seborrheic keratoses are very common benign growths that have a warty or cerebriform,

stuck-on appearance. They are typically pigmented, but may lack pigment in many cases.

An acrochordon is a skin tag. These are usually flesh-colored or pedunculated papules in regions

of the body subjected to friction such as the neck, axillae, and inner thighs.

16. Urinary infection most commonly arises by an ascending route. Sexual intercourse is one of

the most important risk factor for developing uncomplicated UTIs in women due to its

mechanical effect of introducing uropathogens into the bladder (honeymoon cystitis).

17. Blastomycosis is a broad-based budding yeast which is endemic in the south-central and

north-central US. It usually affects the lungs, skin, bones, joint, and prostate. Infection in

immunocompromised hosts is uncommon. Primary pulmonary infection is asymptomatic, or

presents with flu-like symptoms. Cutaneous disease is either verrucous or ulcerative.

Verrucous lesions are initially papulopustular, and then progressively become crusted, heaped

up and warty, with a violaceous hue. These lesions have sharp borders, and may be

surrounded by microabscesses. Wet preparation of purulent material expressed from these

lesions shows the yeast form f the organism.

Histoplasmosis—acute pneumonia, chronic pulmonary histoplasmosis and disseminated

histoplasmosis (HIV).

Coccidioides—pulmonary infection. Cutaneous findings such as erythema multiforme and

erythema nodosum are common.

Invasive aspergillosis—immunocompromised pts, esp transplant pts or those taking cytotoxic

medications.

Nocardia—a subacute pneumonia mimics TB, immunocompromised pts. Actinomyces causes

abscess near the head and neck that drain sulfur granules.

18. Strep pneumoniae is the most common pathogen causing pneumonia in nursing home pts, also

MCC of community-acquired pneumonia in adults. Pts with neurologic disorders (eg.

31

Advanced dementia, parkinson’s disease, and stroke) are at increased risk of aspiration

pneumonia caused by anaerobes.

19. Fever and sore throat in any patient taking antithyroid drug (propylthiouracil, methimazole)

suggests agranulocytosis (immune destruction of granulocytes). Most cases occur within 90

days of tx. Antithyroid drugs should be stopped and WBC count checked. A total WBC count

less than 1,000/cubic mm warrants permanent discontinuation of the drug. Routine WBC

count measurement in pts taking antithyroid drugs is not cost effective.

20. uncomplicated pseudopancreatic cysts smaller than 5cm should be observed for 6 weeks

before any further therapeutic intervention.

21. The MCC of lead poisoning in adults is work exposure. Typical symptoms include GI

complaints, poor concentration, and anemia. The tx is exposure reduction and a lead chelator

such as EDTA or succimer.

22. Hyperparathyroidism is an uncommon cause of secondary hypertension. Asymptomatic

hypercalcemia in hypertensive pt or the presence of ‘stones, bones, groans, and psychiatric

overtones’ should raise suspicion for this condition. DD renal parenchymal disease, renal

artery stenosis, aortic dissections, hypothyroidism, adrenal cortex, medulla

23. Supplementation with folic acid is recommended in all pts with sickle cell anemia to prevent

the occurrence of aplastic crisis. Hydroxyurea produces its beneficial effect by increasing HbF

levels. It is used in pts with sickle cell anemia when their painful episodes occur very

frequently.

24. The odds ratio (OR) is used to interpret a case control study. OR value interpretation>1 The factor being studied is a risk factor for the outcome<1 The factor being studied is a protective factor in respect to the outcome=1 There is no significant difference in outcome in either exposed or unexposed group

25. Non-invasive testing is critical in the work-up of DVT. For pts at moderate to high risk,

compression ultrasonograph is used to determine the need for anticoagulation. In pts at low

risk, the D-dimer level should be checked. If negative, no anticoagulation is necessary. If

elevated, compression ultrasonography is used to determine the need for anticoagulation.

26. COPD is characterized by progressive expiratory airflow limitation which causes air trapping

and increased lung volumes. An FEV1/FVC ration of less than 0.7 on spirometry is

diagnostic.

27. Acute monocytic (FAB M5) leukemia—leukocytosis with many monoblasts, promonocytes

and monocytes. Monoblasts lack Auer rods, and are peroxidasae negative and nonspecific

(alpha-naphthyl) esterase positive.

Acute myeloblastic leukemia with maturation (M2) –predominance of myeloblasts on the

peripheral blood film.

APML (promyelocytic)—many hypergranular promyelocytes. Each cell had many Auer rods.

There is a high incidence of DIC in these pts

Acute lymphoblastic leukemia—predominance of lymphoblasts on the peripheral blood film.

These lymphoblasts are mostly PAS-positive.

Acute erythroleukemia—erythroblasts, which have an irregular outline and a high nuclear-

cytoplasmic ratio.

32

28. The most beneficial therapy to reduce the progessio of diabetic nephropathy in the presence of

renal insufficiency is strict blood pressure control. The data on dietary restriction and

progression of nephropathy are controversial.

29. Massive doses of aspirin and NSAIDs an cause acute erosive gastritis and upper GI bleeding.

Alcohol can aggravate their effect.

30. Know the complication of ventilation with a high PEEP: alveolar damage, tension

pneumothorax and hypotension. Tension pneumothorax may present with sudden-onset

shortness of breath, hypotension, tachycardia, tracheal deviation, and unilateral absence of

breath sounds.

31. Vit B12 deficiency Schilling testA dose of radiolabeled oral B12

An intramuscular injection of unlabeled

B12

radiolabeled B12 with intrinsic factor

Urinary excretion of radiolabeled B12Low dietary intake NormalPernicious anemia diminished NormalMalabsorption (celiac sprue, diphyllobothrium latum infestation)

Low excretion

32. office-based anoscopy/proctoscopy should be the initial procedure in pts less than 50 years old

who present with minimal bright red blood per rectum (bright red blood on toilet paper after

wiping. A few drops of blood in the toilet bowl after defecation, and small amounts of blood

on the surface of the stool) and do not have any risk factors for colon cancer. Blood

intermixed with stool is not included in this category. MCC: hemorrhoids, anal fissure,

polyps, proctitis, rectal ulcers and cancer.

33. Constrictive pericarditis causes hepatomegaly, ascites and increased JVP due to decreased

diastolic filling and impairment of cardiac output. Common causes include radiation therapy

(Hodgkin lymphoma), viral pericarditis, and cardiac surgery.

34. Allergic contact dermatitis is an inflammatory skin condition caused by allergens such as

poison sumac, cosmetics and nickel. It presents days to weeks after exposure with an intensely

pruritic erythematous rash with vesicles at the site of exposure. Secondary infection as a result

of excessive screatching is possible, and is suggested by pus filled vesicles.

Suppurative hidradenitis is a chronic follicular occlusive disease most commonly affecting the

intertriginous skin, including the axillary, groin and inframammary regions. It appears as painful

inflammatory nodules and draining sinus tracts that can last weeks or longer.

35. Any elderly pt who presents with pneumonia, abdominal pain, confusion and hyponatremia

should be suspected for Legionella pneumonia. This should also be suspected in pts who fail

to respond to beta-lactam antibiotics and Gram stain showing many polymorphs with few

visible organisms. Tx of choice is high dose erythromycin or azithromycin.

36. Know the epidemiology of these classic fungal infections. A pt from Arizona/California

should make you think of coccidiodomycosis.

37. Gallstones are common in the ‘fat, female, fertile woman of forty.’ 80% cholesterol + mixed

(radialucent); 3rd type—calcium bilirubinate. The gallbladder hypomotility and stasis

33

associated with pregnancy lead to the formation of biliary sludge, a precursor to gallstones.

Pregnant pts who experience repeated attacks of biliary colic can undergo cholecystectomy,

preferably during the second trimester. Those pts who prefer not to undergo cholecystectomy

may be given a bile salt such as ursodeoxycholic acid to dissolve the cholesterol stones. The

gallstones frequently recur once the treatment is stopped, however. Gallstones are far more

common in the US and other Western countries than in Aisa.

38. An elevated biliary sphincter pressure is highly specific for sphincter of Oddi dysfunction.

Sphincter dysfunction is characterized by right upper quadrant pain and elevation of liver

enzymes during the pain attack, which resolve when the attacks subside. ERCP with a

sphincterotomy is the most appropriate treatment.

39. Portal hypertention is the MCC of ascites. Portal hypertension is usually due to cirrhosis from

chronic liver disease (alcoholic or viral). Intravenous drug abuse predisposes to cirrhosis by

putting individuals at increased risk for chronic infection with HBV or HCV.

40. In the evaluation and management of acute renal failure (ARF—acute oliguria <250cc urine

in 12 hours; azotemia and increased serum creatinine), foley cathererization is a critical first

step. It can resolve various post-renal obstructions in addition to facilitating monitoring of

urine output in pre-renal and intrinsic renal failure.

41. Hypothyroidism is the most common side effect of radioiodine therapy, but it is easily treated

with levothyroxine therapy. The pt’s eye disease may worsen at the beginning of radioiodine

therapy. Extremely high yield!!

42. Eaton-Lambert syndrome is associated with small cell carcinoma of the lung, and results from

autoantibodies directed against the voltage-gated calcium channels in the presynaptic motor

nerve terminal. Electrophysiological studies confirm the diagnosis (the muscle response to

motor nerve stimulation should increase with repetitive stimulation). Tx consists of

plasmapheresis and immunosuppressive drug therapy.

Autoantibodies against postsynaptic receptors cause myasthenia gravis.

Immune-mediated muscle inflammation suggests polymyositis.

Multicentric CNS inflammation and demyelination are indicative of multiple sclerosis.

43. The 3 main categories of diabetic retinopathy are background or simple (microaneurysms,

hemorrhages, exudates, retinal edema), pre-proliferative (cotton wool spots), and proliferative

or malignant (neovascularization). Visual impairment occurs with the development of macular

edema. Argon laser photocoagulation is preformed for the prevention of complications.

44. Mohs surgery is a special type of surgery wherein microscopic shaving is done, such that 1-

2mm of clear margins are excised. This technique currently has the highest cure rate for basal

cell cancer, but is indicated only in pts with high-risk features, as well as those with lesions in

functionally critical areas (e.g. perioral region, nose, lips , ears).

12/21/09

1. Ventricular remodeling in the weeks to months following myocardial infarction can lead to

dilation of the ventricle. This process is lessened by ACEIs—should be initiated within 24

hours of myocardial infarction in all pts without a contiaindiction.

2. Bacillus cereus causes nausea and vomiting after eating rice. Symptom onset is within 1-6

hours. Staph aureus toxin is present in foods such as dairy, salad, meat, and eggs. Symptoms

include nausea, vomiting, diarrhea, and abdominal pain. Symptom onset is within 1-6 hours.

34

3. Disseminated gonococcal infection often presents with a triad of polyarthralgias,

tenosynovitis (pain elicited along the tendon sheaths), and vesiculopustular skin lesions.

4. Suspect babesiosis in any pt from an endemic area who presents with a tick bite. This illness

is caused by the parasite Babesia and is transmitted by the Ixodes tick. It endemic in the

northeastern US. Following a tick bite, the parasite enters the pt’s RBC and causes hemolysis.

Clinical manifestations vary from asymptomatic infection to hemolytic anemia associated

with jaundice, hemoglobinuria, renal failure, and death. Unlike other tick-borne illness, rash is

not a feature of babesiosis, except in severe infection where thrombocytopenia many cause a

secondary petechial or purpuric rash. Clinical significant illness usually occurs in person over

age 40, pts without a spleen, or immunocompromised individuals. Dx: Giemsa-stained thick

and thin blood smear. Labs—intravascular hemolysis, anemia, thrombocytopenia, mild

leucopenia, atypical lymphocytosis, elevated ESR, abnormal LFT, and decreased serum

complement levels. The two most widely used drug regimens are quinine-clindamycin and

atovaquone-azithromycin.

Ehrlichiosis, or “spotless RMSF”, is another tick-borne illness. Clinical features include fever,

malaise, headache, nausea, and vomiting. Labs show leuckopenia and thrombocytopenia.

Hemolysis and jaundice are not common.

Q fever is a zoonosis caused by Coxiella burnetii. The main sources of human infection are

infected cattle, goat, and sheep. People at risk include meat processing workers and veterinarians.

Infection due to C. burnetii occurs in most areas of the world. Manifestation of Q fever many

include a flu-like syndrome, hepatitis, or pneumonia.

5. Zinc deficiency may result form chronic total parenteral nutrition (TPN) or malabsorption.

Symptoms include alopecia, skin lesions, abnormal taste, and impaired wound healing.

Vit A deficiency many be caused by nutritional deficiency or malabsorption. Symptoms include

blindness, dry skin, and impaired immunity.

Selenium deficiency, like zinc deficiency, may result from chronic TPN. It may also result from

malabsorption or malnutrition (food source include nuts, meat, and fish). The most important

feature is cardiomyopathy.

6. Edema in congestive heart failure is due to both mechanical impedance to fluid removal and

increase renal sodium retention.

7. Esophageal cancer may mimic achalasia. Features that favor malignancy over achalasia are: a

short history, rapid weight loss, and inability of the esophagoscope to pass through the lower

esophageal sphincter.

8. Abdominal ultrasound is the study of choice for diagnosis and follow-up of abdominal aortic

aneurysms, as it has nearly 100% sensitivity and specificity of this condition. Drawbacks to

CT imaging as compared to ultrasound include higher cost, requirement for contrast (which

would be C/I in this pt with chronic renal insufficiency), and poor visualization of aortic

branch origin. CT scan can add valuable information when the differential diagnosis of a pt’s

abdominal pain is broad, as it provides a better overall examination of the abdomen. MRI is a

poor imaging choice for pts with renal insufficiency given the association of nephrogenic

dermatopathy with gadolinium administration. It is also much more expensive than

ultrasound.

9. Any pt who return from a developing country and has symptoms suggestive of malabsorption

should be considered for empirical tx with metronidazole for giadiasis. Remember for the

35

pathophysiology (adhesive disks and malabsorption). Extremely high yield!!

10. Warfarin is commonly used for anticoagulation bz it can be administrated orally. However,

during first few days that it’s given, it can have paradoxically pro-coagulant effects in

susceptible pts by inhibiting protein C production. Affected pts may present with skin necrosis

secondary to thrombus formation.

11. It is important to be able to distinguish between different types of tremors. Resting tremor is

often the presenting symptom Parkinson’s disease. It usually starts in one hand before

involving the other extremities.

Essential tremor is the most common type of tremor, affecting up to 5% of the population. It is

familial in up to 50% of cases. It typically presents as an intention tremor in the upper extremities

that is worst at the end of goal-directed activities (eg. reaching for a pen). The head, chin, voice,

and trunk may also be affected.

Cerebellar dysfunction can cause postural and intention tremors and , when severe, resting

tremors. The tremors have a lower frequency than those in Parkinson (5-7 Hz), typically 3-4Hz.

Cerebellar tremors can affect the extremities and whole head. Nystagmus and other signs of

cerebellar dysfunction are usually also present.

Large fiber peripheral neuropathy may be accompanied by tremor. Tremor is more common in

hereditary neuropathies (eg.Charcot-Marie-Tooth), chronic inflammatory demyelinating

polyneuropathy (CIDP), and the recovery phase of Guillain-Barre syndrome. Weakness, loss of

proprioception, and loss of sensation are often also seen.

12. Acute back pain education—strengthening the supporting muscles (including abdominal

muscles) by regular exercise, choosing an appropriate sleeping posture (eg. Avoiding sleeping

on the stomach), and learning proper techniques for bending and lifting objects. Exercises

with repetitive twisting and bending should be avoided. It is important to bend at the knees,

not at the waist. While lifting an object, one should also bend the knees, keeping the back

straight, this techniques is very useful in preventing strains and back injuries. Warm-up

exercises should be done before any sporting activities.

13. Tick-borne paralysis is characterized by rapidly progressive ascending paralysis, absence of

fever, absence of sensory abnormalities and normal CSF examination.

Guillain-Barre—ascending paralysis over days to weeks. Sensation is either mildly or grossly

abnormal. CSF-albumino-cytologic dissocation. Tx-IVIG and plasmapheresis.

Spinal cord tumor may also present with an ascending paralysis over days to weeks. Sensation is

either mildly or grossly abnormal. MRI of the spine may be used to confirm the Dx. Tx-IV

steroids.

14. All pts who are diagnosed with HIV infection should receive the following: 1) detailed history

and physical examination. 2) Routine chemistry and hematology. 3) Two plasma HIV RNA

levels. 4) CD4 T lymphocyte count. 5) VDRL test for syphilis. 6) PPD skin test. 7) Anti-

Toxoplasma antibody titer. 8) Mini Mental Status Examination. 9) Pneumococcal

polysaccharide vaccine, unless the CD4 counts less than 200cm3. 10) HAV, HBV serology.

11) HAV, HBV vaccine if seronegative. 12) HIV counseling. 13) information and assistance

for those who might have been infected by the subject.

Studies before starting antiretroviral tx: 1) Complete history and physical examination. 2) CBC,

complete blood chemistry, serum transaminases, and lipid profile. 3) CD4 T lymphocyte count. 4)

Plasma HIV RNA titer.

36

PCP prophylaxis: CD4<200/cm3

15. Painless jaundice in an elderly pt should make you think about pancreatic head carcinoma.

DD: both AlT and AST should be high incase of Viral hepatitis and hepato-cellular carcinoma.

16. Dacryocystitis is an infection of the lacrimal sac in the medial canthal region of the eye. Staph

aureus and beta-hemolytic strep are the usual infection organisms. Tx-systemic antibiotic.

Episcleritis巩膜外层炎 is an infection of the episcleral tissue between the conjunctiva and sclera.

Pts complain of acute onset of mild to moderate discomfort, photophobia, and watery discharge.

Examination reveals diffuse or localized bulbar conjunctival injection.

Hordeolum麦粒肿(stye) refers to an abscess located over the upper or lower eyelid. It is usually

caused by staph aureus. It appears as a localized red, tender swelling over the eyelid.

Chalazion睑板腺囊肿 presents as lid discomfort. It is a chronic, granulomatous inflammation of

the meibomian gland. It appears as a hard, painless lid nodule.

17. Predominantly found in children who were given aspirin for virus-induced fever, Reye

syndrome can lead to fulminant hepatic failure. Elevation of ammonia levels and

transaminases, vomiting, and mental status changes are the more common clinical

manifestation of this syndrome (diffuse mitochrondrial injury.)

Balloon degeration with polymorphic cellular infiltrates –acute alcoholic hepatitis.

18. Matching is frequently used in case-control studies bz it is an efficient method to control

confounding. Remember: matching variables should always be the potential confounders of

the study (eg. Age race). Cases and controls are then selected based on the matching variables,

such that both groups have a similar distribution in accordance with the variables.

19. The MCC of iron deficiency anemia in an elderly pt is GI blood loss. A single negative fecal

occult blood test is not sufficient to diagnosis occult blood loss. The next step in evaluation

would be colonoscopy. If negative, upper GI endoscopy. Another indication for performing an

upper GI endoscopy is a current or significant history of NSAIDs and/or history of peptic

ulcer disease. If both negative, capsule endoscopy is preformed to look for small intesitine

disease (eg. Arteriovenous malformations).

20. VitB12 deficiency is common after a total or partial gestrectomy. VitB12 is a necessary

cofactor in the pathway leading to purine synthesis. B12 deficiency decrease DNA synthesis

and erythropoiesis, leading to a megaloblastic anemia.

21. Fibromuscular dysplasia can present as new onset hypertension in children. Bruit or venous

hum may be heard at the costovertebral angle. Angiogram reveals the “string of beads” sign.

22. Neurofibromatosis 1—café-su-lait spots, macrocephaly, feeding problems, short stature, and

learning disabilities. Pts may later develop fibromas. Neurofibromas or different tumors.

Neurofibromatosis 2—bilateral acoustic neuromas and cataracts.

23. Helical CT angiogram of the chest is currently the investigation of choice for dx PE in pts

with normal serum creatinine. Ventilation perfusion scan is ordered in pts where iv contrast is

C/I, such as renal failure pts.

24. Gout is a common manifestation of myeloproliferative disorder-induced increases in uric acid

production. Polycythemia vera commonly causes splenomegaly and characteristic pruritis

with hot baths due to histamine release from an increased number of circulating basophils. Up

to 40% pt suffer from gout.

25. Subdural hematoma results from the rupture of bridging veins. Epidural hematoma results

from the rupture of the middle meningeal artery.

37

26. Suspect ischemic colitis in pts who have evidence of atherosclerotic vascular disease, present

with abdominal pain followed by bloody diarrhea, and have minimal abdominal exam

findings. The most commonly involved segment of the colon is the splenic flexure, because it

is supplied by end arteries (watershed). Another watershed zones in colon is the recto-sigmoid

junction, which is supplied by narrow terminal branches of the inferior mesenteric artery.

27. Mx of nephrolithiasis: 1) imaging study—CT scan of the abdomen without contrast is the

investigation of choice bz of its high sensitivity and specificity. Better than KUB in detecting

radiolucent stones. 2) Narcotics and NSAIDs—equally effective in relieving the pain of acute

renal colic, however, in pts with normal renal function, NSAIDs are preferred over narcotics

bx the latter can exacerbate nausea and vomiting. 3) stones <5mm in diameter typically pass

spontaneously with conservative Mx. This includes a fluid intake of greater than 2L daily.

Increased hydration increases the urinary flow rate and lowers the urinary solute

concentration, thus preventing stone formation. 4) Urology referral—urgent urologic

evaluation is warranted in pts with anuria, urosepsis, or abut renal failure. Hydration is the

cornerstone of therapy for renal stone disease. A detailed metabolic evaluation is not needed

when a pt present with his first renal stone.

28. Massive pulmonary embolism is likely in a postoperative pt with jugular venous distension

and new onset right bundle branch block (right heart stain).

29. Cause of widened mediastinum on CXR include aortic dissection (no hypotension),

mediastinal mass, esophageal rupture, SVC syndrome (dyspnea and facial swelling), anthrax

exposure, and mediastinal hemorrhage (hypotension). The clinical history is critical to making

the Dx.

30. A 50-year-old man at average risk for colon cancer, CAD, and prostate cancer should have

colon cancer screening using any number of modalities, cholesterol measurement, and discuss

about prostate cancer screening with PSA measurement.

31. Hyposthenuria 低渗尿 is common in pts with sickle cell anemia and sickle cell trait.

32. Radioiodine therapy is more likely to cause permanent hypothyroidism in pts with Grave’s

disease than those with multinodular goiter or toxic adenoma (bz the whole thyroid glands is

hyperfunctional).

33. Irritable bowel syndrome is a functional disorder of the GI tract characterized by abdominal

pain with diarrhea and/or constipation. There is no lab or pathologic hallmark.

34. Amiodarone is an effective antiarrhythmic but its use is associated with several potential side

effects. In particular, it can cause pulmonary fibrosis, hepatotoxicity, and thyroid dysfunction.

Pulmonary function tests, LFTs and thyroid function tests should be monitored in pts on

chronic amiodarone therapy.

Hydralazine—salt retention, reflex tachycardia, and a lupus-like syndrome.

Metoprolol-impotence, brahycardia, and AV blockade.

Verapamil—constipation, dizziness, and flushing.

Enalapril—hyperkalemia, cough, decrease GFR, rash and angioedema.

35. Duodenal ulcer disease typically presents with epigastric pain that improves with eating. Over

90% pts with duodenal ulcer disease are infected with H. pylori. Tx is acid suppression and

organism eradication with antibiotics.

36. The nocturnal penile tumescence helps differentiate psychogenic from organic causes of male

erectile dysfunction. It is positive in psychogenic causes and negative in organic cuases.

38

37. Think of malignant melanoma in a pt with a metastatic mass that is bleeding inside brain. 4

tumors in the body not been reported to metastasize to the brain—1) non-melanomatous skin

cancer, 2) oropharyngeal cancer, 3) esophageal carcinoma, 4) prostate cancer

38. Increase intragastric pressure during vomiting can cause tears in the mucosa of the cardia 贲

门, an sometimes of the distal esophagus—Mallory-Weiss tears.

39. Acidosis accompanied by hypercarbia and a normal or elevated serum bicarbonate is

diagnositic of a respiratory acidosis. Hypovantilation is a major cause fo respiratory acidosis

and may occur in the post-ictal state.

40. Suspect celiac disease in any pt who presents with malabsorption and iron deficiency anemia,

Celiac disease is associated with anti-endomysial antibodies (IgA).

Symptoms:

1) characteristically bulky, foul-smelling, floating stool

2) loss of muscle mass or subcutaneous fat

3) pallor-iron deficiency anemia

4) born pain-osteomalacia

5) easy bruising-vitK deficiency

6) hyperkeratosis-vitA deficiency

41. MENII consists of medullary carcinoma of the thyroid, hyperparathyroidism and

pheochromocytoma. The serum calcitonin level is elevated in pts with medullary thyroid

cancer.

42. In pts with fever and a cough productive of foul-smelling sputum after an upper GI endoscopy

or any other instrumentation of the upper airway or esophagus suspect an anaerobic lung

infection. Clindamycin is the most commonly used agent for this type of infection.

43. Amaurosis fugax is the painless loss of vision from emboli, and is a warning sign of an

impending stroke. Most emboli occur from the carotid bifurcation; hence, a duplex untrasound

of the neck should be performed.

44. One of the most practical methods of primary prevention of melanoma is wearing protective

clothing such as tightly woven fabrics, hats, and long sleeved shirts. Currently, the available

data shows little to no protection against melanoma with the use of sunscreen lotions with

SPF 15-30; however, these can protect from non-melanoma skin cancers such as squamous

cell cancer.

12/22/09

1. Ankylosing spondylitis is a seronegative spondyloarthropathy that presents with progressive

inflammatory back pain and stiffness. In the setting of suggestive findings on history and

physical, AS is confirmed by pain film x-ray demonstrating fused sacroiliac joints and/or

bamboo spine. Pts often complaint of morning stiffness lasting >30 minutes, and their back

pain typically improves with exercise. Any young pt presenting with progressive low back

pain and spinal stiffness of > 3 months duration should be investigated for this condition.

2. Biphosphonates (Zoledronic acid) are the drugs of choice for mild to moderate hypercalcemia

due to malignancy (bone metastatic breast cancer, multiple myeloma and radiographic lytic

bone disease.) Corticosteroids are used in pts with hypercalcemia due to extra renal

production of 1,25 hydroxy vitamins D (calcitriol) such as carcoidosis.

3. Alcohol hepatitis is characterized by an AST:ALT>2. It is thought that this transaminase

39

imbalance occurs in alcoholic secondary to a hepatic deficiency of pyridoxal-6-phosphate (a

cofactor for ALT enzymatic activity). The absolute values of serum AST and ALT are almost

always less than 500 IU/L in alcoholic liver disease. If higher, concurrent hepatic injury

secondary to viral, ischemia, or acetaminophen usage. Females are more susceptible than

males to hepatic injury form alcohol ingestion. Fatty liver is present in 80-100% of binge or

heavy drinkers .Classical findings—ballooning degeneration, PMN cellular infiltration,

fibrosis, and necrosis. Mallory bodies are often observed in severe cases, but are not specific

for alcoholic hepatitis.

4. The most typical findings of a PE on arterial blood gas sampling are hypoxemia (increased

A-a gradient) and hypocarbia (tahcypnea-respiratory alkalosis).

5. In all pts with COPD, the two modalities that have been shown to decrease mortality are

home oxygen therapy and smoking cessation.

6. The indication for parathyroidectomy is asymptomatic pts with primary

huyperparathyroidism are : 1) serum calcium level at least 1 mg/dL above the upper limit of

normal. 2) 24-hr uninary calcium >400mg. 3) yong age <50. 4) BMD (bone mineral density)

<T-2.5 at any site. 5) difficulty in follow up of the pt.

7. Pancreatic cancer is the primary differential diagnosis in pts with chronic pancreatitis.

Abdominal ultrosonogram is the initial investigation of choice in pts with jaundice; however,

CT scan has a higher sensitivity for suspected pancreatic cancer. ERCP is most useful for pts

in whom there is a high suspicion for chronic prancreatitis, or in cases where CT or USG fails

to reveal a mass lesion within the pancreas. MRI is the investigation of choice in pts in whom

ERCP is not possible due to pancreatic duct obstruction.

8. Acute pyelonephritis can potentially result in gram-negative sepsis. Urine and blood cultures

should be routinely obtained prior to administering antibiotics (ciprofloxacin or ceftriaxone).

9. Hyperkalemia is a medical emergency. Therapy involves three steps: membrane stabilization

with calcium, shifting potassium intracellularly, and decreasing the total body potassium

content. Insulin/glucose administration is the quickest way to decrease the serum potassium

concentration.

10. Reassurance is the most appropriate course of action for a pts with known immunity to HBV

who is exposed to the disease. The HBIG and the HBV vaccination series should be given to

pts with unknown immunity after exposure.

11. Finasteride acts on epithelium and alpha-1 blockers act on smooth muscles of prostate and

bladder base.

12. Diarrhea (either watery or bloody) due to Vibrio parahaemolyticus is usually transmitted by

the ingestion of seafood (including shrimp, crab, and raw oysters). Campylobacter—

undercooked infected poultry. Watery or hemorrhagic diarrhea, along with severe abdominal

pain.

13. The A-a gradient is increased in interstitial lung disease due to reduced diffusing capacity and

V/Q mismatch. In restrictive lung disease, the total lung capacity (TLC), functional residual

capacity and residual volume are all reduced. Flow volumes are also reduced but the ratio of

FEV1/FVC is either normal or increased.

14. Pts with alpha-thalassemia (target cell) minor or beta-thalassemia minor will have an

impressive microcytosis but only modest anemia. Reassurance is the only intervention

necessary for these pts. In contrast, pts with thalassemia major are generally severely

40

symptomatic and transfusion-dependent.

15. Mucosal neuromas are the most distinct feature of multiple endocrine neoplasia type 2b, and

are present in more than 90% of cases.MEN I MEN-2a MEN 2bPituitary adenoma Medullary thyroid cancer Medullary thyroid cancerPancreatic islet cell tumor Pheochromocytoma Pheochromocytomahyperparathyroidism Primary parathyroid hyperplasia Mucosal neuromas and

Marfanoid habitus

16. The majority of human infections with E.granulosus are asymptomatic. Hydatid cysts can be

found in almost any part of the body but liver (MC) is involved in 2/3 of pts and the lungs are

involved in 25% of pts. It is most commonly seen in areas where sheep are raised.

17. The kidney compensates for respiratory alkalosis by preferentially excreting bicarbonate in

the urine. The result is an alkalinized urine (increased urine pH).

18. Amyotrophic lateral sclerosis (ALS) is characterized by the presence of both upper

(spasticity, bulbar symptoms, hyperreflexia) and lower (fasciculations) motor neuron lesions.

19. ACE inhibitors, ARBs, beta-blockers, and spironolactone all confer a survival benefit in

CHF. While digoxin and furosemide (loop diuretics) can reduce CHF symptoms and

hospitalizations, they do not improve survival.

20. Hepatorenal syndrome is a deadly complication of cirrhosis characterized by renal failure that

does not respond to volume resuscitation (1.5L normal saline bolus). It is best treat with liver

transplantation.

21. Microcytic anemia and basophilic stippling in a child are important clues to the Dx of lead

poisoning.

22. Elderly pts are particularly predisposed to dehydration after even minor insults (eg. a minor

febrile illness). Know the classic signs of dehydration (ie. Dry mucosa, marginally high

values for hematocrit and serum electrolytes, BUN/creatinin ration >20). The Tx is

administration of iv sodium containing crystalloid solutions (usually 0.9% NaCl= normal

saline). Colloid solutions (eg. albumin) are usually used in burns or conditions accompanied

by hypoproteinemia.

23. Normal pressure hydrocephalus (NPH) is thought to result from decreased CSF absorption or

transient increases in intracranial pressure that cause permanent ventricular enlargement

without chronically increasing intracranial pressure. It is characterized clinically by dementia,

gait disturbances, and incontinence. Increase in CSF production are not strongly associated

with a particular disease process.

24. Fibrolytic therapy improves neurologic outcomes in pts with ischemic strokes when given

within 3 hours of symptom onset. Before administering tPAm a non-contrast head CT should

be performed to rule out hemorrhage stroke and the pt should be screened for other

contraindications to therapy. Urgent anticoagulation with Heparin is not recommended for

acute ischemic stroke, even when the etiology is cardioembolic, because of the increased the

risk for intracereral hemorrhage. However, anticoagulation with warfarin can be started two

weeks after an acute cardioembolic stroke to prevent recurrence.

25. Proper Tx of mild persistent asthma includes a PRN albuterol inhaler and inhaled

corticosteroids.

26. In general, membranous nephropathy is the most common nephropathy associated with

41

carcinoma, however, nephritic syndrome is a well-known complication of Hodgkin’s

lymphoma, and is usually caused by minimal change disease. Extremely high yield!!

27. Factors independently associated with high rates of liver fibrosis in chronic HCV are: 1) male

gender. 2) Acquiring infection after age of 40. 3) Alcohol intake: alcohol intake in any

amount can hasten the progression of fibrosis.

28. Acute exacerbations of MS are treated with corticosteroids. Beta-interferon or glatiramer

acetate is used to decrease the frequency of exacerbations in pts with relapsing-remitting or

secondary progressive form of MS.

29. DM causes a number of complications, with neuropathy being very common. Diabetic most

commonly have (symmetric) peripheral neuropathy, mononeuropathy, and autonomic

neuropathy. Mononeuropathies are divided into cranial and somatic-among cranial nerves.

CNIII is affected the most often. In diabetes, oculomotor nerve (CNIII) neuropathy is

ischemic. Somatic and parasympathetic fibers of CNIII have separate blood supplies. For this

reason, diabetic CNIII neuropathy affects only somatic fibers, leaving parasympathetic fibers

intact. Symptoms are ptosis and a “down and out” gaze. Accommodation and the pupil’s

response to light remain intact. (Alternatively, if nerve compression causes CNIII neuropathy,

both somatic a parasympathetic fibers will be affected. Pts will have ptosis, a “down and out”

gaze, a fixed, dilated pupil, and no accommodation reflex.)

CNIII (oculomotor nerve) has the following functions:

1) Its somatic component innervates the inferior, superior, and medial rectus, inferior

oblique and levator palpebrae muscles. Ptosis occurs due to paralysis of the levator

palpebrae; whereas, the unopposed action of the lateral rectus (CN VI) and superior

oblique (CN IV) muscle lead to a “down and out” gaze.

2) Parasympathetic fibers of CNIII innervated the sphincter of the iris and the cilliary

muscle. Paralysis of parasympathetic fibers causes a fixed, dilated pupil and causes an

inability of the lens to change shape (loss of accomodation).

30. Suspect toxoplasmosis in AIDS pt with multiple ring-enhancing lesion on CT. Administration

of empirical sulfadiazine and pyrimethamine is both diagnostic and therapeutic.

31. Suspect Creutzfeldt-Jacob disease in an old pt (50-70 yeas old) with rapidly progressive

dementia, myoclonus and periodic synchronous bi-triphasic sharp wave complexes on EEG.

DSM-IV criteria for dx of Alzheimer’s disease:

1) Gradual impairment of cognitive function resulting in social or occupational dysfunction.

2) Impaired recent memory with one or more of the following: impaired executive function,

impaired visual processing, and impairment of skilled motor activities.

3) Absent of other psychiatric, neurologic or systemic diseases.

4) Occurrence of deficits not exclusively in the setting of delirium.

32. Nephrotic syndrome is a hypercoagulable condition which manifests as venous or arterial

thrombosis, and even pulmonary embolism. Renal vein thrombosis is the most frequent

manifestation. Complications of nephritic syndrome include: protein malnutrition, iron-

resistant microcytic hypochromic anemia (transferrin loss), increased susceptibility to

infection, and vitamin D deficiency (increase urinary excretion of cholecalciferol-binding

protein).

33. Xanthelasma are cholesterol-filled yellow plaques that appear on the medial aspects of the

eyelids bilaterally. The upper and lower eyelids may be affected. Xanthelasma may occur in

42

the setting of primary biliary cirrhosis though these lesions are most commonly idiopathic.

34. H1 histamine receptor antagonists decrease nasal inflammation and post-nasal drip in pts

with allergic rhinitis. 5-20% pts treated with ACEIs develop dry cough. This is bc decreased

degradation of bradykinin and substance P. Accumulation of these substances causes cough.

35. In otherwise young healthy pts who develop CHF, myocarditis should be considered thigh on

the differential. Vital infection, especially with Coxsachie B virus, is the MCC.

36. Scabies is a high contagious disease which presents with generalized itching and pruritic

papules over the penis and scrotum in males, and on the areolas and breasts in females. For

adults, the tx is 5% permethrin cream, which is applied from the neck down and left

overnight.

37. Suspect leprosy in an Asian patient with an insensate, hypopigmented patch of skin. Dx is

made by demonstration acid –fast bacilli on skin biopsy. Pregrassive peripheral nerve damage

results in muscle atrophy, with consequent crippling deformities of the hands. The most

common affected sites are the face, ears, wrists, buttocks, knees, and eyebrows.

38. The US Preventive Services Task Force (USPSTF) strongly recommends routine screening

for chlamydial infection in all sexually active women age 24 years and younger, and in other

asymptomatic women at increased risk for this infection.

39. The development of clubbing and sudden-onset joint arthropathy in a chronic smoker is

suggestive of hupertrophic osteroarthropathy. This condition if often associated with lung

cancer, and therefore a chest x-ray is indicated to rule out malignancy and /or other lung

pathology.

40. Gouty arthritis cannot be reliably distinguished from septic arthritis or pseudogout base on

history and exam alone; therefore, synovial fluid analysis is critical to diagnosis.

41. Suspect tropical sprue in pts with malabsorption, along with a history of living in endemic

areas for more than one month. Tropical sprue involves the small intestine; the typical biopsy

is characterized by blunting of villi with infiltration of chronic inflammation cells, including

lymphocytes, plasma cells and eosinophils.

42. Ticks should be removed promptly in order to decrease the risk of developing a tick-borne

illness. The recommended technique is to grasp the tick’s mouth parts with tweezers and

remove the tick using slow, constant pressure.

43. Tumor lysis syndrome is characterized by huperphosphatemia, hypocalcemia, hyperkalemia,

and hyperuricemia. Tumors with high cell turnovers (eg, poorly differentiated lymphoma and

leukemias) are the frequent culprits. Both potassium and phosphate are intracellular ions, and

the breakdown of the cells release excess amounts of these. Release phosphate binds calcium

and causes hypocalcemia. Hypocalcemia is also due to the release of intracellular products by

cell lysis. Degeneration of cellular proteins causes elevation of uric acid levels. Allopurinol

may greatly reduce the possibility of acute urate nephropathy.

44. Alcohol abuse is the MCC of nutritional folate deficiency in US and would cause

megaloblastic anemia.

12/24/09

1. In any dog bite, an attempt is made to capture the dog.

1) If the dog is not captured, it is assumed rabies, and post-exposure prophylaxis is

indicated.

43

2) If the dog is captured and does not show features of rabies, it is kept for observation for

the development of rabies for 10 days. If the dog develops any features of rabies, post-

exposure prophylaxis should be started immediately. The dog’s diagnosis is confirmed by

fluorescent antibody (FA) examination of the brain.

3) Post-exposure prophylaxis should be started immediately for exposures involving the

head and neck.

2. Hypoglycemia is associated with multiple differential diagnoses. Helpful tests used in the

evaluation of hypoglycemia patients are measurements of c-peptide, proinsulin and

sulfonylurea levels (surreptitious use of insulin or sulfonulurea). Hypoglycemia secondary to

insulinoma is assocated with elevated sulin, c-peptide and proinsulin levels.

Non-beta cell tumors, typically large mesenchymal tumors, can lead to hypoglycemia independent

of insulin. Such tumors produce insulin-like growth factor II (IGF II), which has an

insulinomimetic action after binding to insulin receptors. In pts with suspected non-beta cell

tumors, the serum IGF II level can be measured. Pts with this condition characteristically have

suppressed insulin and c-peptide levels.

3. Acute bacterial prostatitis—In young pts, the MCC organisms are sexually transmitted

organisms such as Chlamydia and gonococci. In elderly pts, E.coli is the MCC. Prostatic

massage and urethral catheterization are C/I due to the risk of septicemia.

4. Choriocarcinoma is a metastatic form of gestational trophoblastic disease. It may occur after

molar pregnancy or normal gestation, and the lungs are the most frequent site of metastatic

spread. Suspect choriocarcinoma in any postpartum woman with pulmonary symptoms and

multiple nodules on CXR. An elevated beta hCG helps to confirm the diagnosis.

5. Elderly pts are particularly sensitive to fluid loss, and even mild hypovolemia may predispose

them to orthostatic syncopy, especially upon getting up in the morning. BUN/creatinine ratio

is a useful indicator of dehydration.

6. Precision is the measure of random error (not scattered widely). The tighter the confidence

interval, the more precise the result. Increasing the sample size increases precision. Validity

(Specificity and sensitivity) and accuracy are measurements of systemic error (bias).Accuracy

is reduced if the result does not reflect the true value of the parameter measured.

7. The three major mechanical complications of MI include mitral regurgitation due to papillary

muscle rupture, left ventricle free wall rupture, and interventricular septum rupture. While all

3 of these developments can results in hypotension, a pansystolic murmur heard loudest at the

apex with radiation to the axilla is the classic characterization of mitral regurgitation. Left

ventricle free wall rupture classically results in pericardial tamponade. While this condition

often results in sudden death, pts who survive the initial insult may develop hypotension,

jugular venous distension, distant heart sounds, pericardial rub, and pulsus paradoxus. Like

papillary muscle rupture, interventricular septum rupture results in a pansystolic murmur. It

differs in that the murmur is heard best at the left sternal border, often has an accompanying

thrill, and is less likely to radiate to the axilla.

8. Metaclopramide is a prokinetic agent used to treat nausea, vomiting, and gastroparesis. Pts

taking this medication should be monitored closely for the development of drug-induced

extrapyramidal symptoms—tardive dyskinesia, dystonic reactions, and Parkinsonism (neck

pain an neck muscle are noted to be stiff and tender). Tx for a dystonic reaction involves

discontinuation of the medication and administration of benztropine or diphenhydramine.

44

9. External hordeolum or stye is a common staph abscess of the eyelid. It is treated with warm

compresses. Incision and drainage is performed if resolution does not begin in the next 48

hours. Incision and curettage is the Tx of choice for chalazion, which is a granulomatous

inflammation of the meibomian gland.

10. Pts who have received the equivalent of more than one blood volume of blood transfusions or

packed RBC over 24 hours may develop elevated plasma level of citrate (a substance added to

stored blood). Citrate chelates calcium and magnesium and may reduce their plasma levels,

causing parestesias. Stored RBC gradually loose intracellular potassium to the surrounding

solution. This mechanism has the potential to cause hyperkalemia, but not hypocalcemia.

11. Hypokalemia is a common electrolyte abnormality that causes weakness, fatigue, and muscle

cramps. When severe, it can lead to paralysis and arrhythmia. The ECG may show U waves,

flat and broad T waves, and premature ventricular beats.

12. Water restriction is the first step in managing hyponatremia in pts with SIADH. If the serum

sodium is below 120 or if the pt is seizing, emergency tx is asministration of 3% sodium

chloride solution to raise the serum sodium to 125. If the pt has evidence of fluid overloading,

a history of CHF, or is resistant to tx, loop diuretics may be added as well. Chronic tx may

involve administration of lithium or demeclocycline, which inhibit ADH action.

13. Chemical in the eye deserve immediate attention. The best primary course of action is to flush

the eye with water—best achieved under a faucet of running water—for at least 15 minutes.

Obtaining medical care is also appropriate once this treatment is initiated. For foreign bodies

or cuts/scratches of the eye, obtaining medical care is the first priority.

14. The classic physical exam findings of cardiac tamponade are pulsus paradoxus and Beck’s

triad: hypotension, jugular venous distention, and muffled heart sounds. When the pericardial

space filled with fluid, the ventricles are less able to expand to accept venous return to the

heart. As a result, overall preload to the heart decreases.

15. Hereditary spherocytosis is an autosomal dominant disorder of spectrin that leads to increased

RBC membrane fragility. It is characterized by a positive family history, splenomegaly,

spherocytosis, and can cause cholecystitis due to pigmented gallstones (tenderness in the right

upper quadrant exacerbated by deep inspiration.)

Gaucher’s disease can result in anemia, thrombocytopenia, and hepatosplenomegaly. Pts can also

have painful cortical bone erosions. Pts typically present at an earlier age and their pain is more

commonly ‘bone’ pain. Gaucher cells with eccentric nucleoli and PAS inclusions are diagnostic.

16. Acute pancreatitis is most commonly secondary to gallstones or alcohol abuse. However, it

may be also secondary to drug toxicity, idiopathic etiology, heypertriglyceridemia, infection,

severe trauma or burns, hypercalcemia, pancreatic abnormality, tumors, toxins or due to post

operative or post ERCP etiology. Hypertriglyceridemia>1000mg/dl can cause acute

pancreatitis. Hyperlipidemia types 1 and 5 are associated with acute pancreatitis. Acute

pancreatitis sencondary to hypertriglyceridemia often has normal levels of amylase.

17. Paracentesis provides the most useful diagnostic information in pts with ascites. It may reveal

signs suggestive of an exudates: turbid fluid, >250 leukocytes/mL; protein >2.5 g/L; serum-

to-ascitic albumin gradient <1.1; and specific gravity >1.016 g/L. Gram stain of the peritoneal

fluid should be done. Cultures of the blood and peritoneal fluid should also be done. Empiric

antibiotic therapy with third generation cephalosporins (eg. cefotaxime or ceftriaxone) should

be started immediately after obtaining cultures.

45

18. Analgestic nephropathy (eg. aspirin and naproxen, female, 50-55 years) is the most common

form of drug-induced chronic renal failure. Papillary necrosis and chronic tubulointestitial

nephritis are the most common pathologies seen. Pts with chronic analgesic abuse are also

more likely to develop premature aging, atherosclerotic vascular disease, and urinary tract

cancer. Polyuria and sterile pyuria (WBC casts may also be seen) are early manifestations.

Microscopic hematuria and renal colic may occur following sloughing of renal papilla.

Hyertension, mild proteinuria, and impaired urinary concentration commonly occur as the

disease advances.

19. Anticholinergics (benztropine) are useful for Parkinsonism pts younger than 70 years with

disturbing tremors and minimal bradykinesia. Parkinsonism tremor is a resting tremor.

Anticholinergics should be discouraged in pts who are elderly, have dementia or do not have

any tremor. The side effects are similar to those of other anticholinergics. Amantadine is an

antiviral agent, and its mechanism of action is unknown. It has relatively lower efficacy, and

is used for pts with mild disease and no disability. It has the advantage of low toxicity, and

improves all the features of Parkinsonism. Its side effects include confusion, restlessness, skin

rash, depression, edema, nausea, anorexia and postural hypotension; however, these side

effects are uncommon with the usual dose. Selegiline is a selective monoamine oxidase B

inhibitor and neuroprotective agent that may slow down the progression of the disease. It has

a mile symptomatic action, and may be used as an adjunctive treatment in pts receiving L-

dopa. Clozapine is an atypical antipsychotic agent, and is used when pts with Parkinsonism

have hallucinations. Propranolol is used for the treatment of benign essential familial tremors.

Pts with this condiction may have a family history of tremors, and their tremors are usually

absent at rest and appear with emotional stress. * Recognize the different types of tremor and

know how to evaluate and treat them. This is a big time topic for the USMLE!!

20. Presence of hematuria in a pt with irritative or obstructive voiding should alert the physician

to the possibility of bladder cancer. A long history of analgesic use and cigarette smoking aer

risk factors for the development of bladder cancer. Pts with prostate cancer have induration of

their gland on rectal examination and PSA is greater than 4ng/ml in such pts.

21. Chalazion initially presents as a painful swelling that progresses to nodular rubbery lesion.

This is a chronic granulomatous condition that develops when a meibomian gland becomes

obstructed. Persistent or recurrent chalazion may be due to meibomian gland carcinoma

(sebaceous carcinoma). Additionally, basal cell carcinoma frequently presents as a solitary

nodule on the lid margin, and may initially be clinically difficult to distinguish from a

chalazions. Recurrent chalazion requires histopathologic examination to rule out malignancy.

22. In a pt presenting with sudden onset of severe shortness of breath with stridor and coarse

upper respiratory sounds, laryngeal edema is the most likely diagnosis and the pt should be

managed as such.

23. The sudden onset of a sharply-demarcated, erythematous, edematous, tender skin lesion with

raised borders in a febrile pt suggests erysipelas. The most frequently implicated organism is

group A beta-hemolytic streptococcus.

24. There is a lack of physical exam findings and laboratory values specific for osteoarthritis

(OA). Therefore, the diagnosis of OA is often made based upon the overall clinical impression

and supplemented by radiographic findings. There are six criteria used to establish the

diagnosis of OA in the setting of a painful knee: age >50, crepitus, bony enlargement, bony

46

tenderness, and lack of warmth/morning stiffness. If 3 or more criteria are met, the specificity

for OA is 69%.

25. Whipple’s disease can mimic many illnesses, such hyperthyroidism, connective tissue

diseases, alcoholism and AIDS. It should be suspected in all pts with fever of unknown origin,

generalized lymphadenopathy, arthralgias, weight loss, abdominal pain and diarrhea. The

diagnosis of Whipple’s disease can be readily made using upper GI endoscopy and PAS-

staining of the obtained small intestinal biopsies. The classical findings are PAS-positive

material in the lamina propria and villous strophy. In doubtful cases, electron microscopy may

be required to demonstrate the Whipple bacillus for confirmation of diagnosis.

26. High-grade fever and bony tenderness in an IV drug abuser is considered osteomyelitis until

proven otherwise. Empiric antibiotic therapy should be instituted immediately after blood

cultures are obtained. Plain x-ray film are the initial radiologic study in the evaluation of

possible osteomyelitis; however, x-ray are usually normal in the first 2-3 weeks. If the x-rays

are normal, a three phase technetium bone scan is the next test of choice, except in the

evaluation of the spine or foot where MRI is preferred.

27. Marijuana is the most commonly used illicit drug in the US and is involved in 10-50% of car

accidents. Marijuana use can be recognized based on a history of slowed reaction time,

impaired short term memory and increased appetite. In addition, look for conjunctival

injection and dry mouth, as well as hypertension and tachycardia.

Cocaine intoxication is characterized by elevated blood pressure and heart rate and feelings of

euphoria. Unlike marijuana, it causes increased alertness and does not produce increased appetite,

slowed reaction time or conjunctival injection. Stimulant withdrawal, on the other hand, is

associated with increased appetite.

28. Pts with Meniere’s disease typically present with a combination of vertigo, ear fullness,

tinnitus, and hearing loss. First line therapy consists of environmental and dietary

modifications, including maintenance of a low-salt diet. Medical therapy with diuretics,

antihistamines, or anticholinergics is usually considered if lifestyle modifications are

unsuccessful.

29. Iron poisoning disrupts basic processes, causing systemic manifestations including abdominal

pain, hematemesis, shock and metabolic acidosis. It commonly occurs in children of pregnant

women taking prenatal vitamins because children often confuse brightly colored ion pills for

candy (abdominal imaging shows radioopaque tablets in the stomach). Tx involves

deferoxamine, which binds ferric iron, allowing urinary excretion.

30. It is better to keep systolic pressure < 130 and diastolic <80 mmHg to slow end-organ damage

in pts with diabetes and chronic renal failure.

31. Radiation therapy is useful in managing bone pain in pts with prostate cancer who have

undergone orchiectomy. Flutamide combined with a LHRH agonist, prolongs the survival of

pts with limited disease. However, there have been no demonstrable benefits in pts who

underwent orchiectomy.

32. Aortic dissection causes chest pain that is classically described as sudden, tearing, and

radiating to the back. Hypertension is the most common predisposing factor. The decrescendo

diastolic murmur heard here suggests aortic regurgitation, which can be a consequence of

aortic dissection involving the aortic root. Marfan’s syndrome or Ehlers-Danlos syndrome is

not the most common cause of aortic dissection and would tend to present in younger pts.

47

33. Chlamydial urethritis is suggested by mucopurulent urethral discharge, absent bacteriuria

(urine culture shows less than 100 colonies/ml), and history of multiple sexual partners.

34. Renal vein thrombosis is an important complication of nephritic syndrome, which is most

commonly caused by membranous glomerulonephritis in adults.

35. Breast fibrocystic disease is treated with aspiration of the cyst, which should yield clear fluid

and results in the disappearance of the mass. Afterwards, pts are typically observed for 4-6

weeks. Cytology is warranted if the aspirated fluid is bloody or foul smelling.

36. Anticoagulation with warfarin is the most effective way to decrease the risk of embolic

phenomena in pts with atrial fibrillation. Emboli can cause stroke and limb ischemia. Aspirin

and clopidogrel are less effective than warfarin in decreasing embolic complications in pts

with artial fibrillation.

37. Dihydropyridine calcium channel blockers such as nifedipine can worsen cardiac ischemia

since they cause peripheral vasodilation and reflex tachycardia; as such they are C/I in pts

with acute coronary syndrome.

38. When a seizure has been ongoing for more than 30 min (status epilepticus) and has failed to

respond to the usual anti-seizure medications (iv lorazepam, phenytoin, glucose), general

anesthesia and intubation are the next steps. A neuromascular blocker will greatly aid the pt,

an may also be included in his Mx. Work-up is indicated once the seizure is controlled.

39. Central or secondary hypogonadism (uncontrolled diabetes) in males is characterized by low

levels of testosterone with low or inappropriately normal serum gonagotropin levels.

Measurement of serum prolactin levels is the most important biochemical test to perform in

pts with suspected central hypogonadism. Regardless of the cause, high serum prolactin levels

inhibit the release of GnRH, thereby resulting in hypogonadism. Prolactin-secreting pituitary

tumor is one of the most important causes of elevated prolactin levels. Primary (testicular)

hypogonadism is characterized by elevated serum gonadotrophin levels.

40. Plasma aldosterone to plasma renin activity (PA:PRA) is used as an initial screening test for

primary hyperaldosteronism; a ratio over 30 is suggestive of the diagnosis. Non-

suppressibility of aldosterone levels after iv saline or oral salt loading is used to confirm

autonomous production of aldosterone in primary hyperaldosteronism. Once primary

huperaldosteronism is confirmed, adrenal CT is performed to look for adrenal adenoma.

41. Neuroleptic Malignant Syndrome (NMS) is caused by the initiation of dopaminergic

antagonists and typically presents with fever, muscle rigidity (lead pipe rigidity), autonomic

instability and mental status change. An elevated creatine kinase, leukocytosis and electrolyte

abnormalities are also common.

42. Chronic bronchitis is defined as a chronic productive cough that has been present for at least

three months in each of the past 2 years without an alternative diagnosis. Productive cough

and exertional dyspnea are the most common complaints, and symptoms can worsen with

increased mucous buildup in the airways. Therefore, the dx should be considered in pts who

report waking up around the same time each night with dyspnea that resolves with (yellowish)

sputum production. Pts with left ventricle failure often become dyspneic at night, but this is a

result of lying flat (orthopnea) as opposed to a lack of sputum clearance. Pts with pulmonary

edema would also have pink frothy sputum.

43. The pancreatic pseudocyst is an encapsulated area comprised of enzyme-rich fluid, tissue, and

debris that accumulates within the pancreas and causes an inflammatory response. A

48

complication of both acute and chronic pancreatitis, the pseudocyst is best diagnosed by

ultrasound and tends to resolve spontaneously. Hemorrhagic pancreatitis is a sever, life-

threatening illness characterized by retroperitoneal hemorrhage and pancreatic necrosis.

Physical exam findings may include a bluish discoloration of the flanks (Grey Turner sign) or

periumbilical region (Cullen sign) resulting from blood accumulation in abdominal fascial

planes.

44. Symptoms of giant cell arteritis include headache, jaw claudication, muscle fatigue, and visual

disturbance. On exam, pts have scalp tenderness and decreased temporal artery pulse. ESR is

generally more than 50mm/hr. Aortic aneurysms are well-known complications of giant cell

or temporal arteritis due to the involvement of the branches of the aorta. For this reason, pts

should be followed with serial CXR.

12/25/09

1. the stepwise approach of the tx for ascites: 1) sodium and water restriction. 2) spironolactone.

3) Loop diuretic (not more than 1L/day of diuretics due to the risk of hepato-renal syndrome).

4) frequent abdominal paracentesis (2-4 L/day, renal function should be frequently monitored.

Less aggressive paracentesis is recommended in pts with borderline renal function.

Vascular shunts are generally indicated for symptomatic varices (ie. After first bleeding). Distal

spleno-renal shunting will not improve the ascties, and may actually worsen it. Side-to-side portal-

caval shunt will improve the ascite, but may worsen the encephalopathy. Peritoneo-jugular shunt

is designed for the tx of ascites only. Peritonitis, sepsis, and disseminated intravascular

coagulation are its major drawbacks.

2. MEN 2a is a syndrome of pheochromocytoma, medullary carcinoma of thyroid and

hyperparathyroidism. 1) ↑ serum calcium 2) ↑ calcitonin 3) ↓serum phosphorous 4) ↑ urine

metanephrine 5) ↑ serum alkaline phosphatase 6) ↑ urine catecholamines 7) MRI of abdomen

showing adrenal mass 8) thyroid biopsy showing medullary carcinoma cell 9) adrenal biopsy

showing pheochromocytoma 10) parathyroid biopsy showing tumor or hyperplasia 11)

Radioimmune assay of parathyroid hormone showing increased level

3. Porcelain gallbladder results from the intramural deposition of calcium salts and usually

diagnosed on abdominal rediograph. The condition is associated with an increase risk of

gallbladder carcinoma and requires surgical resetion.

4. Diabetics have a high risk for erectile dysfunction—vascular complications, neuropathy and

medications. The first-line drug of tx are the phosphodiesterase inhibitors (sildenafil).

Remember to consider: 1) Sildenadil is C/I in pts being treated with nitrates, and in those who

are hypersensitive to sildenafil. 2) Sildenafil is used with precaution in conditions

predisposing to priapism. 3) concurrent us of drugs which interfere with metabolism of

sildenafil (eg. erythromycin, cimetidine) may predispose to adverse reactions by prolonging

its plasma half life. 4) While combining with an alpha-blocker (tx for BPH), it is important to

give the drugs with at least 4-hour interval to reduce the risk of hypotension.

5. The patient’s symptoms (ie. Spontaneous pain, odynophagia for cold and hot food) are

suggestive of diffuse esophageal spasm. Resolution of her chest pain after taking nitroglycerin

is also consistent with the diagnosis, because nitrates (and Ca channels blockers) relax not

only the myocytes in the vessels, but also all others, including those in esophagus, thereby

alleviating the pain. Esophagography may or may not show other anomalies (eg. corkscrew

49

shape). Esophageal manometry should reveal repetitive, non-peristaltic, high amplitude

contractions, either spontaneously or after ergonovine stimulation.

6. Sporotrichosis is the gardener’s disease. The initial lesion, reddish nodule that later ulcerates,

appear at the site of the thorn prick or other skin injury. From the site of inoculation, the

fungus spreads along the lymphatics forming subcutaneous nodules and ulcers.

7. Contrast administration has the potential to cause contrast-induced nephropathy, particularly

in pts with renal insufficiency (Cr>1.5) and/or diabetes. Non-ionic contrast agents are

associated with lower incidence of nephropathy than the older ionic hyperosmolar agents. In

addition to using non-ionic contrast agents, adequate IV hydration and acetylcysteine can

decrease the incidence of nephropathy. Predinison can be used to prevent hypersensitivity

reactions to contrast media in pts with known dye allergies. Prednisone does not prevent

contrast-induced nephropathy, and in pts with renal insufficiency, prednisone can actually

cause fluid retention. Prednisone can also cause elevations in blood pressure. NSAIDS should

be withheld before the procedure, as they can cause renal vasoconstriction. Statins are known

to improve endothelial function and reduce oxidative stress and may therefore have some

benefit in preventing contrast nephropathy.

8. Fasting blood glucose measurement is now the recommended screening test for DM. A fasting

blood glucose level of 126mg/dL on two separate occasions is diagnositic of DM.

9. Pts with dressler’s syndrome present weeks after a myocardial infarction with chest pain that

is improved by leaning forward. NSAIDs are the tx of choice. Anticoagulation should be

avoided to prevent development of a hemorrhagic pericardial effusion.

10. Recurrent pneumonia in the same anatomic location is a red flag for lung cancer. Furthermore,

smoking is the primary risk factor for bronchogenic carcinoma. When lung cancer is

suspected, CT scan of the chest is warranted. Bronchoscopic (pericronchial) or CT-guided

(peripheral) biopsy may then be performed depending on whether the lesion had a

pericronchial or peripheral location. Bronchoscopy is an invasive procedure that should be

reserved for pts with masses documented by CT scan.

11. Secondary (AA) amyloidosis is frequently the result of chronic systemic inflammation as may

occur in autoummune disorders, chronic infections, IV drug abuse and various other dieases

(psoriasis, IBD, RA). Amyloidosis is a systemic disease, affecting the kidneys (nephritic

syndrome), GI tract (hepatomegaly), heart (cardiomyopathy), musculoskeletal system

(pseudohypertrophy) and nervous system (peripheral neuropathy).

12. Chronic supraphysiological doses of glucocorticoids suppress CRH release from the

hypothalamus, thereby causing adrenal insufficiency. ACTH acts on the adrenal cortex, and is

responsible for the secretion of cortisol and androgen from the zona fasciculate and zona

reticularis. Aldosterone secretion is relatively preserved. Laboratory studies typically show

low ACTH and low cortisol levels.

13. Radioactive iodine therapy is the preferred tx of most patients with Grave’s disease. The C/I

of radioactive iodine tx are pregnancy and very severe ophthalmopathy.

14. Toxic shock syndrome (TSS) is a potential fetal condition caused by toxins produced by

specific strains of Staph, including epidermal exfoliating toxin. The condition is related to

females using tampons and other intravaginal articles. Symptoms include a sudden onset of a

flu-like syndrome, high fever, hypotension, erythematous rash, and signs of systemic failure.

15. TCA overdose is characterized by CNS depression, hypotension, hyperthermia, and

50

anticholinergic effects including dilated pupils, dry flushed skin, and intestinal ileus. It also

causes QRS prolongation on EKG, leaving the patients susceptible to ventricular arrhythmias.

By obtaining an EKG and evaluating the QRS complex, the physician can obtain valuable

prognostic nformation,; namely the patient’s likelihood of developing arrhythmia and seizure.

Sodium bicarbonate shortens the QRS interval in TCA overdose, decreasing the likelihood of

arrhythmia.

16. The tx of choice for PCP is trimethoprim-sulfamethoxazole, and the alternative drug is

pentamidine. Pentamidine has a number of side effects, with 50% of pts developing

hypoglycemia, hyperglycemia, hypocalcemia, azotemia, or liver dysfunction.

17. Several medications can cause hyperkalemia. Examples of these are: ACEIs, NSAIDs, and

potassium-sparing diuretics, such as spironolatone and amiloride. Immediate tx is indicated if

there is cardiac toxicity (as evidenced by EKG changes), muscle paralysis, or K+ >6.5. 10ml

10% calcium gluconate-insulin and /or B2 agonists-Sodium bicarbonate-loop and thiazide

diurectics-cation exchange. Dialysis should be reserved for pts with renal failure and those

with severe life-threatening hyperkalemia that is unresponsive to the more conservative

measures. In pt is symptomatic and with normal EKG findings (ie. Without indications for

immediate drug therapy), may be treated less aggressively by simply stopping usage of

causative drug (ie. amiloride) and rechecking the blood levels after one week.

Pseudohyperkalemia should always be considered in a symptomatic pts, as occasionally the

lab sample can become hemolysed during venipunctue.

18. Hemarthroses are the most common manifestation of hemophilia. The tx of hemophilia

consists of replacing the missing coagulation factor.

19. Guillain-Barre syndrome (GBS) can lead to respiratory muscle weakness and may

occasionally result in respiratory failure. The best way to monitor respiratory function in such

cases is through serial measurements of bedside vital capacity. * Extremely important

question for USMLE!!

20. Pts with chronic liver disease should receive a number of vaccinations. In addition to being

up-to-date with childhood immunizations, they should be vaccinated against HAV, HBV,

influenza (intramuscular inactivated influenza vaccine is preferred over the live attenuated

intranasal vaccine), and invasive pneumococcal disease. Like all adults, they should get a Td

booster at least every 10 years.

21. Neurofibromatosis—decreased hearing and several subcutaneous nodules, hyperpigmented

spots on the back. Chromosome 22. Nonsense and frameshift mutations are typically more

severe that missense mutations. Silent (same sense) mutation do not affect the structure of the

protein.

22. Acute-onset epigastric pain that radiates to the back and is accompanied by nausea, vomiting,

and tenderness is strongly suggestive of acute pancreastitis. The hypotension that can

complicate acute pancreatitis is thought to arise from intravascular volume loss secondary to

local and systemic vascular endothelial injury, which increases vascular permeability and

transudation of plasma. Systemic vasodilation may also contribute.

23. Increase serum calcium and increased PTH levels (8-24 pg/ml) are suspicious for either

primary hyperparathyroidism or familial hypocalciuric hypercalcemia (FHH). Primary

hyperparathyroidism presents with normal or elevated urinary calcium excretion, while FHH

presents with decreased calcium excretion. Pts with VitD intoxication, multiple myeloma,

51

renal cell carcinoma and sarcoidosis may also present with hypercalcemia, but their PTH

levels are typically low due to feedback inhibition.

24. Metabolic acidosis observed during diabetic ketoacidosis is typically accompanied by

hyperkalemia; this is sometimes called paradoxical hyperkalemia because the body potassium

reserves are actually depleted. All hyperkalemic pts should start receiving potassium once the

serum potassium level goes below 4.5mEq/L. In pts with normal or low potassium levels,

potassium replacement should be started with initiation of iv fluid therapy. Extremely high

yield!!

25. Hyperglycemic, hyperosmolar non-ketotic coma is characterized by very high blood glucose

level, plasma hyperosmolality, normal anion gap and negative serum ketones. Serum

osmolarity can be calcultated using the following formula: 1(serum Na) + blood glucose/18 +

BUN/2.8.

26. The most likely diagnosis in a patient presenting with metabolic acidosis, polyuria,

dehydration, decreased level of consciousness, and diffuse abdominal pain in the setting of an

ongoing infection is diabetic ketoacidosis (DKA). DKA is characterized by an osmotic

diuresis that reduces total body potassium stores, even though the serum potassium level may

be elevated.

27. Ischemic hepatic injury occurs in the setting of hypotension and manifests as acute, massive

increases in the AST and ALT with milder associated increases in the total bilirubin and

alkaline phosphatase. In pts who survive the inciting condition, liver enzymes typically return

to normal within a few weeks. HAV or HBV can present with large AST and ALT elevations,

typically accompanied by significant huperbilirubinemia, nausea, and vomiting.

28. The above patient is most likely suffering from MS. Her history is consistent with patchy

neurological problems (eg. right upper extremity, left lower extremity and optic neuritis),

whick is characteristic for MS. Optic neuritis presents as a painful loss of vision, and is an

important presentation. Patients usually have a central visual field defect, and fundoscopy is

normal. Sensory abnormalities may also occur. MRI is the test of choice to support the clinical

diagnosis of MS.

29. Suspect DKA in stuporous pts with rapid breathing and a history of weight loss, polydipsia

and polyuria.

30. Nucleoside reverse transcriptase inhibitors (particularly zidovudine) may cause lactic acidosis

(low pH, primary decrease in HCO3-, compensartory decrease in PaCO2, AG=26) and

steatosis (elevated transaminases and could be further proven by imaging or liver biopsy) due

to inhitibition of DNA polymerase gamma and depletion of mitochondrial DNA (reduced

oxygen utilization by tissues)—acidosis-steatosis syndrome (LASS).

31. Multiple myeloma is a plasma cell disorder which often presents with lytic lesions,

hypercalcemia and renal failure. Marrow plasmacytosis.

32. The goal of tx of pseudotumor cerebri (idiopathic benign intracranial hypertension) include

prevention of visual loss, reduction of intracranial pressure and symptomatic relief. For early

detection of visual loss, there must be routine monitoring of visual acuity, visual fields and

fundoscopy. Intracranial pressure can be reduced by medical or surgical means. Medical

measures are employed firsht and if they fail, surgery is the next option. Surgical options are

optic nerve sheath decompression and lumboperitoneal shunting. Medical tx includes tx of

underlying disorder, pharmacotherapy with acetazolamide (first line, inhibits choroid plexus

52

carbonic anhydrase and thus reduces CSF production and intracranial hypertension) or

corticosteroids and repeated lumbar punctures.

33. The symptoms of BPH are weak urinary stream, urgency, frequency, and sensation of

incompletes voiding. Unlike prostate cancer, BPH starts in the center of the prostate.

Placement of a Foley catheter is the most appropriate initial step in pts who present with acute

renal failure. The obstruction should be reversed within the first 2 weeks to prevent permanent

kidney damage.

34. Decrease in the synthesis of non-cholesterol products may be responsible for some adverse

effects of statin therapy, for example, reduced CoQ10 production is implicated in the

pathogenesis of statin-induced myopathy.

35. in VF and pulseless VT, defibrillation is of primary importance. Time to defibrillation is

strongly correlated with survival. Once defibrillation has been attempted, epinephrine should

be given. Epinephrine administration is then repeated every 3 minutes while CPR is onging.

After a repeated attempt at defibrillation, use of antiarrhythmics (amiodarone, lidocaine, and

magnesium) is warranted.

36. Infection with HEV had a high rate of progression to fulminant hepatitis in pregnant women,

especially in the 3rd trimester.

37. HIV-infected pts should follow the same immunization regimen for teranus and diphtheria

(Td) as the general population. This means they should receive a Td booster at least every 10

years. Also influenza, S.pneumoniae, HAV and HBV.

38. Pts with DVT in whom anticoagulation is C/I (those with recent surgery, hemorrhagic stroke,

bleeding diathesis, or active bleeding) require placement of an inferior vena cava filter to

prevent PE. Compression stocking decrease venous stasis in the lower extremities and are

effective in reducing postoperative venous thrombosis in low risk surgical patients. While

they may be useful in preventing DVTs, compression stockings offer no benefit in the tx of

existing DVTs.

39. Folic acid supplement is encouraged in pts with high risk for developing aplastic crisis

(spherocytosis, AD). Splenectomy is usually indicated for pts who have moderate to severe

disease in order to prevent the complications of the disorder such as aplastic crisis,

cholelithiasis and chronic leg ulcers. Usually splenectomy should be delayed four to five years

to prevent post splenectomy sepsis. All pts prior to splenectomy should receive vaccinations

against encapsulated organisms such as Harmophilus, pneumococci, and meningococci.

40. Recognize the cutaneous lesions of Kaposi sarcoma. In HIV pts it’s caused by Human

herpesvirus 8. Asymptomatic, elliptical, arranged linearly in legs, face, oral cavity, genitalia,

light brown to violet.

41. Laboratory test used in the evaluation of liver disease either assesss liver functionality (eg.

prothombin time, bilirubin, albumin, cholesterol) or structural integrity and cellular intactness

(eg. transaminases, gamma glutaryl transferase, alkaline phosphatase). A progressive decrease

in transaminase levels signals either recovery from liver injury or that few hepatocytes are

functional.

42. Routine urine cultures are not indicated in wormen with incomplicated cystitis. Oral

trimethoprim-sulfamethoxazole is the preferred empiric tx. Ciprofloxacin or nitrofurantoin is

not a first-line empiric therapy, except when there is resistance or an allergy to trimethoprim-

sulfamethoxazole.

53

43. Focal neurologic deficits in a young adult with asymmetric periventricular white matter signal

changes on MRI are most likely due to MS. Glatiramer acetate is a long-term disease

modifying tx for MS that works by modulating T-cell-mediated autoimmunity to myelin basic

protein. Argatroban is an iv anticoagulant that selectively and reversibly inhibits thrombin. Its

main use is to prevent thrombosis in pts with heparin-induced thrombocytopenia who require

anticoagulation.

44. Cancer pts with new or progressive back pain should have detailed neurological examination

for signs of myelopathy. Mx 1) iv steroids (dexamethasone usually) 2) MRI of spine, if MRI

spine is not available or C/I (pts with pacemaker), CT myelogram should be done. 3) Once the

diagnosis is confirmed—radiotherapy. Extremely high yield!!

12/25/09

1. Pts with streptococcal pharyngitis often have a fever, exudative pharyntitis, and tender

cervical lymphadenopathy in the absence of a rash or rhinorrhea. A rapid streptococcal antigen

test can confirm the diagnosis. Tx with penicillin is recommended to hasten recovery, reduce

the risk of rheumatic fever and prevent transmission to close contacts.

2. Paroxysmal nocturnal hemoglobinuria (PNH) is an acquired disorder of hematopoietic cells. It

is characterized by an intravascular hemolytic anemia, a hypercoagulable state, and bone

marrow aplasia. PNH should be considered in the following situations: 1) pancytopenia

(hypocellular bone marrow) accompanied by hemolytic anemia (increased reticulocyte count

and LDH and low haptoglobin levels). Loss of iron in the urine may result in iron deficiency

anemia. 2) Recurrent thrombosis at unusual sites eg. portal vein thrombosis or Budd Chiari

syndrome. Sucrose lysis test and Ham acid hemolysis test were used in the past as screening

and confirmatory tests, respectively. Currently flow cytometry had replaced—CD55 and

CD59 can be analyzed using monoclonal antibodies.

Aplastic anemia is characterized by a hypocellular bone marrow. The peripheral blood smear does

not show any abnormal morphology of cells. RBCs are normocytic or macrocytic. There is neither

hemolyis of RBCs nor splenomegaly. Reticulocyte count will be very low.

3. This patient’s syphilis infection suggests that he may be involved in high-risk sexual activity,

also putting him at risk for HIV exposure. After proper counseling, HIV screening using

ELISA should be offered.

4. If a test result is negative, the probability of having the disease is 1-negative predictive value.

Other cases and diagnostic tests which are high-yield for USMLE are 1) pulmonary embolism

and perfusion-ventilation scanning; 2) prostate cancer and serum PSA level

5. The usual cause of renal artery stenosis in young adults is fibromuscular dysplasia. In older

pts, the cause is usually an atheromatous plaque. The tx of choice for fibromuscular dysplasia

is percutaneous angioplasty with stent placement. * Extremely high yield!!

6. Salmonella is the MCC of osteomyelitis in pts with sickle cell disease (responsible for up to

70% of cases); Staphylococcus aureus accounts for less than 25% of cases. * whether

Salmonella or S. aureus is the MCC in sickle cell pts had been a source of debate; however,

meta-anlysis of the available studies supported that the majority of cases are due to

Salmonella.

7. The cause of vitB12 deficiency include a strict vegetarian diet (after 3-4 years) and pernicious

anemia. Anti-TB drugs such as INH can cause B6 or pyridoxine deficiency.

54

8. Sodium bicarbonate narrows the QRS complex, preventing the development of arrhythmia in

pts with TCA toxicity by alleviating the cardio-depressant action on sodium channels. Sodium

bicarbonate is a critical part of tx in TCA intoxication. Urine alkalinization is a mechanism by

which Sodium bicarbonate increases salicylate excretion and thus, alleviates aspirin overdose.

9. Pts with end stage renal disease have only two tx options: dialysis or renal transplantation.

Renal transplantation is always preferred due to better survival rates and quality of life. A

living related donor is always preferred, followed by a living non-related donor, and lastly, a

cadaveric donor.

10. Most colon cancers develop from polyps. The risk factors for a polyp progressing into

malignancy are villous adenoma, seeeile adenoma, and size >2.5cm. Only adenomatous

polyps are clearly premalignant, but <1% of such lesions progress to malignancy.

Hyperplastic polyps are non-neoplastic and do not require further work-up. Extremely high

yield!!

11. Whenever a healthcare worker is exposed to HIV, baseline HIV testing should be performed

immediately. Postexposure prophylaxis with two or three antiretroviral drugs should be started

without delay.

12. COPD causes airflow limitation and increase lung volumes. The latter results in

diaphragmatic flattening, evident on CXR, which further exacerbates pts’s dypnea and work

of breathing.

13. A tea and toast type of diet is assocated with folic acid deficiency. Folic acid is heat sensitive.

Folic acid deficiency causes macrocytic anemia.

14. Premature atrial beats are benign and neither requires any follow up nor treatment.

15. MI is one of the differential diagnoses of acute abdominal pain and should be ruled out in pts

with risk factors.

16. There are 3 methods to control confounders in the design stage of a study: matching,

restriction and randomization (clinical trial, control known risk factors, as well as unknown

and difficult-to-meaure confounders). The methods to control confounding in the analysis

stage of a study are stratification and modeling.

Selection bias can be controlled by choosing a representative sample of the population for the

study and achieving high rates of follow-up.

Observer’s bias is controlled by the blinding technique.

Ascertainment bias can be avoided by selecting a strict protocol of case ascertainment.

17. The measurement of serum BNP can help distinguish between CHF and other causes of

dyspnea. A value >100pg/mL diagnoses CHF with a sensitivity, specificity, and predictive

accuracy of 90, 76, and 83 percent, respectively.

18. MCC of anionic gap metabolic acidosis are: 1) Lactic acidosis: hypoxia, poor tissue perfusion,

mitochondrial dysfunction 2) ketoacidosis: Type I DM, starvation or alcoholism 3) Methanol

ingestion: Formic acid accumulation 4) Ethylene glycol ingestion: Glycolic and oxalix acid

accumulation 5) Salicylate poisoning: Causes concomitant respiratory alkalosis 6) Uremia

(ESRD): Failure to excrete H+ as NH4+

19. In anion gap acidosis, the anion gap is increased by the abnormal presence of non-chlorinated

acids in the serum.

20. 3 key principles to the use of pain medications in pts with terminal stages of cancer are as

follows: 1) try non-narcotic measures first, unless you are sure (using your clinical judgement)

55

that the pt is in severe pain. 2) Do not be afraid to give narcotic analgesics. 3) Prescribe

adequate amounts of medication. Short-acting morphine → long-acting

21. Bechet’s syndrome is a multi-systemic inflammatory condition characterized by recurrent oral

and genital ulcers, skin lesions; seen most commonly in the Turkish, Asian and Middle

Eastern population. Some pts have involvemen of the GI, skeletal and vascular systems.

Corticosteroids offer relief from most of the symptoms but do not protect from progression to

dementia or blindness.

22. The proxima deep leg veins (illiofemoral vein) are the MCC of symptomatic pulmonary

embolism. Other less common sources of emboli include the calf veins, renal veins, pelvic

veins, upper extremityh veins, and right heart.

23. Propranolol is the drug of choice for benign essential tremor (head and upper expremities are

affected, legs are usually spared. AD, ↑ at the very end of goal-directed activities).

24. DD for thrombocytopenia: Bone marrow disorders—aplastic anemia, hematologic

malignancies, megaloblastic anemia, alcoholism, myelodysplasia. Non BM—ITP, heparin-

induced thrombocytopenia, hupersplenism, DIC, sepsis, viral infection, HIV, SLE, TTp, and

HUS. Thrombocytopenia is the initial presentation of HIV infection in up to 10% of pts. An

HIV antibody test is the first step in diagnosing this disease.

25. Lactulose and a reduced-protein diet are recommended for the tx of acute hepatic

encephalopathy. Non responder—ornithine-aspartate infusion or oral sodium benzoate; A

further alternative—neomycin (long-term use →ototoxicity and nephrotoxicity).

26. In the mx of pts with hyperthyroidism, propanolol is generally used for symptomatic relief

until the underlying cause is identify and definitively treated. *Extremely important!!

27. Factitous thyrotoxicosis results form ingestion of exogenous thyroid hormone. Pts present

with signs and symptoms of hyperthyroidism, but not goiter or exophthalmos. Thyroid

function tests demonstrate low TSH and elevated T3 and T4. The diagnosis many be

confirmed by a 24-hour radioiodine uotake test showing diffusely decreased iodine uptatke by

the thyroid. Biopsy of the thyroid demonstrates follicular atrophy.

28. Vertigo is a sensation of exceeeive motion compared to physical reality. It is most commonly

due to dysfunction within the vestibular system. Disease of the posterior column of the spinal

cord can occur in syphilis (tabes dorsalis) and vit B12 deficiency. Typical symptoms include

ataxia, devreased proprioception and vibratory sense, and hyporeflexia. Cerebellar

dysfunction presents as ataxia, imbalance, incoordination, and nystagmus. Pts with cerebellar

dysfunction will be unable to perform tandem walking due to their imbalance.

29. Suspect aspirin intoxication in a pt with the triad of fever, tinnitus and tachypnea. Adults with

aspirin toxicity develop a mixed respiratory alkalosis and anion gap metabolic acidosis—

characterized by a near-normal pH (7.36), a primary decrase in PaCO2 , and a concurrent

primary decrease in HCO3-.

30. Beta-blockers can worsen severe peripheral vascular disease by causing beta2-mediated

vasoconstriction of the peripheral arteries.

31. Suspect secondary pneumothorax (dilated apical airspaces) in COPD pts presenting with

catastrophic worsening of there respiratory symptoms.

32. Admimistration of DDAVP after water depression can distinguish between central and

nephrogenic diabetic indipidus. Pts with central DI will have an increase of urine osmolality

by at least 50% following the administration of AVP or DDAVP, whereas pts with

56

nephrogenic DI will not have such an increase. The tx of choice for central DI is

desmopressin, which is usually administered intranasally.

33. The absence of forehead furrows indicates peripheral facial nerve palsy (Bell’s palsy) and

excludes central causes.

34. Congenital cataract is the MCC of while reflex in the pediatric population (more than

retinoblastoma).

35. Pts with generalized resistance to thyroid hormones (receptor defects on the peripheral

tissues) have high serum T4 and T3 levels with normal to mildly elevated TSH levels. Pts

typically have features of hypothyroidism despite having elevated free thyroid hormones. Pts

usually present at an early age with growth and mental retardation, however, pts with milder

defects can present later in life.

36. Approximately 10-20% of pts on isoniazid will develop mild aminotransferse elevation within

first few weeks of treatment. This hepatic injury is typically self-limited and will resolve

without intervention. INH has beeb associated with severe INH hepatitis and a much milder

hepatotoxicity. INH hepatitis similar to those seen with viral hepatitis→discountinue the drug

immediately. Most of the first –line antiTB drugs are hepatotoxic and should be replaced by

second-line drugs in pts who develop significant hepatotoxicity with the initial regimen.

37. Lewy body dementia is characterized by flunctuating cognitive impairment, recurrent visual

hallucinations and motor features of Parkinsonism. Pick’s disease is a fronto-temporal

dementia. The typical features include personality change, (euphoria, disinhibition, apathy),

compulsive behaviors (peculiar eating habits, huperorality), and impaired memory. Visual-

spatial functions usually remain intact.

38. A pt with familiar colonic polyposis has a 100% risk of cancer, if not treated appropriately

with a proctocolectomy at the time of diagnosis.

39. Antihypertensive mx should be the first step in pts with aortic dissection with hypertension.

40. Dopamine-agonists such as bromocriptine or cabergoine (new drug with fewer side effects,

more effective) are the mainstay of tx for most pts with prolactinoma

(microadenoma<10mm).

41. GERD is present in up to 75% of asthma pts, and may be the primary trigger in many. Pts

with adult-onset asthma and symptoms that are worse after meals, exercise, or laying down

are likely to have GERD-induced disease. Where GERD is suspected, a trial of PPI can be

both diagnostic and therapeutic.

42. Decrease free water intake will cause hupernatremia due to obligate renal free water losses

and sodium reabsorption by the kidney. This is particularly common in elderly demented pts.

43. Senroineural hearing loss that occurs with aging is referred to as presbycusis. It is usually first

noticed in the sixth decade of life, and presents with high-frequency, bilateral hearing loss. Pts

with presbycusis often have difficulty hearing in noisy, crowded environments. Otosclerosis is

a type of chronic conductive hearing loss associated with bony overgrowth of the stapes. It

typically begins with low-frequency hearing loss and is often found in middle-aged

individuals.

44. Electromyography and conduction studies are the best diagnositic tests for polyneuropathy.

Repetitive stimulation electromyography can be used for the diagnosis for myasthenia gravis.

Electroencephalography detects brain electrical activity. Evoked potentials are very small

electrical impulses monitored from the scalp ather a motor or sensory peripheral, visual or

57

hearing impulse.

12/26/09

1. It is essential to distinguish between a seizure and a syncopal event. A seizure resulting in loss

of consciousness will often be followed by a post-ictal state of clouded sensorium, whereas

people rapidly return to their baseline mental status after a syncopla event.

2. The MCC of folic acid deficiency is nutritional due to poor diet and/or alcoholism. Folic acid

deficiency can be caused by some drugs, these drugs can impair the absorption of folic acid

(eg, phenytoin, primidone and penobarbital) or antagonize its physiologic effects (eg.

methotrexate, trimethoprim). Extremely high yield!!

3. Cauda equina syndrome (CES) is a seious neurologic disorder of the spinal nerve root. CES

can result from any disorder that comprsses the nerves of the cauda equine, include trauma,

lumbar disk disease, malignancy and abscesses. The cauda quina is particularly sensitive to

damage because the nerves have a poorly developed epineurium. Symptoms of CES include

low back pain, bowl and bladder dysfunction, saddle anesthesia, sciatica, and low extremity

sensory and motor loss. Exam will reveal perineal anesthesia, poor rectal tone, low extremity

muscle weakness, reflex abnormalities, and and elevated post-void residual volume. An

emergent MRI of the spine must be obtained when entertaining a diagnosis of CES.

Peripheral nerve disease presents with lower motor neuron symptoms (weakness, hyporeflexia,

decreased muscle tone, fasciculations) and/or decreased sensation. The most common form of

peripheral neuropathy are due to diabetes and alcohol. Bowl and bladder dysfunction, saddle

anesthesia, and back pain suggest that this is CES and not a generalized peripheral neuropathy.

The presence of upper motor neuron findings (such as hyperreflex, hypertonia, spasticity, and

positive Babinski sign) would indicate that the lesion was in the brain or spinal cord. CES

involves spinal nerve roots, which carry lower rather that upper motor neuron fibers.

4. Transfusion reactions remain relatively common and canbe life-treatening. An acute

hemolytic transfusion reaction is a life-threatening reaction resulting form transfusion of

mismatch blood, typically ABO mismatch. Patients rapidly develop fever, hemolysis, shock

and DIC (bleeding around the iv catheta site).

Reaction to cytokines stored in the transfused blood products is thought to be the leading cause of

febrile nonhemolytic reaction, the most common transfusion reaction. During blood storage,

leukocytes release cytokines, whick when transfused cause transient fevers, chills and malaise.

This reaction can largely be prevented by using leukoreduced blood products, particularly if the

RBCs are washed.

IgA deficiency is a major risk factor for developing an anaphylactic reaction to blood products.

Anaphylaxis typically occurs within seconds to minutes of the infusion start time. It develops

because some IgA deficient pts hava anti-IgA IgG antibodies. It can be prevented by using IgA-

deficient blood products or performing extra washes of the blood product.

Despite rigorous screening standards, a vey small percentage of blood products are contaminatd

iwht infectious organisms. The presentation is variable but most pts develop fever within 24 hours.

Delayed hemolytic transfusion reactions result from an anamnestic antibody response to an RBC

antigen to which the pts has previously encountered. Exposure may occur via pregnancy,

transplant, or transfusion. The antibody is undetectable prior to transfusion but reappears rapidly

following transfusion. It typically causes low-grade hemolysis 2-10 days after the transfusion.

58

5. Prophylactic lidocaine decreases the frequency of VPBs, and diminishes the risk of ventricular

fibrillation, but the overall prognosis is unaffected. The problem is that lidocaine can increase

the risk of asystole in these pts. Lidocaine has little effect on myocardial contractility. So

Lidocaine is not used prophylactically in pts with acute coronary syndrome.

6. Lactose intolerance may result in chronic diarrhea, but fecal fat will not be high in such cause.

Bacterial overgrowth, pancreatic insufficiency, Whipple’s and Crohn’s disease all result in chronic

diarrhea with increase fecal fat content. D-xylose is a simple sugar. It does not need to undergo

any digestive process before it can be absorbed. Its absorption requires an intact mucosa only.

In normal individuals, after ingestion of 25 grams of D-xylose, its urinary excretion will be greater

than 4.5 gram in 5 hours. In pancreatic insufficiency, there is deficiency of pancreatic enzymes,

which lead to impaired digestion of fats. D-xylose absorption remains unimpaired, as there is no

damage to small intestinal mucosa.

In terminal ilea disease, bile salts are decrease due to impaired enterohepatic circulation, and as a

result absorption of fats will be impaired. In bacterial overgrowth there will be both impaired fat

absorption due to bacterial de-conjugation of bile as well as decreased excretion of xylose due to

bacterial consumption of the simple sugar. In such cases, urinary excretion of xylose returns to

normal after treatment with antibiotics, while Whipple’s disease does not.

7. Continued pressure on a bony preminence for more than 2 hours can result in ischemic

necrosis of overlying muscle, subcutaneous tissue and skin leading to formation of a pressue

(decubitus) ulcer.

8. When treating pts with pheochromocytoma, do not give beta-blockers without alpha-blockers

(unopposed). Always give an alpha-blocker first, followed by a beta-blocker; doing this in the

wrong order can precipitate a very dangerous increase in blood pressure.

9. Coarctation can present with rib notching on the CXR. A ‘3’ sign is typically seen with

coarctation of longer duration, implying proximal aortic dilation, constriction and descending

aorta dilatation.

10. Nearly 20% of cocaine oversodoses are complicated by rhabdomyolysis, as indicated by

marked elevations on serum creastine phosphokinease (CPK). The main danger associated

wht CPK levels greater than 20,000U/L is acute renal tubular necrosis due to myoglobinuria.

11. Airway disease is the MCC of both mild and massive hemoptysis. Chronic bronchitis is a

frequent cause of mild hemoptyiss in pts with significant smoking history. A low suspicion of

respiratory tract cancer must be maintained, so obtaining a chest X-ray in this pt would be

reasonable. Bronchiectasis is a well-known cause of hemoptysis but much less common than

chronic bronchitis.

12. In a pt wht increased reticulocyte count, increased bilirubin, negative Cooms test and

spherocytes on peripheral blood films (anisocytosis, some spherocytosis, and

polychromatophilia), the diagnosis of Hereditary Spherocytosis is top on the list. MCV is

normal or slightly decreased while MCH is normal but MCHC is generally greater than 36%.

The osmotic fragility test is the next best test for the dignosis of the pt.

13. Squamouse cell carcinoma of the mucosa of the head and neck is common in people with a

significant history of alcohol and tobacco use. The first manifestation may be a palpable

cervical lymph node. The best initial test is panendoscopy (triple endoscopy=esophagoscopy,

bronchoscopy, laryngoscopy) to detect the primary tumor.

14. In US, acute liver failure and fulminant hepatitis are most commonly due to acetaminophen

59

toxity. Remember that acute hepatic failure is defined as the development of liver failure

within 8 weeks of hepatocellular injury onset. If hepatic encephalopathy is also seen, then the

symptom is described as fulminant hepatic failure.

15. In clinical trials, randomization is said to be successful when a similarity of baseline

characteristics of the pts in the tx and placebo groups is seen.

16. The most common site of colon cancer metastasis is the liver. Liver metastases manifest as

right upper quadrant pain, mildly elevated liver enzyme, and firm hepatomegaly. The

diagnosis is confirmed by CT of the abdomen.

17. Sexually active women with a cevix may have Pap smears every 2-3 years if they have had 3

consecutive negative Pap smears and have the same monogamous partner. Screening may stop

at age 70.

18. No matter what the underlying disease of the pt is, remember that the mx of any pt arriving in

the ER begins with assessment of the pt’s “ABC’s”. Somatostatin or octreotide can be used

when endoscopy is unsuccessful, contraindicated, unavailable, or as an adjuncti before

endoscopy.

19. The diagnosis of Gilbert’s syndrome is suggested in those pts with no apparent liver disease

who have mild unconjugated huperbilirubinemia (a familial disorder of bilirubin

glucuronidation in which the production of UDP glucuronul transferases is reduced—enzymes

that mediate glucuronidation of various substances) thought to be provoked by one of the

classic triggers (hemilysis, fasting or consuming a fat-free diet, physical exertion, febrile

illness, stress, or fatigue). Presumptive diagnosis can be made when the unconjugated

huperbilirubinemia persists with repeat testing, but liver function test, complete blood count,

blood smear, and reticulocyte count are normal. Tx generally considered to be unnecessary in

pts with Gilbert’s syndrome. However, its mode of inheritance should be discussed with pts as

a means of preventing needless testing in similarly affected family members.

Crigler-Najjar syndrome type 1 is an autosomal recessive disorder of bilirubin metabolism

characterized by severe jaundice and neurologic impairment due to kernicterus (bilirubin

encephaopathy). In these infants, the indirect bilirubin levels are typically 20-25mg/dL but can rise

to as high as 50mg/dL. Liver enzymes and histology are normal. If iv phenobarbiral is

administered, the serum bilirubin remains unchanged. Phototheray or plasmapheresis are typically

helpful in the short term, though liver transplant is the only curative option.

Crigler-Najjar syndrome type 2 is an mild autosomal recessive disorder of bilirubin metabolism

characterized by low serum bilirubin levels ( < 20 mg/dL) and survival into adulthood with no

kernicterus or neurologic impairment. Liver enzymes and histology are normal. If iv phenobarbiral

is administered, the serum bilirubin is reduced. Tx is often unnecessary in milder cases, though

periodic administration of phenobarbital or clofibrate can reduce serum bilirubin levels if

necessary.

20. Dipsticks are commercially available kits that detect the presence of leukocyte esterase and

nitrie in the urine of pts with suspected UTI. Positive leukocyte esterase signifies significant

pyuria and positive nitrites indicate the presence of Enterobacteriaceae. However, dipsticks

are associated with a high false positive rate and high false negative rate. For this reason, a

negative dipstick test in a pt with symptoms of UTI should still have urine cultures done.

21. Cardiogenic pulmonary edema results from left heart failure and is characterized by crackles

on pulmonary exam. Nitroglycerin is the most rapid acting medication to relieve the

60

symptoms of pulmonary edema (quicker than morphine or loop diuretics). Beyond NTG, loop

diuretics are the mainstay of therapy for decompensated heart failure and principally work by

reducing total body volume.

22. Hypovolemic shock is characterized by hypotension, tachycardia, decreased central venous

pressure (or pulmonary capillary wedge pressure), decreased cardiac output, and increased

peripheral vascular resistance. *Extremely important!!

23. Dilated cardiomyopathy mayh be seen following viral myocarditis (postpartum), particularly

following Coxsackie B infection. Diagnosis is made by echocardiogram, which typically

shows dialated ventricles and diffuse hypokinesia resulting in systolic dysfunction (low

ejection fraction).

24. Thrombocytopenia and hypercoagulation within days of initiating anticoagulant therapy are

most likely caused by unfractionated heparin. Heparin-induced thrombocytopenia is often

associated wth abnormal intravascular thrombosis. The aPTT can become elevated due to

thrombin consumption. Parenteral enoxaparin and other LMWHs may be used in place of

unfractionated heparin in the tx of DVT. However, enoxaparin does not prolong the aPTT.

25. Differentiating preseptal cellulites and orbital cellulites is important in any patient who

presents with a swollen and painful eye accompanied by fever. Pain with eye movement is

more suggestive of orbital cellulites, and proptosis in addition to decreased visual acuity are

almost always seen in pts with orbital cellulites. CT can be used to confirm the extent of

infection and to identify a potential abscess. Cavernous sinus thrombosis (CST) is also

characterized by periorbital edema, exophthalmos, and chemosis, but fundoscopy typically

reveals papilledema and dilated tortuous retinal veins, Although it is often difficult to

differentiate CST from orbital cellulites, the symptoms of CST are often bilateral, and there is

often involvement of cranial nerves III (resulting in ptosis) and V as well as early visual loss.

26. Consider CMV infection in a pt with mononucleosis-like symptoms, typical lymphocytes on

the blood smear, and a negative monospot test. Unlike EBV-associated mononucleosis, sore

throat and lymphadenopathy are uncommon in CMV infection. Peripheral blood smear shows

large basophilic lymphocytes with a vacuolated appearance.

27. Suspect glucagonoma in a pt with huperglycemia, necrolytic migraroey erythema (an

erythematous, scaly plaque) and diarrhea. It is usually malignant, and metastasis to the liver

may occur. The excess glucagon causes symptoms such as glucose intolerance and

hyperglycemia. The diagnosis is confirmed by elevated levels of fasting glucose and serum

glucagons, as well as pancreatic tumor revealed by CT scan. The preferred treatment is

surgical removal of the tumor. The tumor does not respond to chemotherapy.

28. Herpes zoster (shingles) develops when there is reactivation of VZV. Pts experience pain and

a vesicular rash in a dermatomal distribution, The pain may precede the rash by several days

or they may appear concurrently.

29. Expressive aphasia results from the lesions in dominant frontal lobe (motor area of Broca).

Frontal lobe also contains motor cortex whose damage results in contralateral paresis.

Sensory cortex is present in the parietal lobe and damage to sensory cortex results in contralateral

hemianesthesia. Damage to dominant parietal lobe also results in dysgraphia, dyscalculia and

right/left confusion. Damage to nondominant parietal lobe causes construction apraxia,

contralateral sensory neglect and anosognosia. Unilateral lesion of occipital lobe causes

contralateral hemianopia.

61

30. Damage caused by lye (sodium hydroxide) ingestion occurs nearly nstantaneously and its

effects are most pronounced in the esophagus, where liquefactive necrosis occurs. Efforts to

neutralize the alkali, induce vomiting, or administer charcoal do not improve outcomes and

will only complicate management. Early upper gastrointestinal contrast studies and/or

endoscopy are critical for evaluating the extent of damage and determining further tx.

31. Several well-designed clinical trials have proven that interferon-beta decreases the frequency

of relapse, and reduces disability in pts with the relapsing-remitting form of MS.

32. The diagnosis of hypertension requires at least 3 separate blood pressure readings greater than

140/90 mmHg, preferably measured over a period of months. Once the diagnosis is made, the

goal of therapy is to maintain BP below 140/90 mmHg in pts with uncomplicated

hypertension and below 130/80 in pts with DM or renal disease. For pts with a BP ranging

from 120-139/80-89 (pre-hypertension), lifestyle modification in the form of weight loss,

exercise, and decreased salt intake is recommended. Pts with a BP in excess of 140/90 should

also undergo a trial period of lifestyle modification; if the BP remains elevated despite such

measurement, then it is appropriate to initiate antihypertensive therapy. When the BP is in the

range of 140-159/90-99 (stage I), single-drug therapy should be attempt. Hydrochlorothiazide

is the initial antihypertensive of choice in the absent of specific indications for other

antihypertensives. Thiazide diuretics are particularly beneficial in older pts, African-

Americans, and those afflicted with osteoporosis (↓urinary calcium excretion). Tow-drug

combinations are appropriate when the BP exceeds 160/100 (stage II).

Beta-blocker: young, Caucasian pts, hypertension due to systolic heart failure, post-myocardial

infarction, angina, essential tremor, migraine, huperthyroidism, and atrial fibrillation.

ACEIs: hypertension pts with co-existing DM, proteinuric renal failure, systolic heart failure, or a

history of myocardial infarction.

33. Whipple disease is a multisystem disorder with a varied presentation caused by infection with

the gram+ bacillus Tropheryma Whippelii. Chronic malabsorptive diarrhea, weight loss,

migratory non-deforming arthritis, lymphadenopathy and a low-grade fever are the most

common presenting symptoms.

34. Disseminated histoplasmisis is characterized by palatal ulcers, hepatosplenomegaly, and

pancytopenia.

35. The third heart sound (S3) is a low frequency diastolic sound heard just after S2 that is

associated with left ventricular failure. IV diuretics provide symptomatic benefits ot pts with

decompensated heart failure.

36. The low glucose concentration in exudative effusion is due to the high metabolic activity of

leukocytes within the pleural fliud.

37. Scatter plots are useful for crude analysis of data. These can demonstrate the type of

association (linear, non-linear), if any present.

38. Cervicofacial actinomycosis classically presents as a slowly pregressive, non-tender,

indurated mass, which evolves into multiples abscesses, fistulae, and draining sinus tracts

with sulfur granules, which appear yellow. The tx is high-dose penicillin for 6-12 weeks.

39. The gold standard test for making a diagnosis of acromegaly is measurement of growth

hormone levels following an oral glucose load. Most pts with acromagaly are unable to

suppress growth hormone levels following an oral glucose load. GH leads to increase IGF1

formation and secretion mainly by the liver. Most of effects of GH are mediated through

62

IGF1. IGF1 has a very long half-life, and its level does not fluctuate like GH levels. IGF1 is

therefore a good screening test for acromagely. Nontheless, IGF1 is an indirect measure of

GH, diseases of the liver, kidney, and some other systemic illness can lead to spuriously low

IGF1 levels. So IGF1 only for screening.

40. The most common malignancy of the liver is metastasis form another primary source (normal

AFP, multiple nodules of varying sizes). Hepatocellular carcinoma is far less common.

Imaging of HCC usually reveals one lesion with poorly defined margins instead of several

masses of varying sizes.

41. Drugs with anticholinergic properties (amitryptilin) can cause acute urinary retention by

preventing detrusor muscle contraction and urinary sphincter relaxation. The tx is medication

discontinuation and urinary catheterization.

42. Membranous glomerulonephritis is the most likely diagnosis in pts with both HBV infection

and nephrotic syndrome. HCV-cryoglobulinemia, membranoproliferatvie glomerulonephritis.

Minimal change disease—Hodgkin’s lymphoma. HIV-collapsing and focal segmental

glomerulosclerosis. SLE-diffuse proliferative glomerulonephritis.

43. Encapsulated bateria, esp pheumococcus, are the most common cause of pneumonia in HIV

pts.

44. Spironolactone is the diuretic of choice in treating cirrhotic ascites. Tx of cirrhotic ascites: 1)

diagnostic paracentesis, SAAG (serum albumin minus ascetic albumin) >1.1 suggest portal

hupertension. 2) Salt-restricted diet 3) spironolactone, generally in conjunction with

furosemide. 4) large volume paracentesis. 5) transjugular intrahepatic portosystemic shunt

(TIPS) procedure.

12/27/09

1. Screening for ovarian cancer with annual CA125 and transvaginal ultrasound is a reasonable

option for pts who are at higher than average risk fo developing the disease. CA19-9 is a

marker of GI cancers such as pancreatic, gastric, and colon cancers. Pts with who are positive

for BRCA 1 or 2 mutations are typically recommoned to undergo a prophylactic

oophorectomy by age 35 or after completing child bearing. Prophylactic oophorectomy

decreases mortality from ovarian cancer in such pts.

2. Risk is the probability of getting a disease over a certain period of time. To calculate the risk,

divide the numer of disease subjects by the total number of subjects in the study (all people at

risk). a/a+c

3. Always suspect Crohn’s disease in a young pt with chronic bloody diarrhea. Crohn’s disease

can involve the GI tract anywhere, from the esophagus to anus. Ulcerative colitis nealy always

involves the rectal mucosa.

4. A complex partial seizure is characterized by brief (ie. Lasting for a few minutes) episodes of

impaired consciousness, failure to respond to various stimuli during the episode, staring

spells, automatisms (eg. lip smacking, swallowing, picking movements of the hands), and

post-ictla confusion. The EEG pattern is usually normal or may show brief diacharges. Unlike

in absence seizure, hupervantilation during the EEG cannot simulate a complex partial

seizure.

A typical absence seizure is characterized by brief (ie typically lasting for a few seconds) periods

of impaired consciousness. Pts may also present with automatism, but do not have a post-ictal

63

state. Huperventilation during the EEG reveals a generalized 3Hz spike-and –wave patern on a

normal background.

A atypical absence seizure lasts longer. The characteristic EEG patern is slow apike-and-wave

activity with a frequency less than 2.5 Hz.

Juvenile muoclonic epilepsy presents in adolescents with unilateral or bilateral myoclonic jerks.

The symptoms take place in the morning, and may be precipitated by sleep deprivation.

Lennox-Gastaut syndrome usually presents in childhood as seizure of multiple types, impaired

cognitive function, and slow spike-and-wave activity on EEG.

5. The first step in the evaluation of a pt with dizziness is to classify the symptoms as vertigo

(spinning accompanied by nausea), presyncope or disequilibrium. The next step is to classify

the vertigo as either central (A lesion of VIII cranial nerve) or peripheral (inner ear).

Meniere’s disease is the likely cause of vertigo if the pt also has a sensation of ear fullness

(abnormal accumulation of endolymph within the inner ear). Pt with Meniere’s disease will

often complain of hearing loss and tinnitus.

A lesion of VIII cranial nerve can lead to central vertigo, but not lead to ear fullness.

Pts with cerebellar lesions may have vertigo, but it is usually accompanied by other signs of

cerebellar dysfunction, such as incoordination or imbalance.

Lateral medullary infarction, also known as Wallenberg syndrome, can present with intense

vertigo. However, other nsurologic problems such as gaze abnormalities, limb ataxia, sensory loss,

and Horner’s syndrome are often present as well.

6. Tumors that are metastatic to bone cause local osteolysis by production of cytokines such as

IL-1 and tumor necrosis factors (TNF). The most frequent tumors that produce hypercalcemia

by this mechanism are lung cancer and breast cancer.

The most common cause of hupercalcemia in pts with nonmetastatic solid tumors is production of

parathyroid hormone related peptide (PTHrP). In such cases, serum PTH is typically low.

Ectopic PTH production by tumor cell is a very rare cause of hypercalcemia and has been reported

in ovarian tumors, lung cancer and neuroectodermal tumors.

Hypercalcemia is cases of Hodgkin’s disease is almost always produced by calcitriol.

7. All adults should be immunized against diphtheria and tenanus every 10 year. Adult age 50

and older as well as youger adults with certain comorbidities should also hav annual influenza

vaccinations. Pneumococcal vaccine is recommended for all adults age 65 years as older as

well as in younger adults who are at least risk for invasive pneumococcal disease or its

complications. (This includes pts with chronic illness and immunosuppression as well as

cigarette smokers.)

8. Cardiac tamponade is grave condition characterized by hypotension, tachycardia, and

distended jugular veins. The ECG reveals electrical alternans—the amplitude of the QRS

alternates as the heart moves in the fluid-filled pericardial sac. Tx consists of massive volume

resuscitation and emergent pericardiocentesis.

9. PVCs (premature ventricular complexes) in pts post-myocardial infarction and can be

recognized by there widened QRS (>120msec), bizarre morphology, and compensatory pause.

Even though they may indicate a worse prognosis, tx is not indicated unless the pt is

symptomatic. Suppressing PVCs with antiarrhythmis medications has actually been shown to

worsen survival.

10. High-dose niacin therapy that is used to treat liped abnormalities frequently products

64

cutaneous flushing and prutitis. This side effect is explained by prostaglandin-inducec

peripheral vasodilatation and can be reduced by low-does aspirin.

11. Polymyositis is an inflammatory muscle disease of unknown etiology that presents initially

with slowly progressive proximal weakness of the lower extremities characterized by

difficulty with stair climbing or rising from a seated postion. Muscle tenderness also occurs in

some pts (severe tenderness or tenderness in the sbsence of weakness should lead to the

consideration of other dx such as polymyalgia rheumatica). There is no skin findings in PM,

which helps to distinguish it from dermatomuositis. A muscle biopsy is the best diagnosis

study for polymyositis. Biopsy in PM shows a mononuclear infiltration surrounding necrotic

and regenerating muscle fibers. In more advanced disease, an inflammation infiltrate may be

absent and replacement of muscle with fibrotic tissue may be evident.

12. A recently changed mole is a very strong risk factor for malignancy, and is associated with a

relative risk of at least 10.

13. Immunocomplex disease is primarily responsible for IE (Infective endocarditis) -associated

glomerulonephritis, Roth spots, and Osler’s nodes. Janeway lesions (macular, erythematous,

blanching, non-painful lesions on the palm and soles) result from septic embolism.

Embolization of begetation fragements lead to infection or infarction of remote tissues (eg.

pulmonary and splenic infarction).

14. Placing a consolidated lung segment in the dependent position can increase the right-to-left

shunt in a pt with pneumonia. In an upright individual, both ventilation and perfusion are

greatest at the lung bases (the most dependent regions of the lungs) and leasr at the apices.

When lying on one’s righ or left side, the dependent lung gets the greatest ventilation and

perfusion. In condolidated lung segments the alveoli are filled with exudates and do not

participate in pulmonary gas exchange, so their ventilation is essentially zero, ie, blood

perfusing consolidated areas is not oxygenated.

An example of a situation that would cause hypoxia due to increased dead space would be a

pulmonary embolism, where the lung is properly ventilated, but segments of lung are not perfused.

15. Compression fractures typically occur in the setting of decreased bone mineralization or

density. Diagnosis the underlying cause of compression fractures is important in decreasing

the risk of future fracture. Local tenderness is evident upon percussion of the fourth lumbar

vertebra.

Ligamentous sprains of the back can often be traced to a specific event or action. However, they

are more painful with movement and should feel their best in the morning. Pain is typically

perispinal, not directly over a vertebral body.

16. Folic acid and cobalamin deficiencies both cause a macrocytic anemia with hupersegmented

neutrophils. Deficiencies in either nutrient will result in increased homocysteine levels

because both are involved in homocysteine metabolism. Only cobalamin deficiency, however,

will cause in increase in methylmalonic acid concentration (cobalamin is involved in the

conversion of methylmalonyl-CoA to succinyl-CoA).

17. Development of nephropathy is preceded by development of excessive protein excretion, the

initial stages of which is termed microalbuminuria (30-300 mg/24hr, Normal<30mg/24hr).

Spot urine collection and timed urine collection for the measurement of urine microalbumin to

creatinine ratio are generally accepted as good screening methods. Although 24-hour urine

collection is slightly more accurate in screening for microalbuminuria, its inconvenience to

65

pts make it less preferred by physicians. Routine dipstick testing is not recommended during

the initial stages of nephropathy. It only detect macroalbuminiuria >300mg/24hr.\

18. Iron deficiency is the MCC of anemia in elderly pts. Inflammatory joint disease, and not

degenerative joint disease, causes the anemia of chronic disease. A diet poor in green leafy

vegetables and meat can cause folate or VitB12 deficiency, respectively. *Extremely high

yield!!

19. Slurred speech, unsteady gait and drowdiness can be seen in the overdose of multiple drugs.

Benzodiazepine overdose can be distinguished from opioid overdose by the lack of severe

respiratory depression and the lack of papillary constriction. Furthermore, while alcohol

(nystagmus in addition to slurred speech, sedation, and unsteady gait) and phenitoin

intoxication (horizontal nystagmus, cerebellar ataxia and confusion) also share similarities

with benzodiazepine overdose, they can be distinguished by the presence of nystagmus.

20. Cradle cap, or seborrheic dermatitis, is a common pediatric skin condition. This popular, scaly

rash tends to affect the eyebrows, nasolabial folds, and scalp. Tx consists of moisturizers,

antifungals, and topic steroids. DD: Atopic dermatitis, contact dermatitis, tinea capitis,

psoriasis.

21. Incidental discovery of a pulmonary nodule on radiographic imaging is common. The first

step in the work-up of such nodules is to obtain any previous radiographic lung images.

Absence of growth over time rules out malignancy. If previous films are not available, then

the nodules is assigned either a low, intermediate or high probability of malignancy based on

clinical and radiographic data. Low probability nodules are followed by serial resolution chest

CT (3,6,9,12,18, and 24 months), intermediate probability nodules are further investigated

with PET scan and/or biopsy, and high probability nodules are removed surgically.

22. Klebsiella is an encapsulated gram-negative bacillus and can cause pnrumonia in subjects

with debilitating conditions, esp. alcoholics. Friedlander’s pneumonia generally affects the

upper lobe and is characterized by currant jelly-like sputum.

Pseudomonas is also encapsulated gram-negative bacillus and a very common cause of

nosocomial infection.

23. In hyperthyroid pts not pretreated with antythyroid medications, radioactive iodine treatment

may initially cause exacerbations of the thyrotoxic state (dying tryroid cells release excess

thyroid hormone into the circulation and can temporarily aggravate the hyperthyroid state--

new onset atrial fibrillation and hand tremor). Iopanoic acid (telepaque) is a contrast medium

used in radiography. It has antithyroid effects by virtue of its iodine content. It decreases

thyroid hormone release from the overactive thyroid gland and decrease peripheral conversion

of T4 to T3.

24. pts with CHF due to alcoholic dilated cardiomyopathy: thrombocytopenia, macrocytosis,

elevated transminase, coronary angiography excludes ischemia. Total abstinence from alcohol

is the mainstay of alcoholic cardiomyopathy mx and it may reverse this condition if it is

employed earlier in the course of the disease.

25. CO ↓ PCWP ↑: left ventricle failure

CO N PCWP ↑: volume overload

PCWP ↓: hypovolemia-blood loss, intravascular volume redistribution;

Non-cardiogenic pulmonary edema (ARDS) usually had normal or low PCWP.

* extremely high yield!!

66

26. ARDS causes hupoxemia regractory to high inspired oxygen concentrations. Adequate

oxygenation in ARDS often requires positive end-expiratory pressure (PEEP) delivered via

mechanical ventilation.

27. All pts suspected of having Zollinger-Ellison syndrome (prominent gastric folds, ulcer located

beyond the duodenal bulb) should have fasting serum gastrin levels done. A serum gastrin

value greater than 1000pg/ml is diagnositic of the disorder. Pts with non-diagnostic fasting

serum gastrin levels should have a secretin stimulation test done. Secretin stimulates the

release of gastrin by gastrinoma cells. Normal gastric G cells are inhibited by secretin.

28. Valacyclovir is the drug of choice in the tx of herpes zoster, acyclovir is an alternative.

Postherpetic neuralgia can be prevented and /or treated with tricyclic antidepressants such as

amitriptyline or nortriptyline along with acute antiviral therapy.

29. Cherry hemangiomas (senile hemangiomas) are small red cutaneous papules common in

aging adults. They do not regress spontaneously and typically increase in number with age.

Light microscopy of these lesions reveals proliferation of capillaries and postcapillary venules

in the papillary dermis. They are always cutaneous and are not found on the mucosa or deep

tissues.

Strawberry (infantile) hemangiomas appear during the first week of life, initially grow rapidly,

then frequently regress spontaneously by 5-8 years of age. They are bright red when neart the

epidermis and more violaceous when deeper.

Spider angiomas consist of a bright red central papule surroundied by several ourwardly radiating

vessels. They occur due to dilatation of a central arteriole and its superficial capillary network and

are estrogen-dependent.

Cavernous hemangiomas consist of dilated vascular spaces with thin-walled endothelial cells.

They present as soft blue compressible masses up to a few centimeters in size. They may appear

on the skin, mucosa, deep tissue and viscera; when they appear on the skin, they are most

frequently based in the dermis. They are less likely to regress spontaneously than capillary

hemangiomas. Cavernous hemangiomas of the brain and viscera are associated with von Hippel-

Lindau disease.

Cystic hygromas consist of lymphatic cysts lined by a thin endothelium. These benign tumors are

often present at birth, and are most commonly located on the neck and lateral chest wall. Cystic

hygromas are frequently found in neonates with Turner syndrome and Down syndrome.

30. Ursodeoxycholic acid is the most commonly used drug in the tx of PBC as it relieves

symptoms and lengthens transplant-free survival time. Associated conditions: Sjogren’s

syndrome, Raynaud’s syndrome, scleroderma, autoimmune thyroid disease, hypothyroidism,

and celiac disease. Liver transplantation is the sole definitive cure for progressive PBC, but

indicated only in severe liver damage or cirrhosis.

31. A pt’s data may be used for research purposes only after obtaining informed consent.

32. Vaccination against HBV decrease the incidence of hepatocellular carcinoma, esp in regions

with high level of HBV infection such as Asia and Africa.

33. Syncopal episode (caused by Tordades de pointes) without following disorientation (post-

episode confusion is more characteristic for a seizure), hearing impairment, normal physical

exam, and family history of sudden cardia death should make you think of congenital long

‘QT’ syndrome—Jervell-Lange-Nielson, AR. Asymptomatic—beta-blocker. Symptomatic

(syncope) -- beta-blocker+a DDD pacemaker.

67

34. Isolated systolic hypertension (ISH) is an important cause of hupertension in elderly pts. It is

created by dereased elastic properties of the arterial wall. Always treat ISH, in spite of the fact

that diastolic blood pressure is not elevated. The drug of choice would be hydrochlorothiazide.

35. The Waterhouse-Friderichsen syndrome (adrenal glands hemorrhage) is a very serious

complication of Meningococcal meningitis caused by Neisseria meningitidis.

36. Acute lymphoblastic leukemia is predominantly a disease of children. Lymphoblasts lack

peroxidase positive granules but often contain cytoplasmic aggragates of PAS positive

material. Immunostaining for TdT is positive in more than 95% of pts. TdT is expressed only

by pre B and pre T lymphoblasts.

37. The serum ascites albumin gradient (SAAG) is used to distinguish between portal

hypertension and other causes of ascites. A SAAG of 1.1 g/dL or greater is consistent with

portal hypertension (increased hydrostatic pressure within the liver’s capillary beds).

38. Exudate: Pleural fluid protein/serum ratio>0.5; LDH >0.6; Pleural fluid LDH more than 2/3 of

the upper limit of normal serum LDH. Normal pleural pH 7.64; pH<7.3 indicates pleural

inflammation. pH 7.35-transudate. Glucose <60 mg/dL parapneumonic effusion, TB, or RA.

39. SIADH is characterized by hypotonic hyponatremia with euvolemia. Low plasma osmolality

(<280mOsm/Kg) with high urine osmolality (>100-150mOsm/Kg) in suspected pts is

diagnostic.

40. Until proven otherwise, suspect lung cancer in smokers who present with Horner’s syndrome

(partial ptosis, miosis, anhidrosis). Simple CXR should be the first test.

41. Cavernous sinus thrombosis typically occurs in pts with infections of the skin adjacent to the

eye or nose, but ethmous and sphenoid sinusitis can cause this condition as well. Pts usually

present with a headache, low-grade fever, periorbital edema, and cranial nerve palsies (III, VI,

IV, V1, V2).

42. Mitral regurgitation classically results in a holosystolic murmur heard best at the apex with

radiation to the axilla. Common clinical features of MR include exertional dyspnea, fatigue,

atrial fibrillation, and signs of heart failure.

43. Blastomycosis is a pulmonary fungal infection endemic to the Great Lakes, and Mississipi and

Ohio River basins. Systemic blastomycosis may cause skin and bone lesions in addition to

pulmonary manifestations. Broad-based budding yeast grown from the sputum confirm the

diagnosis. Itraconazole or amphotericin B may be used to trat symptomatic disease.

44. The ophthalmic branch (V1) of the trigeminal nerve controls corneal sensation. When it is

damaged, pts can suffer from corneal injury without awareness.

12/27/09

1. Pts with cerebellar hemorrhage present with atxia, vomiting, occipital headache, gaze palsy,

and facial weakness. There is no hemiparesis. It is crucial to make an early and correct

diagnosis, because emergent surgical decompression may be life-saving in such cases.

Pontine hemorrhage accounts for 5-12% of cases of hypertensive intraparenchymal hemorrhage.

Pts present with a deep coma and paraplegia that developed with a few minutes. The pupils are

pinpoint and reactive to light. There is decerebrate rigidity. There are no horizontal eye movement.

2. Acute iron intoxication occurs in 3 phases, beginning with disturbance of the GI mucosa

(30min to 6 hours, nausea, vomiting, hematemesis, melena, abdominal pain) and potentially

progressing to hepatic failure (12-96 hours, shock, metabolic acidosis) and death. The dx is

68

made when serum iron concentrations are greater than 350mcg/dL. Latent phase occurs 6-24

hours.

3. Low back pain in pts with history of malignancy should always raise suspicion for bone

metastasis. Tumors metastatic to bone typically cause constant progressive pain worsened at

rest.

4. Two-sample t test—compare the means of two groups of subjects. Two sample Z test—two

means, but population (not sample) variances are employed in the calculations. Because

population variances are not usually known, the test has limited applicability. ANOVA—

compare 3 or more means. Chi-square—categorical data and proportions. Meat-analysis—an

epidemiologic method of pooling the data from several studies to do an analysis having a

relatively big statistical power.

5. The basic pathophysiologic mechanism of Paget disease is impaired bone remodeling.

6. Chest pain that is reproducible with palpation suggests a musculoskeletal etiology.

7. Observer bias occurs when the investigator’s decision is adversely affected by knowledge of

the exposure status.

8. Chest pain in a young person with cardiovascular risk factors warrants a thorough cardiac

work-up. Aspirin should be the first drug administered when suspicion of a coronary artery

event is high due to its ability to prevent platelet aggregation and coronary spasm.

9. Optic glioma occurs in 15% of pts with neurofibromatosis, type 1. Retinal hamartoma is

typical for tuberous slcerosis. Optic neuritis is frequently the early manifestation of multiple

sclerosis. Visual loss develops typically faster, and no exophthalmos is present. Pituitary

adenoma results in bitemporal hemianopsia.

10. Most thyroid nodules are benign colloid nodules.

11. Porphyria cutanea tarda is a condition that is characterized by painless blisters, hypertrichosis,

and hyperpigmentation. It si often associated with HCV infection, and can be triggered by the

ingestion of certain substance (eg. ethanol, estrogens), which should be discontinued once

suspected.

12. Fecal occult blood testing is the most commonly used screening test for colon cancer. A

positive FOBT test warrants further evaluation using colonoscopy (better than sigmoidoscopy

and double contrast barium enema).

13. treatment initiation thresholds and goals based upon LDL level

14. Alzheimer’s disease is the most common cause of dementia in the US. It affects up to 50% of

people over age 85 and is the 6th leading cause of death among the elderly. Early signs include

visuospatial deficits (e.g getting lost in one’s own neighborhood) and problems with

anterograde memory formation (old memories tend to be preserved). Personality and

behavioral changes (eg. hupersexualit, agitation) may occur as the disease progresses.

Hallucination, changes in alertness, gait impairment, and urinary incontinence are late

findings.

Unlike Alzheimer’s disease, demential with Lewy bodies presents with alterations in alertness,

visual hallucinations, and extrapyramidal symptoms. Memory deficits occur later in the course of

disease as compared to Alzheimer’s.

15. Hypoparathyroidism (PTH deficiency) is characterized by low calcium and elevated

phosphorus level in the presence of normal renal function. Causes of hypoparathyroidism

include post-surgical, autoimmune parathyroid destruction and defective calcium-sensing

69

receptor (pseudohupoparathyroidism).

16. The adult and pediatric pneumococcal vaccines contain capsular polysaccharides and induce a

T-cell-indipendent B-cell response.

Oral polio vaccine—anti-polioviris IgA antibodies in the GI tract.

17. Phencyclidine is a hallucinogenic drug, which characteristically causes vertical nystagmus. It

can also cause dissociative feelings, phychotic and violent behavior, severe hypertension, and

hyperthermia. Tx includes urine acidification, haloperidol in the case of psychotic behavior,

and close monitoring until the condition resolves.

18. Suspect aortic dissection in a pt with acute retrosternal pain and a normal EKG. Check blood

pressure in both arms and auscultate for diastolic murmur of aoric regurgitation.

Transesophageal echocardiography is the preferred diagnosic tool. Before performing the

TEE, HTN shouldn be controlled.

19. CK-MB is the most useful lab test in assessing for coronary re-occlusion after an MI because

it typically returns to normal levels within 1-2 days. While troponin T is generally a more

sensitive marker for cardiac injury, it takes up to 10 days to return to normal after an MI,

making elevations difficult to interpret in this sort of scenario.

20. Chronic inflammatory diarrhea is typically associated with inflammatory changes in the blood

(anemia, elevated ESR, acut phase reactants. Reactive thrombocytosis). Blood/leukocyte-

positive stool is another important finding.

21. The 3 MCC of aortic stenosis in the general polulation are senile calcific aortic stenosis (>70

years old), bicuspid aortic valve, and rheumatic heart disease (less common than the other 2).

A bicuspid aortic valve is the cause of aortic senosis in the majority of pts under 70 years old.

22. sensitivity, specificity

23. The strongest influence on long-term prognosis following an ST-elevation myocardial

infarction is the duration of time that passes before coronary blood flow is restored (PTCA

–‘door-to-balloon time’ less than 90 min or fibrinolysis—‘door-to-needle time’ 30min).

24. Lactose intolerance is characterized by a positive hydrogen breath test, a positive Clinitest of

stool for reducing substances (undigested lactose), and an increased stool osmotic gap.

25. Risk factors for non-alcoholic steatohepatitis include obesity, DM, huperlipidemia, total

parenteral nutrition, and the usage of certain medications (corticosteroids, amiodarone,

diltiazem, tamoxifen, HAART, endocrinopathies (eg. cushing’s syndrome). Labs show mild

elevation in aminotransferases and alkaline phosphatase, with AST/ALT <1.

26. Excisional biopsy with narrow margins is the preferred study for the dx of melanoma. If the

depth of the lesion is less than 1mm, the melanoma can be excised with a 1mm tumor free

margin and they have a 99% 5-year survival. Tumors greater than 1mm in depth should have a

sentinel lymph node study.

27. Reentrant ventricular arrhythmia (ventricular fibrillation) is the MCC of death in pts with

acute MI. Electromechanical dissociation is typical for pulmonary thromboembolism and

pericardial tamponade.

28. Influenza vaccine—annually, pneumococcal not yearly

29. Primary hyperaldosteronism is characterized by perissitently elevated blood pressure,

hypokalemia, metabolic alkalosis, mile huypernatremia, very low plasma rennin activity and

absent edema. (metabolic alkalosis—K+ move into extracellular to correct hypokalemia, H+

move into cells)

70

30. Consider Clostridium difficile diarrhea in all pts who have received antibiotics and develop

nausea, vomiting, abdominal pain, diarrhea, and elevated while count. Cytotoxin assay in the

stool is a highly sensitive test to diagnosis this condition. *Extremely high yield!!

31. The most common renal vascular lesions seen in hypertension are arteriosclerotic lesions of

afferent and efferent renal arterioles and glomerular capillary tufts. DM nephroparhy is

characterized by increasd extracellular matrix, basement membrane thickening, mesangial

expansion, and fibrosis.

32. Cataract is a vision-impairing disease characterized by blurred vision and a glare. It is due to

progressive thickening of the lens. Definitive tx is lens extraction.

Macular degeneration affects central vision. The two forms are atrophic and exudative. The

atrophic type si characterized by drusen and patchy depigmentation in the macular region. The

exudative form is characterized by new blood vessels which can leak, bleed, and scar the retina.

Dry type ARMD usually presents a progressive, slow loss of vision in one or both eyes, while wet

type ARMD presents as an acute distortion in vision, with wavy lines or loss of central vision,

usually in one eye.

33. Trachoma presents with follicular conjunctivitis and pannus (neovascularizatio) formation in

the cornea.

34. Ankylosing spondylitis is a seronegative spondyloarthropathy that typically affects men under

40 years of age. The classic finding is low back pain that is worse in the morning and

gradually improve during the day. Apophyseal joint arthritis

35. In HIV pts, both HSV and VZV can cause severe, acute retinal necrosis associated with pain,

keratitis, uveitis, and fundoscopic findings of peripheral pale lesions and central retinal

necrosis. In contrast, CMV retinitis is painless, not usually associated with keratitis or

conjunctivitis, and characterized by funduscopic findings of hemorrhages and fluffy or

granular lesions around the retinal vessels.

36. Tartrate-resistant acid phosphatase (TRAP) staining is important in the dx of hairy cell

leukemia.

37. The main mechanism responsible for pain relief in pts with anginal pain treated with

nitroglycerin is dilation of veins and decrease in ventricular preload. (dilation of capacitance

vessels).

38. Any elderly pt with bone pain, renal failure, and hypercalcemia has multiple myeloma until

proven otherwise. Approximately 50% of multiple myeloma pts develop some degree of renal

insufficiency, this is most likely due to obstruction of the distal and collecting tubules by large

laminated casts contining paraproteins (mainly Bence Jones protein).

39. Cauda equine is an acute compression syndrome presenting with acute motor and sendory

loss, loss of rectal tone and urinary retention. This is a surgical emergency.

40. Diuretics are a necessary adjunct to the tx of MI if pulmonary edema is present. Beta-blockers

are a standard therapy in MI, but should be avoided in this scenario as they can worsen acute

heart failure.

41. Rotator cuff tears result from chronic rotator cuff tendonitis and shoulder trauma. Shoulder

pain and weakness when lifting the arm above the head is suggestive of rotator suff pathology.

A lidocaine injection ameliorates the pain and weakness of rotator cuff tendonitis, while it

does not improve symptoms of a rotator cuff tear. MRI is excellent at visualizing soft tissue

structures and is the study of choice for dx rotator cuff tear.

71

42. Thiamine deficiency can cause Wernicke’s encephalopathy, the triad of encephalopathy,

oculomotor dysfunction, and gait ataxia. This condition may be induced iatrogenically in

susceptible pts by administration of glucose without thiamine. Chronic thiamine deficiency

can also cause Korsakoff’s syndrome, characterized by irreversible amnesia, confabulation

and apathy.

43. Fanconi’s anemia (FA) is an autosomal recessive disorder marked by progressive bone

marrow failure (macrocytic anemia), areas of skin hypopigmentation, congenital

abnormalities (microcephaly, abnormal thumbs and hypogonadism), and a predisposition to

cancer. Most pts with FA are diagnosed by age 16. Numerous genes have been implicated, all

believed to involved DNA repair. The definitive tx for aplastic anemia is hematopoietic stem

cell transplantation.

44. In pts with suspected MENIIa syndrome, genetic testing had replaced biochemical

measurement of serum calcitonin as the recommended screening test. If genetic analysis is

positive for RET protooncogene mutation, total thyroidectomy is indicated.

12/28/09

1. CT scan is the best test for the diagnosis of diverticulitis in acute setting.

2. CHF exacerbation can cause tychypnea as left ventricular dysfunction allows fluid to pool in

the lungs, causing a pleura effusion and hypoxemia due to reduced ventilation. Tachypnea

causes hypocapnia and respiratory alkalosis. Exam typically shows signs of fluid overload, S3

and S4 gallops, cardiomegaly, and bibasilar crackles in the lungs.

Pulmonary embolism presents with acute-onset dyspnea, tachypnea, and pleural chest pain. Pts

have hypoxemia and respiratory alkalosis with a widen A-a oxygen gradient.

3. Choledochal cysts are congenital abnormalities of the biliary tree characterized by dilatation

of the intra and /or exta hepatic biliary ducts. The clinical presentation varies with the age. In

infants it may present with jaundice and the passage of acholic stools. In children it causes

abdominal pain, jaundice, and attacks of recurrent pancreatitis, which may be eveident by

increase in the amylase and lipase levels. Adults with choledochal cysts commonly present

with vague epigastric or right upper quadrant abdominal pain or cholangitis. Choledochal cyst

can degenerate into cholangiocarcinoma. The initial investigation of choice is an

ultrasounogram followed by CT scan or magnetic resonance imaging (MRI) as needed.

Caroli’s syndrome is a congenital disorder characterized by intrahepatic dilatation of bile ducts.

Biliary atresia usually presents early in the infancy with marked obstructive jaundice and acholic

stool.

4. The typical clinical scenario for vasovagal syncopy (common faint, neurocardiodiogenic

syncopy) including prodrome (lightheadedness, weakness, and blurred vision), provocation by

an emotional situateion, and repid recovery of consciousness. Vasovagal syncope is frequently

recurrent. Upright tilt table testing with or without pharmacologic provocation (isoproterenol)

may be indicated to confirm the diagnosis.

5. Wenchbech or Mobitz type I heart block—a narrow QRS, pregressive increase in PR interval

until a ventricular beat is dropped, then the sequence is repeated. It is a benign arrhythmia and

it transient. Unless the pts is symptomatic, it requires no tx; symtomatic-atropine.

6. Asbestosis is a form of pneumoconiosis resulting from inhalation of particles involved in

many industrial processes. Progressive dyspnea, clubbing and end-inspiratory crackles are

72

typical signs and symptoms. Pulmonary function tests reveal a restrictive lung disease pattern,

with decreased lung volume, decreased DLCO (diffusion lung capacity), and a normal

FEV1/FVC.

7. Opiod intoxication presents with miosis, depressed mental status, decreased respiratory rate,

decreased bowel sounds, hypotension and bradycardia. Of these, decreased respiratory rate is

the best predictor of intoxication and is also a frequent cause of mortality.

8. Herpex simplex keratitis –corneal vesicles and dendritic ulcers. Herpes zoster ophthalmicus is

an infection caused by varicella-zoster virus. Most episodes occur in the elderly. It presents

with fever malaise and a burning, itching sensation in the periorbital region. Examination

reveals a vescular rash in the distribution of the cutaneous branch of the first division of the

trigeminal nerve. Conjunctivitis and dedriform corneal ulcers characterize the eye

involvement.

9. G6PD deficiency is an X-linked recessive disorder commonly seen in African-American men.

It is characterzied by episodic hemolysis in response to oxidants drugs, or fava beans.

Peripheral blood smear reveals bite cells and red blood cells inclusions (Heiz bodies) seen

after crystal violet staining.

10. Conjugared hyperbilirubinemia is mainly because of intrahepatic or extrahepatic obstruction

or congenital impaired hepatic excretion of bilirubin. It is important to rule out an extra

hepatic obstruction with an ultrosound or a CT scan of abdomen. If these imaging techniques

fail to show an extra hepatic biliary dilatatio then the next step would be to go for an ERCP or

PTC (percutaneous tranhepatic cholangiogram).

11. Suspect medullary cystic disease in adults with recurrent UTI or renal stones and typical

radial arrangement contrast filled cysts demonstrated by IVP. It is autosomal dominant

whereas the juvenile form, also known as nephronophthisis is autosomal recessive. There is

no therapy to prevent the disease progression but in most pts the cysts are small and have no

harmful effects. These pts are usually asymptomatic initially; later, they develop UTI and

renal stones. Renal stone are managed in the same manner as pt without medullary cystic

kidney. These pts must take adequate salt and water to replenish the renal losses. Periodic

screening is required for renal stones, hematuria, and UTI. Pts with medullary cystic disease

and hemi-hypertrophy should be screen for cancer.

12. COPD exacerbation is characterized by an acute worsening of symptoms in a pt with chronic

obstructive lung disease. It is classically caused by an upper respiratory tract infection and

may be preceded by cough and fever. Examination typically shows bilateral wheezes, and

ABG finding include respiratory acidosis and hypoxia.

13. Goodpasture’s disease affects the lungs (causing cough, dyspnea, and hemoptysis) and kidney

(nephritic range proteinuria <1.5g/day, acute renal failure, and dysmorphic red cells/red cell

casts on urinalysis). Systemic symptoms are uncommon. Diagnosis is made by renal biopsy

showing linear IgG antibodies along the glomerular basement membrane.

14. Multiple sclerosis is best diagnosed by MRI. The MRI shows white matter disease, which is

very characteristic. Tonometry is done to measure the pressure in the ocular chambers to

assess the presence of glaucoma.

15. Mild manifestation of a drug allergy (urticaria and pruritis without systemic symptoms) are

usually treated with antihistamines and discontinuation of the offending drug. Tx with

adrenalin and steroids are reserved for more severe reactions accompanied by systemic

73

symptoms (anaphylaxis).

16. Short-acting beta-adrenergic agonists administered 20 min before exercise are the first-line

treatment for isolated exercise-induced asthma. The inciting trigger in exercise-induced

asthma is rapid ventilation of cold, dry air which stimulates mast cell degranulation and

airway constriction. Long acting beta-adrenergic agonists may be used in children or athletes

who engage in athletic activities throughout the day.

17. Acetaminophen should be the initial tx for pts with mild to moderate pain from osteoarthritis.

It has proven efficacy and a favorable side effect profile.

18. A copious amount of purulent drainage in newborns who are 2-5 days old is most consistent

with gonococcal conjunctivitis. Conjunctivitis casued by Chlamydia trachomatis is usually

milder and presents more than 5 days after birth. Chemical conjuctivitis can result from

prophylactic silver nitrate eye drops, but it is usually fairly mild.

19. All pts with suspected bacterial pneumonia should have a CXR done as the first step, and

antibiotics should be administrated as soon as possible without waiting for sputum gram stain

or cultures.

20. First-degree heart block is completely benign arrhythmia and requires no tx.

21. CT scan is the best test for diagnosisng and evaluating the abdomen of pts during an acute

episode of diverticulitis.

22. Non-selective beta-blockers (propranolol) are used for the primary and secondary prevention

of variceal bleeding in cirrhotic pts who have portal hypertension with esophegeal varices.

23. ACE inhibitors are the drug of choice for lowering blood pressure to target levels (<130/80

mmHg) in DM.

24. Gastric outlet obstruction typically presents with persistent vomiting, which leads to

hypokalemic, hypochloremic metabolic alkalosis. Emergent tx includes nasogastric suctioning

to deccompress the stomach, hydration with sodium chloride, and potassium replacement.

25. case control study

26. Post cholecystectomy pain most commonly occurs due to one of three reasons: common bile

duct stone, sphincter of Oddi dysfunction or functional causes (no obvious organic basis and

should treated symptomatically with analgesics and reassurance).

27. Aortic regurgitation produces an early diastolic murmur and can be associated with several

physical signs caused by a hyperdynamic pulse, including bounding or “water hammer”

peripheral pulses.

28. A confounder is an extraneous factor which had properties linking it with the exposure and

outcome of interest.

29. Decreased esophageal body peristalsis and poor relaxation of the LES on manometry are

typical for achalasia. Achalasia presents with progressive dysphagia, chest pain, food

regurgitation and aspiration. Barium swallow shows a dilated esophagus and a “bird beak”

deformity of the LES.

30. Indications for surgery in pts with AAA (abdominal aortic aneurysms) include: diameter

greater than about 5cm, presence of symptoms, or rapid rate of growth. For pts being

medically managed, smoking cessation is the intervention with the greatest likelihood of

slowing AAA progression.

31. Condyloma acuminata are verrucous, papilliform lesions located in the anogenital region.

Podophyllin is one of the available tx options.

74

32. Atheroembolism results from dislodgement of cholesterol plaques from the aortic root. “Blue

toe syndrome”, in which emboli to the pedal circulation cause cyanotic and painful toes with

intact pulses, is one common presentation of cather-induced atheroembolism.

33. Histopathologically, demonstration of invasion of the capsule and blood vessels is required for

differentiating follicular cancers from follicular adenomas. Follicular thyroid cancers have the

propensity to invade blood vessels and metastasize to distal organs.

34. Suspect hemochromatosis in a patient with new-onset diabetes, arthropathy, and

hepatomegaly.

35. Suspect malignant otitis extrena in any diabetic pt with severe ear pain, otorrhea, and evidence

of granulation tissue in the ear canal. Pseudomonas aeruginosa is the most frequent cause of

malignant otitix externa.

36. Inflammatory bowel disease may occur in association with an inflammatory arthritis.

Ankylosing spondylitis and IBD are both associated with HLA-B27 and may occur in

association with one another. Both conditions may also be associated with a positive p-ANCA

despite the absence of vasculitis in both conditions.

37. Polymyalgia rheumatica (PMR) is characterized by pain and stiffness in the neck, shoulders

and pelvic girdle in a patient over 50 years old with an elevated ESR and morning stiffness

lasting over one hour. The treatment of choice for PMR when it is not associated with giant

cell arteritis is low-dose prednisone. In PMR, the physical examinaiton is frequently

unremarkable with patients denying tenderness with active or passive range of motion. Signs

of inflammation in the joints are absent. When asked to identify the location of their pain, pts

typically indicate that it is in the soft tissues and not the joints.

38. Suspect ehrlichiosis in any pt from an endemic region with a history of tick bite, systemic

symptoms, leukopenia and/or thrombocytopenia, and elevated aminotransferses. The drug of

choice is doxycycline.

39. Always consider malaria in patients from endemic areas with high-grade periodic fever and

chills. Anemia and splenomegaly are the clinical clues.

40. The most likely diagnosis of an asymptomatic (ie. Lesion-free), immunocompetent adult pt

with a non-healing, isolated ulcer in the vermilion zone of the lower lip and a significant

history of sun exposure is squamous cell carcinoma.

41. Raising the cut-off point of a diagnostic test decreases its sensitivity, but increases its

specificity. Consequently, TN is increased and FP is decreased.

42. Heparin-induced thrombocytopenia (HIT) is an immune-mediated process that causes

thrombocytopenia several days after heparin therapy is started. Somewhat paradoxically, HIT

is highly thrombogenic and pts must be monitored closely for arterial and venous clots.

43. Plain film of an osteoarthritic knee reveal narrowed joint space, osteophytes, and subchondral

sclerosis or cysts. Obesity is the most readily modifiable risk factor of OA, and weight loss

decreases joint pain, increases function, and slows progression of the disease. Pharmacologic

therapy and exercise also play integral roles in OA therapy.

44. Pharygoesophageal (Zenker’s) diverticulum develops immediately above the upper

esophageal sphincter by herniating posteriorly between the fibers of cricopharyngeal muscle.

Motor dysfunction and incoordination are responsible for the preblem. * The surgical tx of the

disorder includes excision and frequently cricopharyngeal myotomy. * Extremely high yield!!

75

12/29/09

1. Tinea corporis is charecterized by ring-shaped scaly patches with central clearing and dintinct

borders. Topical tx with 2% antifungal lotions and creams (eg. terbinafine), or systemic tx

with griseofulvin (for extensive dieasea) offer good relief.

2. The incidence of a disease is not changed by any kind of treatment. On the other hand, the

prevalence may be affected by tx of the diease. In this case, tx of an acute and rapid fatal

diease (eg. lung cancer) prolonged the suevival of pts; however, such tx did not cure the

disease. This will result in more people having the said disease at one point in time; hence, the

prevalence will increase.

3. Pts with Hashimot’s thyroiditis (postive anti-thyroperoxidase) are at increased risk for

developing thyroid lymphoma.

4. Choose the appropriate initial antihypertensive therapy in a pt with asthma.

Hydrochlorothiazide is the initial drug of choice the general population, and beta-blockers are

relatively C/I with asthma.

5. Neurofibromatosis type 1 is an autosomal dominant disorder.

6. ACEIs have been shown in multiple studies to improve mortality in pts with CHF. Other

commonly used medications with a mortality benefit in CHF include beta-blockers,

angiotensin II receptors blockers, and spironolactone.

7. Raising cut-off point (eg. increasing the inclusion criteria) of a screening test results in an

increase in speicificity and decease in sensitivity.

8. Ethylene glycol is associated with hypocalcemia and calcium oxalate deposition in the

kidneys. The leads to flank pain, hematuria, acute renal failure and anion gap metaboic

acidosis. Tx involves administration fomepizile or ethanol to achieve ADH dehydrogenase

inhibition, sodium bicarbonate to allevate acidosis and hemodialysis in the case of acidosis

and /or end organ damage.

9. Viral infection may present with symmetric arthritis. Viral arthritis is distinguished form RA

and other causes of symmetric arthritis by its acute onset, lack of elevated inflammation

markers, and resolution with two months.

10. Achalasia—the recent onset of hysphagia to both solids and liquids is characteristic. Reflux of

food particles from the esophagus at night may lead to aspiration symptoms, as in this case.

The typical rediologic finding shows a dilated esophagus with an air fliud level. The dx is

made by manometry; however, endoscopy is required to ensure that there is no malignancy.

11. Strawberry hemaniomas (capillary hemaniomas) are benign vascular tumor of children. They

appear during the first weeks of life; initially grow rapidly, and typically regress by 5-8 years

old. These tumors are composed of capillaries separated by connective tissue.

12. PEA Mx--Any pt in cardiac arrest with a non-shockable ryhthm (anything other than VF/VT)

should immediately receive CPR including manual chest compressions, establishment of a

definitive airway and ventilation with 100% oxygen. IV access should also be established so

ACLS medications such as epinephrine, vasopressin and stropine can be adminstered. It is

important to make the dx of PEA as there are a variety of potentially treatable cauese (6Hs

and 6Ts): Hs: hypotension, hypoxia, hydrogen ions (acidosis), hypothemia, hypoglycemia,

hyper/hypokalemia. Ts: tamponade (cardiac), tension pneumothorax, thrombosis (MI, PE),

trauma (hypovolemia), tablets (drugs) and toxins.

13. Crohn’s disease can involve any component of the GI tract from the mouth to the anus, and

76

charateristically has skip areas of involvement. Although nonspecific, aphthous ulcers in the

mouth can be seen in Crohn’s disease. Granulomas are identified pathologically in up to 30%

of pts.

14. FFP contains all clotting factors and is the first line tx for bleeding pts with a coagulopathy.

15. Chronic GI blood loss is the MCC of iron-deficiency anemia in an adult male or a post-

menopausal woman.

16. Post-splenectomy pts are at increased risk for sepsis from encapsulated organisms due to

impaired antibody-mediated opsonization in phagocytosis.

17. Immediate anticoagulation with heparin and surgical intervention (embolectomy) are crucial

to prevent tissue death in a pt with onging ischemia of the limb.

18. Gout can result from overprodiction or underexcretion of uric aric. Induction chemotherapy

results in rapid tumor cell lysis and release uric acid into the circulation. It is important to

distinguish between the prevention and tx of gout. For prevention, allpurinol and probenecid

ar used, while colchicine, NSAIDs, and steroids are used during acute attacks.

19. Transjugular intrahepatic portosystemic shunt precedure is used in cirrhotic pt for refractory

ascites, refractory hydrothoras, and surgical mx of acute recurrent variceal bleeding.

Pleurodesis is a choice for refractory pleural effusion secondary to maligancy but not for a

hydrothorax secondary to cirrhosis.

20. NPV is the probability of being free of a disease if the test result in negative. The NPV will

vary with the pretest probability of a disease. A pt with a high probability of having a disease

will have a low NPV, and a pt with a low probability of having a disease will have a high

NPV.

21. Staph. Aureus causes toxin-induced gastroenteritis. Because the illness is due to preformed

toxin, emesis is a predominant feature and symptoms start in less the 6 hours. Mayonnaise-

containing salads are often implicated.

22. Lyme arthritis is a late manifestation of Lyme infection; suspect in pts with history of travel to

endemic areas (Connecticut, Rhode Island, New York, Pennsylvania, New Jersey, Delaware,

Maryland and Wisconsin). Intermittent inflammatory arthritis is a typical presentation.

23. Poison ivy and nickel jewelry can cause allgergic contact dermatitis, which is a type IV

hypersensitivity reaction.

24. Hepatic encephalopathy is an alteration in CNS function due to poor hepatic clearance of

toxin. There is typically a clear precipitant and symptoms may range from mild cofusion to

coma. Tx includes lactulose, neomycin or rifaximin, and laxatives.

25. Digoxin is a cardiac glycoside whose adverse effects include nause, vomiting, diarrhea, vision

changes, and arrhythmias. Pt’s digoxin levels must be followed closely.

26. CMV retinitis occurs when the CD4 count is less than 50/ul. Pts may be asymtpmatic. The

characterisic fundoscopic findings of yellow-white patches of retinal opacification and

hemorrhages are diagnostic.

27. Vigorous hydration with IV normal saline is the first step in managing acute hypercalcemic

crisis (vomiting, oliguria, anuria, somnolence and eventually, coma.)

28. Suspect Huntington’s disease in a pt with mood disturbances, dementia, chorea (Whithing

movement) and a family history of similar symptoms.

29. Wilson’s disease is the likely diagnosis is a pt younger than 30 years old with unexplained

chronic hepatitis. The presence of low serum ceruloplasmin and increased urinary copper

77

excretion or Kayser Fleischer rings confirms the dx.

30. Contrast-induced nephropathy presents as a transient spike in creatinine with 24 hours of

contrast administration, with a return to normal renal function with 5-7 days. Pts with diabets

and elevated baseline creatine are at especially high risk. Adequate IV hydration with isotonic

bicarbonate and administration of acetylcysteine help to minimize the risk of contrast-induced

nepropathy.

31. Tinea versicolor (Malassezia furfur) is characterized by pale, velvety pin or whitish,

hypopigmented macules that do not tan and do not appear scaly, but scale on scraping.

Topical tx with selenium sulfide lotion and ketoconazole shampoo is recommeded. *

Extremely important!!

32. Rapid developing huperandrogenism with virilization is highly suggestive of an androgen-

secreting neoplasm of the ovary or adrenal. Serum testosterone (ovarian source) and DHEAS

level (adrenal source) are very helpful in delineating the site of excess angrogen production.

33. A bronchodilator response test is used to demonstrate reversibility of airway obstruction. It

help to differentiate between COPD and asthma (significantly improvement in FEV1),

although a subset of pts with COPD may also demonstrate airway reactivity.

34. Hypertension and spontaneous hypokalemia are characteristic for both primary

hyperasdosteronism and renovascular hypertension. The plasma renin activity (PRA) can

distinguish between these two diagnoses; pts with primary hyperasdosteronism have low

PRA, while pts with renovascular hypertension have high PRA.

Bartter’s syndrome

35. Chornic lymphocytic leukemia is almost always seen in elderly pts. The dx features are

marked leukocytosis with predominant lymphocytosis and smudge cells.

36. Spinal cord compression is characterized by signs asdn symptoms of upper motor neuron

dysfunction distal to the site of compression. These include weakness, huperreflexia, and an

extensor plantar response. Cord compression is a medical emergency requiring prompt

diagnosis by a spinal MRI.

37. Diabetic ketoacidosis causes an anion gap metabolic acidosis. Typical symptoms of DKA

include polyuria, polydipsia, nausea, vomiting and abdominal pain that may mimic a surgical

abdomen.

38. The most serious side effect of hydroxychloroquine is retinopathy. Eye examiniations every 6

months are the only form of routine screening required with hydroxychloroquine therapy.

39. Cushing’s syndrome due to extopic ACTH production is characrized by the rapid development

of symptoms, hypokalemic alkalosis, pigmentation and hypertension. Serum cortisol and 24

hour urine cortisol levels are not supressed following the administration of high dose

dexamethasone in most pts with ectopic ACTH syndrome.

40. Anasarca 全身水肿 typically results from organ failure or hypoalcuminemia. In pts with

glomerulonephritis, edema results from the decreased glomerular filtration rate as well as

proteinuria in some cases.

41. Macrovascular hemolysis can cause microcytic anemia in pts with artificial heart valves or

severely calcified aortic valves.

42. Normal pressure hydrocephalus is characterized by the triad of gait disturbance, demantia and

urinary incontinence. Lumbar puncture reveals the normal CSF pressures, and MRI shows the

enlarged ventricles.

78

43. Chronic GERD and Barrett’s esophagus are risk factors for adenocarcinoma of the esophagus.

The major risk factors for squamous cell cancer of the esophagus are smoking and alcohol.

44. Paget’s disease of the bone is characterized by increased bone remodeling and abnormal

osteoid formation. It may be asymptomatic or accompanied by pain and deafness. Alkaline

phosphatase will be elevated but serum calcium, phosphorus, and other liver function tests

will be normal.

12/30/09

1. Pericardial effusions will appear as an enlarged, ‘water bottle’ shaped cardiac silhouette on

CXR. Physical exam findings with pericardial effusions without cardiac tamponade include

deminished heart sounds on auscultion and a difficult to palpate point of the maximal impulse.

Pts has a pericardial effusion may not have cardica tamponade.

Pulsus bisferiens, or biphasis pulse, refers to two strong systolic impulses with a midsystoli dip. It

can be seen in aortic regurgitation with or without aortic stenosis and in hypertension obstructive

cardiomyopathy.

2. Age-dependent idiopathic sclerocalcific changes are the most frequent cause of isolated aortic

stenosis in elderly pts. These changes are common and usually have minimal hemodynamic

significance, but sometimes may be severe.

3. Microangiopathic hemolytic anemias are disorders in which intravascular fragmentation of

red blood cells occurs. One cause is shearing of RBCs on artificial heart valve. Schistocytes

are the classic finding on peripheral blood smear.

4. Systolic heart failure is characterized by depressed cardiac index (CI) accompanied by

inceased total peripheral resistance (TPR) and left ventricular end-diastolic volume (LVEDV).

5. Constrictive pericarditis results in pericardial fibrosis and diastolic dysfunction. Pts presents

with sings of decreased cardiac output (fatigue, dyspnea on exertion, muscle wasting) and

venous overload (elevated JVP, positive Kussmaul’s sign, pedia edema). Sharp ‘X’ and ‘Y’

descents on central venous tracing are characteristic of constrictive pericarditis as is the

preence of a pericardial knock (early heart sound after S2).

6. Massive pulmonary embolism can result in pulmonary hypertension, evidence by elevated

right atrial and pulmonary artery pressures. While the right heart is strained by massive PE,

the left heart is not. Massive PE may also result in shock. A right atrial pressure >10 and

pulmonary artery systolic pressure of 40 are the criteria used to diagnose massive embolism.

7. Rhadomyolysis should be suspected in the following situations: 1) Presence of risk factors

such us alcoholism, cocaine use, and electrolyte abnormalities (eg. hypokalemia,

hypophosphatemia) 2) Disproportionate elevation of creatinine as compared with BUN. 3)

Urine dipstick positive for blood but no RBC on microscopic examination. The underlying

pathology is acute tubular necrosis. Serun CK should be measured in suspected patients. The

tx is aggressive iv hydration and alkalinization of urine, In some cases, forced diuresis with

mannitol may be required.

8. ARDS is a form of noncardiogenic pulmonary edema caused by inflammatory damage to

alveoli. Sepsis, severe bleeding, trauma, and toxic injury may result in ARDS. Criteria for

diagnosing ARDS include: acute dyspnea, an inciting condition, bilateral infiltrates on CXR,

PCWP<18, and PaO2/FiO2 ratio<200.

9. The murmur of aortic regurgitation is typically decribed as a diastolic decrescendo murmur at

79

the left third intercostal apace. Medical therapy consists of afterload reduction with calcium

channel blockers or ACE inhitors.

10. Toxic epidemal necrolysis is a severe mucocutaneous exfoliative disease. It is characterized

by an erythematous morbilliform eruption that rapidly evolves into exfoliation of the skin.

Positive Nikolsky’s sign

11. Folate and cobalamin deficiency both result in megaloblastic, macrocytic anemia. Folate

supplementation can improve the anemia of either disorder but will not improve the

neurologic changes of cobalamin deficiency. It is important to distinguish between the two

prior to initiating therapy.

12. Lung consolidation causes dullness to percussion, increased fremitus, bronchophony,

egophony, and whispered pectoriloquy on physical exam.

13. Elderly pts with poor oral intake living in nursing homes and taking medications such

NSAIDs, ACEIs and diuretics (blunt the renal response to low intravascular volume.) should

be suspected of having prerenal azotemia due to intravascular volume depletion and poor

renal perfusion.

14. Low serum iron, ferritin, and increase TIBC indicates iron deficiency anemia.

15. Aspirin sensitivity syndrome is believed to be a pseudo-allergic reaction. It results from

aspirin-induced prostaglandin/leukotriene misbalance in susceptible individuals. Tx includes

avoidance of NSAIDs and the use of leukotriene receptor antagonists (drug of choice).

Extremely high yield!!

16. Mucormycosis requires aggrassive surgical debridement plus early systemic chemotherapy

with amphotericin B.

17. Pulmonary TB presents with productive cough, hemoptysis, and recurrent fevers. Weight loss

may also be present. Imaging most commonly reveals upper lobe cavity lesions (CT picture).

TB must be suspected in pts from endemic regions with subacute or chornic pulmonary

complaints.

18. Acute venous thromboemboli should be treated with a combination of heparin and warfarin

initially. After 5 days the heparin may be discontinued if the pt’s INR is therapeutic (2-3).

Warfarin should be continued for at least 6 months in most pts with a first-time clot. In

patients with a second episode of clot, lifetime anticoagulation is necessary.

19. GVHD is caused by recognition of host major and minor HLA-antigens by donor T-cell and

consequent cell-mediated immune response. The organs that are typically affected include the

skin, intesine and liver.

20. Pulmonary emboli classically present with sudden-onset pleuritic chest pain, cough, dyspnea,

and hemoptysis. Chest CT showing wedge-shaped infarction is virtually pathognomonic.

21. Unacceptability bias occurs when participants purposely give desirable response which lead to

the underestimation of risk factors.

Admission rate bias refers to a distortion in risk ratio due to different hospital’s admission of

certain cases. For instance, pts with cardiac diseases may prefer to be admitted to a particular

hospital.

22. Asbestos exposure increases the risk of malignancy, pulmonary fibrosis and pleural plaques.

In terms of malignancies, both bronchogenic carcinoma and mesothelioma are linked to

asbestos exposure, with bronchogenic cacinoma being the MCC.

23. Diastolic dysfunction refers to impaired ventricular filling due to poor myocardial relaxation

80

or diminished ventricular compliance. It is usually due to hypertension and is treated with

diuretics and antihypertension. In severe cases, diastolic dysfunction can cause

decompensated heart failure.

High-output heart failure refers to supranormal vantricular function that still cannot meet the

body’s metabolic demands. Causes include anemia, hyperthyroidism, beriberi, Paget’s disease, and

arteriovenous fistulas.

24. Factor V leiden is the most common inherited disorder causing hypercoagulability and

predisposition to thromboses, especially DVT of lower extremities. Factor V Leiden is the

result of point mutation in a gene coding for the coagulation factor V. As a result of this

mutation, Factor V becomes resistant to inactivation by protein C, an important

counterbalance factor in hemostatic cascade. Other disorders causing inherited predisposition

to thrombosis-like protein C deficiency, protein S dificiency, antithrombin III deficiency, and

plasminogen disorders are quite rare.

25. Intravenous nafcillin or cefazolin is the tx of choice for cellulitis with systemic signs. Many

cases of cellulitis are due to Staphylococcus aureus. For this reason, IV aqueous crystalline

penicillin G (almost 99% resistance) is not an effective tx for cellulitis.

26. Oligoclonal bands are present in 85-90% of cases of multiple sclerosis, CSF pressure, protein

and cell count are grossly normal. Elevated immunoglobulin levels may be found in other

diseases such as neuropathies, chronic central nerivous system infections or viral syndromes.

The presence of oligoclonal bands is not considered dx for MS.

27. Post TURP (transurethral resection of the prostate) hyponatremia is caused by absorption of

free water from the nonconductive irrigation solutions (containing glycine, mannitol, or

sorbitol to flush) used during the procedure. The decrese in sodium concentration is rapid and

should be corrected quickly if the patient is symptomatic.

28. TSH-secreting pituitary adenoma leads to excessive TSH levels, which stimulate the thyroid

gland and thereby lead to a hyperthyroid state. It is biochemically characterized by elevated

circulating thyroid hormone levels with inappropriately normal or elevated TSH levels. Most

of the TSH-secresting pituitary adenomas are macroadenomas. Pts with this condition

typically have goiter due to the effect of TSH on the growth of the thyroid follicles. Pts don’t

have extrathyroidal manifestation of Grave’s disease, such as infiltrative ophthalmopathy and

pretibial myxedema.

29. Creutzfeldt-Jakob disease is characterized by repidly progressive dementia, myoclonus and

sharp, triphasis, synchromous discharges on EEG. This spongiform encephalopathy is caused

by a slow virus (prion).

30. Polycythemia Vera (PCV) is distinguished from the other kind of myeloproliferative disease

(MPD) by the remarkable increase in RBC mass and total blood volume. A typical pts is an

old plethoric 多血症的 male who may complain of prutitus after bathing. Symptoms such as

headache, dizziness, and parethesias are frequently related to hyperviscosity. Both thrombosis

and bleeding can occur due to elevated platelet count and impaired platelet function.

Reversible moderate hypertension frequently occurs as a result of expanded blood volume.

They also have granulocytosis and thrombocytosis on peripheral blood smear, and

aplenomegaly. Bone marrow is virtually always hypercullular. There is an elevated leukocyte

alkaline phosphatase, normal oxygen saturation, and low erythropoetin level. A urinary

erythropoetin assay reveals the absence of measurable erythropoetin in the urine.

81

31. Amyloidosis can result in a restrictive cardiomyopathy with thickened ventricular walls and

preserved ventricular dimensions, as well as involvement of the liver (inhibit the synthesis of

coagulation factors—increased bruisability) and kidneys (proteinuria). Alcohol causes a

dilated cardiomyopathy. Hemochromatosis, Sarcoidosis—restrictive cardiomyopathy.

32. Wegener’s granulomatosis is defined clinically as the triad of systemic vasculitis, upper and

lower airway granulomatous inflammtion, and glomerulonephritis. Nasal cartilage destruction

and vasculitic cutaneous lesions (tender nodules, palpable purpura, ulcerations) are common

external manifestation.

33. Asymptomatic gallstones should not be treated. Laparoscopic cholecystectomy is the tx of

choice of symptomatic gallstone disease.

34. Invasive aspergillosis occurs in immunocompromised pts, who may present with fever, cough,

dyspnea or hemoptysis. CXR may show cavity lesions, and CT scan shows pulmonary

nodules with a halo sign or lesions with an air crescent.

35. CSF findings of elevated white count, elevated protein, and decreased glucose levels are

suggestive of bacterial meningitis. Typical skin lesions during meningococcal infection are

petechiae and purpura.

36. Imatinib has changed the diagnosis of pts with chronic myelogenous leukemia. It is a tyrosine

kinase inhibitor and works by blocking signals within cancer cells and preventing a series of

chemical reactions that cause the cancer cells to grow and divide.

37. 3 tx options for myasthenia gravis: orall anticholinesterase is usually the initial tx of choice

(pyridostigmine or neostigmine, not edrophonium—for dx). Atropine can be used to prevent

muscarinic side effects of anticholinesterase therapy in pts with myasthenia gravis.

Anticholinesterases provide symptomatic benefit, but do not induce remission.

Immunouppressive agents and thymectomy may induce remission. All pts should be evaluate

for a thymectomy. Immunouppressive agents that are used in the tx of mysthenia gravis

include prednisolone, zazthioprine and cyclosporine. Corticosteroids are used in (1) pts who

are over 60 years of age, and (2) pts who show poor response to anticholinesterase and have

already undergone thymectomy. Plasmapheresis removes acetylcholine receptor antibodies

from the circulation. It is used in seriously ill pts when other treatments are not effective or

C/I. Its effect is transient and cannot be used on a long-ter basis. Other indications of

plasmapheresis include (1) stabilizaion of the pt before thymectomy, and (2) myasthenia

crisis. IV immunoglobulins act by an unknown mechanism, and are used in the same setting

as plasmapheresis.

38. Adrenal insufficincy and adrenal failure are characterized by nonspecific symptoms and signs

including anorexia, fatigue, GI complaints, weight loss, and hypotension. Hyponatremia is the

most common associated electrolyts abnormality. Hyperkalemia is also common.

39. Pica refers to an appetite for nonnutritive substances, such as ice, clay, dirt, and paper

products. Pica, esp.for ice (Pagophagia), can be indicative of iron deficiency (in old person,

chronic bleeding). Pica may also be present as a manifestation of psychiatric diseaseas and

people may eat very odd items, such as light bulbs or hair.

40. Agents used to shift potassium intracellularly include insulin and glucose, sodium

bicarbonate, and beta-2 (albuterol) agonists.

41. Primary polydipsia is a problem of excessive water ingestion. The kidneys continue to work

appropriately, excreting dilute urine. The dx of primary polydipsia is confirmed by

82

withholding water from the pt and monitoring the urine. Increases in urine osmolality and

specific gravity confirm the dx. Primary polydipsia is relatively common among pts with

schizophrenia. Simutaneous administration of phenothiazines to schizophrenic pts with

primary polydipsia can worsed the hyponatremia because dry mouth is a side effect that

further stimulates thirst. Carbamazepine intensifies the kidney’s response to ADH, causing

water retention, hyponatremia, and inappropriatedly concentrated urine, as in SIADH.

42. S. aureus pneumonia is a serious complication of influenze pneumonia. It can occur in both

children and adults. Pts must be treated with anti-staph antibiotics when post-influenza

pneumonia is suspected.

43. The median is the value that is located in the middle of a datasheet. It divides the right half of

the data from the left half. If the number of observation is even, find the middle two values,

add them together, and divide by two.

44. Steven Johnson syndrome (erythema multiforme major) is characterized by the sudden onset

of target-shaped, mucocutaneous lesions (oral and conjuctival) and systemic signs of toxicity.

The pathology involves immune complex mediated hypersensitivity. The most commonly

involved drugs are sulfonamides, NSAIDs, and phentoin. Tx is similar to burns—primarily

supportive, with regular assessment of hemodynamic stability, fluid status, and pain control.

Erythema multifome minor (under a spectrum of disease with ‘major’) is characterized by

sudden onset erythematous rash and target lesions, and usually occurs after a herpes simplex

infection. Mucosal involvement is rare, and systemic symptoms are not so severe as in

‘major’.

1/1/10

1. Suspect pseudodementia in elderly pts with dementia in the setting of severe depression.

Antidepressants are the tx of choice (reversible with tx). Donepezil is an acetyle-

cholinesterase inhibitor used in the tx of Alzheimer’s dementia. A good distinguishing factor

to remember is that pts with pseudodementia seem significantly concerned about their

impaired memory, whereas, pts with Alzheimer’s dementia seem relatively unconcerned.

2. The 2 initial tests, which are recommeded in all the pts with possible benign prostatic

hyperplasia, are serum creatinine and urinalysis. Such symptoms are not always due to BPH,

other diseases: bladder cancer, prostate cancer, urethral stricture, neurogenic bladder, and

urinary tract infection. Urinalysis helps to rule out urinary tract infection and serum creatinine

detects renal insufficiency that might due to bladder outlet obstruction. Serum creatinine is a

better index of renal function than blood urea nitrogen and therefore is the preferred test.

3. Paget’s disease presents as a persistent dermatitis of the nipple. The lesion is red, oozing,

crusted, and often unresponsive to topical steroids and anbiotics. Biopsy shows malignant

cells that invade the epidermis and have abundant pale-staining cytoplasm surrounding a

hyperchromatic nucleus with prominetn nucleoli.

4. The 3 cardinal signs of Parkinson disease (accumulation of alpha-synuclein within the

neurons of the substantia nigra pars compacta) are rest tremor, rigidity, and bradykinesia. The

presence of at least two of these signs on physical examination is gounds for a clinica

diagnosis. To date, there are no imaging or lab tests that can be used to confirm the dx with

any greater accuracy than physical exam.

5. Women who were treated for cervical intraepithelial neoplasia II/III should have Pap smear

83

with or without colposcopy and endocervical curettage every 6 months until 3 negative results

are obtained. Pts may resume standard screening after that milestone is met (annual Pap smear

testing).

6. The most common renal stones are calcium stones. CT scan of the abdomen without contrast

is the dianostic precedure of choice because it can detect redioopaque (eg. calcium) as well as

radiolucent (eg. uric acid) stones. (KUB or abdominal radiograph is not the best test!). The

dietary recommendations for pts with renal culculi are: 1) ↓ dietray protein and oxalate. 2) ↓ sodium intake (↑sodium intake enhances calcium excretion). 3) ↑fluid intake. 4)↑dietary

calcium. Mega doses of VitC lead to increased formation of oxalate stones.

7. A nail puncture would in an adult resulting in osteomyelitis in most likely due to

Pseudomonas aeruginosa.

8. The best screening test for pts with suspected primary adrenal insufficiency is the cosyntropin

(analogue of ACTH) stimulation test. An increase in serum cortisol levels above 20 mcg/dl

30-60 min after the administration of 250mcg of cosyntropin virtually rules out primary

adrenocortical insufficiency (addison’s disease). Measurement of plasma ACTH is performed

to distinguish between primary and secondary adrenocortical deficiency once the dx of

Addison’s disease has been made. Plasma ACTH levels greater than 50pg/ml indicate primary

adrenocortical deficiency, whereas levels less than 50pg/ml indicate secondary adrenocortical

deficiency.

24hr urinary free cortisol measurment and low-dose dexamethasone suppression test are screeing

tests ofr Cushing syndrome.

9. TCA overdose is characterized by CNS depression, hypotension, and other anticholinergic

effects including dilated pupils, hyperthermia and intesitnal ileus. It can also cause QRS

prolongation on EKG, leaving the pt susceptible to ventricular arrhythmias. Pts suspected of

TCA overdose should first undergo the ABC’s and sodium bicarbonate should then be

administrated to improve blood pressure, shorten the QRS interval, and prevent arrhythmia.

10. Amoxicillin-clavulanate is the antibiotic of choice for prophylaxis/treatment of infections

caused by a human bite (polymicrobial—G+, G-, anaerobes). This is also the drug of choice

of dog bites. Clindamycin is effective against G+ and anaerobes, typically used for treating

lung abscesses and infections of the female genital tract. Ciproflaxacin does not have good

coverage against anaerobes. If the pt were allergic to penicillin, a combination of

ciprofloxacin and clindamycin would be a possible regimen.

11. If a pt presents with priapism, always check his medications first. This symptom is often drug-

induced. The most common drug that causes Priapism is prazosin, although for the boards, it

is important to remember the association of priapism with trazodone. The common causes of

priapism: 1) sickle cell disease and leukemia—usually in children or adolescents. 2) Perineal

or genital trauma—results in laceration of the cavernous artery. 3) Neurogenic lesions—such

as spinal cord injury, cauda equina compression, etc. 4) medications.

12. Proctosigmoidoscopy with biopsy establishes the dx of ulcerative colitis. One of its

advantages is that this dx test can be performed in acute settings. Fulminant colitis is a serious

complication of ulcerative colitis; plain radiographs can usually reveal the distended colon.

13. Brainstem lesons typically involve the cranial nerves and sensory loss of one half of the face

and the contralateral half of the body. Lesions of the thalamus or cortex are associated with

sensory loss of one half of the face and the same half of the body. Cortical lesions are

84

associated with findings specific to the cerebral cortex (eg. aphasia, neglect, abnormal

graphesthesia or stereognosis).

Medial medullary syndrome is typically caused by an occlusion of the vertebral artery or one of its

branches. As a result, the pt will demonstrate contralateral paralysis of the arm, leg, contralateral

loss of tactile, vibratory, and position sense, and tongue deviation to the injured side.

Lateral medulla—Wallenberg syndrome. Ipilateral Horner syndrome, loss of pain and temperature

sensation of the face, weakness of the palate, pharynx, and vocal cords; and cerebellar ataxia.

There is also loss of pain and temperature sensation on the contralateral side of the body.

Medial pons—medial midpontine syndrome. Common characteristics include ipsilateral limb

ataxia and contralateral eye deviation and paralysis of the face, arm, and leg; impairment of touch

and position sense is variable.

Lateral pons—lateral midpontine syndrome, which results in impaired sensory and motor function

of CNV with accompanying limb ataxia.

Central midbrain—a number of different syndromes, including Weber’s syndrome, Benedikt’s

syndrome, Claude’s syndrome, Nothnagel’s syndrome, and Parinaud’s syndrome. Oculomotor

paresis and othe abnormalities of CNIII function are common to all of thers syndromes, and

cerebellar ataxia and contralateral hemiplegia amy be noted as well.

14. Pts with acute pyelonephtitis susally present with fever, chills, nausea, vomiting and flank or

suprapubic pain. Physical examination shows costovertebral angle tenderness. Hospitalization

and iv antiboitics are indicated in complicated acute phylonephritis.

15. Pneumococcal vaccine is recommeded for all HIV-infected pts whose CD4 count is above 200

cells/ml. Annual influenza vaccination is also recommoded for all HIV-infected pts.

16. In any pt, the PH and PaCO2 are the 2 lab values that provide the test picture of acid-base

status; the HCO3- can be calculated form these values using the Hendeson-Hasselbalch

equation. PH=6.1+log {(0.03xPaCO2)/[HCO3-]}

17. Acute pancreatitis classically causes nausea, vomiting and epigastric pain that radiates to the

back. The pt’s clinical history, physical exam findings, serum lipase and amylase levels all

help to make the diagnosis. Alcohol abuse is the MCC of acute pancreatitis in men and

gallstones are the MCC in women. Potential complications of acute pancreatitis include

exudative left-sided pleural effusions when the amylase concentration is high, abdominal

compartment syndrome, intraabdominal hemorrhage, shock, diabetes, pancreatic pseudocyst

formation, and abdominal pseudoaneurysm.

18. Dementia with lewy bodies causes alterations in alterness, visual hallucintions, and

extrapyramidal symptoms. At autopsy, ‘lewy bodies’, or eosinophilic nuclear inclusions

representing accumulations of alpha-synuclein protein, may be seen in neurons of the

substantia nigra, locus ceruleus, dorsal raphe, and substantia innominata. Lewy bodies are a

pathologic finding also present in Parkinson’s disease. The key distinction between these two

conditions is the early appearance of dementia in Lewy body disease and of motor symptoms

in Parkinson.

19. Proteus species produce urease, which makes the urine alkaline (PH 8.5). This infection is

particularly common in pts who live in long-term care facilities and have chronic indwelling

catheters. Candida, Psudomonas, and Klebsiella infections are also common in pts with

chronic indwelling catheters but they do not produce alkaline urine. E. Coli ist he MCC of

UTIs, but it does not produce urease and thus does not alter the normal acidic pH of urine.

85

20. Suspect trichinellosis in a pt who present with GI complaints followed by the characteristic

triad of periorbital edema, myositis, and eosinophilia. Other clues include subungual splinter

hemorrhage and conjunctival or retinal hemorrhages.

21. A non-productive cough is observed in 5-20% of pts receiving ACE inhibitors. It is caused by

an accumulation of kinins, and possibly by the activation of the arachidonic acid pathway.

22. Alveolar hypoventilation, as in a COPD exacerbation, can cause CO2 retention and a resultant

lethargy, somnolence, seizures, coma, and even death.

23. Cholesterol screening in men above age 35 and in women >45 when there is average risk for

coronary heart disease. Screening should begin at younger ages (men 20-35, women 20-45)

for individuals at increased risk.

24. Recurrent bacterial infections in an adult pt may indicate a humoral immunity defect.

Quantitative measurement of serum Ig levels helps to establish the diagnosis.

25. The overall incidence of vertical transmission of HCV is approximately 2-5%. All pts

including pregnant pts, with chronic hepatitis C should receive vaccinations againt HAV, HBV

if not already immune. * Extremely important!

26. PE cause dyspnea, tachypnea, pleuritic chest pain, and tachycardia. While there are suggestive

CXR is normal in most cases. The most common ECG finding is sinus tachycardia.

27. Most pts who are initially controlled with one antidiabetic medication eventually require the

addition of more antidiabetic drugs to achieve optimal glycemic control. Combining

antidiabetic agents with different mechanisms of action is typically done to achieve better

glycemic control. Metformin is the only antidiabetic drug that causes some weight loss.

28. Atrial myxoma is the most common primary intracardiac tumor, and is usually located in the

left atrium. It can casue systemic symtoms such as fever and weight loss, neurologic

symptoms sue to tumor embolization, and presents as a mass on echocariography.

Myxomatous valve degeneration –mitral valve prolapse.

29. Dipyridamole can be used during myocardial perfusion scanning to reveal the areas of

restricted myocardial perfusion. The redictribtution of the coronary blood flow to ‘non-

diseased’ segments induced by this drug is called coronary steal phenomenon. Dipyridamol

and adenosine are coronary vasodilators. Infusion of these substances in pts without CAD,

increases coronary blood flow 3-5 times above the baseline levels. However, in pts with CAD,

the diseased vessels distal to the obstruction are already maximally dilated, and their ability to

increase myocardial perfusion is limited, therefore, redistribution of coranary blood flow to

‘non-diseased’ areas occurs, and the perfusion of ‘diseased’ segments diminishes.

30. Newborns of mothers with active HBV infection should be passively immunized at birth with

HBIG followed by active immunization with recombinant HBV vaccine.

31. Postictal lactic acidosis commonly occurs following a tonic-clonic seizure, It is a transient

anion gap metabolic acidosis that resolves without tx with 60-90 mins following resolution of

seizure activity.

32. Platelet dysfunction is the MCC of abnormal hemostasis in pts with CRF. PT, PTT and

platelet count are normal. BT is prolonged. DDAVP is usually the tx of choice, if needed.

DDAVP increases the release of factor VIII:VWF multimers from endothelial storage sites.

Platelet transfusion in not indicated because the transfused platelets quickly become inactive.

* Extremely high yield!

33. Chloroquine-resistant plasmodium falciparum is particularly common in Sub-Saharan Africa

86

and the India subcontinent (eg. India, Pakisan, and Bangldesh). Mefloquine is the drug of

chioce for chemoprophylaxis against chloroquine-resistant malaria. The use of primaquine

(both for prophylaxis and tx) is indicated in settings where malaria is due to Plasmodium

vavax or plasmodium ovale; there organisms cause persistent infection in the liver.

34. Acute confusion, extreme huperthermia >40.5 (105F), tachycardia, and coagulopathic

bleeding after heavy work under direct sunlight are most likely due to exertional heat stroke.

Malignant hyperthemia affects genetically susceptible individuals during anesthesia involving

agents like halothane and succinylcholine.

A hypothalamic stroke may disrupt normal neural thermoregulatory mechanisms and result in

neurogenic fever (NF). In NF the hypothalamic set-point becomes pathologically elevated.

35. Cystoscopy can cause an enterococcal bacteremia in pts with chronic genitourinary infections.

Pts with underlying valvular lesions are at risk for endocarditis in this setting.

36. Antiphospholipid antibody syndrome (VDRL positive FTA-ABS Negative) places a pregnant

pt at increased risk of first and second trimester abortin. The use of heparin and aspirin

reduces this risk. Warfarin is not used due to its teratogenicity. Type I: false-positive syphilis

serology; Type II is lupus anticoagulant, and it falsely elevates the APTT level; Type III is

anticardiolipin antibody.

37. The initial tx of both acute and chronic anal fissures includes dietary modification (eg. high-

fiber diet and large amounts of fluids), a stool sofener, and a local anesthetic. Colonoscopy in

not needed in the pt as the bright red blood on defecation is clearly due to the fissure. His

stool caliber is also normal. However, if the fissue heals and the pt still has blood in the stool,

then colonoscopy is warranted. * Extremely high yield!!

38. Degenerative joint disease (osteoarthritis) presents in adults over age-40 with indolent

progressive anterior hip pain worsened by activity and relieved by rest. The hip is not tender

and systmic symptoms are absent. Short-lived morning stiffness may occur.

39. Macular degeneration is the most common cause of blindness in industrialized nations.

Activities that require fine visual acuity are usually the first affected, and pt with this

condition may report that straight grid lines appear curved.

40. The first line of medical therapy for hypertrophic cardiomyopathy is either beta-blockers or a

cardiac acting CCB such diltiazem since they promote diastolic relaxation. Amlodipine is a

peripherally acting CCB, so it does not have the diastole-prolong effects of a drug such as

diltizem. It acts primarily as an arterial vasodilatior.

41. Emtamoeba histolytica is a protozoan, which can cause amebic liver abscess. Remember the

Mexico trip.

42. Childre with cystic fibrosis should adhere to routine vaccination schedules with a few

additions. They should receive yearly influenza vaccinations and may require pneumococcal

booster shots. In pts with egg allergies, the influenza and yellow fever vaccines should not be

given, and MMP should be given with caution.

43. Acyclovir can cause crystalline nephropathy if adequate hydration is not also provided.

44. Norcadiosis is dx by the presence of crooked, branching, beaded, G+, partially acid-fast

filaments on microscopy. Trimethoprim-sulfamethoxazole is the tx of chioce. Picture!

1/1/10

1. A mobile cavitary mass in the lung, which presents with intermittent hemoptysis, is usually

87

indicative of asperggilloma. The CXR may demonstrate a crescent radiolucency next to a

rounded mass. Cavitary lesions may form because of destruction of the underlying pulmonary

parenchema, and debris and hyphae may coalesce and form a fungus ball, which lies free in

the cavity and moves around with position change.

2. Intermittent catheterization is an effctive measure to reduce the risk of UTI in pts with

neurogenic bladder.

3. In heat stroke the temperature is usually above 40.5C (105F). Heat stroke results from an

insufficient evaporative cooling machanism. Therefore, tx involves induction of evaporative

cooling to reverse hyperthermia.

4. When isolated proteinuria occurs, the evaluation of the pt should begin by testing the urine on

at least two other occasions.

5. Orthostatic hypotension is characterized by a drop in blood pressure after standing and is

typically preceded by a lighthearded or presyncopal sensation.

6. Atypical pneumonia presents with a more indolent course and higher incidence of

extrapulmonary manifestations than pyogenic pneumonia. CXR findings may be out of

proportion to findings on physical exam. Erythema multiforme is a characteristic

extrapulmonary manifestations of Mycoplasma pneumonia, the MCC of atypical pneumonia.

7. ERCP is the investigation of choice for pts with recurrent pancreatitis with no obvious cause

—ductal abnormalities including pancreatic divisum, choledochal cyst, and common bile duct

stones. It would also allow for gall bladder aspiration of bile to look for biliary crystals or

microlithiasis.

8. Chronic liver disease or cirrhosis from almost any cause is a risk factor for hepatocellular

cancer (hepatoma). Hepatocellular cancer is responsible for 30% of deaths in pts with

hemochromatosis.

9. Ischemic cardiac pain can sometimes be mistaken for epigastric pain, but should remain high

on the differential, especially in the setting of symptoms worsened with exertion. An exercise

strss test without imaging is the most reasonable first step if the baseline resting EKG in

normal (pt’s heart burn has exertional nature).

10. Pts presenting to the ER with confusion or coma should receive empiric tx with dextrose,

oxygen, naloxone, and thiamine. Thiamine will help to prevent or reverse Wernick’s

encephalopathy in alcoholics, and should be administered bofore dextrose.

11. Aortic dissections are life-threatening emergencies that classically present with tearing chest

pain radiating to the back. Sequelae include cardiac tamponade, acute aortic regurgitation,

stroke, and renal failure.

12. After 48-72 hours of parenteral therapy for uncomplicated pyelonephritis, the pt can be

usually switched to an oral agent. Oral therapy (TMP/SMX) is more convenient and less

expensive; if the results of antibiotic susceptibilit testing are known, the appropriate antibiotic

can be easily chosen.

13. The MCC of pulmonary complications in pts with systemic sclerosis is interstitial fibrosis.

14. Herpes zoster is a painful reactivation of the varicella zonster virus that causes a vesicular

rash in a dermatomal distribution. Acyclovir and other antivirals may be used to decrease both

the duration of disease and the incidence of post-herpetic neuralgia.

Topical 5-FU—actinic keratoses, superficial basal cell carcinoma, Bowen’s disease, and solar

keratosis.

88

Topical cortisosteroids—numerous inflammatory skin conditions.

Mupirocin—superficial skin infection and to eradicate methicillin-resistant Staph aureus

colonization.

Fluconazole—superficial and systemic fungal infections.

15. An aortic aneurysm of the descending aorta may appear on a CXR as a well-circumscribed

lesion. These aneurysms are due to atherosclerosis.

16. Cocaine intoxication is a leading cause of myocardial infarctions in young people, and

myocardial ischemia associated with cocaine use should be treated initially with

benzodiazepine, aspirin, and nitrates. Elevated blood pressure may be controlled with alpha-

blockers, but beta-blockers are C/I.

17. Obesity hypoventilation syndrome is a long-term consequence of severe obesity and untreated

obstructive sleep apnea. It causes chronic hypercapnic/hypoxic respiratory failure, secondary

erythrocytosis, pulmonary hypertension, and cor pulmonale. In a compensatory effort to

maintain a normal PH, the kidney increases bicarbonate retention and decrease chloride

reabsorption.

18. ARDS is a form of noncardiogic pulmonary edema caused by inflammatory damage to the

alvioli. Criteria for the dx include: acute onset of respiratory distress in the setting of a

predisposing condition (sepsis, pneumonia, etc.), PaO2/FiO2 ratio<200, bilateral infiltrates on

CXR, and a normal PCWP.

19. All COPD pt with PaO2<55mmHg or SaO2 <88% are candidates for long-term home oxygen

tx. Pt with signs of pulmonary hypertension or hemotocrit >55% should be start on home

oxygen when the PaO2 < 60mmHg.

20. SIADH is characterized by euvolemic hypoatremia, decreased serum osmolarity, elevated

urine osmolarity (often greater than serum osmolarity), increase urine sodium concentration,

and failure to correct with normal saline infusion.Serum osmolarity Extracellular

volumeUrine findings Causes of hupomatremia

Normal - - Hyperproteinemiahyperlipidemia

High(>295 mOsm/kg) HyperglycemiaExogenous solutes

(radiocontrast, mannitol)Low(<280 mOsm/kg) Hypovolemic UNa < 10 mEq/L Nonrenal salt loss

Dehydration

Vomiting

DiarrhreaUNa >20 mEq/L Renal salt loss

Diuretics

ACEIs

Mineralocoricoid deficiencyEuvolemic UNa >20 mEq/L

Uosm < 300Psychogeic polydipsiaBeer potomania

UNa >20 mEq/LUosm >300

SIADH

Hypervolemic CHF

89

Hepatic failure

Nephrotic syndrome

21. Autoimmune platelet destruction is a common cause of thrombocytopenia and should be

suspected in patients with ecchymoses, petechiae and mucosal bleeding without signs or

symptoms of TTP/HUS, pancytopenia, marrow failure or splenomegaly.

22. Severe symptomatic hyponatremia requires ts with hypertonic (3%) saline with or without

furosemide. Rapid correction of serum sodium levels can lead to central pontine myelinosis.

Mx of hyponatremia due to SIADH:

Mild (asymptomatic with sodium 120-130 meq/L) = Fluid restriction

Moderate (asymptomatic with sodium 110-120 meq/L) = Loop diuretic + normal saline

Severe (symptomatic) = Hypertonic saline

23. Pick’s disease is a fronto-temporal dementia that is characterized by personality changes,

compulsive behavior, and impaired memory.

24. Whipple’s disease is a multi-system illness characterized by arthralgias, weight loss, fever,

diarrhea and abdominal pain. PAS-positive material in the lamina propria of the small

intestine is a classical biopsy finding.

25. Aldosterone deficiency, which occurs in Addison disease, causes a non-anion gap,

hyperkalemic, hyponatremic metabolic acidosis. TB is a common cause of primary adrenal

failure in geographic regions where this organism is endemic (CXR shows a right upper lobe

cavity).

26. Injection drug users are prone to get tricuspid endocarditis caused by S. aureus. Fragments of

the vegetation can embolize to the lungs, causing the characteristic nodules infiltrate with

cavitation.

27. Measles (paramyxovirus) is characterized by a prodrome of cough, coryza, conjunctivitis,

followed by Koplik’s spots and maculopapular rash initially appearing on the face.

28. Always suspect sickle cell trait in a young black male who presents with painless hematuria—

papillary ischemia-necrosis.

29. Lumbosacral stain is the MCC of acute back pain. The typical clinical scenario includes acute

onset of back pain after physical exertion absence of radiation, presence of paravertebral

tenderness, negative straight-leg raising test, and normal neurologic exam.

30. Inhale albuterol and systemic steroids are appropriate tx for acute asthma attacks. Pts on high

doses of beta-2 agonists may develop hypokalemia (by driving potassium into cells), which

may present with muscle weakness, arrhythmias and EKG abnormalities. Other common side

effects of beta-2 agonists include tremor, palpitations and headache.

31. Impairment of daily functioning is essential in distinguishign between demantia and normal

changes of aging. Pts with dementia have functional impairments. Normal aging DementiaIndependence in daily activities perserved Person becomes dependent on others for ADLPerson complains of memory loss but can provide details about incidents of forgetfulness

May complain of memory problems only if asked, unable to remember specific instances where memory loss was noticed (by others)

Patient is more concerned about memory loss Close family members are more concerned about memory loss

Recent memory for important events and conversationd intact

Notable decline in memory for recent important events and conversations

90

Occasional word-finding difficulties (expressive aphasia)

Frequent word-finding difficulty and substitutions, also some receptive aphasia

Does not get lost in familiar territory, may have to pause briefly to reorient

Can get lost for hours in familiar territory while walking or driving

Able to operate common appliances Becomes unable to operate common appliancesMaintain previous interpersonal social skills Shows loss of interest in social activities and

inappropriate behaviorNormal performance on mental status examination

Abnormal mental status exam

32. Unrecognized bowel ischemia is one of the common causes of lactic acidosis (results from the

production of lactate as an end product of anaerobic metabolism in the ischemic tissue) in pts

with severe atherosclerotic disease or atrial fibrillation.

33. Wilson’s disease is diagnosed by decreased serum ceruloplasmin, increased urinary copper,

and Kayser-Fleischer rings seen on slit lamp exam of the eye. Liver biopsy may resemble

alcoholic—mallory body

34. The persence of antibodies to ds-DNA is highly specific for and confirms the dx of SLE.

35. The indications for aortic valve replacement are: 1) all aymptomatic pts with AS. 2) pts with

severe AS undergoing CABG or other valvular surgery. 3) asymptomatic pts with severe AS

and either poor LV systolic function, LV hypertrophy>15mm, valve area <0.6 cm2 or

abnormal response to exercise. Aortic valvotomy is not nearly as effective as aortic valva

replacement. It should only be considered as a bridge to surgery in the hemodynamically

unstable pt or for poor surgical candidates.

36. Pts with carcinoid syndrome are at risk of developing niacin deficiency, owing to the

increased formation of serotomin from tryptophan. The classic triad of carcinoid syndrome

includes flushing, valvular heart disease, and diarrhea. Carcinoid tumors with hepatic

metastasis may be associated with this syndrome. Isolated tumors without metastasis do not

produce carcinoid syndrome. Niacin deficiency—3 D’s diarrhea, dermatitis and dementia

37. The leukocyte alkaline phosphatase score is high in leukemoid reactions, and usually low in

chornic myeloid leukemia.

38. Echocardiogram is necessary for the definitive dx of AS. Symptomatic AS requires valve

replacement.

39. Long-term cyclophosphamide use is associated with the increased incidence of acute

hemorrhageic cyctitis and bladder carcinoma.

40. Pts with non-inflammatory chronic prostatitis are afebrile and have irritative voiding

symptoms. Expressed prostatic secretions show a normal number of leukocytes and culture of

these secrestions is negative for bacteria.

41. up to 70% of pts with mitral stenosis will develop atrial fibrillation because of the significant

left atrial dilatation.

42. Of the vacterial pathogens know to cause secondary pneumonia complicating a viral upper

respiratory infection, only Staph aureus is associated with necrotizing bronchopneumonia

resulting in pneumatocoeles.

43. Clinical scenario describing a woman with chronic headache who presents with painless

hematuria is typical for anagesic nephropathy. Papillary necrosis is the cause of hematuria.

44. PPV increases with an increase in prevalence of disease in the study population. For NPV, the

inverse is true.

91

1/2/10

1. Nocardiosis in the setting of chronic corticosteroid use for RA. Nocardia asteroides is a G+

partially acid –fast, filamentous aerobe that is found in soil. Nocardia may initially be

confused with TB or Actinomyces infection. The most common symptoms include weight

loss, fever, and night sweats. Pulmonary involvement is characterized by cough productive of

purulent sputum. CXR findings include alveolar infiltrated and nodules, often with cavitation.

Chest wall invasion may be seen. Disseminated disease may involve any organ, but is most

likely to manifest as subcutaneous or brain abscesses. Nocardiosis usually occurs in

immunocompromised pts, including those on chronic corticosteroid therapy. Tx of choice—

trimethoprim-sulfamethoxazole. Penicillin G is the tx of choice for Actinomyces, another

filamentous G+ bacterium. 2 diseases may present similarly but there are several key

difference: 1) Actinomyces is anaerobic, not aerobic; 2) Actinomyces is more likely to cause

cervicofacial disease and sinus tracts than Nocardia; and 3) sulfur granules may be found with

Actinomyces.

2. Unexplained hemolytic anemia and thrombocytopenia in a pt with renal failure and neurologic

symptoms should raise strong suspicious for thrombotic thrombocytopenic purpura-hemolytic

uremic syndrome (TTP-HUS). The presence of fragmentated cells in the peripheral smear

suggested macioangiopathic hemolytic anemia (MAHA), a characteristic finding in TTP-

HUS. Idiopathic TTP-HUS is thought to be due to a deficiency of or autoantibody against a

specific von Willebrand factor-cleaving protease (ADAMTS-13). This causes the

accumulation of large von-Willebrand factor multimers and platelet aggregation. TTP-HUS is

fatal in 80% of cases if appropriate therapy is not promptly insituted, so a high degree of

clinical suspicion is necessary. Plasmapheresis (plasma exchange) is the tx of choice and

should be started as soon as possible. Plasmapheresis removes the offending autoantibodies

and repletes the deficient enzyme. Renal function impairment and peripheral blood smear

schistocytes may persist for several weeks following clinical recovery.

3. Elderly pts with depression often present with memory loss (pseudodementia). This can

mimic other causes of dementia, including Alzheimer’s dementia. CT findings are usually

normal and non-contributory for the differential diagnosis, whereas DST may be of value.

4. Lowering the cut-off point will consequently increase the sensitivity of a test. The true

positives will increase. However, the false positives will have a relatively large increase. This

results in a decrease in the PPV, as well as the FN.

5. In low risk pts (ie. <40 years and non smokers) a solitary pulmonary nodule is not a sign of

immediate alarm. The best approach is to ask for an old x-ray.

6. Carpal tunnel syndrome is the most common mononeuropathy of the upper extremity,

paresthesias of the first three-and-a-half digits and occasionally thenar eminence atrophy are

typical.

7. Pts with RA are at increased risk of developing osteopenia and osteoporosis due to the

combined effects of the disease process itself, glucocorticoid therapy, female sex and

decreased ability to perform weight-bearing exercises that prevent osteoporosis. Osteitis

fibrosis cystica (Von Recklinghausen disease of bone) is a condition in which osteoclastic

resorption of bone leads to replacement with fibrous tissue (brown tumors).

Hyperparathyriodism is the cause. Avascular necrosis is most commonly seen with the use of

92

systemic glucocortcoids and alcohol.

8. BAL is most useful in the evaluation of suspected malignancy and opportunistic infection.

BAL is >90% sensitive and specific for PCP. BAL is of less diagnostic utility in evaluating

other forms of interstitial lung disease, such as interstitial pulmonary fibrosis, sarcoidosis, and

connective tissue diseases.

9. Respiratory alkalosis is characterized by an increased pH and a primary decrease in the

PaCO2. Typically causes include huperventilation due to pneumonia, high altitude or

salicylate intoxication.

10. Hemorrhage is the most common complication of peptic ulcer disease.

11. Systemic sclerosis is a form of scleroderma with widespread organ involvement. GERD, right

heart failure and hypertension result from involvement of the esophagus, pulmonary atreries

and kidneys, respectively. Antinuclear autoantibodies and anti-topoisomerase-I antibodies

would most likely be present.

12. Any pt with an acute, severe disease may have an abnormal thyroid function test. This

condition is called sick euthyroid syndrome, and the most common thyroid hormone pattern in

such pts is a fall in total and free T3 levels with normal T4 and TSH levels. (Low T3

syndrome).

13. Pupillary dilation and blood at the external nares鼻孔 in a pt with chest pain can be a clue to

cocaine-induced vasospasm. ST elevation myocardial infarctions can occur as a consequence

of cocaine use. Cocaine-induced ST elevation myocardial infarction (STEMIs) are treated the

same as classic STEMIs, with PTCA or thrombolysis. Aspirin and nitrates are also

appropriate, but beta-blockers should be avoided. Beta-blocker therapy administered to

cocaine-using individuals allows unopposed alpha agonist activiy that can worsen vasospasm.

CCBs and alpha blockers like phentolamine can also help reduce vasospasm.

14. Alzheimer’s disease is the most common cause of dementia in the western world. It is initally

characterized by memory loss, language difficulties and apraxia, followed by impaired

judgement and personality changes. CT scan shows generalized cortical atrophy.

15. Inactive of pancreatic enzymes by increased production of stomach acid may lead to

malabsorption in pts with Zollinger-Ellison syndrome (multiple duodenal ulcers are typical,

and a jejunal ulcer is almost pathognomic for this condition.).

16. Neutropenia is defined as an absoute neutrophil count (ANC) <1500/ml; this pt’s ANC is 280.

Susceptibility to infection increases when ANC falls below 1000/ml; ability to control

endogenous flora is lost and risk of death is markedly increased when the ANC falls below

500/ml. Fever in a neutrophenic pt is defined as a single temperature reading of greater than

38.3 (100.9F) ir a sustained temperature of greater than 38C (100.4F) over one hour. Bacteria,

fungi and viruses can all cause infection in neutropenia pts. Bacteria infections are the most

common and are frequently caused by endogenous skin or colon flora. Over the past decade,

there has been a shift from G- to G+ bacteria being the most frequent cause of neutropenic

infection. Febrile neutropenia is considered a medical emergency, thus, empiric antibiotics

should be started immediately. Empiric therapy should be broad-spectrum and should cover

Psudomonas aeruginosa. Either monotherapy or combination therapy can be employed.

Monotherapy consists of ceftazidime, imipenem, cefepime or meropenem. Combination

therapy is equally effective, and consists of an aminoglycoside plus an anti-pseudomonal beta-

lactam. Vancomycin is added to the empiric regimen when the pt is hypotensive, has severe

93

mucositis, had evidence of skin or line infection, has a history of colonization with resistant

strains of S.aureus or pneumococcus, or has had recent prophylaxis with fluoroquolones.

Fungal infections occur quite commonly in pts with prolonged neutropenia. When neutropenic

fever persists despite empiric antibacterial therapy, antifungal agents such as amphotericin B

are added to the empiric regimen.

17. The first step in the mx of a pt with heparin-induced thrombocytopenia (HIT) is immediate

cessation of all exposure to heparin, including low-molecular weight heparin. Subsequently,

an alternative means of anticoagulation is usually given, since most pts remain at risk of

thrombosis and require ongoing anticoagulation (eg. those with mechanical heart valves).

Currently, the 2 recommended alternative tx are danaparoid and a direct thrombin inhibitor

(lepirudin, argatroban). The use of warfarin without other anticoagulants should be avoid in

pts with HIT until the platelet count rises above 100,000/ml. Without heparin, warfarin

therapy alone may increase the risk of venous limb gangrene in pts with deep vein thrombosis.

18. Anemia can be due to decreased RBC production, increased RBC destruction, and frank blood

loss. The anemia of lymphoproliferative disorders is due to bone marrow infiltration with

cancerous cells.

19. Disseminated gonococcemia causes high fever, chills, tenosynovitis, migratory polyarthralgias

and a small number of hemorrhagic pustular lesions on the extremities. Routine blood and

pustule cultures are typically negative due to the growth requirements of the organism.Toxic

shock syndrome doesn’t have pustules,

20. Outflow obstruction in hypertrophic cardiomyopathy results from both septal hypertrophy and

systolic anterior motion (SAM) of the mitral valve.

21. If a pt has mild acne (non-inflammatory comedones), topical retinoids are usually tried first.

Oral antiboitics are usually used if papular and inflammatory acne (moderate-to-severe

inflammation) is present. Oral isotretinioin is reseved for pts with nodulocystic and scarring

acne.

22. Chronic HCV is associated with number of extra hepatic complications like 1)

cryoglobulinemia, 2) B cell lymphomas. 3) Plasmacytomas, 4) Autoimmune diseases like

Sjogren’s syndrome and thyroiditis, 5) Lichen planus, 6) Porphyria cutanea tarda, 7)

Idiopathic thrombocytopenic purpura. Some of these complications are immune complex

mediated, however immune complex mediated extrahepatic complications are more common

with HBV than with HCV except for cryoglobulinemia.

23. Chronic corticosteroid use and chronic excessive ingestion of alcohol account for over 90% of

cases of avascular necrosis of bone (osteonecrosis). In the hip, pts present with slowly

progressive anterior hip pain with limitation of range of motion.

24. Always suspect multiple sclerosis (MS) in a female with multiple neurologic presentations

that are interspaced between time periods. Optic neuritis can result in blurring of vision, and is

often associated with retrobulbar pain.

25. Abdominal CT scan is the next diagnostic test when abdominal ultrasound does not explain

cholestatic jaundice.

26. The two most common causes of painless GI bleeding in elderly pt over 65 years of age are

diveriticulosis and angiodysplasia (vascular ectasia, Painless bleeding). There is a well-

defined association between aortic stenosis and angiodysplasia. Another well-recognized

association is renal failure. Ectasia can present thoughout the colon; however, the most often

94

site of bleeding is the cecum or ascending colon. The colonoscopic finding of a cherry red

fern-like pattern of blood vessels that appear to radiate from a central feeding vessel is

diagnostic. Pts with diverticulosis will have colonoscopic findings of multiple diverticuli with

a diverticular bleed. Furthermore, most diverticula are located in the sigmoid colon; therefore,

sigmoidscopy would have shown these. *Extremely high Yield!!

27. Pts with rheumatoid arthritis and those who have joint damage are an increased risk of

developing septic arthritis, particularly with Staph aureus. Inflammatory monoarthritis

presents acutely with a red, swollen joint and motion limited by pain; the differential

diagnosis includes septic arthritis, crystal-induced arthritis or trauma.

28. Damage to the lateral spinothalamic tracts causes contralateral loss of pain and temperature

sensation beginning two levels below the level of the lesion.

29. Hypocalcemia with concordant changes of serum calcium and phosphate levels are usually

caused by vit D deficiency (eg. due to malabsorption in alcoholic). Hypoalbuminemia is a

common cause of decreased serum calcium level, but ionized calcium level remains

unchanged; therefore, no symptoms of hypocalcemia are usually present. Primary

hypoparathyroidism and pseudohypoparathyroidism cause discordant changes in serum

calcium (low) and phosphate levels (high).

30. Loop or thiazide diuretic use is one of the MCC of metabolic alkalosis. Such pts are typically

volume depleted and hypokalemic. Other causes of metabolic alkalosis include

hyperaldosteronism and vomiting.

31. Primary syphilic (chancre) is best diagnosed with dark field microscopy. Serological tests

(RPR & VDRL) are usually negative in primary syphilis but are positive in almost all cases of

secondary syphilis.

32. Seborrheic dermatitis is characterized by waxy scales with underlying erythema on the scalp,

central face, presternal region, interscapular areas, umbilicus and body folds. It may be seen in

association with Parkinsonism or HIV. Lichen simplex chronicus is the result of chronic

scratching, picking or rubbing of the skin. Manifestation include thickened plaques on the

skin with increased skin markings and hyperpigmentation or thickened papules with

excoriated centers.

33. Anterior artery stroke : contralateral motor and /or sensory deficits which are more pronouced

in the lower limb than the upper limb. Other features that may be seen include urinary

incontinence, gait apraxia, primitive reflexes (eg. grasp and sucking), abulia, and paratonic

rigidity.

Middle cerebral artery stroke: contralateral motor and /or sensory deficit (more pronounced in the

upper limb than lower limb) and homonymous hemianopia. If the dominant lobe (left) is involved,

the pt may have aphasia; whereas, if the non-dominant lobe (right) is involved, the pt may have

neglect and/or anosognosia.

Posterior cerebral artery stroke: homonymous hemianopia, alexia without agraphia (dominat

hemisphere), visual hallucinations (calcarine cotex), sensory symptoms (thalamus), third nerve

palsy with pareses of vertical eye movement, and motor deficits (cerebral peduncle, middle brain).

Internal carotid artery generally results in signs and symptoms of middle cerebral artery occlusion,

along with visual symptoms, such as smaurosis fugax (temporary monocular blindness).

Lacunar infarcts are small noncortical infarcts cuased by occlusion of a single penetrating branch

of a large cerebral artery. The associated risk factors are hypertension, DM, or polycythemia.

95

Lacunar infarcts usually do not produce both sensory and motor deficits, and instead have well

recognized syndromes (eg. pure motor hemiparesis, pure sensory stroke, dysarthria-clumsy hand,

ataxic hemiparesis).

34. Glomerular hyperfiltration is the earliest renal abnormality seen in diabetic nephropathy. It is

also the major pathophysiologic mechanism of glomerular injury in these pts. Thickening of

the glomerular basement membrane is the first change that can be quantitated.

35. An increase in serum levels of both insulin and C-peptide and hypoglycemia is diagnostic of

insulinoma. The whipple triad of insulinoma clinically: 1) episodic hypoglycemia, 2) central

nervous system dysfunction temporally related to hypoglycemia (confusion, anxiety, stupor,

convulsions, coma). 3) dramatic reversal of CNS abnormalities by glucose administration.

36. Necrotizing fasciitis is characterized by purpish discoloration of the skin with gangrenous

changes and systemic signs of toxicity. Thorough surgical debridement of all the necrotic

tissues is the most important therapy. Empiric antibiotic therapy should cover aerobic and

anaerobic organisms (ampicillin dn sulbactam plus clindamycin). CT scan is useful in

identifying the involved site, which reveals necrosis, asymmetrical fascial thickening and gas

in the tissues. Thrombophlebitis is characterized by palpable, indurated, cord-like, tender,

subcutaneous venous segments.

37. Progressively decreasing baroreceptor sensitivity and defects in the myocardial response to

this reflex are the main reasons for the increased incidence of orthostatic hypotension in the

elderly.

38. Pregnancy causes a physiological chronic compensated respiratory alkalosis due to the

stimularoty effect of progesterone on the medullary respiratory center.

39. Mitral stenosis classically presents during pregnancy. It is most often due to rheumatic fever

and occurs much more often in countries with limited access to antibiotics.

40. Diphenhydramine overdose leads to anti-histamine effects including drowsiness, confusion

and anticholinergic effects including dry mouth, dilated pupils, blurred vision, reduced bowel

sounds, and urinary retention. Physostigmine, a cholinesterase inhibitor can counteract these

anti-cholinergic effects. Serotonin syndrome tends to occur as a result of intentional overdose,

or as a consequence of drug interactions (classically when MAOI’s are mixed with other

serotonergic agonists). Presentation includes tachycardia, diaphoresis, dilated pupils,

hypertension, and hyperthemia. The other anticholinergic symptoms/signs are uncommon in

this syndrome.

41. Cervical cancer screening with a Pap smear should begin at age 21 or 3 years after the

initiation of sexual intercourse, whichever comes first. The interval of screening can increase

in women with 2-3 consecutive negative smears and the same monogamous partners. Women

without a cervix and any history of gynecologic malignancy do not need screening.

42. All pts who are start on anti-TB therapy should also be started on vitamin supplements,

especially pyridoxin (10ng/day), to avoid peripheral neuropathy and other neurological

complications. Hepatitis is another known side effect of isoniazide.

43. Acute bronchitis is a common cause of blood-tinged spurum and is usually viral in etiology. In

an afebrile pt with new-onset blood-tinged sputum without significant signs of more serious

disease, observation and close clinical follow-up is the best tx strategy.

44. 1) pts with central retinal vein occlusion usually present with subacute monocular visual loss.

The diagnossi is usually made by finding a “blood and thunder” appearance on fundoscopic

96

exam consisting of optic disk swelling, retinal hemorrhage, dilated veins, and cotton wool

spots. 2) Central retinal artery occlusion is characterized by sudden, painless loss of vision in

one eye, and the funduscopic exam reveals pallor of the optic disk, a cherry red fovea, and

boxcar segmentation of blood in the retinal veins.

1/3/09

1. Malignant otitis externa (MOE) is a serious infection of the ear seen in elderly pts with poorly

controlled diabetes, and is most commonly caused by Peudomonas aeruginosa. The

characteristic presentation consists of ear pain and ear drainage, and granulation tissue may be

seen within the ear canal on examination. Progression of the infection can lead to

osteomyelitis of the skull base and destruction of the facial nerve. Pts with poorly controlled

diabetes are susceptible to Rhizopus infections, but these infections typically begin in the

paranasal sinuses and extend into the orbit and brain.

2. Suspect the following conditions whenever a pt presents with hypokalemia, alkalosis and

normotension: 1) Surreptitious vomiting, 2) Diuretics abuse, 3) Bartter syndrome, 4)

Gitelman’s syndrome. Physical findings that are characteristic of surreptitious vomiting are

scars/calluses on the dorsum of the hands, and dental erosions. The dorsal scars result form

repeated chemical/mechanical injury as the pt uses his/her hands to induce vomiting. Dental

erosions result deu to increased exposure to gastric acid. Surreptitious vomiting may also

result in hypovolemia and hypochloremia, which in turn lead to a low urine chloride

concentration.

Pts with diuretic abuse and Bartter/Gitelman’s Syndrome may also present with hypokalemia,

alkalosis and normotension, but their urine chloride concentrations are high. Pts with diuretic

abuse may give a history of diuretic abuse and have a positive urine assay for diuretics.

Pts with chornic diarrhea have metabolic acidosis and hypokalemia (loss of bicarbonate in the

stool).

Pts with primary hyperaldosteronism have hypokalemia, metabolic alkalosis and hypertension.

3. TIA can be due to intrinsic blood vessel abnormalities (eg. atherosclerosis, inflammation,

dissection, etc.), embolic disease or decreased perfusion pressure. In elderly pts, the MCC of

TIAs are atherosclerosis and emboli. In younger pts, atherosclerosis is less likely, and one

should consider other causes such as emboli, vasculitis, dissection, malformations and

hypercoagulable states—most likely investigations- transthoracic echocardiogram

4. Histoplasma capsulatum is a common and usually asymptomatic infection in endemic areas

like Mississippi and Ohio River velleys and Central America. It is usually found in soil with a

high concentration of bird or bat guano droppings.

5. Anserine bursitis presents with sharply localized pain over the anteromedial part of the tibial

plateau just below the joint line of the knee. Valgus stress test fails to reproduce the pain,

thereby ruling out damage to the medial collateral ligament, and radiographs are classically

normal.

Prepatellar bursitis presents with pain and swelling directly over the parella. Examination shows

cystic swelling over the patella with variable signs of inflammation.

Patellofemoral syndrome is a common overuse pain syndrome of the knee. Pts present with

peripatellar pain worsened by activity or prolonged sitting (due to sustained flexion) and may also

complain of crepitus with motion of the patella.

97

6. If a pt presents with 3 hrs after the onset of an ischemic stroke, thrombolytic therapy with tPA

(after CT scan) should be started. Many trials have demonstrated improved neurological

outcomes with this approach. In the first 24-hrs the tole of CT scan is essentially to rule out

any hemorrhage in which case aspirin and tPA are C/I.

Nimodipien is used in the mx of subarachnoid hemorrhage (SAH). It is a selective cerebrovascular

dilator and reduces the vasospasms that are induced by the SAH.

7. Ewing’s sarcoma is often confused with osteomyelitis; however, characteristic x-ray findings

can point to the diagnosis. The lesion is usually lytic, central, and accompanied by endosteal

scalloping. The ‘onion -skin’ appearance is often followed with a “moth-eaten” or mottle

appearance and extension into soft tissue. The tx includes surgery, radiation, and multi-drug

chemotherapy. Radiation or chemotherapy is used preoperatively.

8. Pulmonary embolism is a common cause of pleuritic chest pain. It should be high in the

differential of a young person presenting with pleuritic chest pain, tachycardia, and dyspnea,

particularly if the pt is taking an OCP. One common cause of pleuritic chest pain is a

pulmonary embolism with subsequent minor or massive pulmonary infarction.

9. Acute acalculous cholecystitis is an acute inflammation of the gallbladder in the absence of

gallstones, most commonly seen in hospitalized and severely ill pts. 1) extensive burns, 2)

Severe trauma, 3) Prolonged TPN, 4) Prolonged fasting, 5) Mechanical ventilation. Involved

pathophysiology is localized or generalized ischemia, biliary stasis, infection, or external

compression of the cytic duct. It generally presents with right upper quadrant abdominal pain

with fever, leukocytosis, and abnormal live function tests. It can lead to complications like

gangrene, perforation, and emphysematous cholecystitis. The initial investigation of choice is

an USG, which shows signs of acute cholecystitis and no gallstones; however, CT scan and

HIDA are more sensitive and specific for the dx. It is important to have a high degree of

suspicion in high-risk pts in order to correctly identify this problem.

10. Emphysematous cholecystitis is a common form of acute cholecystitis in elderly diabetic

males. It arised due to infection of the gallbladder wall with gas-forming bacteria (eg.

Clostridium, Escherichia, Staph, Strep, Pseudomonoas, and Klebsiella). Predisposing factors

include vascular compromise (by obstruction or stenosis of the cystic artery, which is the sole

arterial supply to the gallbladder), immunosuppression (as seen with DM), gallstones, and

infection with gas-forming bacteria. Clinical manifestations of emphysematous cholecystitis

include right upper quadrant pain, nausea, vomiting, and low-grade fever. Crepitus in the

abdominal wall adjacent to the gallbladder is occasionally detectable. But peritoneal signs are

absent. Complications include gangrene and perforation. If perforation occurs, the abdominal

pain is relieved but peritonitis results. Dx is confirmed with an abdominal radiograph

demonstrating air fluid levels in the gallbladder or an ultrasound showing curvilinea gas

shadowing in the gallbladder. CT scan may be of help in confirming the dx if other imaging

modalities are unclear. Laboratory findings may include a mild to moderate unconjugated

huperbilirubinemia or a small elevation in aminotransferases. Tx includes immediate fluid and

electrolyte resuscitation, early cholecystectomy, and parenteral antibiotics therapy that is

effective against G+ anaerobic Clostridium sp. (ampicillin-sulbactam, piperacillin-

tazobactam, or the combination of an aminoglycoside or quinolone with clindamycin or

metronidazole).

11. Hypercalcemia typically causes nonspecific symptoms such as abdominal pain and

98

constipation, polyuria and neuropsychiatric disturbances. The acute tx of hypercalcemia is iv

0.9% saline hydration followed by a loop diuretic. Bisphosphonates (Pamidronate) decrease

plasma calcium concentrations by decreasing osteoclast activity. They are more effective than

calcitonin and are the 2nd-line agent used for pts with moderate to severe hypercalcemia.

12. The power of a study is its ability to detect a difference between two groups (treated versus

non-treated, exposed versus non-exposed). Increasing the sample size increases the power of a

study, and consequently makes the confidence interval of the point estimate (eg. relative risk)

tighter.

13. Surgical resection followed by whole brain radiation is the standard of practice in the mx of

solitary brain metastasis with stable extracranial disease. Multiple brain metastases are best

treated with palliative whole brain radiation. * Extremely high-yield!!

14. Chronic HBV pts with persistently elevated ALT levels (one of the most important predictors

of response to both interferon and lamivudine), detectable serum HbsAg, HbeAg, and HBV

DNA should be treated with interferon or lamivudine.

15. Brain death refers to a total loss of brain function and is a legally acceptable definition of

death. Criteria include: absent cranial nerve reflexes, fixed and dilated pupils, no spontaneous

breaths, and agreement of two physicians. People who die within 24 hours of presnetation, die

from an unknow cause, die from a medical complication or due to suspected illegal activities

must be reported to the coroner or medical examiner.

16. Infective endocarditis is especially common in IV drug abusers. It can be fatal if initial

empirical tx (eg. vancomycin+gentamicin, covers Staph, Enterococci, and Strep) is delayed

17. Amidarone is a class III antiarrhythmic agent; well know for causing pulmonary fibrosis.

Thyroid dysfunction (hypo and hyper), hepatotoxicity corneal deposits and skin discoloration

(blue-grey) are other potential side effects.

18. Acute exacerbation of COPD is treated with a combination of inhaled/neubulized

bronchodilators and systemic steroids. N-acetylcysteine is a mucolytic agent, which is no

linger used in exacerbation of COPD, as they are even implicated in worsening

bronchospasm. Aminophyllin is proven to inferior to a combination of bronchodilator and

corticosteroids for the tx of acute exacerbation of COPD.

19. Trimethoprim/Sulfamethoxazole is used in HIV-positive pts to prevent opportunistic

infections caused by pneumocystis jiroveci and toxoplasma gondii.

20. A detailed medical history and physical examination is the most effective way to screen a low

risk population for the presence of underlying cardiac disease (eg. hypertrophic

cardiomyopathy).

21. Squamous cell carcinoma is the second most common form of non-melanoma skin cancer.

The single most important risk factor for the development of squamous cell carcinoma is

exposure to sunlight.

22. Mammograms should be performed every 1-2 years starting at age 40 in women at average

risk for breast cancer. There is no clear stop time established but many experts recommended

discontinuing mammograms no sooner than age 70.

23. Suspect absence seizures in a 4- to 8-year-old child with frequent daydreaming episodes, brief

staring spells, decline in school performance, and no post-ictal phase. The preferred tx is

ethosuximide or valproate.

24. When evaluating a pt suspicious for pseudotumor cerebri, do a lumbar puncture only after

99

completely ruling out a space-occupying brain lesion with neuroimaging (CT/MRI). Most pts

with pseudotumor cerebri have an empty sella seen on neuroimaging, this is probably caused

by the downward herniation of arachnocele due to the high CSF pressure. The following

criteria is used for the dx (exclusion) of idiopathic intracranial hypertension: 1) presence of

features of increased intracranial pressure in an alert pt. 2) absence of focal neurological signs

except sixth nerve palsy. 3) Normal CSF examination except increased CSF pressure. 4)

Absence of any ventricular abnormality other than enlargement on neuroimaging and absence

of other causes of increasd intracranial pressure.

25. Seminoma—PLAP, Embryonal Carcinoma—AFP+ beta-hCG (50%), Choriocarcinoma: beta-

hCG

26. Screening for bladder cancer is not recommeded, even in pts who are at risk of developing the

disease.

27. Empyema most commonly occurs in the setting of untreated pneumonia due to bacteria

seeding of a pleural effusion. Inflammation and collection of pus in the pleural cavity leads to

the formation of a loculated, abnormally contoured effusion with adjacnet pulmonary

consolidation. A bronchopleural fistula would most likely have radiographic evidence of air in

the pleural space.

28. Polycystic kidney disease is a relatively common autosomal dominant disease. It typically

presents with hematuria and hypertension but may also cause abdominal masses and pain. It is

associated with intracerebral aneurysms.

29. Enterotoxigenic E. Coli is the most frequent cause of traveler’s diarrhea. Travelers with

abdominal cramps, diarrhea, and malaise are likely to be infected with this organism.

30. Diagnosis of bullous pemphigoid is made by skin biopsy which shows deposits of IgG

(autoantibodies against the basement membrane glycoproteins BP230, BP180), and

complement in a linear pattern at the dermal-epidermal junction. It is characterized by tense

blisters in the flexural areas. This condition usually occurs in elderly pts (>60) with an

antecedent history of months of urticaria, and no oral lesions. Pts usually present due to the

pruritis. The identified precipitating factors are ultra-violet rays, NSAIDs, and antibiotics.

Pemphigus vulgaris is characterized by IgG deposits intercellularly in the epidermis. This

condition is an intra-epidemal blistering diease with autoantibodies to adhesion molecules

expressed in skin and mucous membranes. The bullae are usually flaccid.

Cicatricial 瘢痕的 pemphigoid is characterized by IgG deposits in a linear band at the dermal-

epidermal junction. This is an autoimmune blistering disease that affects the mucous membranes,

including the mouth, oropharynx, conjunctiva, nares, and genitalia.

Herpes gestationis is characterized by C3 the basement membrane zone. This condition usually

presents as sub-epidermal blisters in the second trimester of pregnancy.

Intradermal edema with leukocyte infiltraion is a feature of urticaria.

31. Brain death is a clinical diagnosis. The characteristic findings are absent cortical and brain

stem functions. The spinal cord may still be functioning; therefore, deep tendon reflexes may

be present.

32. Anabolic steroids are commonly abused by athletes in an effort or improve performance. Side

effects of these drugs include suppressed endogenous testicular function (resulting in reduced

fertility), gynecomastia, erythrocytosis, hepatotoxicity, psychological disturbance, cardiac

disease, dyslipidemia (lowered HDL and elevated LDL), increased coagulation, and

100

premature peiphyseal function (which stunts growth). Virilization is often seen in females who

abuse steroids.

33. Staph aureus is the most frequent cause of prosthetic joint septic arthritis.

34. Treat acute glaucoma emergently with mannitol, acetazolamide, pilocarpine or timolol. Avoid

mydriatic agent such as atropine.

35. The dx of pernicious anemia is confirmed by achlorhydria, decreased serum vit B12, positive

IF antibodies, and extremely elevated LDH. Elevated bilirubin and serum iron are also seen

but they are not specific.

36. Beta blocker overdose causes AV block, bradycardia, hypotension, wheezing (bronchospasm,

beta2 blockade), and potential cardiogenic shock. Atropine and IV fliuds are the first line of

therapy, and if they do not completely reverse the cardiac symptoms, glucogon should be

administered. Intoxication of CCBs, digoxin, and cholinergic agents would cause some of the

above symptoms, but wheezing is indicative of beta-blocker toxicity.

37. Respiratory acidosis is characterized by a decreased pH and a primary increase in the PaCO2.

Alveolar hypoventilaton is an important cause of respiratory acidosis. Acute pulmonary

embolism, atelectasis, pulmonary edema and pleural effusion can cause a decrease in PaO2

(hypoxemia), but these conditions typically also cause a decrease in PaCO2 (respiratory

alkalosis) due to concomittant tachypnea.

38. CPK levels should be checked in any pt on a statin who presents with myalgias. If highly

elevated, the first step is to discontinue the statin.

39. Cardiovascular disease is the MCC of death in dialyssi pts. It accounts for approximately 50%

of all deaths in this population. Cardiovascular disease is also the MCC of death in renal

transplant pts.

40. Pseudomonas is commonly responsible for nosocomial pneumonia in intubated pts. Fourth

generation cephalosporins (cefepime, ceftazidime) have been used successfully for tx. Other

effective medications include aztreonam, ciprofloxacin, imipenem/cilastin, tobramycin,

gentamicin, and amikacin. Piperacillin-tazobactam plus tobramycin is also highly effective.

41. Bilateral lower extremity pitting edema may be a sign of hypoalbuminenia secondary to

malnutrition. Decreased oncotic pressure causes the edema.

42. Severe recurrent respiratory infections and positive sweat chloride test are characteristic for

cystic fibrosis. Fat malabsorption is common in pts with cystic fibrosis and is due to exocrine

deficiency of the pancreas; therefore, deficiency of fat-soluble vitamins, including vitamin K,

is frequently present if no vitamin supplementation is provided. Vitmin K is an important

cofactor in posttranslational modification of several coagulation factors: Factors II, VII, IX,

and X as well as protein S and C.

43. Neisseria gonorrhea is the MCC of arthritis in young, sexually active pts. Gonococcal septic

arthritis may present in one of two ways: as an asymmetric polyarthritis (often associated with

tenosynovitis and skin rash) or as an isolated purulent arthritis affecting one or a few joints.

Diagnosis may be confirmed by gram stain of the synovial fluid, blood cultures, urethral

cultures or skin cultures. Although skin rash and tenosynovitis are not seen in this pt, purulent

arthritis in as sexually active individual is gonococcal arthritis until proven otherwise.

Reactive arthritis may be caused by genitourinary infection with Chlamydia trachomatis or by

certain GI infections. It presents with a triad of arthritis, conjunctivitis, and urethritis.

44. CT scan without contrast should be done in all pts with a stroke to distinguish between

101

hemorrhagic or ischemic stroke. If there is evidence of ischemic stroke, carotid Doppler and

TEE are performed to evaluate the possible source of embolism. Stroke can be subdivided

into subarachnoid hemorrhage, ischemic stroke and intracerebral hemorrhage. Subarachnoid

hemorrhage is usually accompanied by a sudden, dramatic onset of a severe headache.

Ischemic strokes are usually accompanied by a history of previous transient ischemic attacks.

Pts with ischemic stroke usually don’t have headache and impaired consciousness.

Hemorrhagic stroke is characterized by focal neurological signs that develop suddenly and

gradually worsen over minutes or hours. The onset of symptoms is not abrupt as in

subarachnoid hemorrhage or embolic stroke. Symptoms usually start during normal activity or

may be precipitated by sex or strenuous activity. As the hemorrhage expands, headache,

vomiting and altered mental status develop. Hypertension is the most important risk factor for

hemorrhagic stroke.

1/4/10

1. Vasospasm is the major cause of morbidity and mortality in pts with subarachnoid

hemorrhage (SAH). CCBs (nimodipine) are used to prevent vasospasm in pts with SAH.

2. Renal transplant dysfunction in the early post-operative period can be explained by a variety

of causes, including ureteral obstruction, acute rejection (biopsy shows heavy lymphocyte

infiltration and vascular involvement with swelling of the intima), cyclosporine toxicity,

vascular obstruction, and acute tubular necrosis. Radioidotpy scanning, renal ultrasound,

MRI, and renal biopsy can be employed in conducting a differential diagnosis. Acute rejection

is best treated with iv steroids. Cyclosporine toxicity does not present with graft tenderness.

The transplant’s function is usually restored when the dose of cyclosporin is decreased. In this

case, the pt’s clinical features (ie. Normal serum cyclosporine level, renal biopsy findings) are

more consistent with acute rejection. Administration of IV diuretics may be employded in

acute tubular necrosis. * Extremely high yield!!

3. Pts with Huntington’s chorea usually present in their forties or fifties with chorea and/or

behavioral disturbance. Atrophy of the caudate nucleus is a characteristix feature. Diffuse

atrophy of the cerebral cortex is a feature of Alzheimer’s disease. Atrophy of the lenticular

nucleus is most marked in pts with Wilson’s disease. Atrophy of the frontal and/or temporal

lobes is a feature of Pick’s disease, which presents with dementia and personality changes.

4. The p value is the probability that the result of a study was obtained by chance alone. A study

is considered statistically significant when the ‘p’ value used is less than 0.05. Know how to

interpret the ‘p’value and its relationship with the confidence interval.

5. High serum alpha-fetoprotein (>500 ng/ml) in an adult with liver disease and no obvious GI

malignancy is strongly suggestive of hepatocellular carcinoma.

6. Cholangiocarcinoma can complicate primary sclerosing cholangitis, especially in pts who

smoke and have ulcerative bolitis. Next best step in Mx—order biopsy of the prominent

stricture.

7. Macrocytic or megaloblastic anemia is charaterized by the following RBC indices: elevated

MCV, elevated MCH (in 90% of cases), and normal MCHC. The peripheral blood smear

typically shows macroovalocyte RBCs and hypersegmented neutrophils with a mean lobe

count greater than 4. Anisocytosis, poikilobytosis, and basophilic stippling (not only seen in

lead poisoning—microcytic anemia, MCV<80) are also seen.

102

Lab tests Pernicious Anemia (B12 deficiency)

Folic acid deficiency

Serum B12 level Decreased NormalSerum folic acid level Normal DecreasedSerum LDH level Increased NormalAchlorhydria Present AbsentSchilling test Positive NegativeMethyl malonyl CoA in urine Present AbsentNeurological signs Present Absent

8. The tx of Clostridium difficile diarrhea includes discontinuation of the offending antibiotic

(usually clindamycin) and prompt administration of metronidazole.

9. Pts with nephrotic syndrome are at incrased risk for developing hypercholesterolemia

(accelerated atherogenesis), hypertriglyceridemia, and hypercoagulable state.

10. Lumbar spinal stenosis is a common cause of back pain in pts over 60 years of age. It is

characterized by back pain radiating to the buttocks and thighs that interfere with walking and

lumbar extension. Lumbar flexion alleviated the symptoms. The prdal pulses should remain

normal, as arterial perfusion is not affected. The dx is confirmed with MRI. Lumbar disk

herniation typically presents with the acute onset of back pain with or without radiation down

one leg. Pts usually recall an inciting event. As with lumbar spinal stenosis, the pain may be

radiating and be associated with neurologic symptoms. However, in disc herniation, lumbar

flexion and sitting will make the pain worse.

11. Ankylosis spndylitis occurs most often in adults aged 20-30 years. Pts suffer from limited

spine mobility and progressive back pain of greater than 3 months duration. The pain and

stiffness are worse in the morning and improve with exercise. Bilateral ascroiliitis on plain

film of a pt with the above symptoms is diagnostic. The most prominent extraarticular

manifestation of AS is anterior uveitis.

12. HSV most frequently affects the temporal lobes of the brain. As a result, features such as

bizarre behavior and hallucinations may be present. The disease is usually abrupt in onset,

with fever and impaired mental status. Maningeal signs are frequently absent. Cerebral fluid

(CSF) findings are nonspecific, with low glucose levels and pleocytosis. The diagnostic test of

choice is CSF PCR for HSV DNA. However, IV acyclovir should be started without delay if

there is a suspicion of this dx.

13. Angle closure glaucoma occurs predominanty in people aged 55-70 years. It presents with an

acute onset of severe eye pain and blurred vision associated with nausea and vomiting.

Examination reveals a red eye with steamy cornea and moderately dilated pupil that is non

reactive to light.

14. Effect modification occurs when the effect of a main exposure on an outcome is modified by

another variable. In this scenario, the effect of oral contraceptives on the incidence of breast

cancer is modified by the family history: women with a positive family history have an

increased risk, while women without a positive family history do not have an increased risk.

There are other well-known examples of effect modification, such as: studying the effect of

estrogens on the risk of venous thrombosis (modified by smoking), and studying the risk of

lung cancer in people exposed to asbestos (greatly depends on /modified by smoking status).

Effect modification is NOT a bias!

103

15. Jaundice in the 3rd trimester of pregnancy should be evaluated specifically for those hepatic

disorders associated with pregnancy. Marked pruritus and elevated of total bile acids are

suggestive of intrahepatic cholestasis of pregnancy. Tx is based on symptom releif and

preventing complications in the mother and fetus. Ursodeoxycholic acid is most promising, as

it increases bile flow and can releive prutitus. Hydroxyzine and cholestyramine are alternative

therapies, though they appear to be less effective and can have concerning side effects.

The maternal prognosis of those with ICP is good, as the condition resolves shortly after delivery.

There are no hepatic sequelae. ICP may recur in subsequent pregnancies, and affected women are

also at increase risk of developing gallstones.

ICP poses more danger to the fetus than the mother, as fetal prematurity, meconium-stained

amniotic fluid, and intrauterine demise are all known comolications. Therefore, the preferred

approach in managing the pregnancies of women with ICP includes an early delivery once fetal

lung maturity is established.

Normal physiological changes of pregnancy include a mild elevation of alkaline phosphatase (due

to placental secretion of the enzyme) but normal bilirubin, aminotransferases, and GGT.

Acute fatty liver of pregnancy is a complication of the 3rd trimester of pregnancy. Pts are

sometimes asymptomatic, but may also complain of sever nausea, malaise, headache, and

abdominal pain. Over half of pts also have preeclampsia, and 60% will develop acute renal failure.

This condition can also cuase a prolonged prothrombin time and moderate to severe elevation of

transaminas levels. Histologically, acute fatty liver of pregnancy is characterized byh

microvesicular fat deposition in the hepatocytes without evidence of inflammation or necrosis.

16. Metformin should not be given to pts with renal failure, hepatic failure or sepsis, as these

conditions all worsen a pt’s lactic acidosis.

17. Vertebral osteomyelitis is usually hematogenous, and the lumbar area is most commonly

affected. Back pain accompanied by low-grade fever and elevated ESR is the typical

presentation. High-grade fever and chills are uncommon. Physical examination may

demonstrate local tenderness on percussion over the affected vertebrae and paravertebral

muscular spasm. Early diagnosis is very important in pts with this condition, because epidural

abscess and spinal cord compression may develop if the tx is delayed. MRI of the spine is the

most sensitive test for identification of vertebral osteomyelitis. Since MRI is excellent for

evaluation of soft tissues, it will also reveal whether there is any abscess or cord compression.

Blood cultures are also important in dx, and allow directed antibiotic therapy.

18. This pt most likely has hereditary telangiectasia (Osler-Weber-Rendu syndrome), an

autosomal dominant disorder characterized by diffuse telangiectasias, recurrent epistaxis, and

widespread AV malformations (AVMs). In hereditary telangiectasia, AVMs tend to occur in

the mucous membranes, skin and GI track, but may also be present in the liver, brain, and

lung. AVMs in the lungs can shunt blood from the right to the left side of the heart, causing

chronic hypoxemia and a reactive polycythemia. Pulmonary AVMs can also present as

massive, sometimes fatal, hemoptysis. (On physical exam, there are several ruby-colored

papules on his lips that blanch partially with pressure).

19. Diabetic foot infection can be classified as follows: 1) Non-limb-threatening infections are

superficial, lack systemic toxicity and have minimal cellulitis extending less than 2cm from

the portal of entry. Ulceration, if present, does not fully extend through the skin, lacking

significant ischemia. S. aureus is the major pathogen followed by facultative streptococci;

104

however, facultative G- bacilli and anaerobes are not uncommon. 2) Limb-threatening

infections have more extensive cellulitis, lymphangitis, ulcers penetrating through the skin

into subcutaneous tissue, and prominent ischemia. Such infections are commonly

polymicrobial with S. aureus, Group B streptococci, Enterococcus and facultative G- bacilli

being major pathogens along with anaerobic G+ cocci and Bacteroides species.

IV cefotetan, ampicillin/sulbactam, or the combination of clindamycin and a fluroquinolone is the

appropriate empirical tx for limb-threatening infections; whereas mild, or non-limb threatening,

infections can be treated with oral antibiotics like cephalosprin, clindamycin,

amoxicillin/clavulanate and fluoroquinolones.

20. Restless leg syndrome (RLS) had 4 cardinal symptoms: 1) an uncomfortable sensation or urge

to move the legs. Discomfort--2) which worsens in the evening or during sleep, 3) worsens at

rest, and 4) alleviated by movement of the affected limb(s). The lower extremity discomfort

may also get better after massage or warming. RLS is more common in middle aged and older

pts, as well as in those with chronic kidney disease and/or iron deficiency anemia. It can have

a profoundly negative impact on the sleep of pts and their sleeping partners. The mechanism

of primary RLS is unclear but it may involve abnormalities of dopaminergic transmission in

the central nervous system. Dopaminergic agonists (pramipexole and ropinerole) or levodopa

can be effective in reducing symptoms and improving sleep quality.

21. Essential tremor is characterized by a tremor that is suppressed at rest and exacerbated toward

the end of a goal-directed movement. Affected pts typically have difficulty holding a

newspaper, writing legibly, drinking a cup of coffee, of feeding themselves.

22. First line tx for essential tremor if propranolol, esp if the pt is also hypertension. Alternative

medications include primidone or topiramate

23. Mild intermittent asthma, the least severe form, is defined as symptom <= 2x/week, <=e

nighttiem awakenings/month, with a normal FEV1 and no limitations on activity. For this

form of asthma, only a PRN albuterol inhaler is required. Daily controller corticosteroids are

reserved for persistent asthma.

24. Rubella is characterized by maculopapular rash, posterior cervical and posterior auricular

lymphadenopathies, and polyarthralgia.

25. Obstructive uropathy causes flank pain, low volume voids with or without occasional high

volume voids, and renal dysfunction. Renal artery stenosis can cause hypertension with or

without renal dysfunction. It does not cause flank pain or urinary symptoms, however.

26. In the tx of cocaine-related cardiac ischemia, the first-line drugs are benzodiazepines, nitrates,

and aspirin.

27. The anemia of chronic kidney disease is due to erythropoietin deficiency. One must be careful

to ensure adequate iron stores prior to erythropoietin replacement because the erythropoietin-

induced surge in RBC production can precipitate an iron-deficient state.

28. Autosomal dominant polycystic kidney disease is a potential cause of hypertension.

Intracranial berry aneurysm is a common complication (5-10% cases). The other major extra-

renal complications are: 1) hepatic cysts-most common; 2) valvular heart disease-most often

mitral valve prolapse and aortic regurgitation; 3) colonic diverticula; 4) abdominal wall and

inguinal hernia. Although such aneurysms are common and dangerous when coupled with

hypertension, routine screening for intracranial aneurysms is not recommended.

29. Primary pulmonary hypertension (>25 at rest or >30 mmHg with exercise) can be seen in

105

middle-aged pts, and it presents with exertional breathlessness. Lungs will be clear to

auscultation. CXR would show enlargement of the pulmonary arteries with rapid tapering of

the distal vessels (pruning) and enlargement of the right ventricle.

30. Early morning hyperglycemia is sometimes, but not always, due to inadequate insulin dosage.

Another cause is nocturnal hypoglycemia, which leads to morning hyperglycemia (Somogyi

effect). Measurement of the 3AM blood glucose level is very helpful for determing the

particular etiology of early morning hyperglycemia.

In this case, the pt’s 3:00am blood glucose level shows that her morning 7:00an hyperglycemia is

due to the Somogyi effect, most probably due to a high NPH dosage at night. The effect of NPH

peaks between 4-10 hours after injection; with this in mind, it becomes evident that the time of

peak effect of this pt’s suppertime NPH coincides with (and most probably causes) the occurrence

of her hypoglycemia at 3am. This low blood glucose level results in an increase in the levels of

counterregulatory hormones (ie epinephrine, norepinephrine, glucagon) in the circulation, which

consequently increase the rate of glycogenolysis and gluconeogenesis, thereby eventually causing

the evelation of the morning blood glucose level.

Dawn phonomenon is responsible for morning hyperglycemia due to decrease insulin sensitivity

between 3:00-8:00am. Pts typically have elevated blood glucose levels (hyperglycemia) at 3am

and 7am. The underlying pathology involves spikes of growth hormone release, which occurs

shortly after falling sleep.

Waning of circulating insulin levels can also result in morning hyperglycemia; however, the

recorded blood glucose level at 3am is usually normal. Dawn phenomenon and waning of insulin

action often coexist.

31. Cholecystectomy is indicated in all pts with symptomatic gallstones who are medically stable

enough to undergo surgery.

32. Excessive alcohol intake can lead to aspiration pneumonia. Risk factors for aspiratory

pneumonia are: 1) altered consciousness-seizure, alcoholism, drug overdose, and CVA; 2)

dysphagia-esophageal reflux, diverticula, obstruction; 3) Neurologic disorder-advanced

dementia, Parkinsonism, myasthenia; 4) Sedation to precedures, such as bronchoscopy,

intubation, endoscopy.

33. Td should be given at least every 10 years. Recently, CDC have been recommending a one-

time Tdap booster for adults aged 19-64 years.

Influenza vaccination is recommended for adults who are within the ages of 19-49 years, and have

close contact with children aged 0-59 months. Either intranasal or injected influenza vaccines may

be used. All adults over 50 years of age and those with certain comorbid conditions should receive

the intramuscular influenza vaccine.

The HPV vaccine decreases the incidence of genital warts, abnormal Pap smears, and cervical

cancer. It may be given to females aged 9-26 years, and is most effective if given before coitarche.

34. Schistocytes (helmet cells) are fragmented erythrocytes. They occur in microangiopathic

hemolytic anemias (DIC, HUS, TTP) and due to RBC destruction by prosthetic cardiac

valves. Hemolytic anemias are characterized by a decreased serum haptoglobin level as well

as in an increased LDH (erythrocyte destruction) and bilirubin. Haptoglobin is a serum protein

that binds free hemoglobin and promotes its excretion by the reticuloendothelial system. In

intravascular hemolysis, the amount of free hemoglobin in the serum exceeds the binding

capacity of haptoglobin thereby decreasing the level of haptoglobin.

106

35. Febrile reaction is a common transfusion reaction that is caused by antibodies in the pt’s

plasma reacting with the donor’s leukocytes. It is characterized by fever and chills that usually

respond to NSAIDs and acetaminophen. Leukocyte depletion techniques, like cell washing,

decrease the probability of febrile, nonhemolytic transfunction reaction. Calcium gluconate

infusion is employed in rare cases of severe hypocalcaemia following massive blood

transfusion. Warming the blood is recommended only during rapid massive transfusion to

prevent hypothemia.

36. Saline-responsive metabolic alkalosis has a urine chloride concentration less than 20mEq/L

and is typically due to conditions that cause hypovolemia or GI proton loss. It is treated with

infusion of isotonic saline.

37. Cardiac tamponade is one of the most deadly consequences of aortic dissection. It is

characteried by hypotension, tachycardia, and heart failure.

38. Age-related macular degeneration is usually seen in pts above 50 years of age. It presents with

progressive and bilateral loss of central vision. Navigational vision is preserved. Open angle

glaucoma usually presents as a gradual loss of peripheral vision (over a period of years) and

consequent tunnel vision. Central vision is spared.

39. Influenza presents as the acute onset of fever, chills, cough, malaise, and myalgias. Influenza

pheumonia CXR-intersitial or alveolar pattern; physical exam--wheezes, crackles, and course

breath sounds. Antiviral therapy must be started withi 48 hours to signigicantly decrease the

disease duration and severity. The newest therapies are the neuraminidase inhibitors,

oseltamivir and zanamivir. Rimantadine and amantadine are other options, but only effective

against influenza A.

40. HIV pts with dysphagia should receive one to two weeks of empiric oral fluconazole therapy,

since candidal esophagitis is the most likely dx. If symptoms persist, endoscopy with biopsy

should be performed. The ulcers of HSV esophagitis are usually multiple, well circumscribed,

and have a “volcano-like” (small and deep) appearance (aciclovir), where the ulcers seen in

CMV infection tend to be large, shallow, and superficial (tx is ganciclovir).

41. SLE is an autoimmune disease that is most commonly diagnosed in young females. Systemic

manifestations include fever, fatigue, weight loss, proteinuria, rash, arthralgias and anemia.

Greater than 90% of pts have joint involvement most commonly affecting the hands. Joint

pain is common, but the arthritis induced by SLE is considered non-deforming. Psteophytes

and cartilage degradation—OA. Subluxation of the cervical vertebrae and tendon damage—

RA. Distal phalangeal resorption is prominent in the arthritis mutilans variant of psoriatic

arthritis leading to classic “pencil-in-cup” deformities.

42. Agranulocytosis is a serious side effect of antithyroid drug (ATD) therapy. It occurs in

approximately 0.3% of pts taking ATDs.

43. Chronic alcoholics typically present with multiple electrolyte abnormalities such as

hypokalemia, hypomagnesemia and hypophosphatemia. Hypomagnesemia, causes refractory

hypokalemia; therefore it is important to correct the magnesium along with the potassium

levels to be albe to correct electrolyte abnormalities of such pts.

44. Pts with a T-score of less than -1.5 plus risk factors for osteoporosis or a T-score of less than

-2.0 should receive preventative medications. The preferred agents are oral bisphosphonates

or raloxifene. Estrogen is also an effective medication in preventing osteoporosis; however, its

vascular and embolic risks make it a much less desirable option at this time. Calcium and vit

107

D should be given to all pts who osteopenia or osteoporosis but they are not nearly as

effective in preventing or treating osteoporosis as bisphosphonates or raloxifene.

1/5/10

1. Autoimmune hemolytic disease and hereditary spherocytosis are both extravascular hemolytic

anemias; however, the former is acquired and the latter has an autosomal dominant

transmission. A negative family history and positive Coomb’s test are thus suggestive of

autoimmune hemolytic disease, whereas a positive family history is more suggestive of

hereditary spheocytosis. The peripheral blood smear in both conditions may show

spherocytosis.

2. Syphilis consists of 3 stages. Primary syphilis is characterized by a painless ulcer on the

genitalia, perianal area, rectum, lip, tongue, or elsewhere, 2-6 weeks after exposure. Reginal

LN will be enlarged but non-tender. Serology for syphilis (RPR, VDRL) may or may not

positive at this time. Secondary syphilis is marked by a generalized maculopapular skin rash

that spread to the palms, soles, and mucous membranes, and is accompanied by generalized

lymphadenopathy. Pts may also suffer from fever, arthritis, iritis, hepatitis, meningitis, and

ostitis. Serological tests are always positive by this point. Tertiary syphilis is characterized by

tumors (gummas) that infiltrate that bones, liver, and skin. Aortitis, aneurysms, and aortic

regurgitiation may be seen. CNS disorders (neurosyphilis) may occur.

3. Tetracyclines are an important cuase of phototoxic drug eruptions. These eruptions manifest

as exaggerated sunburn reactions with erythema, edema and vesicles over sun-exposed areas.

Photoallergic contact dermatitis can occur, and certain sunscreens are ironically well-known

causes of this eruption. The skin manifestations are typically more eczematous in appearance, and

prior sensitization is required.

Stevens-Johnson syndrome is an epidermal necrolysis syndrome typically caused by drugs (ie

TMP-SMX, NSAIDs, anticonvulsants). The mucosal surfaces are almost always involved.

The common side effects of benzoyl peroxide are irritation, contact dermatitis, dryness, erythema,

peeling and stinging.

4. Digoxin is medication with a narrow therapeutic window. Several drugs, including verapamil,

increase digoxin concentrations and predispose to toxicity. The most common side effects of

digoxin toxicity are GI (anorexia, nausea, vomiting).

5. Be overly suspicious for an intraocular foreign body in pts with high-velocity injuries

(drilling, grinding, etc). If the initial pen light examination does not reveal any conjunctival

and corneal abrasions or foreign bodies, proceed with fluorescein examination.

6. Malignancy is the most common cause of SVC syndrome (Superior vena cave syndrome: a

condition where obstruction of the SVC impedes venous return from the head, neck and arms

to the heart. Signs and symptoms include dyspnea, venous congestion, and swelling of the

head, neck and arms.). Lung cancer (particularly small cell lung cancer) and NHL are often

implicated. Other possible causes include fibrosing mediastinitis (secondary to histoplasmosis

or Tb infection) or thrombosis secondary to indwelling central venous devices. When the

history and physical examination are suggestive, CXR is warranted.

7. Autosomal dominant polycystic kidney disease is a heritable form of renal disease

characterized by multiple renal cysts and intermittent flank pain, hematuria, urinary tract

infections, and nephrolithiasis. (Palpable mass doesn’t have to be at both sides of flank,

108

maybe only one side). RCC alone would not give renal failure, but ADPKD does.

8. An enlarged left atrium in mitral stenosis can cause a persistent cough and elevation of the left

main stem bronchus.

9. Amoxicillin is the preferred tx for pregnant or lactating pts with early localized lyme disease.

Doxycycline is the first-line tx for early localized lyme disease in pts without C/I to its use.

Doxycycline has the advantage of simultaneously treating possible coexistent human

ehrlichiosis, since this condition has the same vector as lyme disease. Doxycycline is avoided

in children and pregant/lactating women because it can cause skeletal and dental peoblems in

exposed children and fetuses. IV 3rd generation cephalosporins (ceftriaxone and cefotaxime)

are indicated for early disseminated and late lyme disease.

10. The typical CT/MRI findings in high-grade astrocytoma are heterogenous and serpiginous

contrast enhancement. Recognize the classic butterfly appearance of glioblastoma multiforme

(GBM).

11. Several values related to pulmonary mechanics can be calculated in mechanically ventilated

pts. Lung compliance is calculated by performing the end-inspiratory hold maneuver. Airway

resistance is calculated by examinating the peak airway pressures. Positive end-expiratory

pressure (PEEP) is calculated with the end-expiratroy hold maneuver.

12. Features of atrial fibrillation on EKG include an irregular irregular R-R interval with absent P

waves and narrow QRS complexes. In unstable pts immediate DC cardioversion should be

performed. It is a common complication of CABG, occurring in up to 40% of pts. In CABG

with aortic valve replacement, the incidence jumps to 50%.

13. Alcohol withdrawl should be suspected in any hospitalized pt with a history of significant

alcohol abuse. Earl symptoms include anxiety, insomnia, tremors and diaphoresis (complains

of bugs crawling on skin). Delirium tremens, which generally occurs 48-96 hours after the last

drink, involves hupertension, hyperthermia, agitation, hallucinations, and possible death. All

pts suspected of alcohol withdrawal should be placed in a protetive environment, and treated

with benzodiazepines (Chlordiazepoxide-librium).

Haloperidol is often used ot control agitation and aggression in psychiatric emergencies, including

those caused by certain drugs of abuse. However, it is not used in pts undergoing alcohol

withdrawal because it does not exhibit cross-tolerance with alcohol (as benzodiazepines do), and it

also lowers the seizure threshold.

14. Erythema Nodosum (EN) is a condition of painful, subcutaneous, pretibial nodules. It can be a

symptom of more serious disease processes including sarcoidosis, TB, histoplasmosis, recent

streptococcal infection, and inflammatory bowel disease. The association of EN with

sarcoidosis is particularly strong in young, African-American women. Cough, arthritis, uveitis

and hilar adenopathy on CXR are also associated with sarcoidosis.

15. Molluscum contagiosum is characterized by multiple dome-shaped lesions with central

umbilication. It is caused by poxvirus, and occurs as an opportunistic skin infection in HIV-

infected pts though it also commonly affects otherwise healthy individuals as well.

16. Carbamazepine is used for the tx of atypical bipolar depression and trigeminal neuralgia. It is

the most effective tx for trigeminal neuralgia, being effective in approximately 80% of such

individuals. Aplastic anemia can occur with prolonged use; therefore, routine CBC is included

in the follow-up management of such pts. When medication fails to control the pain, surgical

gangliolysis or suboccipital craniectomy for decompression of the trigeminal nerve are

109

options.

17. “Cerebral salt-wasting syndrome” may occur in pts with SAH. The pathology involves: 1) an

inappropriate secretion of vasopressin, which causes water retension, and 2) an increased

secretion of atrial/brain natriuretic peptide, which causes cerebral salt-wasting. These changes

result in hyponatremia, which usually resolves within 1-2 weeks. SIADH is also commonly

seen in pts with intracranial hemorrhage. SIADH also results in hyponatremia, for which

water restriction is the initial treatment of choice.

18. Metabolic alkalosis can be classified into two broad categories, chloride-sensitive and

chloride-resistant based on urinary chloride levels and ECF volume status. Chloride sensitive

metabolic alkalosis is associated with low urinary chloride excretion and volume contraction.

19. The presence of diffuse musculoskeletal pain in the absence of joint swelling, muscle

weakness or laboratory abnormalities is suggestive of fibromyalgia. Fibromyalgia is the most

common cause of generalized musculoskeletal pain in women ages 20-55. The pathogenesis is

made based on the presence of widespread musculoskeletal pain and exerssive tenderness on

palpation of at least 11 of 18 predefined soft tissue locations during physical examination.

These sites include the upper quadrants of the buttocks and medial aspect of the knees, as well

as the sternocleidomastoid and trapezius muscles. The absence of joint swelling or muscle

weakness is also characteristic of fibromyalgia, as is the worsening of symptoms with

exercise.

20. A pt with known CHD or a CHD risk equivalent should be treated with lifestyle modification

and statins to reach a goal LDL of less than 100mg/L.

21. Newer antipneumococcal quinolones, like levofloxacin or gatifloxacin, are the drugs of choice

for in-pt tx of community-acquired pneumonia (caused by a host of bacteria and viruses, tx is

empirical. Most common pneucoccus, others-hemophilus, moraxella). For out-pt therapy,

either azithromycin or doxycycline can be used.

22. Aortic dissection is the most dangerous complication of Marfan’s syndrome. It typically

presents as tearing chest pain that radiates to the back and neck, and it must be identified

immediately to decrease the risk of death. One common exam finding is an early diastolic

murmur as the dissection causes aortic regurgitation.

Fixed splitting of the second heart sound (S2) is associated with atrial septal defects.

Kussmaul’s sign is an increase in jugular venous pressure that occurs in response to deep

inspiration. It occurs in situations that cause right ventricular failure like constrictive pericarditis

and right ventricular infarction.

Pulsus parvus et tardus refers to a carotid pulse that is slow (parvus) and late (tardus). It is

classically observed in aortic stenosis, not aortic regurgitation or aortic dissection.

23. Intracranial hypertension is diagnosed when the intracranial pressure equals or exceeds

20mmHg. It may be due to a number of different causes and typically presents with

headaches, vision changes, nausea and vomiting, changes in awareness, and /or focal

neurologic deficits.

24. Asymptomatic pts with carotid artery stenoses of 60-99% are considered to have a proven

indication for carotid endarterectomy. Complete occlusion (100% stenosis) of the carotid

artery is a C/I to surgery.

25. Chronic HCV classically presents with waxing and waning transaminase levels but few

symptoms. Pts may complain of arthralgia or myalgias. Extrahepatic sequelae include

110

cryoglobulinemia, porphyria cutanea tarda, and glomerulonephritis.

26. Subacromial bursitis is the result of repetitive overhead motions. Pts complain of pain with

active range of motion of the shoulder, and passive internal rotation and forward flexion at the

shoulder also elicits tenderness.

27. The most likely explanation for the isolated urinalysis finding of mild to moderate proteinuria

in a middle-aged adult with renal insufficiency and a history of longstanding diabetes is

diabetic glomerulosclerosis secondary to diabetic microangiopathy.

28. ‘Smudge cell’—the pathologist reports the presence of “leukocytes that have undergone

breakdown during preparation of a stained smear or tissue section, because of their greater

fragility”—CLL. The staging system is directly related to the prognosis.Stage Clinical feature Prognosis0 Lymphocytosis only GoodI Lymphocytosis + adenopathy FairII Splenomegaly FairIII Anemia IntermediateIV Thrombocytopenia Poor

29. Hemi-neglect syndrome is characterized by ignoring the left side of a space, and involes the

right (non-dominant) parietal lobe.

30. A generalized tonic clonic seizure is initially characterized by an aura, followed by stiffness

and loss of consciousness. The next phase involves tonic clonic jerky movements of the whole

body. Biting the tongue during the seizure and subsequent confusion with incontinence are

typical.

31. Back pain is one of the most common compaints in adult medicine. It is important to identify

signs that suggest a serious etiology, such as pain not relieved by rest, night pain, constant or

dull pain, fevers, lack of exacerbation with movement or palpation, and presence of

neurologic changes. Cancer pain is often no tender to palpation.

32. Normal saline is the initial fluid of choice in hypotensive, dehydration pts with DI. Hypotonic

fluids (0.45% saline or D5%W) can be started when the iv volume improves. Water

deprivation should not be performed when a pt is hypovolemic.

33. The most important steps in the mx of lactic acidosis from septic shock are IV normal saline

with or without vasopressor therapy to maintain the intravascular pressure and antibiotics to

correct the underlying infection.

34. Nitrates cause venodilation, which improves cardiac chest pain by reducing cardiac preload

and thus decreasing myocardial oxygen demand.

35. Ostiomalacia is characterized by defective mineralization of the bone. Rickets is characterzied

by defective mineralization of both bone and growth plate cartilage. Disordered skeletla

remodeling in focal areas of the bone is the underlyng pathophysiology of Paget’s disease of

the bone. Osteoporosis is characterized by low bone mass, but the bone that is present is

normally mineralized per unit volume.

36. Erythema chronicum migrans in a pt with a tick bite history warrants immediate tx with

doxycycline. Serological testign is not necessary to confirm a Lyme disease dx if the pt

presents with classic EM.

37. Ethosuximide is used most exclusively for children absence seizure.

38. Loop diuretics cause hypokalemia and hypomagnesemia. These eletrolyte abnormalities can

111

cause ventricular tachycardia, and also potentiate the side effects of digoxin.

39. Factors that improve the prognosis in a pt with chronic renal failure are protein restriction and

the use of ACE inhibitors. ACE inhibitors are likely to worsen renal failure when serum

creatinine levels are greater than 3-3.5 mg/dl.

40. Charcot’s joint, is a precess of developing neurogenic arthropathy starts with decreased pain,

proprioception, and temperature perception, which can occur due to diabetes, peripheral nerve

damage, syringomyelia, spinal cord injury, B12 deficiency, or tabes dorsalis. As normal

neurologic input is lost, pts unknowingly traumatize their weight bearing joints. This causes

secondary degeneration joint disease, joint deformation, and functional limitations. X-rays

will reveal loss of cartilage, osteophyte development, and loose bodies. Associated pain is

typically mild. Mx of a Charcot’s joint involves treating the underlying disease and providing

mechanical devices (eg. special shoes) to assist in weight bearing and to decrease further

trauma.

41. Warfar-induced skin necrosis presents with pain, followed by bullae formation and skin

necrosis. The breasts, buttocks, thighs, and abdomen are commonly involved.

42. The resolution of muscular weakness with rest is a hallmark feature of myasthenia gravis.

43. Pellagra, a niacin deficiency syndrome is characterized by a triad of 3 Ds of Diarrhea,

Dermatitis, and Dementia and if untreated eventually leading to Death (4th D). Causes: dietary

deficiency, alcoholics, carcinoid syndrome and Harnup’s disease. The skin rash is present in

sun-exposed areas (similar to SLE). Acute intermittent porphyria is a subtype of the

porphyrias, it has an acute presentation, occurs more commonly in females and is episodic

(episodes triggered by certain drugs). The episodes are characterized by abdominal pain,

nausea, vomiting, diarrhea, and sweating. It also causes agitiation, anxiety, paraesthesia, and

confusion. Unlike the other porphyrias, there is an absence of photosensitivity in this subtype.

44. CREST syndrome refers to a constellation of findings including Calcinosis cutis, Raynaud

phenomenon, Esophageal dysmotility, Sclerodactyly and Telangiectasias.

1/6/09

1. Testing for both HbsAg and anti-HBc offers the best screening for acute HBV infection, as it

won’t miss the window perios when HbsAg has disappeared but anti-HBS has not yet appeard

in the serum.

2. In pts with suspected stroke, non-contrast head CT is the initial diagnosis test of chioce,

critical for guiding further mx.

3. Pts with rotator cuff tendonitis complain of shoulder pain aggravated by activities such as

reaching or lifting arm over the head. The condition results from repetitive activity above

shoulder height and is most common in middle-aged and older individuals. For this reason,

painters are particularly prone to developing rotator cuff tendonitis. Impingement of preent in

all pts with rotator cuff tendonitis. It is comfirmed on physical exam by performing the Neer

test (passive motion of the arm above the head). Pain and guarding during the Neer test

confirmed impingement. To distinguish rotator cuff tendinitis from other forms of rotator cuff

pathology, lidocaine is injected into the joint. Improved range of motion and pain relief after

the injection corroborates the diagnosis of rotator cuff tendonitis. MRI is used for definitive

dx.

Rotator cuff tear may result from trauma (eg. falling on an outstretched arm) or as the end result of

112

chronic impingement and tendonitis. Similar to pts with rotator cuff tendonitis, pts often complain

of pain upon reaching and lifting the arm over the head. However, weakness of the shoulder is

more common in rotator cuff tears, and symptoms do not improve with lidocaine injection.

Adhesive capsulitis, or frozen shoulder, is an idiopathic condition characterized by pain and

contracture. This condition presents with an inability to lift the arm above the head. Even after

injection of lidocaine, the arm still cannot be lifted above the head due to fibrosis of the shoulder

capsule.

Vascular compression may occur in thoracic outlet syndrome. This most commonly presents with

a combination of numbness, weakness and swelling due to compression of the subclavian vessels

and lower trunk of the branchial plexus. A weakened radial pulse and reproduction of symptoms

with specific arm movements supports the dx.

4. HAV vaccine or serum immune globin should be given to all non-immunized travelers to

endemic countries. If travel will occur in less the 4 weeks, serum immune globulin should be

given. If travel will occur in greater the 4 weeks, HAV vaccine should be given instead as it

offers long-term protection.

5. Hepato-jugular reflex is a useful tool that can be used to differentiate between heart and liver

disease-related causes of lower extremity edema. Positive reflex indicated that the venous

pressure is elevated and suggests that the heart disease-related edema is present. Negative is in

pt with the liver disease-related edema.

6. Caustic poisoning does not cause alteration in consciousness. It presents with dysphagia,

severe pain, heavy salivation and mouth burns. The damage is the result of necrosis of the

tissue that lines the GI tract. In severe cases, perforation of the stomach or esophagus can

occur, causing peritonis or mediastinitis.

7. For frostbite 冻伤 injuries, the best tx is rapid re-warming with warm water. Whenever

frostbite or cold injuries are diagnosed, no attempts should be made to debride any tissue

initially. Rapid re-warming with dry heat (like a fan) is not effective for frostbite.

8. Acute pancreatitis can cause an acute abdomen. It should be managed conservatively with

anagesics, intravenous fliuds, and nothing by mouth.

9. In bronchiectasis, bronchial dilation causes impaired clearance of secretions and resultant

airway obstruction. Acquired bronchiectasis is ofter secondary to prior Tb or recurrent

pneumonia, while inherited bronchiectasis is most often due to cystic fibrosis. Symptoms

include cough, mucopurulent sputum, and atelectasis. HRCT is used for definitive diagnosis.

Bronchoscopy and alveolar lavage (BAL) if often used to establish diagnosis like

pneumocystis pneumonia and certain lung cancers. Rigid bronchoscopy may occasionally be

used in bronchiectasis, but non-invasive HRCT is preferable.

10. Tumor burden is the single most important prognostic consideration in the tx of pts with

breast cancer. It is based on TNM staging. ER+ and PR+ are good prognostic features.

Overexpression of the Her-2/neu is related to a worse prognosis. The histological grade also

tends to reflect the outcome. Poorly differentiated tumors have the worst prognosis.

11. An outlier is defined as an extreme and unusual observed in a datasheet. The mean is very

sensitive to outliers and easily shifts toward them. The median and mode are more resistant to

outliers.

12. The dx of cholelithiasis is best confirmed with abdominal ultrasound. Once diagnosed, the tx

of choice for symptomatic cholelithiasis is laparoscopic cholecystectomy. Those pts who

113

prefer not to undergo cholecystectomy may be given a bile salt such as ursodeoxycholic acid,

which decreases the cholesterol content of the bile by reducing the hepatic secretion and

intestinal reabsorption of cheolesterol. However, this medication is very costly and associated

with a high risk of relapse when therapy is halted.

13. Breaking bad news is a very sensitive issue to both pts and physicians. Step-wise approach: 1.

Make sure the pt is in a quiet, private, and comfortable enviroment. 2. Ask the pt how much

he knows, or what he thinks he might have. This will give you an idea of what his

expectations are. “What do you think of your symptoms?” 3. Ask the pt how much he wants

to know. “How much would you like to know about your condition?” 4. Give him a warning

shot. “Unfortunately, the situation is more serious than what I earlier thought.” 5. Break the

news if he wants you to. “The results show that you have advanced lung cancer.” 6. Give his

prognosis, but always keep him aware of all the options available to make his life as

comfortable as possible. 7. Try to explain everything as clearly and simple as possible. *

High-yield!!

14. Impetigo initially prestns as an erythematous macule, which repidly evovlves into vesicles

and pustules that rupture and leave honey-colored, crusted exudates.

15. Postexposure prophylaxis for HBV infection includes administration of the immune globulin

(HBIG) and 3 shots of HBV vaccine given at set intervals.

16. Amitriptyline and cyclobenzaprine have been shown to be effective in the tx of fibromyalgia

as they are able to increase the amount of restorative phase 4 sleep a pt gets. This is helpful

b/c one possible etiology of fibromyalgia is fatigue due to lack of phase 4 sleep.

17. Cough can be a presenting symptom of GERD. A 24-hour pH recording is the most specific

test available for acidic gastroesophageal reflux. It is usually employed to diagnose the cause

of chest pain or supra esophageal complications of GERD in pts with negative esiphagoscopy.

18. Parvovirus causes arthritis of the MCP, PIP, wrist and ankle joints. Anti-B19 IgM is the

dignostic study of choice when Parvovirus infection is clinially suspected.

19. Mitral regurgitation (mitral valve prolapse—most common in US) is the most common

valvular abnormality observed in pts with infective endocarditis not related to IV drug abuse

(tricuspid regurgitation).

20. Temporomandibular joint (TMJ) dysfunction can result in referred pain the ear that is

worsened with chewing. Pts typically report a history of nocturnal grinding.

Ramsay Hunter syndrome is a form of herpes zoster infection that causes Bell’s palsy. In this

condition, vesicles are typically seen on the outer ear.

21. Lumbar spinal stensis is most commonly casued by degenerative disk disease and presents

with low back and leg pain. The leg pain of spinal stenosis (“neurogenic claudication”) can be

confused with that of PVD (peripheral vascular disease) -induced claudication. In both cases,

leg pain worsens with walking; however, the leg pain of neurogenic claudication is position-

dependent and persists while standing still. PVD-induced claudication is exertion dependent

and resolve with standing still. On exam, normal ankle-brachial index and arterial pulses are

seen in spinal stenosis.

22. The most important factor when diagnosis carbon monoxide poisoning is the hsitory, b/c

physical symptoms can be vague. Suspect CO poisoning in cases of smoke inhalation and

when multiple people from the same confined quarters present with headache, nausea, and

abdominal discomfort. Pinkish-red skin hue is noted on examination and diagnosis is

114

confirmed by carboxyhemoglobin level.

23. A reliable test gives similar results on repeat measurements. Reliability is maximal when

random error is minimal. Validity or accuracy is defined as the test’s ability to measure what it

is supposed to measure. In order to determine the validity of a test, the results are compared to

those obtained from the gold standard test.

24. Unexplained hemolytix anemia and thrombocytopenia in a pt with renal failure and

neurologic symptoms should raise strong suspicions for TTP-HUS. HIV increases the risk for

TTP. A key clue to the dx of TTP-HUS in this pt is his reticulocyosis. A peripheral smear

with>1% schistocytes would be virtually diagnostic of microangiopathic hemolytic anemia

(MAHA), a component of TTP-HUS. However, MAHA can also occur in DIC and malignant

hypertension. Coagulation test would help to rule out DIC. (DIC pts tend to bleed and have

abnormal coagulation studies. TTP-HUS pts do not bleed despite their low platelet count).

Malignant hypertension should be considered if there is a history of hypertension and

evidence of hypertensive retinopathy on fundoscopic exam.

25. IV adenosine is the drug of choice for paroxysmal SVT (HR>140/min, regular, loss of ‘P’

waves and narrow QRS complex). Know ECG. If the pt is hemodynamically unstable,

electrical cardioversion should be performed immediately. However, if the pt is stable, vagal

manuvers should be attempt initially. If these fail to convert him to normal sunus rhythm, IV

adenosine push is the drugh of choice.

26. Open angle glaucoma is more common in African-Americans. It is generally asymptomatic in

the initial stages, followed by a gradual loss of peripheral vision over a period of years, and

eventual tunnel vision. On exam, the intraocular pressure is high. There may be cupping of the

optic disc with loss of peripheral vision. Annual exam in high-risk populations is of great

benefit in prevention of the condition. Beta-blockers such as Timolol eye drops are effective

in the initial mx of the pt. Laser trabeculoplasty is used as an ajunctive measure. If there is a

continuous increase in intraocular pressure, surgical traceculectomy is done.

27. Suspect syringomyelia in a pt with areflexia weakness in the upper extremities and dissociated

anesthesia in a “cap” distribution. The presence of a cord cavity is the most characteristic

feature. Caudal displacement of the fourth ventricle or cerebellar tonsils, and focal cord

enlargement may occur. * Just know how to diagnose.

28. Normal age-related cognitive changes include tiredness, occational forgetfulness, occasional

word finding difficulty, and trouble falling sleep. Dementia cannot be diagnosed unless there

are functional impairments.

29. Hereditary hemochromatosis can cause cirrhosis, pancreatic fibrosis (diabetes) and increased

skin pigmentation due to melanin and hemosiderin deposion (bronze diabetes), as well as

abnormalities of cardiac conduction (conduction block).

30. Ankylosing spondylitis presents with low back pain and stiffness typically in young HLA-B27

positive males. These pts may experience many systemic effects including fatigue, uveitis and

pulmonary disease. The pulmonary symptoms result from of the costovertebral jounts

resulting in chest wall motion restriction and a restrictive pattern on pulmonary function

testing.

31. Always suspext malignant melanoma in a pt who presents with a changing mole. The

ABCDEs of melanoma (Asymmetry, Border irregularities, Color variegations, Diameter

greater than 6mm and Enlargement) help in screening and early detection.

115

Melanocytic nevi are common, benign lesions found in the integument包皮 of most individuals.

They typically obey the ABCDE rules.

32. Bartonella henselae and Bartonell quintana cause bacillary angiomatosis in

immunocompromised individuals. Pts present with cutaneous and visceral angioma-like blood

vessel growths. Antibiotic tx causes lesion regression.

33. Bacteria overgrowth is a malabsorption syndrome which can be associated with a history of

abdominal surgery. Symptoms may be nonspecific and include abdominal pain, watery

diarrhea, dyspepsia, and weight loss. In severe and advanced cases, pts may present with

tetany (hypocalcemia sue to vit D deficiency), night blindness (Vit A deficiency), neutopathy

(B12 deficiency), dermatitis, arthritis, and hepatic injury. Physical exam may reveal

abdominal distention with identifiable succussion 振荡 splash (due to palpable soft, fluid-

filled loops of bowel). Macrocytic anmia (vitB12 malabsorption) can be seen on alb exam.

34. Membraneoproliferative glomerulonephritis, type 2, is a unique glomerulopathy that is caused

by persistent activation of the alternative complement pathway. Dense intramembranous

deposits that stain for C3 is a characteristic microscopic finding.

35. Barium swallow followed by endoscopy is usually done when a pt is suspected of having an

esophageal cancer (heartburn, significant weight loss, fatigue, smoking history).

36. Spontaneous bacterial peritonitis (SBP) should be suspected immediately in cirrhotic pts with

ascitis who develop fever or abdominal pain. The dx of SBP is confirmed by a positive ascitis

fluid bacterial culture and an elevated ascitic fluid absolute neutrophil (PML) count of more

than 250 cells/mm3. Emipirical tx: cefotaxime (or ceftriaxone), a third-generation

cephalosporin, ampicillin may be added to cover enterococcus. Antibiotix therapy is usually

successful, but because SBP ofhter recurs, liver transplantation is the most effective tx.

37. Drug-induced pancreatitis is mild and usually resolves with supportive care. CT scan is

diagnostic for pancreatitis. Scenarios for drug-induced pancreatitis: pt on 1) diuretics—

furosemide, thiazides. 2) inflammtory bowel disease—sulphasalazine, 5-ASA. 3)

immunosuppressive agents—azathioprine, L-asparaginase. 4) history of seizure or bipolar

disorder—valproic acid. 5) AIDs—didanosine, perntamidine. 6) antibiotics—metronidazole,

tetracycline.

38. Spontaneous subconjunctival hemorrhage is a benign finding, and does not require any tx. It

may be due to simple trauma from rubbing the eyes vigorously, violent coughing spells,

hypertensive episodes or coagulopathy.

39. AV fistula (history of trauma) causes high output cardiac failure by shunting the blood form

the arterial to venous side, thereby increaseing cardiac preload. The pt develops heart failure

despite maintaining a normal or high cardiac output b/c the circulation is unable to meet the

oxygen demand of the peripheral tissues. Clinical signs include widened pulse pressue, strong

peripheral arterial pulsation (brisk carotid upstroke), and tachycardia. The extremities are

typically flushed. The left ventricle hypertrophies, and the point of maximal impulse is

displaced to the left. EKG shows left ventricular hypertrophy.

40. Mitral stenosis in left atrial dilation and a risk of AF and cardiac emboli. The pressue is also

transmitted to the pulmonary vasculature, which can result in dyspnea, cough and hemoptysis.

41. Waldenstrom’s macroglobulinemia is characterized by hyperviscosity of the blood owing to

the excess production of IgM. 2 important diagnostic clues for this disorder are: 1) an IgM

spike on electrophoresis, and 2) hyperviscosity. Signs and symptoms: 1) increased size of the

116

spleen, liver and some lymph nodes. 2) Tiredness, usually due to anemia (too few RBC). 3)

Tendency to bleed and bruise easily. 4) Night sweats. 5) Headache and dizziness. 6) Various

visual problems. 7) Pain and numbness in the extremities due to a predominantly

demyelinating sensorimotor neurophathy.

Multiple myeloma—present with similar symptoms, but the involved Igs are usually either IgG or

IgA. Furthermore, such pts rarely have hyperviscosity syndromes such as retinal vein

engorgement.

Monoclonal gammopathy of undetermined dignificance (MGUS) are mostly asymptomatic or

have mild symptoms.

Heavy chain disease present like those with abdominal lymphoma; however, IgA is the main Ig

involved.

42. Presbyopia is common age-related decrease in lens elasticity that leads to difficulty with near

vision. A history of a middle-aged individual who has to hold books at an arms length to read

is classic.

43. Pseudotumor cerebri is a condition that presents with headache, blurry vision, papilledema,

and/or cranial nerve palsies. CSF examination reveals increased opening pressure. Young,

obese women are msot commonly affected. Vit A and isotretinoin can also predispose.

44. All pts with atherothrombotic TIA should receive an antiplatelet agent if there is no C/I to its

use. Aspirin is the initial agent of choice for this purpose.

1/7/10

1. Recommended therapy ofr an acute bacterial exacerbation of COPD includes supplement

oxygen, inhaled bronchodilators (beta-2 agonists and anticholinergics), broad-spectrum

antibiotics, a two-week corticosteroid taper and smoking cessation.

2. Latex allergy can manifest as an anaphylactic reaction during exposure to latex-containing

products such as surgical and condoms.

3. Autonomic neuropathy may lead to a denervated bladder, resulting in overflow incontinence

(due to detrusor weakness). This condition is characterzied by a high post-void residual

volume. Aside from strict glycemic control, tx opitons include intermittent catheterization and

cholingergic medications such as bethanechol. Heavy alcohol intake may contribute to the

effects of autonomic neuropathy; therefore, advising this pt to stop drinking alcohol beverages

may be helpful.

The pt has a normal sized prostate; therefore, overflow incontinence due to bladder outlet

obstruction is unlikely. Overflow incontinence due to medication is usually seen with the use of

anticholinergics, antipsychotics, tricyclic antidepressants, and sedative-hypnotics. Pelvic floor

weakness is a common cause of stress incontinence in women, although it may occur in older men

after a radical or transurethral prostatectomy.

4. Pancytopenia is common in pts with SLE. It occurs due to the formation of autoantibodies

against blood cells, a form of type II hypersensitivity.

5. Sudden onset right upper quadrant abdominal pain, fever, vomiting, and leukocytosis are

highly suggestive of acute cholecustitis. The symptoms are caused by impaction of gallstone

in cystic duct. Ingestion of fatty foods then stimulates the contraction of the gallbladder

against the obstructed cystic duct, causing severe colichy pain. The tissue behind the duct

obstruction becomes inflammed, typically as a result of bacterial overgrowth secondary to

117

stasis. The subsequent ischemic changes can cause gangrene and perforation, with generalized

peritonitis or a well-circumscribed abscess the most common outcomes. Other potential

complications include cholangitis and chronic cholecystitis. Tx for acute cholecystitis includes

supportive care (nothing by mouth, with iv antibiotics, alimentation, and analgestics).

Laparoscopic cholecystectomy is recommended shortly after hospitalization and should be

performed immediately in cases of perforation or gangrene.

6. Opacification of a single lung with shifting of the mediastinum toward the affected lung

indicates a collapsed lung due to atelectatic volume loss. Urgent bronchoscopy is required to

identify and possibly treat the obstructing bronchial lesion (mucus plugging, tumor, foreign

body or external compression).

7. Graves ophthalmopathy is the most common cause of exophthalmos in adults. This proptosis

occurs secondary to autoimmune lymphocytic infiltration of the extraocular muscles resulting

in fibroblast proliferation, hyaluronic acid deposition, edema and fibrosis.

8. Total parenteral nutrition (TPN) causes gall bladder stasis (decrease CCK) and predisposes to

both gall stone formation and bile sludging, both of which may lead to cholecystitis.

9. Zenker’s diverticulum is defined as a herniation of mucosa through the fibers of the

cricopharyngeal muscle. Barium esophagograph is the confirmatory test of choice.

Esophagoscopy should be avoided due to the associated risk of perforation.

10. A pelvic fracture with urethral injury is commonly accompanied by erectile dysfunction (ED).

The causes of ED in this case are nerve injury and latered arterial supply.

11. Many pts with influenza are treated with bed rest and simple analgesia (eg. acetaminophen).

Antiviral medications can reduce the duration of influenza symptoms by 2-3 days; however,

these drugs are only effective if administered within 48 hours of the onset of illness.

Amantadine and rimentadine are only active against Influenza A. The neuraminidase

inhibitors (ie. Zanamivir and oseltamivir) are active against both influenza A and B. *

extremely important!!

12. The tx of disseminated histoplasmosis in HIV pts is IV amphotericin B, followed by lifelong

tx with itraconazole. Initial tx with IV amphotericin B decreases the fungal load, and lifelong

tx with itraconazole prevents relapse.

13. In transient ischemis attack (TIA), the associated focal symptoms resolve in less than 24

hours. In reversible ischemic neurologic deficit (RIND), the associated focal symptoms

resolve in 24 hours to one week.

14. Craniopharyngiomas are benign suprasellar tumors which usually present with signs of

hypopituitarism, headaches and bitemporal blindness. Although they are infrequently seen

adults, craniopharyngiomas actually have a bimodal age distribution—children and 55-65

years age group. * Extemely high yield!!

15. Primary CNS lymphoma—the presence of EBV DNA in the CSF is specific for this condition.

MRI reveals a weakly ring-enhancing mass that is usually solitary and periventricular.

Toxoplasmosis is a good differential diagnosis; however, it is unlikely in this pt since he is

receiving trimethoprim-sulfamethoxazole. MRI reveals multiple, ring-enhancing, spherical

lesions in the basal ganglia. A postive Toxoplasma serology is quite common in normal

subjects in the US, and is therefore not specific.

16. Osteoarthritis predominantly involves the DIP joints. The major radiographic features include:

joint space narrowing, subchondral sclerosis, osteophytes, and subchondral cysts. Heberden’s

118

nodules are seen at the DIP joints, Boucher’s nodules are seen at the PIP joints.

RA involves the MCP and PIP joints. Bony erosions are classically seen.

Hemochromatosis-associated arthropathy include: squared-off bone ends and hook-like

osteophytes in the second and third MCP joints.

17. Thymectomy may induce remission in pts with myasthenia gravis, and should be considered

in all pts who are between puberty and 60, and in those with disease that is not confined to the

extraocular muscles.

18. Diffuse esophageal spasm is usually seen in young females and can present with intermittent

episodes of chest pain and dysphagia. Barium swallow may reveal a ‘corkscrew’ esophagus.

The tx is supportive. * Extremely high yield!!

19. Hypertrophic cardiomayopathy—Carotid pulse with dual upstroke, strong apical impulse and

a systolic ejection-type murmur along the left sternal border. The mormur is worsened by

maneuvers that decrease preload (eg. Valsava, standing); while decrease by increase preload

(squatting, recumbency, leg raising). Hand grip (increase afterload) is typically used to

differentiate between the murmurs of aortic stenosis and mitral regurgitation where it causes a

decrease and increase, respectively, in the severity of the murmur.

20. When a motility disorder of the esophagus is suggested by contrast studies, the next step is

usually esophagoscopy to exclude mechanical causes of dysphagia, such as stricture or

esophageal cancer; then followed by manometry, which confirms the dx.

21. Anemia of chronic disease is a disorder of iron utilization that most commonly occurs in the

setting of chornic inflammation. It is typically a normocytic anemia with a decreased serum

rion, decreased TIBC, and decreased iron saturation, as well as a normal or elevated ferritin

level.

22. Blood in the chest, if it is evacuated, can get infected. The majority of pts will present with a

low-grade fever, dyspnea, and chest pain. Surgery is required to remove the clotted blood and

fibrinous peel (when the empyema is localized, complex, and has a thick rim).

23. Abdominal CT scan is a very sensitive and specific tool used in the diagnosis of pancreatic

carcinoma.

24. Measurement of glycosylated hemoglobin is an excellent way to monitor chronic gylcemic

control. It is reflective of the pt’s average glucose levels over the preceding 100-120 days

(which correlates with RBC survival time). Generally, every 1% increase in HbA1c

corresponds with a 35mg/dl increase in the mean plasma glucose level.

25. VIPomas (pancreatic cholera) are cancerous tumors that affect cells in the pancreas that

produce vasoactive intestinal peptide (VIP). VIPoma cause diarrhea, hypokalemia resulting in

leg cramps, and a decrease in the amount of acid in the stomach. The cause is not known.

VIPomas are diagnosed most commonly at age 50 or so. Women ar more likely to be affected

than men. Other symptoms include dehydration, abdominal pain and cramping, weight loss,

facial flushing and redness. High level of VIP in the blood si diagnostic. A CT scan or MRI is

ordered to determine the location of the tumor.

26. Aortic dissection presents as sudden, tearing chest pain that radiates to the back. Mediastinal

widening is ofter, though not always, seen on CXR. Leg weakness can occur if the dissection

progresses to involve the arteries feeding the spinal cord.

27. Always suspect hypophosphatemic rickets in pts of rackets who had normal serum clacium,

normal serun alkaline phosphatase and normal 25-OH vitamin D.

119

28. A lesion in the upper thoracic spinal cord results in paraplegia, bladder and rectal

incontinency, and absent sensation from the nipple downwards. A lesion in the lower thoracic

spinal cord causes absent sensation from the umbilicus downwards.

29. Acut pericarditis typically occurs in the first several days after myocardial infarction. It is

characterized by sharp, pleuritic pain that is worse in the supine position and improved by

sitting up and leaning forwards. Diffuse ST elevations, especilly with PR depressions, are

typically ECG findings.

30. A consolidation (or fluid-filled area) persents with dullness to percussion, bronchial breath

sounds, and egophony (asking the pt to say the letter E and listening with a stethoscope; it

sounds like “A” with a nasal or bleating quality).

31. Vitamin B6, folate, and Vitamin B12 are involved in the metabolism of homocysteine.

Vitamin B6 lowers homocysteine levels by acting as a cofactor for the enzyme cystathionine

beta-synthase, which metabolizes homocysteine into cystathionine.

32. Hemolytic anemia in a pt with a maligant lymphoproliferative disorders likely to be of the

warm autoimmune type, caused by anti-RBC IgG antibodies. If prednisone therapy is

ineffective, splenectomy is usually indicated.

33. The ophthalmic artery (central retinal A. and ciliary branches) is the first intracranial branch

of the internal carotid artery. Central retinal artery occlusion is emergently treated with an

ocular massage and high-flow oxygen administration.

34. Suspect G6PD deficiency in a pt who develops acute hemolysis after ingesting primaquine or

sulfa drugs, G6PD levels are often normal during the hemolytic episode. Pyruvate kinase

deficiency can also lead to a similar clinical picure of hemolytic anemia, however, the

hemolysis in such cases is not precipitated by sulfa drugs. Furthermore, the typical peripheral

smear does not include bite cells.

35. Firm, flesh-colored, dome-shaped, umbilical papules are typical of molluscum contagiosum.

Cellular immunodeficiency, corticosteroid use, and chemotherapy predispose pts to

molluscum contagiosum. It is particulary common in pts with HIV.

36. SIADH is often seen with small cell carcinoma of the lung.

37. The US Preventive Services Task Force (USPSTF) recommeds screening male active or

former smokers aged 65-75 years with a one-time abdominal ultrasound to evaluate for an

abdominal aortic aneurysm.

38. Hypercalcemia of malignancy is due to several machanisms, including osteolytic metastasis,

increased production of PTHrP, increased production of 1,25-dihydroxyvitamin D, and

increased IL-6 levels. PTH levels are suppressed in most pts with hupercalcemia of

malignancy. Serum calcium levels are generally higher in pts with hypercalcemia of

malignancy than in those with primary huperparathyroidism. * High yield!!

39. Cessation of alcohol and staying on a low purine diet are important measures in the

prevention of future attacks in pts with acute gouty arthritis.

40. Suspect optic neuritis in a pt with central scotoma, afferent pupillary defect, changes in color

perception and decreased visual acuity. Remner the association between optic neuritis and MS

(USMLE love this topic!!).

41. PCP can occur when a pt’s CD4 count <200/ml. The typical clinical features include fever, dry

cough, and exertional dyspnea. Chest radiograph will show diffuse bilateral interstitial

infiltrates. Prednisone is used in combination with trimethoprim-sulfamethoxazole when the

120

PaO2 is less the 70 mmHg in pts with PCP.

42. Most pts with cat-scratch disease (CSD) have a gradual resolution of symptoms, even without

specific antibiotic therapy; however, tender regional lymphadenopathy and systemic

symptoms may be debilitating. Thereforem a short course of antibiotic therapy (5 days of

azithromycin) is recommended for pts with CSD.

43. Orthotopic liver transplantation remains the only effective mode of tx of fulminant hepatic

failure and should be considered in any pt presenting with this condition, regardless of

etiology.

44. Alpha-glucosidase inhibitors (acarbose, miglitol) block dietary carbohydrate breakdown in the

intestinal tract. The most significant side effects are GI disturbances due to the increased

undigested carbohydrate concentration in the stool.

1/9/10

1. Anorexia is a common problem of terminally ill pts. In palliative care, the rule-of-thumb is

that it is always better to have oral rather than parenteral nutrition. All efforts need to be made

to encourage the pt to eat adequate meals (eg. giving the pt his preferred meal, presenting the

food attactively, giving small and frequent meals). The next step is pharmacologic. The drug

of choice for cancer-associated anorexia is megestrol acetate, which is a synthetic progestin

with progestional effects similar to those of progesterone. It is currently used as an appetite

stimulant in pts with advanced malignancies. Prednisone may be used in addition to this drug.

Dimenhydrinate, ondansetron, and metoclopramide are anti-emetics which are usually given to pts

who are undergoing chemotherapy.

2. Anti-thyroid peroxidase and anti-thyroglobulin antibodies are present in more than 90% of pts

with Hashimoto’s thyroiditis.

3. Cor Pulmonale is a term for right-sided heart failure most commonly due to pulmonary

disease. Signs of right-sided heart failure include: jugular venous distension, right-sided S3,

right ventricular heave, hepatomegaly, ascites, and dependent edema.

4. Risk factors for pancreatic cancer are: 1) Male sex. 2) Increasing age (50 years). 3) Black

race. 4) Cigarette smoking; it is the most consistent risk factor. Pancreatic cancer is 2-3 times

more likely in heavy smokers than in non-smokers. 5) Chronic pancreatitis. 6) Long-standing

diabetes. 7) Obesity. 8) Familial pancreatitis. 9) Pancreatic cancer in a close relative. The

following are not risk factors for pancreatic cancer: 1) alcohol consumption. 2) gall stones. 3)

coffee intake.

5. Anti-mitochondrial antibodies are present in 90% of pts with primary biliary cirrhosis, a

chronic liver disease characterized by autoimmune destruction of the intrahepatic bile ducts

and cholestasis. Anti-smooth muscle antibodies and anti-LKM antibodies are associated with

acute and chronic hepatitis. The highest titers are linked with chronic active hepatitis. P-

ANCA is associated with Churg-Strauss syndrome and microscopic polyangiitis.

6. The most common cause of death in pts with acromegaly is cardiovascular, accounting for

approximately 38-62% of deaths.

7. In pts with hyperthyroidism-related tachysystolic atrial fibrillation, a beta-blocker is the drug

of choice.

8. Hehatic adenomas are uncommon benign epithelial tumors of the liver that usually arise as a

solitary mass in the right upper lobe. They are found predominantly in young and middle-aged

121

women who have a lengthy history of oral contraceptive usage. Hepatic adenomas may also

be seen is association with anabolic androgen use, glycogen storage disease, pregnancy, and

DM. A hormonally mediated pathogenesis is suspected, but the mechanism is not well

understood.

Histologically, hepatic adenomas contain sheets of enlarged adenoma cells that contain glycogen

and lipid. While the nuclei are small and regular, the normal hepatic archetecture is absent (with

septa, portal tracts, and bile ductules missing).

Hepatic adenomas are typically identified when pts complain of abdominal pain in the epigastrium

or right upper quadrant, when imaging is obtained for unrelated issues, or when an individual

suddenly collapses because of rupture and intraabdominal bleeding. The risk of such bleeding is

thougth to approach 40% in symptomatic patients, which is why obtaining biopsy of these masses

is normally C/I.

Common physical finding include a palpable abdominal mass, hepatomegaly, or jaundice. Liver

function tesets are usually normal, but are elevated on occasion. Alkaline phosphatase and gamma

glutamyl transpeptidase (GGT) are most commonly elevated in those pts with intratumoral

bleeding or multiple adenomas. Alpha-fetoprotein is normal unless malignant transformation of

the tumor has occurred, which happens in 8-13% among all pts with hepatic adenomas.

The diagnosis is beased upon the clinical presentation, imaging studies (typically CT scan or

ultrasound), and /or surgical resection. There is some controversy regarding preferred tx of this

tumor, but the general consensus is that all symptomatic hepatic adenoma should be resected.

Those adenomas that are not symptomatic can be initially managed ocnservatively with

discontinuation of oral contraceptives and careful observation with repeated imaging and serial

AFP measurements. Complete regression of the tumor can happen with consevative therapy, but

growth, rupture, and malignant transformation are also possibilities. Because of these risks, some

experts recommended the resection of all hepatic adenomas, regardless of size or symptoms.

Focal nodular hyperplasia (FNH) is a very common non-malignant hepatic tumor that is not of

vascular origin. It arises as a hyperplastic response to hyperperfusion by anomalous arteries that

are persent in the center of the nodule. The biopsy usually shows sinusoids and Kupffer cells,

neither of which is seen with hepatic adenomas. FNH is not associated with the use of oral

contraceptives.

9. In a cross-sectional study, exposure and outcome are measured simultaneously at a particular

point of time (remember: snapshot study). In other study designs, a certain time period

separates the exposure from the outcome.

10. Bonr marrow iron stain is the most definitive way to dignose iron deficiency anemia.

11. In diabetic pts, concurrent vascular and neurological diseases contribute to foot problems,

which are important causes of morbidity. The lifetime risk of foot ulcer in pts with diabetes is

15%. Foot ulcers can be classified as follows:

Grade 0: High-risk foot without an ulcer

Grade1: Superficial ulcer with full skin thickness involvement, but not involvement of underlying

tissue.

Grade 2: Deep ulcer penetrating to ligament or muscle, but not bone involvement or abscess

formation.

Grade 3: Deep ulcer with cellulitis, abscess formation or osteomyelitis

Garde 4: Localized gangrene

122

Grade 5: Extensive gangrene involving that whole foot.

Mx: 1) off-loading, 2) debridement, 3) wound dressings 4) antibiotics, 5) revascularization 6)

amputation. The key steps in the mx of grade 1 and 2 diabetic ulcers are proper wound care and

debridement. Debridement and removal of all the infected and necrotic tissue is essential in such

pts.

12. Erosion joint disease in RA is a clear-cut indication for the use of diseases modifying anti-

rheumatic drug (DMARDs). Methotraxate is the initial drug of choice for this purpose.

Glucocorticoids offer rapid improvement in the clinical and radiographic findings of RA, but

due to their significant adverse effects they are typically not used for long term mx. Celecoxib

is a selective COX2 inhibitor and its anti-inflammatory and analgestic action is comparable to

conventional NSAIDs. In contrast to conventional NSAIDs, celecoxib does not affect

hemostasis and does not produce GI side effect. COX-2 inhibitors are avoided currently due to

their ability to cause an increased risk of heart disease. Azathioprine is a purine analog that is

used as an immunosuppressive and a DMARD. It is not as efficacious as methotrexate;

therefore, it is not used as a first-line agent in RA. Etanercept and infliximab are TNF

inhibitors. They are very effective in pts who have disease refractory to methotrexate. These

agents, however, are very expensive and do not have the long safety record that methotrexate

had because they are relatively new. Reports of increased risk of lymphoma, TB reactivation,

CHF and demyelineating disease have been associated with these agents. They are not yet

indicated as first-line tx.

13. Gastric contents are rich in hydrogen, chloride, and potassium. Therefore vomiting causes

hypochloremic metabolic alkalosis and hypokalemia. Bicarbonate levels rise as a result of

hydrogen loss and activation of the renin-angiotensin system. The administration of isotonic

sodium chloride and potassium is used to reverse these eletrolyte abnormalities.

14. Right ventricular failure will cause elevated jugular venous pressure, hepatosplenomegaly,

ascites, and peripheral edema (increased pulmonary artery systolic pressue), but is not a

common cause of pulmonary edema.

15. CMV pneumonitis should be considered in the differential diagnosis of any bone marrow

transplant (BMT) recipient with both lung and intestinal involvement. Risk factors include

certain types of immunosuppressive therapy, older age, and seropositively before

transplantation. The median time of development of CMV pneumonitis after BMT is about 45

days (range of 2-4 weeks). Typical chest x-ray findings include multifocal diffuse patchy

infiltrates. High resolution CT scan show parenchymal opacification or multiple small

nodules. Bronchoalveolar lavage is diagnosis in most cases. Othe than pneumonitis, CMV

infection in post-BMT pts also manifests as upper and lower GI ulcers, bone marrow

suppression, arthralgias, myalgias, and esophagitis.

16. Unfortunately, no serologic or radiographic test has proven effective in screening for

pancreatic cancer in asymptomatic adults.

17. The most probable diagnosis in this pt is acquired sideroblastic anemia. This condition results

from defective heme synthesis, most commonly due to pyridoxine-dependent impairment in

early steps of protoporphyrin synthesis. This pt is taking isoniazid, a well-known pyridoxine

antagonist that can be responsible for this condition. Acquired sideroblastic anemia frequently

manifests as microcytic hypochromic anemia simulating iron-deficiency anemia. Usually two

groups of RBC can be demonstrated on microscopy-hypochromic and normochromic

123

(“dimorphic” RBC population). Besides that, iron studies typically reveal increased serum

iron concentration and decreased total iron binding capacity (TIBC), which helps to

differentiate sideroblestic anemia from iron-deficiency anemia. In this pt, with an easily

identifiable cause of Vit B6 deficiency, administration of pyridoxine is the most reasonable

next step. Bone marrow sampling can confirm the dx by demonstrating ‘ringed sideroblasts,’

(a specific finding in this type of anemia), but it si not necessary in this pt.

18. Pt with acute, severe pain should reveive the same standard of pain management regardless of

drug history. IV morphine is appropriate tx for acute, severe pain. Physicians should never

underteat pain, even if there is a risk of abuse. In the cases of concern for abuse, frequent

reassessment, putpatient follow-up, and referral to a pain specialist is appropriate.

19. Pts with Trousseau’s syndrome (migratory thrombophlebitis) usually have an occult tumor,

which may not always be detectable at the time of presentation. The most common tumor is

an adenocarcinoma. Based on the reviews, 24% -pancreatic carcinoma, 20% lung carcinoma,

13% prostate cancer, 12% stomach cancer, 9% acute leukemia, and 5% colon cancer. The

thrombophlebitis of the atypical sites such as arm and chest is an important clue for the

underlying carcinoma. So, the presence of this along with symptoms of upper GI discomfort

immediately leads to the possibility of pancreatic carcinoma that needs to be thoroughly

investigated. Abdominal CT scan is the test of chioce.

20. Cluster headache is characterized by intense unilateral retroorbital pain which starts suddenly

(usually at night, wakes the pt from sleep), peaks rapidly, and lasts for approximately 2 hours.

It is more common in men. It may be accompanied by redness of the ipsilateral eye, tearing,

stuff or runny nose, and ipsilateral Horner’s syndrome. The attacks occur in clusters, daily, for

6-8 weeks, followed by remission lasting for up to a year. Since the attacks are short but

severe, prophylaxis is the key to mx. The recommended prophylactic medications include

verapamil, lithium and ergotamine. The tx for an acute attack is inhalation of 100% oxygen

and sbcutaneous sumatriptan. * Extremely high yield!!

21. Albendazole or mebendazole is the first-line tx ofr enterobius vermicularis infection (not in

pregnant pts). Pyrantel pamoate is an alternative (second line).

22. Recurrent peptic ulcers with hypercalcemia are best explained by MEN type I. Serum

parathyroid levels and gastrin levels should be performed first.

23. The most common form of congenital adrenal hyperplasia (CAH) is 21-hydroxylase

deficiency, accounting for more than 90% of cases. The metabolite that accumulates us 17-

alpha-hydroxyprogesterone. Elevated levels of 17-alpha-hydroxyprogesterone serve as

precursors of adrenal androgens, thereby leading to hyperandrogenism.

Complete deficiency of 21-hydroxylase presents at birth with virilization of the female child and

excessive androgenization of the male child, there is also a decreased production of

mineralocorticoids and glucocorticoids, which causes salt wasting. Partial deficiency is also know

as non-classical CAH pts usually present during puberty or adulthood with hyperandrogenism,

which is not usually associated with salt wasting. The dx is suggested by increased levels of 17-

alpha-hydroxyprogesterone, and confirmed with an ACTH stimulation test.

17-hydroxylase deficiency leads to delayed puberty and mineralocorticoid excess.

11-alpha-hydroxylase deficiency results in androgen and mineralocorticoid excess.

3-beta-hydroxysteroid dehydrogenase deficiency results in DHEA-S excess and decreased

testosterone and mineralocorticoids.

124

Cystathionine synthase deficiency or homocystinuria is characterized by Marfan body habitus,

dislocation of the lenses, fair skin and vascular thrombosis.

24. TB test:

1. indutation >5mm

HIV-positive persons

Individuals with recent contact with a TB-positive person

Individuals with signs of TB on CXR

Organ transplant pts, pts on immunosuppressive therapy

2. >10mm

Individuals who have recently emigrated from a location where TB is endemic

Injection drug users

Residents/employees of high-risk settings (eg. prisons, homeless shelters)

Pts with diabetes, chronic kidney disease, hematologic malignancies, or fibrotic lung disease

Children less than 4 years of age, teens exposed to high-risk adults

3. >15mm

Healty individuals with no risk factors for TB infection

25. Hypertrophic cardiomyopathy can lead to ventricular arrhythmias, and should be suspected in

a sudden death of an otherwise healthy young individual.

26. Pharmacologic mx of variant angina (transient ST elevation on ECG at night) involves

calcium channel blockers and/or nitrates to prevent coronary vasoconstriction. Nonselective

beta-blockers and aspirin should be avoided b/c they can promote vasoconstriction.

27. The classic symptom triad of normal pressure hydrocephalus (NPH) is abnormal gait,

incontinence, and dementia. NPH is treated with large volume lumbar punctures and, if

successful, ventriculoperitoneal shunting.

28. Anticoagulation with heparin should be initiated immediately in any pt with a high likelihood

of pulmonary embolism and respiratory distress before diagnosis tests are done to confirm the

dx.

29. Cerebellar tumors usually produce ipsilateral ataxia, nystagmus, intention tremors, and loss of

coordination.

30. Arterial pH or anion gap is the most reliable indicator of metabolic recovery in pts with

diabetic ketoacidosis. Serum ketones level correct later than arterial blood pH or anion gap do.

31. Venous insufficiency results from incompetence of the valves within the lower extremity

veins. Complications include chronic edema, stasis dermatitis, and ulceration. Therapies

include leg elevation, compression, stockings, and wound care.

32. The Chi-square test is used to compare proportions. A 2x2 table may be used to compare the

observed values with the expected values.

33. Glucocorticoids cause neutrophilia by increasing the bone marrow release and mobilizing the

marginated neutrophil pool. Eosinophils and lymphocytes are decreased. * Extremely high

yield!!

34. Serous otitis media is defined as the presenct of a middle ear effusion without signs of active

infection. Examniation commonly reveals a dull tympanic membrane that is hypomobile on

pneumatic otoscopy. It is the most common middle ear pathology in pts with AIDS.

35. Agents available for Parkinsonism are L-dopa, dopamine agonist, anticholinergics and

125

amentadine. The most effective symptomastic therapy of Parkinsonism is L-dopa. L-dopa is

associated with a greater risk of dyskinesia than dopamine agonist; while dopamine agonists

have slightly less efficacy than L-dopa. Younger pts are usually initially treated with

dopamine agonist, and older pts are given L-dopa. Amantadine has relatively lower efficacy

and is used for pts with mild disease who have no disability. Selegiline is a selective

monoamine oxidase B inhibitor and a neuroprotective agent that may slow down the

progression of disease. It has a mild symptomatic action, and may be used as an adjunctive tx

in pts receiveing L-dopa.

36. Postoperative endophthalmitis is the most common form of endophthalmitis. It usually occurs

within 6 weeks of surgery. Pts usually present with pain and decreased visual acuity.

Examination reveals swollen eyelids and conjunctiva, hypopyon, corneal edema and infection

(esp vitreous). Based on the severity, intravitreal antibiotic injection or vitrectomy is done.

37. Pulmonary-renal syndromes incude a variety of disorders with simutaneous involvement of

the lung and kidney. Quick differential dx is important b/c the mx differs per disease.

Emergency plasmapheresis is required in pts with Goodpasture’s syndrome. Wegener’s

granulomatosis is treated with a combination of cyclophosphamide and steroids.

38. Legionella pneumophilia is an intracellular G- organism commonly spread by cooling towers

and water supplies. Travel-associated infections is well-documented. Symptoms suggestive of

Legionnaire’s disease include cough, fever >39C, GI symptoms, and confusion. Dx is

confirmed by urine antigen testing or culture on charcoal agar. Tx with azithromycin or

levofloxacin is appropriate.

39. IV amphotericin plus flucytosine is the antibiotics regimen of choice for central nervous

system cryptococcal infection in AIDS pts.

40. Rifampin causes red to orange discoloration of body fluids. “Red urine” in a pt taking

rifampin is usually a benign effect.

41. Suspect neurofibromatosis type II in a young pt with acoustic neuroma and multiple café-au-

lait spots. MRI with gadolinium enhancement is the best method to diagnose acoustic

neuromas.

42. Type 4 renal tubular acidosis should be suspected in a diabetic pt with a non-anion gap

metabolic acidosis, persistent hyperkalemia and renal insufficiency.

43. Melena is usually a manifestation of upper GI bleeding. Pain from duodenal ulcers usually

improves with food, whereas pain from gastric ulcers tends to worsen with food.

44. The MCC of asymptomatic elevation of alkaline phosphatase in an elderly pts is Paget’s

disease of bone (osteitis deformans). Pts with this condition are typically asymptomatic at the

time of dx; the dx is made by incidentally finding an isolated elevated alkaline phosphatase on

routine lab testing.

1/9/10

1. Cat bites are of significant concern because these often result in deep puncture wounds.

Furthermore, infection of such wounds with Pasteurella multicida tends to develop quickly,

and is associated with considerable pain, erythema, and swelling. Localized cellulitis can

develop subacutely, and in some cases, systemic effects (eg. fever and lymphadenopathy) may

arise. Prophylactic tx is thus recommended for such cases. For tx of minor cat bite wounds,

amoxicillin/clavulanate for 5 days is recommended.

126

2. Tricyclic antidepressants (amitryptiline, desipramine, nortriptyline) are the drugs of choice for

diabetic neuropathy. TCAs can worsen urinary symptoms (due to cystopathy) and orthostatic

hypotension (due to cardiovascular autonomic neuropathy). Gabapentin is an alternative for

these pts.

3. The main substances of gluconeogenesis are: alanine, lactate and glycerol 3-phosphate.

Pyruvate is an intermediate of alanine during the process of gluconeogenesis.

4. Mixed acid-base disorders may cause inappropriately normal lab values. Calculate the

expected change in the PaCO2 or HCO3 and compare that to the pt’s measured values to

determine if a mixed disorder exists or if normal physiology compensation has occurred.

PaCO2 =1.5 ( HCO3)+8

5. A massive pulmonary embolism can cause right ventricular dilatation and failure that results

in hypotension, tachycardia, syncope, and shortness of breath. Hemodynamic instability is an

absolute indication, and right ventricular strain a relative indication, for thrombolytics in the

setting of PE.

6. Wegener’s granulomatosis is a vasculitis affecting small and medium-sized arteries. Pts

present with a combination of glomerulonephritis and upper and lower respiratory tract

disease. Diagnosis is made by C-ANCA positivity and tissue biopsy. Tx involves high-dose

corticosteroids and cytotoxic agents.

7. Pericardial cysts are usually found in the middle mediastinum. Thymoma is usually found in

the anterior mediastinum. All neurogenic tumors are located in the posterior mediastinum.

8. Internuclear ophthalmoplegia is a pathognomonic finding of multiple sclerosis, and is due to

demyelination of the medial longitudinal fasciculus.

9. Premature atrial complex (PACs) maybe a manifestation of underlying heart disease or may

be idiopathic. Tobacco and alcohol are reversible risk factors for PACs. Beta-blockers are

often helpful in pts who are symptomatic.

10. Pts suffering from ankylosing spondylitis for two decades or longer are at an increased risk of

vertebral fracture due to decreased bone mineral density. Vertebral fractures in these pts may

occur with minimal trauma; therefore, the clinical suspicion for vertebral fracture in pts with

longstanding ankylosing spondylitis must be high.

11. Lifestyle modification should always be a part of the mx of hypertension. All pts should be

encouraged to lose weight, reduce salt in the diet, avoid excessive alcohol intake, and stop

smoking.

12. Muddy brown granular cast—Acute tubular necrosis

RBC casts—Glomerulonephritis

WBC casts—interstitial nephritis and pyelonephritis

Fatty casts—nephrotic syndrome

Broad and waxy casts—Chronic renal failure * Extremely high yield!!

13. how to calculate predictive values of a test

14. Pts with hemochromatosis are vulnerable to Listeria monocytogenes infections. Iron overload

is also a risk factor for infection with Yersinia enterocolitica and septicemia form Vibrio

vulnificus, both of which are iron-loving bacteria.

15. Adenocarcinoma of lung has the least association with smoking. It is usually located

peripheraly and consists of columnar cells growing along the septa. It presents as a solitary

nodule and may be detected incidentally.

127

16. Lumbar spinal stenosis is most commonly caused by degenerative joint disease (DJD, disc

herniation and facet osteophytes impine upon the spinal cord). The term “neuropathic

claudication” is often used to describe lumbar stenosis. This refers to the exacerbation of leg

symptoms with walking (similar to peripheral vascular disease-PVD). Howeve, unlike PVD,

the symptoms are positional and remain while standing still. Pain is relieved by flexion of the

spine (widening of the spinal canal). Dx is made based on clinical history and classic findings

on spinal MRI.

17. In developed countries, more than 80% of pts have primary adrenal insulficiency due to

autoimmune adrenalitis. Pts with autoimmune adrenalitis have autoimmune involvement of

other endocrine glands, such as the thyroid, papathyroid, and ovaries.

18. Neurofibromatosis is a neurocutaneous syndrome which has a tendency to form tumors in the

central nervous system, peripheral nervous system, skin and viscera. Other neurocutaneous

symptoms include: 1) Sturge-Weber syndrome (port wine stains in the V1 cranial nerve

distribution, angiomatous malformations of the brain, seizures, hemiparesis), and 2) Tuberous

slcerosis (ash leaf hypopigentation, cardiac rhabdomyomas, kidney angioleiomyomas, mental

retardation, seizure).

19. Uremic pericarditis is an absolute indication for dialysis in pts with chronic renal failure.

Know the indications and C/I of dialysis in uremic pts. Remember that the decision to start

dialysis is not strictly based on serum creatinine or BUN levels. Absolute indications: 1) Fluid

overload not responsive to medical tx. 2) hyperkalemia not response to medical mx. 3)

Uremic pericarditis. 4) refractory metabolic acidosis. Relative indictions: 1) Glomerular

filtration rate <10 ml/min (<15 ml/min in diabetics). 2) serum creatinine >8mg/dl (> 6mg/dl in

diabetics). 3) severe uremic symptoms (Uremic seizure and coma is an indication of

immediate dialysis). C/I: 1) debilitating chronic disease. 2) severe irreversible dementia.

20. Moderate to severe asthma exacerbations are treated with a combination of beta2-agonists,

inhaled anticholinergics, and intraveous steroids. Pts who develop respiratory failure, marked

by hypercapnia, or severe hypoxia, require intubation and mechanical ventilation.

21. A negative D-dimer is very helpful in excluding the dx of pulmonary venous

thromboembolism. A positive D-dimer, however, is far from diagnostic and must be followed

by more specific studies.

22. Bladder tumors are the MCC of painless hematuria in adults. More the renal cell carcinoma.

23. Cushing’s syndrome is caused by corticosteroid excess (eg asthma pt). Hypokalemia and

hypernatremia are the electrolyte abnormalities most common observed (have some

mineralocorticoid activity and will bind to aldosterone receptors in the kidney).

24. Cystinuria is an inherited disease causing recurrent renal stone formation. Look for a personal

history of recurrent kidney stones for childhood and a positive family history. The

characteristic stones are hard and radioopaque. Urinalysis shows typical hexagonal crystals.

The urinary cyanide nitroprussid test is widely used as a qualitative screening procedure. The

problem lies with the defective transport of dibasic amino acids (cystine, lysine, arginine, and

ornithine) by the brush borders of renal tubular and intestinal epithelial cells. Cystine is

poorly soluble in water; this leads to the formation of hard, redioopaque renal stones.

25. Pts with a retropharyngeal abscess often complain of a sore throat, fever, difficulty

swallowing (dysphagia), pain with swallowing (odynophagia), and pain with certain neck

movements, particularly neck extension. Tx must be initiated rapidly in order to prevent the

128

spread of infection into the mediastinum.

26. Cutaneous larva migrans, or creeping eruption, is a helminthic disease caused by the

infective-stage larvae of Ancylostoma brazilliense, the dog and cat hookworm. Infection

occurs after skin contact with soil contaminated with dog or cat feces containing the infective

larvae. It is common in travelers in tropical regions, and is characterized by pruritic, elevated,

serpiginous匐行的 lesions on the skin. Infection is often acquired through contact with sand.

27. SLE is an autoimmune disease that most commonly affects African-American women aged

20-40 years. Systemic manifestations include fatigue, fever, weight loss, non-deforming

arthritis, oral ulcers, serositis, hematologic abnormalities, proteinuria, and rash. Greater than

90% of pts have arthritis, most commonly affecting the MCP and PIP joints of the hands.

28. Cyclosporine and tacrolimus have the same mechanism of action (calcineurin-inhibitors,

inhibiting the transcription of IL-2 and several other cytokines). The major side effects of

cyclosporine include nephrotoxicity, hyperkalemia, hypertension, gum hypertrophy,

hirsutism, and tremor. Tacrolimus has similar toxicities, except for hirsutism and gum

hypertrophy. Major toxicity of azathioprine (purine analog) is dose-related diarrhea,

leukopenia, and hepatoxicity. Mycophenolate is a reversible inhibitor of inosine

monophosphate dehydrogenase (IMPDH), which is that rate-limiting enzyme in de novo

purine synthesis. The major toxcity of mycophenolate is bone marrow suppressio.

29. Consider 3 possibilities when a flat film of the abdomen and pelvis does not show a stone in a

pt with typical renal colic: 1) radiolucent stone disease (uric acid stones). 2) calcium stones

less than 1-3m in diameter. 3) Non-stone causes (eg. obstruction by blood clot or tumor). Uric

acid stones accounts for approximately 10-15% of cases of total nephrolithiasis. These are

most commonly seen in pts with unsusally low urine PH levels (which may be due to a defect

in renal ammonia secretion) and hyperuricosuria. These are radiolucent, but can be seen on

USG and CT scan. Tx includes hydration, alkalinization of urine, and a low-purine diet

with/without allopurinol, depending on the presence of hyperuricosuria. Since uric acid

stones are highly soluble in alkaline urine, alkalinization of urine to pH>6.5 with oral sodium

bicarbonate or sodium citrate is indicated.

30. In general, immunosuppressed pts and those with immunodeficiencies should receive an

extensive panel of vaccinations, but should not receive live vaccines: BCG, varicella

(chickpox), varicella zoster (shingles), anthrax, oral typhoid, intranasal influenza, oral polio,

and yellow fever vaccines. The only exception is the MMR, which may be used in pts

without evidence of immunity if their CD4+ count is >200/mm3 and they have no history or

evidence of an AIDS-defining illness.

31. Acute inflammatory arthritis caused by CPPD (calcium pyrophosphate dehydrate) crystals is

called pseudogout. It often occurs in the setting of recent surgery or medical illness.

Pseudogout is diagnosis by the presence of rhomboid-shaped, positively birefringent crystals

on joint aspiration microscopy and radiographic evidence of chondrocalcinosis (calcified

articular cartilage).

32. receiver-operating characteristic curve (ROC)

33. Suspect acute glaucoma in a pt with a sudden onset of eye pain, photophobia, and a mid-

dilated pupil. Tonometry is the best diagnostic test.

34. The concept of latent period is an important issue in chronic disease epidemiology. Exposure

must be continuously present for a centain peroid to time (called latent period) to influence

129

the outcome.

35. Exercise EKG testing is recommended for pts with an intermediate pre-test probability of

angina based on clinical features and risk factors. Medications that should be withheld prior

to testing include anti-ischemic medications, digoxin and medications that slow the heart (eg.

beta-blockers).

36. Cholesterol embolization usually follows surgical or interventional manipulation of the

arterial tree. Renal failure, livedo reticularis, systemic eosinophilia, and low complement

levels should make you think of cholesterol embolism. Renal failure in the presenct of

systemic eosinophilia should make you think of cholesterol embolism, allegic interstitial

nephritis, or polyarteritis nodosa. Acute allergic interstitial nephropathy is a drug-induced

hypersensitivity reaction characterized by rash, renal failure, eosinophilia, and eosinophiluria

(Hansel stain). The common medications that you should remember of the exam are: 1)

antibiotics (most common is methicillin group). 2) NSAIDs (often cause heavy proteinuria).

3) Thiazides 4) phenytoin 5) allopurinol * Extremely high yield!!

37. Acute otitis media should be considered in any pt with symptoms of ear drainage and

difficulty hearing. Ear pain is also common, but may be absent in young pts. Nonspecific

systemic symptoms including fever, irritability, and diarrhea can sometimes be present as

well. Cholesteatoma can present with hearing loss and ear drainage, but fever and systemic

symptoms are not usually present.

38. Succinylcholine is a depolarizing neuromuscular blocker that can cause life-threatening

hyperkalemia. It should not be used in pts with or at high risk for hyperkalemia, such as burn

and cruch injury pts and pts with prolonged demyelination.

39. The classic findings in pts with amyloidosis are renal amyloid deposits that show apple-green

birefrigence under polarized light after staining with Congo red. Clues—RA pt with enlarged

kidney and hepatomegaly.

40. Many expert group including the US Preventative Services Task Force recommended a one-

time screening of all women who are 65 years and older with a DEXA scan—ostepporosis.

41. “Soap bubble appearance in the epiphyseal end of long bone”—giant cell tumor of bone. The

classical pt is 20-40 yr old female with knee pain and some mass. The tumor cells are oval or

spindle shaped intermingled with numerous multinuclear giant cells in fibrous stroma. It is

benign but locally aggressive. It frequently recurs even after local currettage. The best way to

handle this tumor is to recognize it and leave it for experts.

42. The most feared complication of a retropharyngeal abscess is spread of infection into the

mediastinum, which can lead to acute necrotizing mediastinitis. The major complication of an

infection in the parapharyngeal space is involvement of the carotid sheathm which may lead

to erosion of the carotid artery and jugular thrombophlebitis.

43. Crystal-induced nephropathy is a well-know side effect of indinavir (a protease inhitibor)

therapy. Remember: 1) didanosine-induced pancreatitis. 2) abacavir—related hypersensitivity

synrome. 3) Lactic acidosis sencondary to the use of any of the NRTIs. 4) Stevens-Johnson

syndrome secondary to the use of any of the NNRTIs. 5) nevirapine—associated liver failure.

44. Alcoholism is the most common cause of cirrhosis in the US. Infection with HCV is the

second MCC of cirrhosis in the US.

1/10/10

130

1. IgA nephropathy is the most common cause of glomerulonephritis in adults. Pts have

recurrent episodes of gross hematuria, beginning 1-3 days after an upper respiratory infection.

Serum complement levels are normal. The latent peroid from infection to hematuria in post-

streptococcal glomerulonepharitis averages 10 days for pharyngitis and 21 days for impetigo,

while it is less than 5 days for IgA nephropathy. Serum complement levels are low in cases in

post-streptococcal glomerulonepharitis.

2. In chronic tophaceous gout, urate crystals can be deposited in the skin resulting in the

formation of tumor with a chalky white appearance.

3. Viridans group strepcocci are the most likely cause of endocarditis in native valves following

dental procedure. 4 members of the viridans group cause infective endocarditis: Streptococcus

mitis, S. sanguis, S. Mutans and S. salivarius. S. mutans also causes dental caries.

S. bovis is a normal inhabitant of the GI tract, and S. bovis bacteria is associated with colon

cancer. Colonscopy should be perfomed when this organism is isolated from blood culture.

S.epidermidis is an important cause of prosthetic valve endocarditis. It is also seen in infants with

IE sencondary to umbilical venous catheter infection in neonatal intensive care units.

Enterococci are normal inhabitants of the GI tract and also occasionally colonize the anterior

urethra. Enterococcal endocarditis generally affects older men after genitourinary manipulation or

young women after obstetric procedures.

4. Guillain-Barre syndrome is an acute or subacute ascending papalysis, CSF analysis reveals an

elevated protein level with normal glucose levels, WBC and RBC counts.

5. Gallbladder carcinoma (GBC) is a rare malignancy that most often arises in Hispanic and

Southwestern Native American females who have a history of gallstones. It is typically

diagnosed during or after cholecystectomy. Because the symptoms of gallbladder carcinoma

are vague and nonspecific (eg. pain, anorexia, nausea, vomiting, malaise) and the gallbladder

is nestled among other organs, GBC tends to be late-presenting and to have spread by the time

of diagnosis. Radical surgical interventio for pts with advanced disease includes

cholecystectomy and removal of the gallbladder bed, pancreatic head, LN, and sometimes

portions of the liver or biliary tree. If the GBC is discovered while still confined to the lamina

propria of the gallbladder, however, simple cholecystectomy results in a cure rate of 73%-

100% and is considered sufficient tx. Chronic choledochoithiasis is a risk factor for GBC

because if causes inflammation and irritation of the gallbladder epithelium, thereby inviting

malignant change.

6. Hawthorne effect is the tendency of the study population to affect the outcome since they are

aware they are being studied.

Sample distotion is seen when the estimate of exposure and outcome association is biased b/c the

study sample is not representative of the target populationn with respect to the joint distribution of

exposure and outcome.

Information bias occurs due to the imperfect assessment of association between the exposure and

outcome as a result of errors in the measurement of exposure and outcome status. It can be

minimized by using standardized techniques for surveillance and measurement of outcomes, as

well as trained observers to measure the exposure and outcome.

Confounding bias occurs due to the presence of one or more variables associated independently

with both the exposure and the outcome. For example, cigarette smoking can be a confounding

factor in studying the association between maternal alcohol drinking and low birth weight babies,

131

as cigarette smoking is independently associated with both alcohol consumption and low birth

weight babies.

7. The most common site of ulnar nerve entrapment is the elbow where the ulnar nerve lies at

the medial epicondylar groove. Prolonged, inadvertent compression of the nerve by leaning on

the lebows while working at a desk or table is the typical scenario. * Extremely high yield!!

8. 70% of cases with interstitial nephritis are caused by drugs such as cephalosporines,

penicillins, sulfonamide containing diuretics, NSAIDs, rifampin, phenytoin, and allopurinol.

Discontinuing the offending agent is the tx of drug-induced interstitial nephritis. Steroids may

hasten recovery in cases of drug-induced interstitial nephritis, but they may aggrevate the

underlying infection.

9. Otimally, the first step in treating a pt with a witnessed cardiac arrest is defibrillation. When a

cardiac arrest is unwitnessed or when there is a greater than 4-5 minute lapse between arrest

and arrival of the defibrillator, a trial of cardiopulmonary resuscitation (CPR) should precede

defibrillation.

10. Hodgkin’s disease is a curable lymphoma that tends to affect young pts. Unfortunately, pts can

develop secondary malignancies form chemotherapy and/or radiation, especially those treated

before the age of 30. The risk of secondary malignancy is highest when chemotherapy is

combined with radiation, reaching up to 3.2% within 20 years of tx. Lung and breast cancers

are among the most common secondary malignancies affecting pts previously treated for

Hodgkin’s disease.

11. Wafarin is a vit K antagonist used for anticoagulation in numerous clinical settings. Foods rich

in vits K (eg. dark green vegetables: brussels sprouts, green tea, spinach) will decrease its

efficacy whereas numerous other foods, supplements, and medications will increase its

activity. Examples of agents that increase warfarin activity include alcohol, vit E, garlic,

ginkgo biloba, ginseng, St. John’s wort, and several types of antibioticss.

12. IgA deficiency significantly increases the risk of developing an anaphylactic reaction to

transfused blooc products. The risk of anaphylaxia may be reduced in susceptible pts by

providing IgA-deficient blood products or by preforming additional washes on RBC or

platelet products.

13. Clear lung fields, hypotension, and jugular venous distension in the setting of an inferior wall

MI are suggestive of an RV infarct. Fluid resuscitation is the appropriate mx, and nitrates

should be avoided. Nitrates are not indicated in the setting of aortic stenosis, recent

phosphodiesterase inhibitor use, or right ventricular infarction.

14. It is important to distiguish between the different patterns of muscle weakness. The hallmark

of myasthenia gravis is decreasing muscular strength with continued contraction. While this

can affect any skeletal muscle, extraocular symptoms (eg. diplopia) are most common.

15. Normally, more than 95% of circulating bilirubin is the unconjugated fraction, which is highly

insoluble in water and is tightly bound to albumin. When bound, this unconjugated bilirubin

cannot be filtered by the glomerulus and is therefore not excreted in the urine. In the contrast,

the conjugated bilirubin is water soluble, loosely bound to albumin, and excreted in urine

when presnt in excess. Therefore, the presence of bilirubin in urine is indicative of a

conjugated hyperbilirubinemia (urine dipstick)—eg. Rotor syndrome.

16. Leukoplakia presents as hard to remove white patches in the oral mucosa and may lead to

squamous cell carcinoma. It is usually caused by chronic irritation to the oral mucosa due to

132

smoking, alcohol, or ill-fitting dentures.

17. First generation H1-antihistamines (diphenhydramine, chlorpheniramine, doxepine and

hydroxyzine) have potent anticholinergic effects and may cause eye and oropharyngeal

dryness as well as urinary retention, especially in older males who may have some underlying

BPH. Urine retention caused by anticholinergic agents results from failure of detrusor

contraction.

18. Pts at average risk of developing colon cancer should begin screening at age 50 with fecal

occult blood testing (FOBT), flexible sigmoudoscopy, a combination of the two, colonoscopy,

or double-contrast barium enema. Pts with an affected first-degree relative should begin

screening 10 years before the age of relative was diagnosed.

19. Facial nerve palsy and classical erythema migrans (EM) indicate Lyme disease, which is a

tick-bite disease.

20. The first step in the evaluation of a pt with a thyroid nodule is measurement of the TSH level.

If the TSH level is elevated, the levels of thyroid antibodies and thyroxine (T4) are measured b/c

pts with Hashimoto’s thyroiditis can have thyroid nodule formation. Since thyroid cancer can

occur with Hashimoto’s thyroiditis, FNAB is usually done if the nodule is larger than 1-1.5cm.

There is no real pathognomic sign that can conclusively differentiate benign form malignant

thyroid nodules; however, a rapid decrease in nodule size with levothyroxine tx is very reassuring

for a bening process.

If the TSH level is suppressed or below normal, radioiodine uptake and scan are typically

performed. If the scan shows a hot nodule (increased uptake of the tracer in the nodule with

decreased uptake in rest of the tryroid gland), FNAB is not performed b/c the chances of

malignancy in a hot nodule are extremely low.

If the TSH level is normal, FNAB is the next step.

21. Muscle weakness in paraneoplastic syndrome can be due to a variety of processes at different

levels. A good history and physical exam can help localize the process in most cases.

Myopathy association with lung cancer--The muscle involvement may be due to

inflammation, necrosis, fiber atrophy, and antibody-mendiated damage. Proximal muscles are

typically affected, and muscle strength is diminished symmetrically. Reflexes are normal, and

no sensation abnormality is present. Elevated CK level and myopathic electromyograph help

to confirm the dx. The pathologic precess in localized in muscle membrane.

22. The dx of cystic fibrosis is made on the basis of compatible clinical findings with laboratory

comfirmation. Sweat chloride test ( >60 mEq/L) is the gold standard test for the diagnosis of

cystic fibrosis and preferred over direct mutation analysis as more than 1,250 mutations in

CFTR can lead to cystic fibrosis.

23. There is no evidence that regular abdominal ultrasounds can help decrease mortality from

ovarian cancer.

24. The body compensates for chronic hypercarpnia by increasing renal bicarbonate retention.

Typically 0.1 mEq/L of bicarbonate is retained for every 1 mmHg increase in pCO2.

25. Senile purpura is characterized by ecchymoses that occur on elderly pts’ extensor surfaces due

to perivascular connective tissue atrophy. The lesions develop rapidly and resolve over several

days. Typically leaving a brownish discoloration from hemosiderin deposition. Senile purpura

is not a dangerous condition and requires no further investigation.

26. Psudogout cannot be reliably distinguished from gout and septic arthritis based on history and

133

physical alone. It is diagnosed by the presence of rhomboid, positively birefringent crystals on

synovial fluid analysis, and radiographic evidence of chondrocalcinosis.

27. It is important to recognize that oral contraceptives can be a potential cause of hypertension

(an estrogen-mediated increase in the synthesis of angiotensinogen in the liver), and simply

discontinuing its use can correct the problem.

28. Chronic pancreatitis is a serious disease and often leads to severe disability. Surgical

interventions in the case of chronic pancreatitis are indicated in case of chronic pseudocyst,

debilitating pain or to treat other complications. This pt has severe debilitating pain affecting

her work and daily life. She is an ideal candidate for ERCP and stone removal with stent

insertion, which would relieve the pain in 80% of such pts.

29. Turcot’s syndrome refers to an association between brain tumor (primarily medulloblastomas

and gliomas) and FAP (Familiar Adenomatous polypsis) or HNPCC (Hereditary nonpolyposis

colorectal cancer). The majority of FAP-associated brain tumors are medulloblastomas, but

gliomas have also been described. Pts with HNPCC are prone to high-grade gliomas. It is

autosomal recessive and mainly occurs in teens.

Gardner’s syndrome is autosomal dominant where colonic polyps are seen with prominent

extraintestinal lesions. These include desmoid tumors, sebaceous or epidermoid cysts, lipomas,

osteomas (especially of the mandible), supernumerary teeth, gastric polyps, and juvenile

nasopharyngeal angiofibromas.

Peutz-Jeghers syndrome is an autosomal dominant inherited disorder characterized by intestinal

hamatomatous polyps in association with mucocutaneous melanocytic macules.

Multiple hamartoma syndrome (Cowden syndrome) is associated of GI tract hamartomas with

breast Ca, thyroid Ca, and nodular gingival hyperplasia.

Cronkhite-Canada syndrome is association of juvenile-type polyps and ectodermal abnormalities

like alopecia, hyperpigmentation, and nail loss (onycholysis).

30. Malignant otitis externa (MOE) is a potentially serious infection of the external ear that is

usually caused by Pseudomonas aeruginosa. It is typically seen in elderly pts with poorly

controlled diabetes and presents with ear pain, drainage, and granulation tissue within the ear

canal on otoscopic exam. IV ciprofloxacin is the most effective tx.

31. Opioid intoxication does not always present with miosis. Coingestions can lead to normal

pupil size or even mydriasis and certain opioids (meperidine, propoxyphene) do not reliably

cause miosis even when taken alone. As a result, pupil examination is not as reliable as the

recognition of bradypnea in opioid intoxication. Tx—naloxone. Buprenorphine is an

alternative opioid agonist used in long-term mx of opioid addiction. However, it is not helpful

in cases of acute intoxication.

32. In a normal (bell-shaped) distribution:

68% --within 1 SD from the mean

95% --within 2 SD from the mean

99.7% --within 3 SD from the mean

33. Cryptosporidium parvum is a major cause of chronic diarrhea in HIV-infected pts with CD4

counts less than 180 cells/mm3. A modified acid-fast stain showing oocysts in the stool is very

suggestive.

34. Parkinsonism is caused by overactivity of cholinergic neurons and underactivity of

dopaminergic neurons in the substanta nigra. A shuffling gait (ie. The pt appears as if he was

134

chasing his center of gravity) is characteristic of the disease. –hypokinetic gait * Extremely

high yield!! Other syndroms—multiple system atrophy, spinocerebellar atrophies, and

multiple-infarct gait disorder. Wide-based gait is also seen in sensory ataxia, some cerebellar

disorders and muscular dystrophies.

Cerebellar ataxia is usually ipsilateral (ie. The pt tends to fall towards the side of the lesion). Other

features of cerebellar disorders include nystagmus, hypotonia, dysarthria, loss of coordination, and

the inability to perform rapid, alternating movements.

Muscular dystrophy results in a waddling gait 蹒跚而行 owing to weakness of the gluteal

muscles.

Spastic gait is seen with lesion of the upper motor neuron (eg. spinal cord injury or cerebral

palsy). The movements of the affected extremities are slow, stiff, and effortful.

Gait disequilibrium results from disorders of the frontal lobe and multiple sensory systems.

Sensory ataxia is seen with lesions involving the peripheral nerves, dorsal roots or posterior

columns. Loss of proprioception results in a wide-based, high-stepping gait. Romberg’s sign may

be positive (the pt sway on standing with his feet together and eyes closed).

Vestibular ataxia results in an en-bloc gait, with minimal movements of the head during walking.

It is typically a staggering gait蹒跚 and accompanied by vertigo and nystagmus.

35. Petit mal (absence) seizures are characterized by a sudden cessation of mental activity. An

episode is very short, but may occur repeatedly throughout the day. There are no associated

complex automatisms or tonic-clonic activity. The diagnosis is best confirmed by EEG studies

(activation procedures—hyperventilation, photic stimulation, sleep).

36. ARP (arributable risk percent) represents the excess risk in the exposed population that can be

attributed to the risk factor. ARP= (risk in exposed –risk in unexposed)/risk in exposed. It can

be easily derived from the relative risk: ARP = (RR-1)/RR

37. Fluid replacement is the most important step in the mx of non-ketotic hyperglycemic coma.

5% dextrose is given once blood glucose level has been lowered to 250 mg/dl by insulin

therapy. It prevents the development of cerebral edema. Regular insulin should be

administrated in all cases of nonketonic hyperglycemic coma, but fluid replacement alone can

reduce hyperglycemia significantly.

38. Abdominal ultrasound is the best tool for the initial investigation of gallbladder pathology.

While asymptomatic pts typically do not require tx, laparoscopic cholecystectomy (not open

cholecystectomy) is the tx of choice of those with symptomatic gallbladder disease.

39. Renal artery stenosis is a common cause of resistant hypertension in a pt with advanced

atherosclerosis. Carefully auscultate the periumbilical area of such a pt to reveal continuous

(or systolic with diastolic component) murmur characteristic of renal artery stenosis.

40. Syncope in a young pt with a crescendo-decrescendo murmur at the lower left sternal border

without carotid radiation is most likely due to hypertropic cardiomyopathy. Syncope in

hypertrophic cardiomyopathy is often multifactorial and inappropriately causes vasodilation.

41. Cholesteatomas in children can either be congenital or acquired secondary to chronic middle

ear disease. New-onset hearing loss or chronic ear drainage despite antibiotic therapy are

typically presenting symptoms of cholesteatomas, and granulation tissue and skin debris may

be seen within retraction pockets of the tympanic membrane on otoscopy.

42. Volume resuscitation with normal saline will correct contraction alkalosis (loss of gastric fluid

and activation of the renin-angiotensin-aldosterone system). Hypokalemia shound be treated

135

well.

43. Opioid withdrawal presents with symptoms of nausea, vomiting, abdominal pain, diarrhea,

restlessness, arthralgias and myalgias. Signs of opioid withdrawal on exam can include

increased bowl sounds, mydriasis and piloerection. Unlike ethanol and sedative withdrawal,

opioid withdrawal does not cause seizures. Methadone is proven effective for tx of opioid

withdrawal (not iv morphine).

44. Sjogren syndrome is diagnosed when subjective and objective evidence of dry eyes

(xerophthalmia) and mouth (xerostomia) exists in the presence of either histologic evidence of

lymphocytic infiltration of the salivary glands or serum autoantibodies against SSA (Ro)

and /or SSB (La).

1/10/10

1. Neurocardiogenic, or vasovagal, syncope occurs due to excessive vagal tone. Episodes are

preceded by nausea, diaphoresis, tachycardia, and pallor. Pain, stress, and situations like

medical needles and urination can all precipitate vasovagal syncope.

2. All pt with chronic liver disease should be immunized against HAV and HBV.

3. The correlation coefficient shows the strength and direction (positive, negative) of linear

association between two variables. It does not necessarily imply causality.

4. Lateral epicondylitis (tennis elbow) manifests as pain with supination or extension of the wrist

and point tenderness just distal to the lateral epicondyle.

5. Pts with infectious mononucleosis are at risk for splenic rupture. All pts with splenomegaly

should avoid excessive physical activity, particularly contact sports, until their spleen

regresses in size and is no longer palpable (usually after one to three months).

6. Viridans group streptococci are a frequent cause of subacute bacterial endocarditis (SBE) in

pts with preexisting valvular disease. Staphylococcus aureus is the major cause of acute

infective endocarditis and if often seen in IV drug abusers.

7. Allergic conjunctivitis is an acute hypersensitivity reaction that is caused by enviromental

exposure to allergens. It is characterized by intense itching, hyperemia, tearing, conjunctival

edema and eyelid edema.

8. In a pt with an MI who develops a cold leg, one has to get an ECHO to rule out a thrombus in

the left ventricle. Venous duplex study is an error. The leg is cold and without pulses which

indicates an arterial occlusion. Venous obstruction will present with a swollen leg and the

pulses will be present. In the setting of an MI and administration of heparin, the chances of a

DVT are low.

9. PCP is an important cause of progressive dyspnea and hypoxia in pts with HIV/AIDS.

Hypoxia results from an increased alveolar-arterial oxygen gradient.

10. 100% oxygen (tx of chioce) is an effective and rapid method used to abort an acute attack of

cluster headache. Verapamil is used to prophylaxis of cluster headache. * Extremely high

yield!!

11. The extent of a malignancy determined the most appropriate, timely, and individualized pt

care. CT is a standard diagnostic tool employed in pts with newly diagnosed gastric cancer to

evaluate the extent of the disease. Surgical removal of the affected tissues remains as the

mainstay of therapy. * Extremely high yield!!

12. Always suspect infectious mononucleosis (IM) in a young pt with a sore throat and

136

nonspecific symptoms. Pts with IM can develop autoimmune hemolytic anemia and

thrombocytopenia.

13. The characteristic lab findings of iron deficiency anemia are decreased serum iron level,

decreased percent saturation (serum iron/TIBC), and increased TIBC. Sideroblastic anemia is

characterized by increased serum iron levels and normal TIBC.

14. A thiazide diuretic such as hydrochlorothiazide is the first-line pharmacologic agent for mx of

hypertension in a pt without significant comorbidities. ACEIs are the first-line agent for mx of

hypertension in pts with DM, chronic kidneu disease and CHF. Beta-blockers such as

altenolol are indicated as a first-line antihypertension in pts with angina, low ejection fraction,

or status post-MI.

15. Lacunar strokes are due to microatheroma and lipohyalinosis in small penetrating arteries of

the brain. They often affect the internal capsule and result in pure motor dysfunction. Lacunes

comprise 25% of ischemic strokes. Hypertension and diabetes are two major risk factors.

Because of their small size, lacunes are often not appreciated on non-contrast CT scans

obtained during or shortly after the event.

Todd’s palsy is transient paralysis that occurs during the post-ictal state.

16. Warfarin doding should be adjusted to maintain a goal INR appropriate for the condition

being treated. For pts with idiopathic VTE (venous thromboembolism) or AF, a target INR of

2.0-3.0 provides adequate anticoagulation without an excessive risk of bleeding.

17. Hyperthyroidism is characterized by numerous symptoms including weight change, tremor,

fatigue, anxiety, and weakness. Hyperthyriod myopathy presents as progressive proximal

muscle weakness. It is slowly progressive but reverses quickly when the underlying

hyperthyroidism is treated. Inflammatory muscle diseases include polymyositis,

dermatomyositis, and inclusion body myositis. Tremor is not seen in these conditions.

18. CHF causes decreased cardiac output, which in turn lead to increased hydrostatic pressue in

the pulmonary vasculature. This results in transudation of fluid into the air spaces of the lungs,

which can lead to pulmonary edema and pleural effusion. A pleural effusion presents with

absence of lung sounds and dullness to percussion at the involved lung bases.

19. Most pts will require pharmacotherapy for blood pressure control, but life style changes can

make a difference. Weight control is the most important intervention, followed by physical

activity, salt restriction, and moderation of alcohol consumption. Every 10 kg reduction in

weight can decrease the systolic blood pressure by 5-20 mmHg. For all pts, the goal BMI is

18.5-24.9 kg/m2.

20. COPD, in general, is characterized by reduced FEV1 and repid decline in FEV1. FEV1/FVC

is decreased in COPD as opposed to normal in restrictive lung disease. DLCO is decreased in

emphysema and normal in chronic bronchitis.

21. Paroxysmal, lightning-like pain on the face is usually due to trigeminal neuralgia.

Carbamazepine is the drug of choice.

Carotidynia is a neurological condition caused by inflammation of the carotids and the vagus. The

pain is sharp and localized to the carotid artery distribution in the neck. Usually, the intensity of

pain is much less. The dx is clinical, although an MRI is sometimes required.

Burning mouth syndrome is a rare cause of facial pain. It is caused by a virus and the individual

has reddened mucosa and has significant pain. The condition is aggravated by dryness. It is treated

by supportive care.

137

22. Suspect hepato-renal syndrome in a pt with severe liver disease, hypotension, hyponatremia,

azotemia, and oliguria with normal urinalysis. Take measures to withhold all possible

precipitating factors. Initial mx includes careful volume loading and discontinuation of

furosemide and spironolactone.

23. Acute, symptomatic hyponatremia is a medical emergency requiring a prompt increase in the

serum sodium concentration (3% saline). In contrast, chronic hyponatremia requires slow

correction of the serum sodium concentration (<0.5 mEq/L/h) b/c the cells of the brain have

adjusted to the hyponatremia milieu and rapid increases in the serum sodium can cause

osmotic demyelination syndrome.

24. Staphylococcus aureus is the most common cause of osteomyelitis in both infants and

children.

Staphylococcus epidermidis—prosthetic devices

Salmonella—sickle cell anemia

G- rods (Klebsiella and psudomonas)—history of UTI or urinary tract instrumentation

25. Calcium gluconate is the most appropriate initial tx for hyperkalemic pts with significant

EKG abnormalities.

26. Firm, solitary LN are highly suspicious for LN metastases, particularly in older pts with a

significant smoking history. Non-tender, solitary nodes in the head and neck are concerning

for squamous cell carcinoma.

27. A normal or increased FEV1/FVC ratio with a low VC is seen in restrictive lung disease. The

Dlco is decreased with interstitial lung disease. Neuromuscular disorders are not associated

with a decreased Dlco.

28. Studies have shown that the risk for sepsis is present up to 30 years and probably longer after

spenectomy, Current recommendations state that pts should receive anti-pneumococcal,

harmophilus, and meningococcal vaccines several weeks before the operation, and daily oral

penicillin prophylaxis for 3 -5 years following splenectomy. * Extremely important.

29. Gallstones and alcoholism are the two most common causes of acute pancreatitis in the US.

Abdominal ultrasound should be used to search for gallstones in all pts experiencing a first

attack of acute pancreatitis. Abdominal CT scan is used to confirm the dx of acute

pancreatitis.

30. Prophylaxis against Mycobacterium avium complex with azithromycin is given to HIV-

infected pts when their CD4 count is less than 50/mm3. Dapsone is an alternative agent for

PCP prophylaxis. Clarithromycin, in combination with ethambutol, is used as tx rather than

prophylaxis for Mycobacterium avium complex infection.

31. Amebic abcesses within the liver contain debris charaterized as “anchovy paste” in

appearance, though cyst aspiration is not typically recommended b/c of the associated risk. Tx

is with oral metronidazole.

32. Rule out bladder cancer (urinary cytology and cystoscopy) in all elderly pts with irritative

voiding symptoms and have negative urine culture.

33. Surgical tx of hyperparathyroidism with parathyroidectomy should be reserved for pts who

are less than 50 years old, those with significantly worsening renal function, those with

osteoporosis, and those with serum calcium greater the 1mg/dl over the upper limit of normal.

34. Psoriatic arthritis occurs in 5-30% of psoriasis pts. The classic presentation involves the distal

interphalangeal joints. Morning stiffness, deformity, dactylitis (sausage digit) and nail

138

involvement are common. NSAIDs, anti-TNF agents and methotrexate are used for tx of

psoriatic arthritis; systemic corticosteroids are relatively C/I.

35. Spontaneous rupture of the esophagus (Boerhaave’s syndrome) is a rare and dangerous

condition. It typically occurs after severe retching and presents as chest or epigastric pain,

nausea, vomiting, fever, and unilateral pleural effusion (have a high amylase concentration,

low pH, and may contain particles of food).

36. Colicky flank pain with radiation to the grooin indicates renal colic. 75% -90% of the kidney

stones are composed by calcium oxalate. Small bowel disease, surgical resection or chronic

diarrhea can lead to malabsorption of fatty acids and bile salts; this in turn predisposes to the

formation calcium oxalate stone. Calcium phosphate stones are common in primary

hyperparathyroidism and renal tubular acidosis.

37. Sarcoid can present in a black female with bilateral hilar adenopathy. Recognize the

association with erythema nodosum. Biopsy typically shows non-caseating granulomas

(inflammatory). Serum Ca and ACE levels are elevated. Steroids are the tx of choice in

symptomatic pts.

38. Warfarin is an anticoagulant agent that functions by inhibiting the synthesis of vitK-dependent

factors II, VII, IX, and X (prothrombin), protein C and protein S.

39. New clubbing in pts with COPD often indicates the development of lung cancer.

40. Tamoxifen has a mixed agonist and antagonist activity on estrogen receptors. Estrogenenic

effects of tamoxifen increase the risk of endometrial cancer and venous thrombosis.

41. Thiazide diuretics have some unfavorable metabolic side effects including hyperglycemia,

increased LDL cholesterol, and plasma triglycerides. Electrolyte abnormalities that can be

induced by thiazide diuretisc include hyponatremia, hypokalemia, and hypercalcemia. *

Extremely high yield!!

42. In diabetic pts, the pathogenic mechanism of osteomyelitis adjacent to a foot ulcer is

continguous spread of infection.

43. GERD predisposes to Barret’s esophatus, erosive seophagitis and peptic stricture formation.

Peptic strictures cause symmetric and circumferential narrowing of the involved esophagus

and dysphagia to solids, but not weight loss. Other causes of peptic strictures include

radiation, scleroderma, and caustic ingestions.

44. Germ cell tumors typically affect young pts and display aggressive biologic behavior.

Nonsemnomatous germ cell tumors typically produce both AFP and beta-HCG as tumor

markers.

1/11/10

1. In a post-MI hypertensive pt, beta-blockers and ACEIs are perferred over diuretics and CCBs.

Beta blockers (esp cardio selective, matoprolol, better than propranolol) decrease myocardial

oxygen demand by reducing heart rate and contractility. ACEIs are indecated when EF is

decreased. right ventricle infarction, leading to right sided heat failure. Hypotension, jugular

venous distention and clear lung fields should raise suspicion for right ventriclar infarction.

Tx—IV fluids and avoidance of preload reducing medications such as nitrates and diuretics.

2. In high-risk pts presenting with fevers, chills, and evidence of septic emboli or abscesses (CT

scan of the abdomen reveals a fliud collection within the spleen), infections endocarditis must

be high on the differentia.

139

3. Gonorrhea is a common organism which causes sexually tranmitted disease. It is also a cause

of pharyngitis, which is acquired during oral sex.

4. Lactic acidosis resulting from poor oxygen delivery to the tissues is known as type A lactic

acidosis. Causes include CO poisoning (binds hemoglobin and a left shift of the oxygen

disscociation curve) and circulating failure (shock).

5. Lowering the cut-off point of a diagnostic test will increase its sensitivity.

6. Suspect primary hyperaldosteronism in a young pt with hypertension, muscle weakness and

numbness; the most specific lab value for pts with primary hyperaldosteronism is high

aldosterone/renin ratio, indicating autonomic aldosterone secretion.

7. PAO2 = FiO2 (760-47)-PaCO2/0.8 PAO2 = (0.21* 713)-PaCO2/0.8 = 150- PaCO2/0.8

PaCO2 normal range 33-45mmHg PAO2 93.75-108.75

A normal A-a gradient is <15 in a young person. Values increase with age, but an A-a gradient

>30 is considered elevated regardless of age.Hypoxia types PaCO2 A-a

gradientCorrect with 100% oxygen

Hypoventilation ↑ N Excessive anesthesiaLow inspirated oxygen N NShunting N ↑ No Pulmonary edema, pneumonia

and vascular shuntV/Q mismatch N ↑ Yes PE, excerbation of bronchial

asthma

8. The classic triad of carcinoid syndrome includes flushing, diarrhea, and wheezing. They are

exhibited by pts with carcinoid tumors. Such carcinoid tumors may occur in the small

intestine, colon, bronchial tubes, or appendix. Surgery with complete removal of the tumor

tissue is usually the first-line tx. The mainstay of tx for advanced carcinoid tumors that cannot

be removed surgically is octreotide injection. Pathognomonic plaque-like deposits of fibrous

tissue occur most commonly on the endocardium on the right side of the heart.

9. A pt with asthma-like symptoms that occur only at night and not during the day may actually

be suffering from GERD. Such pts should adopt anti-GERD lifestyle modifications and

reveive a trial of a PPI (eg. pantoprazole).

10. Bright red, firm, friable, exophytic nodules in an HIV infected pt are most likely bacillary

angiomatosis (picture). It is caused by Bartonella, a G- bacillus. Oral erythormycin is the

antibiotic of chioce.

11. It is important to be able to identify the signs and symptoms of anticholinergic toxicity and to

know common possible causes. The classic findings are dry skin, dry mouth, constipation,

urinary retention, flusing, vision change, and confusion. Medications for Parkinson’s disease –

trihexyphenidyl, benzathopine (excess) can cause anticholingeric symptoms.

12. AF fibrillation presents on EKG as an irregularly irregular, narrow QRS complex techycardia

that lacks P waves. The causes of acute AF are numerous and include cardiac, pulmonary,

metabolic (hyperthyroidism), and drug-related (alcohol, cocaine, amphetamines, theophylline)

etiologies.

13. Suspect vit B12 deficiency in strict vegetarians with anemia and neurologic complications. In

such pts, folic acid supplementation may correct the anemia, but not the neurologic

complications.

140

14. Pleural effusions are classified as transudative or exudative based on Light’s criteria. Pts with

tuberculosis may develop exudative pleural effisions due to increased capillary permeablility.

This fluid characteristically has a high adenosine deaminase concentration.

15. Calcium oxalate crystals (rectangular, envelope-shaped crystals) are seen in pts with ethylene

glycol (anti-freeze) poisoning. Ethylene glycol, methanol and ethanol intoxication cause

metabolic acidosis with both an anion gap and an osmolar gap.

16. Several randomized controlled clinical trials have demonstrated the beneficil effect of ACE

inhitibors on slowing the progression of diabetic nephropathy. The target limit of HBA1c is

7%. * the use of ACEIs in pts with DM is a very high-yield topic in the USMLE.

17. Ischemic damage in the setting of MI may lead to diastrolic dysfunction and a stiffened left

ventricle, resulting in an atrial gallop (S4).

18. Severe pain in a pt with a mild urinary obstruction, such as BPH, may cause urinary retention

due to inability to Valsalva. Classic symptoms of disk prolapse include unilateral redicular

pain in a dermatomal distribution. Back tenderness due to spasm of the paraspinous muscles is

common, and symptoms are usually worsened with straight leg testing.

19. Barium swallow is the initial test of choice for all pts with dysphagia. Subsequent endoscopy

is dependent on the barium swallow findings (ie. The outline of the pts current esophageal

anatomy). Motility studies are indicated once organic causes have been rule out by upper

endoscopy. If Zenker’s diverticulum is found on esophagography, endoscopy should be

avoided.

20. Diastolic and continous murmur as well as loud systolic murmurs revealed on cardiac

auscultation should always be investigated using transthoracic Doppler echocardiography.

Midsystolic soft murmurs (grade I-II/IV) in an asymptomatic young pt are usually benign and

need no further work-up.

21. Controlling the rhythm or rate in pts with prolonged tachysystolic AF usually improves the

LV function significantly, sometimes even dramatically.

22. Pts who abuse IV drugs are at increased risk for subacute bacterial endocarditis (SBE) of both

right and left heart. If the tricuspid or pulmonary valve is affected, SBE can present with

embolic phenomena to the lung. SBE must be in the defferential for all IV drug abusers

presenting with fever and malaise. Tricuspid regurgitation is common in individuala with

endocarditis of the tricuspid valve, and cause a systolic murmur that increase on inspiration.

Aortic regurgitation produces a diastolic murmur that is best heard when the pts sits up.

Paradoxical splitting of the second heart sound (S2) occurs in pts with left bundle branch

block (LBBB) due to delayed closure of the aortic valve.

23. An isolated liver mass is much more likely to be the result of metastatic disease than a

primary malignancy of the liver. Primary tumors of the GI tract, lung and breast are the most

common diseases causing liver metastases. Liver metastases are generally asymptomatic; pts

tend to present instead with symptoms related to their primary malignancy.

24. Tearing pain with radiation to the back and a difference in BP of greater than 30mmHg

between the arms are important clinical clue for aortic dissection. Transesophageal

echocardiography or CT with contrast are the two diagnostic studies of choice for suspected

aortic dissection.

25. When an acute pulmonary embolus occurs with hemodynamic compromise, the best tx is

fibrinolytic therapy. But if the pt has C/I (a fresh postoperative pt) to the use of fibrinolytic

141

therapy, an embolectomy is the tx of choice.

26. Type 2 diabetics are prone to the development of a hyperosmolar hyperglycemic state without

ketoacidosis. Decreased consciousness is the most common symptom. Other reversible

neurologic abnormalities, such as blurred vision, can also be caused be hyperglycemic

hyperosmolarity (a myopic increase in lens thickness and intraocular hypotension sencondary

to hyperosmolarity).

27. Inhaled anti-muscarinic agents such ipratropium are the mainstay of symptom mx in COPD.

These anti-cholinergic medications may be combined with short-acting beta-adrenergic

agonists for greater symptom relief.

28. The Rumack-Matthew nomogram provides the likelihood of hepatoxic effects from

acetaminophen overdose and the need for N-acetylcysteine. The first data point on the curve is

at 4 hours. The decision of whether or not to administer the antidote can be made after 4

hours, based on the actaminophen level. As long as it is administered within 8 hours of

ingestion, the outcome will not be adversely affected.

29. Cusing’s syndrome is an important cause of secondary hypertension. In addition to blood

pressure elevation, high systemic cortisol concentrations cause hyperglycemia, hypokalemia,

proximal muscle weakness, central adiposity, thinning of the skin, weight gain, and

psychiatric problems (eg. sleep disturbances, depression, and psychosis).

30. Ondensetron is the drug of choice for chemotherapy-induced emesis. Methoclopramide and

prochlorperazine are classic anti-emetic agents that can be given either alone or in

combination with ondansetron; however, these are not the best drugs for chemotherapy-

induced emesis.

31. Ultrasound of the kidney, ureter and bladder should be done in pts of BPH whose serum

creatinine is elevated. All pts with irritative or obstructive voiding symptoms should have

their urinalysis and serum creatinine done; as such symptoms are not always due to benign

prostatic hyperplasia. Such symptoms may be produced by multiple causes like BPH, bladder

cancer, prostate cancer, urethral stricture, neurogenic bladder and urinary tract infections.

Urinalysis helps to rule out urinary tract infection and serum creatinine detects renal

insufficiency that might be due to bladder outlet obstruction.

32. Hypoxemia-induced increases in erythropoietin production are responsible for the

polycythemia observed in obstructive sleep apnea. Tx of the OSA cause causes the

polycythemia to improve.

33. Suspect progressive multifocal leukoencephalopathy (PML) in an HIV-infected pt with focal

neurological signs and multiple non-enhancing lesions with no mass effect on the CT scan. It

is caused by JC virus, a human polyomavirus. PML predominantly involves the cortical white

matter, but the brain stem and cerebellum may also be involved. The most common presenting

symptoms are hemiparesis and disturbances in speech, vision and gait. Cranial nerve deficits

may occasionally develop. Classical MRI findings in PML consist of multiple demyelinating,

non-enhancing lesions with no mass effects. There is no effective tx for PML, and the mean

duration of survival from the time of dx six 6 months.

In pts with AIDS dementia complex imaging studies reveal cortical and subcortical atrophy and

secondary ventricular enlargement.

34. PCP is likely if an HIV pt has a non-productive cough, exertional dyspnea, fever, severe

hypoxia, bilateral interstitial infiltrates on CXR, and a normal white count. PCP usually

142

affects HIV pts when wheir CD4 counts falls below 200/ml. TMP-SMX is the drug of choice.

Steroids have been shown to decrease mortality in pts with severe PCP infection. Indications

for steroid use in PCP include PaO2 <70mmHg or A-a gradient >35mmHg.

35. The most common site of hypertensive hemorrhages in the putamen (35%). The internal

capsule that lies adjacent to the putamen is almost always involved, thereby leading to

hemiparesis.

36. Dry and rough skin with horny plates over the extensor surfaces of the limbs is a hallmark of

ichthyosis (Lizard skin).

37. A case-control study is used to compare the exposure of people with the disease (cases) to the

exposure of the people without the disease (controls). The main measure of association is the

exposure odd ratio. Prevalence odds ratio is calculated in cross-sectional studies to compare

the prevalence of a disease between different populations.

38. Always consider ACEI therapy as a potential cause of chronic cough. Simple discontinuation

of the drug should precede any diagnostic testing in pts with chronic dry cough who are taking

an ACEI. * Extremely high yield!!

39. Delirium is an acute confusional state due to organic illness which is commonly superimposed

upon dementia in the elderly. Urinary tract sepsis and metabolic disturbance are common

precipitants in older pts. The initial work-up of delirium should include urinalysis and

measurement of serum electrolytes.

40. Asymptomatic diverticulosis needs only dietary modifications in the form of high fiber intake.

41. Acanthosis nigricans presents with thickening and hyperpigmentation of skin of the flexural

areas. It has a classic ‘velvety’ texture. It is commonly associated with insulin resistant states

(DM, acromegaly, abesity and others) as well as GI malignancies (middle-aged or elderly).

42. Methotrexate is a DMARD. It works by inhibiting dihydrofolate reductase. Macrocytic

anemia is a common side effect. Other side effect include: nausea, stomatitis, rash,

hepatotoxicity, interstitial lung disease, alopecia and fever.

43. The most likely culprit 犯人 lesion for acute inferior wall myocardial infarction is right

coronary artery occlusion, especially if it is complicated by right ventricular infarction

(hypotension) and bradycardia (SA node). * Extremely high yield!!

44. The tx of acute cholangitis includes provision of supportive care and broad-spectrum

antibiotics. Pts who do not respond to this tx regimen should undergo biliary drainage with

ERCP.

1/12/10

1. This pt has postsperative cholestasis, a benign condition that often develops after a major

surgery characterized by hypotension, extensive blood loss into tissue, and massive blood

replacement. The jaundice is thought to develop secondary to 3 factors: increased pigment

load (caused by the transfusion), decreased liver functionality (caused by hypotension), and

decreased renal bilirubin excretion (caused by tubular necrosis). Generally, jaundice is evident

by the second or third day after a prolonged surgery, with the bilirubin levels peaking at 10-40

mg/dL by the 10th postoperative day. Although alkaline phosphatase levels can also be

markedly elevated, the AST and ALT levels are typically normal or only mildly elevated.

2. The renin-angiotensin-aldosterone system is integrally involved in the pathophysiolocy and tx

of hypertension. Numerous antihypertensives perturb this system and understaning how

143

different agents affect it important.

3. Streptococcus bovis endocarditis is associated with colorectal cancer. Colonoscopy should be

pursued for futher evaluation.

4. Atherothrombotic strokes occur at rest (unable to wake up in the morning) and have a gradual

onset. Often, the pt experience successive strokes with increasing frequency. Atherosclerotic

risk factors are frequently present in such pts.

5. Epidemiological difference (eg. incubation peroid, route of transmission, risk factors) offer

some of the best means of identifying the virus responsible for a pt’s hepatitis. Confirmation

can be established with serological testing.

6. Nonketotic hyperosmolar syndrome (NKHS) occurs in type 2 DM because the level of insulin

in these pts is sufficient to prevent ketosis, but not hyperglycemia. In most cases, severe

hyperglycemia develops, thereby resulting in glycosuria and severe dehydration. The severe

hyperosmolality is also responsible for lethargy, weakness, altered mental status, focal

neurological deficits and eventual coma. Left arm weakness may make you think of a

cerebrovascular event, but keep in your mind that focal neurological deficits are common in

NKHS. A head CT or MRI may be considered afterwards to rule out a cerebrovascular

process, especially if the blood glucose turns out to be normal.

7. This pt’s presentation and radiologic feature (ie. Symmetrical ‘loose zones’ or

pseudofractures, blurring of the spine) are classic for osteomalacia. Low or low-normal serum

calcium, low serum phosphate, increased serum parathyroid hormone, and low plasma 25-OH

vit D levels are characteristic. Vit D deficiency leads to decreased intestinal calcium and

phosphorus absorption, thereby resulting to hypocalcemia and hypophosphatemia.

Hypocalcemia then stimulates the parathyroid glands. The resultant secondary

hyperthyroidism brings the serum calcium levels to normal or near-normal (esp. in the early

stage), and increases urinary phosphate excretion, thereby worsening the hypophosphatemia.

Vit D deficiency thus causes more marked hypophosphatemia than hypocalcemia, esp in the

early stage. Normal serum calcium, phosphate and parathyroid hormone levels are seen in pts

with osteoporosis and Paget’s disease. The serum alkaline phosphate level is markedly

elevated in Paget’s disease. Hypercalcemia can occur in pts with moderately severe Paget’s

disease following immobilization.

8. Whenever an infective endocarditis is suspected empiric antibiotics should be administered

after drawing blood culture. TEE should be performed after.

9. Symtomatic sinus bradycardia should be treated with iv atropine, followed by transcutaneous

pacing. Medications that slow the HR should be replaced with alternative therapeutic

regimens. If the bradycardia does not resolve, a permanent pacemaker may be necessary.

10. Second degree heart block is identified by prolongation of the PR interval with occasional

dropped beats. In Mobitz type I, the PR interval gradually lengthens until a beat is dropped. In

type II, the PR intervals are of constant length before the dropped beat. The prognosis is better

for Mobitz type I, which is the result of AV node dysfunction (impaired conduction).

Ventricular preexcitation usually results in premature ventricular complexes (PVCs). Impaired

AS node automaticity (SSS) most often results from fibrosis of the sinus node or disease of

the SA nodal artery. Pts may present with bradycardia, lightheadedness, or syncope. On EKG,

this often appears as tachycardia-bradycardia syndrome (bursts of atrial tachyarrhythmia

followed by bradycardia). Atrial reentry leads to atrial tachycardia of abrupt onset and

144

termination. Bundle branch block occurs below the AV node and impedes ventricular

depolarization. The result is a prolonged QRS complex.

11. Hepatic encephalopathy is a central nervous system complication of liver failure secondary to

accumulation of ammonia in blood because of inability of liver to detoxify ammonia into

urea. It is characterized by reversal of sleep cycle, asterixis, progressive coma, and

characteristic delta waves on EEG.

12. Neuroleptic Malignant Syndrome (NMS) typically presents with fever, muscle rigidity,

autonomic instability and mental status change. Serum CK is often elevated. Based on clinical

evidence, dantrolene, a muscle relaxant, is the most comon drug used to the condition,

followed by bromocriptine (a dopamine) agonist and amentadine (an antiviral drug with

dopaminergic properties). Cessation of all dopaminergic antagonists.

13. Sympathetic ophthalmia is also known as “spared eye injury”. It is characterized by an

immune-mediated inflammation of one eye (the sympathetic eye) after a penetrating injury to

the other eye. The typical manifestation is anterior uveitis, but panuveitis, paplillary edema,

and blindness may develop. The pathophysiological mechanism is believed to be the

uncovering of “hidden” antigens. Some antigens contained within the eye are protected from

immunologic recognition by natural barriers. Breaking these barriers results in the uncovering

of “hidden” antigens. An immune response against these antigens can involve autoantibodies

as well as a cell-meidated reaction.

14. Septic arthritis of the hand is a rapidly destructive infection that causes intense pain, swelling,

erythema, and tenderness with active and passive movement. It is diagnosis via joint

aspiration. Tx is with drainage and antibiotics.

Osteomyelitis may complicated puncture wounds, but would be unlikely to cause this pattern of

joint pain and swelling.

Flexor tenosynovitis is inflammation of the flexor tendon sheath on the hand. Pts present with a

finger held in slight flexsion, fusiform swelling, tenderness along the flexor sheath, and pain with

passive extension. In contrast, this pt had pain with flexion.

A midpalmar space infeciton would affect the potential space between the middle, ring and small

finger flexor tendons and the volar interosseous muscles. This condition typically results from

direct trauma.

Paronychia is characterized by pain, swelling, and erythema at the nail folds. It is the most

common kind of hand infection.

15. CT of the chest should be done to look for a thymoma in all newly-dignosed myasthemia

gravis pts.

16. ACEIs at the most common cause of acquired angioedema, which typically begins within one

week of starting the drug. Other adverse effects of ACEIs are cough, hyperkalemia, and

precipitation of acute renal failure in pts with bilateral renal artery stenosis. Pts present with

edema in the face, mouth, lips, tongue, glottis and larynx. Laryngeal edema can cause airway

obstruction and be life threatening. Angioedema occurs due to the pro-inflammatory action of

bradykinin, which promotes edema, inflammation and the sensation of pain.Angiotensin

converting enzyme (ACE) is also known as kininase; it functions to degrade bradykinin. The

first step in mx of angioedema is to check for airway compromise and vasomotor instability,

which require subcutaneous epinephrine administration if present. If airway obstruction fails

to respond to epinephrine, an emergency tracheostomy is done. The ACEIs should be stopped

145

immediately.

17. The lupus anticoagulant, or anti-phospholipid antibody, is a prothrombotic immunoglobulin

that causes a spuriously prolonged PTT. It is an IgM or IgG immunoglobulin that prolongs

PTT by binding the phospholipids used in this assay. It needs to be considered in pts with

clotting disorders and prolonged PTT. Hence, it is strictly a lab artifact and does not actually

promote bleeding. The Russel viper venom test is designed specifically to test for the lupus

anticoagulant and will be prolonged in the disease. With the lupus anticoagulant: 1) The PT

will be normal or slightly prolonged. 2) VWF, bleeding time, platelet count will be normal. 3)

D-dimer will be normal or high. D-dimer is typically elevated in the presence of blood clots.

18. Any HIV-positive pt with bloody diarrhea and a normal stool examination should have a

coloscopy with biopsy to look for CMV colitis. CMV colitis is characterized by bloody

diarrhea with abdominal pain. Colonoscopy shows multiple ulcers and mucosal erosions.

Biopsy demonstrates the presence of large cells with eosinophilic intranuclear and basophilic

intracytoplasmic inclusions (“owl” eye “effect”). The tx of choice is ganciclovir. Foscarnet is

used in case of ganciclovir failure or intolerance.

19. HIV often presents with non-specific symptoms such as weight loss, malaise, and neurologic

changes. Physicians must maintain a high index of suspicion for HIV infection, especially in

pts with known risk factors (drug use).

20. Most cases of active TB in US occur in people who have immigrated to his country from

endemic geographic regions.

21. Pts with Wolf-Parkonson-White syndrome who develop atrial fibrillation with a rapid

ventricular rate should be treated with cardioversion or antiarrhythmics like procainamide. AV

nodal blockers like beta-blockers, CCBs, digoxin and adenosine should be avoided b/c they

can cause increased conductance through the accessory pathway (conducts depolarization

directly from the atria to the ventricles without traversing the AV node.).

22. Neurocysticercosis is the most common parasitic infection of the brain. It is mose prevalent in

rural areas with poorer sanitary conditions and where pigs are raised. Autopsy shows multiple

fluid-filled cysts in the brain parechyma. Creutzfeldt-Jacob disease and Kuru are prion

diseases that cause spongiform encephalopathy, in which intracytoplasmic vacuoles are

evident diffusely throughout the gray matter on microscopy.

23. Campylobacter is the MCC of bloody diarrhea in the US (uncooked poultry-- chicken burger).

The presence of simutaneous severe abdominal pain is a helpful clue. Shigella is very

common cause of diarrhea in the US, and is the second MCC of food-borne illness. Diarrhea

due to Shigella is often bloody and usually occurs in daycare centers or other institutional

settings. E coli—uncooked hamburger. Yersinia enterocolitica—uncooked pork

24. Atopic dermatitis in infancy presents with pruritus and skin lesions typically distributed

symmetrically over the face, scalp, chest and extensor surfaces of the extremities. The diaper

region is typically spared. Lesions usually begin with prutitus alone and evolve to

erythematous excoriated 表 皮 脱 落 papules and plaques that may weep and become

secondarily imperiginized. Atopic dermatitis is the result of decreased skin barrier function

due to improper synthesis of components of the epidermal cornified cell envelop. This allows

allergens ready access to the deeper levels of the epidemis where they may generate the

immune response characteristic of atopic dermatitis. Tx is with improvement of skin barrier

function through the use of mild cleansers and thick, bland emollients and in addition to mild

146

topical anti-inflammatory ointments.

25. In evaluation the asymptomatic evaluation of aminotransferases, the first step is to take a

thorough history to rule out the more common hepatitis risk factors (eg. alcohol or drug use,

travel outside of the country, blood transfusions, high-risk sexual practices).

26. Dapsone is an effective tx for dermatitis herpetiformis (commly occurs in association with

celiac sprue). 1% lindane solution is used as a topical agent in the tx of scabies. Azithioprine

—blistering autoimmune diseases, such as pemphigus vulgaris.

27. The differential dx for chest pain is broad and varies dignificantly depending upon a pt’s age

and risk factors. In young women, anxiety and other psychosocial factors are s common cause

of chest pain (next step mx--reassurance).

28. Lynch syndrome II is distinctly associated with a high risk of extracolonic tumors, the most

common of which is endometrial carcinoma, which develops in up to 43% of females in

affected families. HNPCC subgroups: 1) hereditary site specific colon cancer (Lynch

syndrome I); 2) cancer family syndrome or (Lynch syndrome II).

29. Guillain-Barre syndrome is characterized by ascending paralysis, antecedent history of

infection, and CSF finding of albumino-cytologic dissociation. The tx includes IV

immunoglobulin and plasmapheresis (not corticosteroids).

30. Aortic regurgitation causes widening of the pulse pressure, which can be felt as a “water

hammer” pulse. Lying down and turning to the left brings the heart close to the chest wall and

can make the pt more aware of the forceful heartbeat.

31. Condyloma lata and bilatery symmetrical maculopapular rash involving the entire trunk and

extremities are characteristic of second stage of syphilis. Remember the rash is present on

palms and soles. Serological tests are positive in secondary syphilis.

32. Retinal detachment usually presents with a sudden onset of photopsia and floaters. The most

classic description is that of “a curtain down over my eyes”.

33. Hemodynamically stable supraventricular tachycardia should be treated with vagal maneuvers

followed by adenosine and AV nodal blockers. Unstable pts with supraventricular tachycardia

should undergo DC cardioversion.

34. Constrictive pericarditis and restrictive cardiomyopathy can be difficult to distinguish. A low

voltage EKG can occur in either condition. The pt’s other issue are key to making the dx.

New-onset diabetes mellitus and liver test abnormalities along with congestive heart failure

suggest hemochromatosis, which can cause restrictive cardiomyopathy. Iron studies and an

echocardiogram would help confirm this dx.

35. Laxative abuse is characterized by very frequent, watery, nocturnal diarrhea. The diagnosis

can be confirmed with the characteristic biopsy finding of dark brown discoloration of the

colon with lymph follicles shining through as pale patches (melanosis coli).

36. Always suspect lacunar stroke if a pt with a limited neurologic deficit. The typical lacunar

stroke scenarios are pure motor stroke, pure sensory stroke (thalamus), ataxia-hemiparesis,

and dysarthria clumsy hand syndrome. The principle cause of lacunar stroke is hypertension.

Memorize 4 common lacunar syndromes:Syndrome Pathology PresentationPure motor hemiparesis

Lacunar infarction in the posterior lime of the internal capsule

Unilateral motor deficit (face, arm, and, to a lesser extent, leg); mild dysarthria; NO sensory, visual or higher cortical dysfunction

Pure sensory Stroke in the Unilateral numbness, paresthesias, and

147

stroke ventroposterolateral nucleus of the thalamus

hemisensory deficit involving the face, arm, trunk, and leg

Ataxic-hemiparesis Lacunar infarction in the posterior limb of the internal capsule

Weakness that is more prominent in the lower extremity, along with ipsilateral arm and leg incoorfination

Dysarthria-clumsy hand syndrome

Lacunar stroke at the base of pontis

Hand weakness, mild motor aphasia, NO sensory abnormalities

37. UTI is common in diabetes, the elderly, and postoperative pts. Elderly pts with UTI may

present with confusion and an absent of geintourinary complaints. The finding on UA, taken

in the context of the history and physical exam, help to confirm UTI dx.

Trimethoprim/sulfamethoxazile is appropriate antibiotic therapy for UTI.

38. Chronic pancreatitis is an inflammatory condition characterized by chronic abdominal pain

and pancreatic insufficiency. A fecal elastase study is the most sensitive and specific test to

diagnose pancreatic exocrine failure.

39. The S4 is a low frequency sound heard at the end of diastole just before S1 that is commonly

associated with left ventricular hypertrophy from prolonged hypertension.

40. Migratory thrombophlebitis and atypical venous thromboses are suggestive for chronic DIC,

most likely due to some visceral malignancy (lung, pancreas, stomach and prostate).

41. Zollinger-Ellison syndrome (gastrinoma) is associated with MEN I (primary

hyperparathyroidism, pituitary tumors and enteropancreatic tumors).

42. Paracentesis can be used for both dx and therapeutic purpose in pts with ascites.

43. Corticosteroid-induced avascular necrosis of the femoral head usually presents as progressive

hip or groin pain without restriction of motion range and normal radiograph on early stages.

MRI is gold standard for the dx of avascular necrosis of hip.

44. In elderly pts with multiple medications, it is very important ot regularly assess their general

well-being and current medication list, as well as to promote pt adherence to the appropriate

regimen. Hemorrhages are seen as hyperdense areas on CT scan, while infarcts

charateristically have hypodense parenchymal areas on CT scan.

1/13/10

1. Diabetic cystopathy manifests as overflow incontinience. The pathology involves weak

detrusor contraction secondary to autonomic neutopathy. The initial treatment usually

involves strict voluntary urinary voiding schedule and bethanechol; if there is no response,

intermittent catheterization is recommended. Oxybutynin is an antispasmodic agent used for

urge bladder incontinence. Tamsulosin is an alpha-1 blocker that is used for BPH.

2. Suspect squamous cell carcinoma of the lungs in a pt with a significant smoking history (ie.

More than 45 years), hypercalcemia and a hilar mass. Small cell carcinoma of the lung usually

causes papaneoplastic syndrome such as ACTH production and SIADH. Adenocarcinoma is

usually a peripheral lesion that is usually not associated with hypercalcemia. It is typically

associated with hypertrophic pulmonary osteoarthropathy.

3. Lifestyle modification should be the first-line intervention for newly diagnosted stage 1

hypertension. Reduction of alcohol and sodium intake, weight loss, and aerobic exercise can

all decrease BP. Quitting smoking will decrease a pt’s risk of cardiovascular disease, but does

not improve BP itself.

148

4. The differential dx of normotensive pts with hypokalemia and metabolic alkalosis include: 1.

Diuretic use; 2) Surreptitious 秘 密 的 vomiting; 3) Bartter’s syndrome; 4) Gitelman’s

syndrome. Classic Bartter’s syndrome ussually presents early in life as polyuria, polydipsia,

and growth and mental retardation; howwver, such presentation can occur much later. The

underlying pathology is defective sodium and chloride reabsorption in the thick ascending

limb of the loop of Henle, thereby resultig in hypovolemia and consequent activation of the

renin-angiotensin aldosterone system (RAAS). Activated RAAS then causes an increase in

potassium and hydrogen ion secretion, which eventually leads to hypokalemia and metabolic

alkalosis. Pt with Batter’s syndrome have hypokalemia, urinary chloride level above 20mEq/L

(most often above 40 mEq/L), metabolic alkalosis, and normal BP.

Primary hyperaldosteronism and renin-secreting tumors are characterized by hypertension,

metabolic alkalossi and hypokalemia. Measurement of the plasma renin activity (PRA) and

plasma aldosterone levels (PA) may be used to distinguish between these two diagnoses. In

primary hyperadosteronism, PRA is supressed and PA is elevated; whereas, in renin-secreting

tumors, both PRA and PA are elevated.

The MCC of hypokalemia seen in clinical practice is the use of diuretics. It may be very difficult

to differentiate this diagnosis from Batter’s syndrome, especially if there is no documented

diuretic use in pt’s medical records. If diuretic abuse if strongly suspected, measurement of the

urine diuretic level can be performed.

5. Drug induced intersitital nephritis is usually cuased by cephalosporins, penicillins,

sulfonamides, NSAIDs, rifampin, phenytoin and allopurinol. Pts present with althralgias, rash,

renal failure and the urinalysis will show eosinophiluria.

6. Pemphigus vulgaris is a mucocutaneous blistering disease that is characterized by flaccid

bullae and intercellular IgG deposits in the epidermis. Autoantibodies are formed against

desomoglein, an adhesion moelcule. Bullous impetigo is a contagious infection of the skin

caused by Staph, Strep, or both. The bullous type is associated with Staphy aureus infection.

The lesion consist os macules, vesicles, bullae, and honey colored crusts, which leave red

denuded areas when removed.

7. Until proven otherwise, suspect squamous cell carcinoa in any pt with a chronic scar that

deveops into a non-healing, painless, bleeding ulcer. Punch biopsy is used to confirm the

diagnosis of squamous cell carcinoma of skin.

8. Nasal polyps are often associated with chronic rhinosinusitis, asthma, and aspirin- or

NSAIDs-induced bronchospasm in a condition known as aspirin-excerbated respiratory

disease (AERD). The frequently cause symptoms of bilateral nasal obstruction, nasal

discharge, and anosmia.

9. Shingles is characterized by a vesicular eruption that occurs in a dermatomal distrubution and

is often preceded by pain. It is caused by the varicella zoster virus, which also causes chicken

pox.

10. Riluzole is a glutamate inhibitor that is currently apporved for use in pts with amyotrophic

lateral sclerosis (a neurodegerative disease). Although it cannot arrest the underlying

pathological process, it may prolong surviving and the time to tracheostomy. Its side effects

are dizziness, nausea, weight loss, elevated liver enzyme and skeletal weakness.

Corticosteroids, iv IgG and cyclophosphamide have no role in its mx.

11. Acute deep vein thromboses should be treated with a combination of warfarin and heparin for

149

the first 5 days. After 5 days, the heparin may be discontinued if the pt’s INR is therapeutic.

Warfarin should be continued for at least 6 months in most pts with a first-time clot. A

progressing clot in a pt with a sub-therapeutic INR requires bridging heparin until the INR is

therapeutic. Thrombolytics are indicated for pts with hemodynamically significnat pulmonary

emboli. They are not routinely used for pt with DVTs. Inferior vena cava filters may be used

in pts with lower ectremity DVT where anticoagulation is C/I. This pt has no C/I to

anticoagulation.

12. The absence of peristaltis waves is the lower two-thirds of the esophagus and significant

decrease in lower esophageal sphincter (LES) tone are characteristic for esophageal

dysmotility associated with scleroderma.

13. The clues for dx of syncope caused by arrhthmia include sudden onset of syncope without

warning signs, presence of structural heart disease (post-infarction scar and probable mitral

regurgitation because of the characteristic murmur), and frequent ectopic beats. Another

disturbances predisposing to ventricular arrhthmia,

14. It appears that this pt developed parkinsonian features from the illicit agent MPTP.

Amantadine was first in India to treat influenza epidemics. It was noticed that the drug has a

beneficial effect in some Parkinsonism pts. The drug then changed its indication from treating

influenza to treating Parkinson’s disease as well. The problem is that its effects are not long

lasting. It can also cause depression and hypotension.

15. An isolated, round, smooth-borded, ring-enhancing intracranial lesion on contrast CT scan of

the brain in an immunocompetent pt with a known extracranial bacterial infection is most

likely a brani abscess. Aerobic and anaerobc streptococci and Bacteroides species (anaerobes)

are the MCC organisms (more than strap).

16. Iron studies are indicated in pts who present with microyctic/hypochromic anemia. These are

useful in confirming the dx of iron-deficiency anemia, which is the usual cause of

microcytic/hypochromic anemia, and in ruling out other causes. microcytic/hypochromic anemias Findings in Iron studies1. Iron-deficiency anemia Typically depressed serum iron level, increaed TIBC, and

decreased serum ferritin level2. Thalassemia Normal to high serum iron and ferritin levels3. Anemia of chornic disease Below normal TIBC, Normal or increased serum ferritin

level4. Sideroblastic anemia Normal to high serum iron and ferritin values

17. Simple renal cysts are almost benign and do not require further evaluation. Know how to

recognize these on a CT scan. For dx, make sure does not have the following findings: 1) A

multilocular mass; 2) Thickened, irregular walls; 3) Thickened septae within the mass; 4)

contrast enhancement.

18. The most common organisms responsible for the development of epiglottitis, esp in the adult

population, are Haemophilus influenzae and Streptococcus pyogenes. Epiglottitis is a medical

emergency, and rapid tx must be initiated in order to prevent obstruction of the airway.

19. Paget disease of bone (osteitis deformans) is associated with normal serum calcium and

phosphate levels and increased alkaline phosphatase and urinary hydroxyproline levels.

20. Suspect HUS in a child who has recently recovered from a diarrheal illness and presents with

acute renal failure, microangiopathic hemolytic anemia, fever, thrombocytopenia and

150

characteristic peripheral smear finding of schistocytes. ITP is characterzied by an isolated

thrombocytopenia. On peripheral smear, the morphology of red cells is normal. * Extremely

high yield!!

21. Acute epididymitis in younger pts is usually caused by sexually transmitted organisms such as

C trachomatis or N. gonorrhea. In older men it is usually non-sexually transmitted and is

caused by G- rods (E.coli).

22. Beta-thalassemia minor occurs in people heterozygous for the beta-hemoglobin gene. It is

common in people in Mediterranean (Greek) decent and manifests as a hypochromic

microcytic anemia of moderate severity.

23. GERD is characterized by a retrosternal burning sensation after eating and with lying down. It

may also be accompanied by hoarseness and chronic cough, esp while recumbent. The initial

tx is and H2-receptor antagonist or a proton pump inhibitor.

24. Learn the clinical presentation, pattern of organ involvement, and complications of mumps.

Orchitis is one of the most frequent complications of mumps in adults.

25. The major cause of anemia in pts with end stage renal disease is deficiency of erythropoietin.

The anemia is normocytic and normochromic. The tx of choice is recombinant erythropoietin,

which is started if the Hb<10g/dL. The most common side effects are worsening of

hypertension (30% of pt), headaches (15% of pts) and flu-like syndrome (5%); Red cell

aplasia (a rare, but potential side effect).

26. All chronic hepatitis C pts with elevated ALT, detectable HCV RNA and histologic evidence

of chronic hepatitis of at least moderate grade are candicates for antiviral therapy with

interferon and ribavirin. Combined therapy with interferon and lamivudine is used in tx of

HBV.

27. Primary hyperparathyroidism is the MCC of hypercalcemia in ambulatory pts. Hypercalcemia

due to primary hyperparathyroidism is associated with elevated or inappropriately normal

serum PTH levels (normal 8-24 pg/ml).

28. In acute appendicitis, the initial peri-umbilical pain is referred pain and visceral in nature;

however, pain shifts to the right lower quadrant with involvement of the parietal pritoneum

and becomes somatic in nature.

29. Third degree, or complete, heart block is marked by complete independence of the P waves

and QRS complexes, both of which contract at completely separate rates. Tx for 3rd degree

heart block is expeditious pacemaker placement, as the rhythm can degenerate into ventricular

tychycardia or ventricular fibrillation.

30. Primidone is an anticonvulsant agent which can be used to treat benign essential tremors. Its

administration can precipitate acute intermittent porphyria (colichy abdominal pain,

confusion, headaches, hallucination and dizziness), which can be diagnosed by checking for

urine porphobilinogen.

31. Hypertension, hypokalemia and suppressed plasma renin activity are highly suggestive of

primary hyperaldosteronism. The MCC of primary hyperaldosteronism is aldosterone-

producing adenoma.

32. Prolactin-producing lactotroph adenoma is the most common primary pituitary tumor.

33. Cellulitis is an infection of the skin and subcutaneous tissue. Predisposing risk factors for

cellulitis include obesity, venous insufficiency, disruption of cutaneous barrier, and previous

cellulitis. Red, edematous skin that is hot to the touch is consistent with cellulitis. Fever,

151

lymphagitis and regional lymphadenopathy are often seen. Group A Streptococcus and

Staphylococcus aureus are the most common infectious agents.

34. Suspect pseudotumor cerebri in a young obese female with a headache that is suggestive of a

brain tumor, but with normal neuroimaging and elevated CSF pressure. The tx includes

weight reduction and acetazolamide. Shutting or optic nerve sheath fenestration may be

performed to prevent blindness (the most significant complication if left untreated).

35. Free air underneath the diaphragm signifies bowel perforation and requires urgent laparptomy.

36. Amidarone can cause pulmonary toxicity and should be avoided in pts with preexisting lung

disease since they have less pulmonary reserve.

37. Insulin resistance plays a central role in the pathophysiology of non-alcoholic fatty liver

(biopsy shows macrovascular fat deposition with displacement of the nucleus to the

peripheral) disease by increasing the rate lipolysis and elevating the circulating insulin levels.

38. Fibromyalgia is a chronic widespread pain disorder associated with fatigue, poor sleep, and

depression. Pts have multiple trigger points of tenderness.

39. Sarcoidosis is most common in African-American woman during the third and fourth decades

of life. The lungs are the most commonly affected organ system. When symptomatic, cough,

erythema nodosum, anterior uveitis, and arthritis may be seen. Hilar adenopathy and reticular

opacities on CXR are classic findings.

40. Currently, quantitative estimation of stool fat (72-hour fecal fat collection) is the gold standard

for the diagnosis of steatorrhea.

41. Long-term supplement oxygen therapy has been proven to prolong survival in pts with COPD

and hypoxemia. Criteria for therapy include PaO2 <55, SaO2 <88%, erythrocytosis

(hematocrit >55), or evidence of cor pulmonale.

42. Pts with multiple myeloma are at increased risk for infection due to both a total decrease in

functional antibodies and leukopenia that develops as the bone marrow is filled with

malignant plasma cells. Multiple myeloms mnemonic—CRAB: clacium (hypercalcemia),

renal impairment, anemia, and bones (bone pain, lytic lesions, fractures).

43. HIV pts with CD4 cell counts less than 50/mL require MAC prophylaxis with azithromycin or

clarithromycin. Oral ganciclovir may be effective for prophylaxis against CMV infection in

HIV pts with CD4 cell counts below 50/mL, however, it is not currently recommended for this

purpose.

44. Molluscum contagiosum is characterized by single or multiple rounded, dome-shaped papules

with central umbilication. The causative agent is poxvirus. It is ussally a self-limited infection

and resolves spontaneously in immunocompetent individuals. Genital lesions should be

treated to prevent sexual transmission. The best form of tx is curettage or application of liquid

nitrogen.

1/16/10

1. A combination of HBV immune globulin and lamivudine is the most effective measure to

prevent recurrent HBV infection after liver transplantation.

2. Pts with severe aortic stenosis can have anginal chest pain due to increased myocardial

oxygen demand.

3. The dx of chronic mesenteric ischemia is suspected in pts with unexplained chronic

abdominal pain, weight loss, and food aversion. Evidence of associated atherosclerosis

152

disease is usually present. Physical findings are usually nonspecific. Abdominal exam may

reveal a bruit (50% of pts).

4. Tamoxifen reduces the risk of breast cancer in pts who have an increased risk of developing

breast cancer.

5. Noninvasive test for H.pylori should be the first step in the mx of pts with dyspepsia under 45

years of age who do not have alarming symptoms. GI referral is required in pts older than 45

years of age or those with alarming symptoms at any age, due to the small risk of gastric

cancer. Such pts should be considered for endoscopy and H.pylori testing.

6. Osteromyelitis most often presents as focal pain, fever, chills, and a grossly elevated ESR. It

is particularly common in iv drug abusers and, in that setting, often involves the spine.

7. Appropriate pt education about photo-protection, esp in the high-risk group, is important. Sun

avoidance remains as the best method of photo-protection. Know this: Sunscreens should be

applied 15-60 min prior to sun exposure to allow enough time fro protective film

development. * Extremely high yield!!

8. Collapsing focal and segmental glomerulosclerosis is the most common form of

glomerulopathy associated with HIV. Typical presentation of focal segmental

glomerulosclerosis includes nephritic range proteinuria, azotemia, and normal sized kidneys.

* Extremely high yield!!

9. A MMSE (mini-mental state examination) score of less than 24 points (total 30) points is

suggestive of dementia. CT finding of diffuse cortical and subcortical atrophy are

characteristic for Alzheimer’s dementia.

10. Lithium is a common cause of nephrogenic diabetes insipidus. Lithium-induced nephrogenic

DI is treated with salt restriction and discontinuation of lithium. Other drugs: demeclocycline,

foscarnet, cidofovir, and amphotericin.

11. Consider the natural history of a disease when evaluating the effectiveness of a drug in a trial.

12. Cardiac dysfunction associated with hemochromatosis can be reversed with early

identificantion of the disease and tx (phlebotomy). Symmetric thickening of the ventricle

helps to differentiate restrictive from hypertrophic cardiomyopathy, in which the

interventricular septum is thickest.

13. Lowering LDL cholesterol levels is the primary goal of lipid modification, and should be

prioritized over increasing HDL levels. Fibrates (increase HDL by 10-20%, decrease LDL

modestly, and decrease triglycerides significantly) and nicotinic acid (niacin, elevates HDL

levels by 15-35% and lowers LDL and triglyceride level as well) are the drugs of chice for

raising HDL. Fibrates are often preferred because they have fewer side effects.

14. Ventricular tachycardia in the presence of a stable BP does not warrant cardioversion. The

best tx is loading lidocaine or amidarone (drug of choice).

15. Hairy cell leukemia is characterized by lymphocytes with fine, hair-like irregular projections

and a tartrate-resistant acid phosphatase (TRAP) stain. The bone marrow may become

fibrotic; thus leading to dry taps. Cladribine is the drug of choice for pts with hairy cell

leukemia.

Psychiatry

1. Pts who develop acute dystonia from the use of antipsychotics should be treated with

anticholinergic (benztropine or trihexyphenidyl) or antihistamines (diphenhydramine). It is

153

characterized by muscle spasms or stiffness, tongue protrusions or twisting, opisthotonus and

oculogyric crisis (a forced, sustained elevation of the eyes in an upward position).

Beta-blockers such as propranolol can be used to treat akathisia, which is a condition that can

occur at any time during the course of antipsychotic use. These pts report a subjective feeling of

restlessness and may pace constantly, unable to sit still.

Dantrolene is used to treat neuroleptic malignant syndrome (NMS).

When Parkinsonism develops as a side effect from antipsychotic use (usually within 4 days to 4

months of tx), the drug of choice is an anticholinergic agent such as benztropine.

2. Pts taking monoamine oxidate inhibitors like phenelzine should avoid foods high in tyramine,

as the combination can result in hypertensive crisis (most appropriate to monitor--BP).

3. If a pt’ family disagrees with his living will and demands care that contradicts the pt’s written

wishes, the best initial step is to discuss the matter with the family. If a discussion fails to

resolve the situation, then the hospital’s ethics committee should be consulted.

4. For the dx of major depressive disorder, pts should have at least 5 of 8 depressive symptoms

for at least 2 weeks, with the symptoms causing significant functional impairment. Primary

insomnia is characterized by the isolated symptom of having difficultu in falling or staying

asleep.

5. Rationalization is the offering of a rational, logical reason for an upsetting event rather than

the real reason. Intellectualization is a neurotic defense mechanism that allows the individual

to behave in a detached manner by separating emotional concerns from intellectual curiosity

about an issue that causes anxiety. An example would be a recently laid of woman who then

starts researching the political causes of the country’s recent economic downturn.

6. The antidepressant of choice in depressed pts suffering from sexual dysfunction (whether as a

medication side effect or as a pre-existing condition) is bupropion.

7. When choosing between medications, look for a medication that is effective for more than one

problem. Buproprion is an example of a medication with two uses, as it is an antidepressant

and a smoking cessation aid.

8. Pts with psychiatric dx can give informed consent as long as their judgment and decision-

making abilities are determined to be intact.

9. Propranolo is the drug of choice for treating performance-related anxiety. Buspirone is used to

treat generalized anxiety disorder, but not specific phobias. Clomipramine is a TCA that is

often used in the tx of OCD.

10. Methylphenidate is a CNS stimulant that is frequently used to treat attention deficit

hyperactivity disorder. Common side effects include nervousness, decreased appetite, wight

loss, insomnia, and abdominal pain. Prolonged therapy has been shown to cause mild growth

retardation or weight loss. Methylphenidate should not be used in children younger than 6

years old because safety and efficacy in this age group have not been evaluated.

11. If a medical error occurs, the physician must take responsibility for the situation, inform the pt

of precisely what happened, and offer a sincere apology to the pt and his family.

12. Odd behavior, magical thinking , and a lack of close friends are common features of

schizotypal personality disorder. While individual with schizoid personality disorder also lack

close friends and have a restricted range of emotional expression, they do not have eccentric

behavior or odd thinking. Those with avoidant personality disorder want friends but fear

ridicule.

154

13. Altruism involves minimizing internal fears by serving others. It is an example of a mature

defense mechanism in which individuals provide a helpful, gratifying service to others as a

means of quelling their own anxieties.

Reaction formation is the transformation of an unwanted thought or feeling into its opposite. An

example would be having a bias against a particular race and then embracing that race as ideal and

wonderful.

Sublimation is a mature defense mechanism that allows for unacceptable impulses to be channeled

into more acceptable activities. An example would be a man with a fiery temper who channels shi

anger into athletic pursuits.

14. Benzodiazepines and other sedative hypnotics are contraindicated in pts with breathing-

related sleep disorders.

15. When a woman presents complaining of dyspareunia, the presence of a general medical

condition or underlying psychological stressors should first be ruled out. Medical causes of

dyspareunia include endometriosis, local infections, vulvar or vagianl growthsm and estrogen

deficiency. Psychological causes include somatization disorder, pain disorder, or dyspareunia

without associated symptoms. Tx should target the underlying cause.

16. Bupropion is an antidepressant that produces its effects primarily through the inhibition of the

re-uptake of norepinephrine, dopamine, and serotonin. Contraindications to bupropion use

include seizure disorders, eathing disorders, or other conditions predisposing to seizure (eg.

concurrent alcohol or benzodiazepine use). Individuals with anorexia nervosa or bulimia

nervosa frequently develop electrolytes abnormalities that can precipitate seizures.

17. Pathological gambling is the likely dx in an individual with a chronic history of gambling and

a seeming inability to stop. Significant financial losses or damaged relationships are common

consequences of this behavior.

18. First line therapy for major depression is prescription of a SSRI such as sertraline. Major

depression is diagnosed when at least five of the following symptoms are present in a 2-week

period (including either depressed mood or anhedonia). Depressed mood; Sleep disorder;

Interest deficit (anhedonia); Guilt (worthlessness, hopelessness, regret), Energy deficit;

Concentration deficit; Appetite disorder; Psychomotor retardation or agitation; Suicidality

(SIGECAPS). Because TCAs cause a number of significant side effects—including those

considered particularly undesirable in the elderly (eg. orthostatic hypotension)—they are not

recommended for this pt. Moreover, there is no indication for prescribing bezodiazepines in

the tx of depression.

19. Pts who are acute dangers to themselves should be hospitalized (involuntarily, if necessary)

for tx and stabilization. This principle applies to minors as well, even when parents or

guardians disagree.

20. Reaction formation is the transformation of an unwanted thought or feeling into its opposite.

21. Always have a high index of suspicion for physical/sexual abuse in children (esp females)

with sudden behavioral problems, families with unstable economic backgrounds, or parents

with a history of drug/alcohol abuse.

22. Displacement is a psychological defense machenism in which unacceptable feelings about one

object or person are displaced onto anther, “safer” object or person.

23. The grandiose delusions, racing thoughts, distractibility, and irritablility demonstrated by this

pt are highly suggestive of mania. Lithium is a first-line medication in the tx of bipolar

155

disorder. This pt, however, has impaired renal function as evidence by his elevated BUN and

creatinine levels, and since lithium is excreted though the kidneys and has been shown to

cause nephrogenic diabetes insipidus after prolonged use, impaired renal function is a C/I for

lithium therapy. Suitable alternatives to lithium include valporic acid or carbarmazepine, both

of which are metabolized by the liver.

24. Pts have the right to know their diagnosis. If a family member requests that the dx not be

revealed to the pt, the underlying reasions should be ecplored before deciding how to preceed.

25. All atypical antipsychotics, including olanzapine, can cause weight gain, hyperglycemia,

dyslipidemia, and hypertension. Routine monitoring for the development of these side effects

is recommended in pts taking these medications.

26. Indecisiveness, a fear of being alone, and clingy and submissive behavior are all suggestive of

dependent personality disorder.

27. Confidential pt information should only be disclosed to fellow health care workers who are

directly involved in the pt’s care. Additionally, care should be taken to avoid discussing a pt’s

medical condition in public areas where comments might be overheard. Inappropriate

inquiries form colleagues curious about a pt’s medical condition should be politely but firmely

rebuffed.

28. One of the most common side effects of ECT is amnesia, both retrograde and antegrade.

29. Antipsychotics cause hyperprolactinemia by blocking dopamine activity along the

tuberoinfundibular pathway.

The nigrostriatal pathway extends form the substania nigra to the basal ganglia, and is involved in

the coordination of movement. Increase dopamine—chorea and tics; decrease—Prakinson.

The mesolimbic pathway extends from the ventral tegmental area to the limbic system. Increased

dopamine—euphoria accompanying drug use, as well as delusion and hallucinations experienced

by pts with schizophrenia; decrease—therapeutic effects of antipsychotics.

30. Lithium exposure in the 1st trimester of pregnancy causes a twenty-fold increase in the risk of

Ebstein’s anomaly, a cardiac malformation (a malformed and inferiorly attached tricuspid

valve that causes atrialization of the upper right ventricle and a decrease in the size of the

functional right ventricle). In the later trimester, goiter and transient neonatal neuromuscular

dysfunction are of concern.

31. Depot antipsychotics such as fluphenazine and haloperidol are the tx of choice for

schizophrenic pts who suffer replases due to tx noncompliance.

32. Cocaine abuse should be suspected in an individual presenting with weight loss, behavioral

changes, and erythrema of the turbinates and nasal septum.

33. Schizophreniform disorder can be differentiated from schizophrenia by the duration of

symptoms. To diagnosis schizophreniform disorder symptoms must last for more than one

month but less than six months, while the dx of schizophrenia requires symptoms to be

present for at least six months.

34. The likelihood a schizophrenic pt will relapse is decreased if conflicts and stressors in the

home environment are kept to a minimum.

35. Pts of all ages should be given the opportunity to speak with the doctor alone.

36. When child abuse is suspected, the following steps should be performed: 1) Complete

physical examination. 2) Radiographic skeletal survery, if needed. 3) Coagulation profile (if

multiple bruises are present). 4) Report to Child Protective Service. 5) Admit to hospital if

156

necessary. 6) Consult psychiatrist and evaluate family dynamics.

37. The disscociative disorders are characterized by forgetfulness and disscociation. Dissociative

fugue is the only condition within this group that is associated with travel.

Dissociative identity disorder, formerly known as multiple personality disorder, is characterized

by the presence of two or more distinct identities that alternatively assume control of the preson’s

behavior. Amnesia regarding important personal information about some of the identities is

observed.

Dissociative amnesia is characterized by the presence of one or more episodes of inability to recall

important personal information. The memory disturbance is usually related to a traumatic or

stressful event and is too extensive to be considered ordinary forgetfulness.

Derealization disorder describes the state of experiencing familiar perosons and surroundings as if

they were strange or unreal.

38. Always rule out hypothyroidism in pts who present with symptoms of depression.

39. Obsessive-compulsive personality disorder is characterized not by obsessions or compulsions,

but more by a need for perfection, over-attention to detail, stubbornness, and a need to strictly

follow rules. Individuals often do not identify their behavior as abnormal. OCD is marked by

obsessions that cause anxiety, resulting in compulsive behaviors to decrease the anxiety.

40. The greatest risk factor for committing suicide is a past history of suicide attempt(s).

41. Adjustment disorder is characterized by the development of emotional or behavioral

symptoms in response to an identifiable stressor that causes a significant impairment in the

pt’s life.

Acute stress disorder is very similar to PTSD, with anxiety symptoms developing after

experiencing or witnessing an event that involved actual or threatened death or serious injury. The

symptoms last not longer than one month.

PTSD is characterized by symptoms of anxiety that develop after experiencing or witnessing an

event that involved actual or threatened death or serious injury. Pts with PTSD often re-live the

event through flashbacks or nightmares. Avoid stimuli realated to the trauma, may hypervigilance.

Symptoms of PTSD last longer than one month.

42. Generalized anxiety disorder is characterized by excessive worry over many aspects of one’s

life that causes significant impairment in functioning. Additional symptoms such fatigue,

restlessness, difficulty concertrating, irritabilitym and sleep disturbance are commonly seen in

pts with GAD.

43. Delusional disorder is an uncommon condition characterized by isolatedm non-bizarre

delusions in an otherwise high-functioning individual. Hallucinations, disorganized though,

and functional deterioration do not manifest in pts with delusional disorder, when present,

they are suggestive of a more expansive psychosis. Antipsychotic medications are often used

to treat pts with delusional disorder, but have not proven very effective.

Brief psychotic disorder is characterized by delusions, hallucinations, and disorganized thought

that last for no more than one month.

44. Eating disorder, not otherwise specified is a DSM-IV category that includes all eating

disorders that do not meet the criteria for any one specific eating disorder.

1/17/10

1. Pts with somatization disorder benefit from regularly scheduled appointments intended to

157

reduce the underlying psychological distress.

2. The DSM-IV criteria for many psychiatric conditions required that symptoms be severe

enough to cause significant functional impairment of a specified duration of time. (His levevel

of functioning at work and at home has not been impaired.)

3. Altered levels of the neurotransmitter serotomin play an important role in the development of

OCD.

4. A pregnant woman has the right to refuse tx, even if it places her unborn child at risk.

5. All depressed pts should be screened for suicidal ideation. Actively suidical pts will often

need to be hospitalized for stabilization and to maintain their safety.

6. A delusion is a fixed, false belief not consistent with culture norms. Individuals with

grandiose delusions typically believe they have special powers, extraordinary

accomplishments, or a special relationship with God.

7. PCP and LSD intoxication present similarly, but agitation and aggression occur more often in

pts using PCP. Visual hallucinations and intensified perceptions are hallmarks of LSD use. Pts

wth PCP intoxication usually present with distinct behavioral changes such as physical

aggression, severe agitation, impulsivity, impaired judgement, psychosis, paranoia, or

hallucinations. These symptoms develop shortly after ingestion of the drug. Physical signs

such as nystagmus, hypertension, tachycardia, ataxia, dysarthria, muscle rigidity, seizure, or

coma may also be evident. LSD is a hallucinogen. Shortly after ingesting this drug,

individuals may experience mood impairment, hallucinations, subjective perceptual

inrensification (ie, colors are richer, tastes are heightened, and sensation is enhanced),

depersonalization, and illusion. In addition, two or more of the following signs are often

present: sweating, tachycardia, papillary dilatation, palpitations, tremors, and poor

coordination.

8. Narcissistic personality disorder is characterized by an exaggerated sense of self-importance,

feelings of entitlement, egocentrism, and a lack of empathy for others.

9. Some of the more serious adverse effects associated with lithium include nephrogenic

diabetes insipidus, hypothyroidism, and Ebstein’s anomaly in the fetus.

10. Childhood disintegrative disorder is a rare pervasive developmental disorder that occurs more

commonly in males. It is characterized by a period of normal development for at least two

years, followed by a loss of previously skills in at least two of the following areas: expressive

or receptive language, social skills, bowel or bladder control, or play and motor skills.

Autism is another type of pervasive developmental disorder that is also more frequently seen in

males. The onset of symptoms always occurs before the age of 3 years. The condition is

characterized by qualitative impairments in communication and social interaction. These pts tend

to indulge in repetitive, stereotyped behavior with stange preoccupations. Although the child in

this scenario does have autistic symptoms, his normal development until the age of 4 supports a dx

of childhood disintegrative disorder.

Occurring almost exclusively in female, Rett syndrome is a rare pervasive developmental disorder.

The condition is characterized by an initial period of normal development (typically until six

months of age) followed by the loss of hand coordination and the development of peculiar

stereotyped hand movements. Other common features include a deceleration of head growth, poor

coordination, seizures, ataxia, mental retardation, and diminished social interaction.

Asperger syndrome is characterized by a qualitative impairment in social interaction and

158

restricted, repetitive, and stereotyped patterns of behavior. Unlike pts with autictic disorder,

children suffering form Asperger syndrome have normal cognitive and language development.

ADHD presents before age seven years and is characterized by inattention, hostile behavior.

Children with this condition are disobedient, irritable, spiteful 恶意的 and argumentative. They

often blame others for their failings.

11. If a pt presents with refractory mania despite therapy with a mood stabilizer, a urine

toxicology screen (to ensure had not recently taken cocaine or amphetamine, as usage of

either stimulant can result in a manic presentation) and mood stabilizer drug levels should be

obtained in the initial evaluation.

12. Pregnant women with a current of previous dx of anorexia nervosa are at risk for numerous

complications, including miscarriage, intrauterine growth retardation, hyperemeiss, premature

birth, cesarean delivery, and postpartum depression (not postpartum psychosis). Osteoprosis is

also a common finding in anorexic pt, whether pregnant or not. * Extremely high yield!!

13. The following signs and symptoms are indications that psychiatric hospitalization is

necessary: homicidal ideation, suicidal ideation, grave disability, gross disorganization;

agitated or threatening behavior, or severe symptoms of substance intoxication or withdrawl.

14. Pt confidentiality should not be maintained if it would endanger the health and welfare of

others. In cases of sexually transmitted diseases, public health laws require the reporting of

the pt and any known sexual partners to the local health department authorities.

15. In severely depressed pts with active suicidal thoughts, antidepressants should be started

immediately. This is also true for depressed pts suffering from terminal illnesses.

16. Weight gain is a significant adverse effect associated with olanzapine. * Extremely high

yield!!

17. Adolescent seeking a physician’s care for contraception, prenatal care, sexually transmitted

diseases, or substance abuse may be treated without parental consent and are entitled to strict

patient confidentiality.

18. Children with oppositional defiant disorder are disobedient and argumentative. Although they

may be hostile, they do not seriously violate the rights of others. The presence of four or more

of the following symptoms is required for dx of ODD: short temper, argumentativeness with

adults, noncompliance with commands or rules, blaming others, and vindictiveness.

Conduct disorder is characterized by a persistent pattern of behavior in which the basic rules of

society and the rights of others are violated. Children with conduct disorder frequently lie, steal,

destroy property, set fires, and demonstrate cruelty to animals and people.

Antisocial personality disorder presents in the same fasion as conduct disorder but occurs in adults

ages 18 years or older.

19. Avoid benzodiazepine use in pts with PTSD (b/c PTSD is strongly associated with substance

abuse, the prescribing of addictive medications like benzodiazepine is strongly avoided when

possible). Tx of PTSD is best accomplished with a combination of SSRI and exposure or

cognitive therapy. Cognitive therapy involves separating the recurrent flashbacks from the

anxiety they produce. This form of therapy is particularly helpful for those pts who avoid

stimuli that are reminders of the traumatic event.

20. When a pt refuses potentially life-saving tx, it is important to fully discuss the specific reasons

for his decision before honoring it. If the pt continues to refuse medical intervention despite

clarification and reassurance from the physician, then his decision should be respected and the

159

tx withheld.

21. A rare sleep disorder, narcolepsy is characterized by episodes of irresistible, refreshing sleep

during the daytime and one or more of the following traits: cataplexy (diffuse muscle

weakness leading to collapse), hypnogogic and hypnopompic hallucination (hallucinations

wihile falling asleep and waking), and sleep papalysis. Tx options for narcolepsy include:

scheduled daytime naps, psychostimulants (eg. modafinil or methylphenidate), or a

combination of antidepressants and psychostimulants.

Obesity is a common cause of obstructive sleep apnea syndrome, which is characterized by

excessive daytime sleepiness and a history of snoring and frequent waking at night. There is no

history of cataplexy or sleep paralysis, and complaints of fatigue and early morning headaches are

common. Tx options include weight reduction and nasal continuous positive airway pressure

(CPAP).

22. If no significant harm is likely ot result from withholding therapy, parental wishes regarding

the medical care of a child should be honored and the discussion documented in the chart.

23. The extrapyramidal side effects of antipsychotics can be treated with anticholinergic

medications like benzitropine.

24. When treating a single episode of major depression, the antidepressant should be continued

for a period of six months following the pt’s response. If multiple episodes of depression have

occurred, maintained therapy will likely need to be continued for a long period of time.

Nausea is a side effect of most antidepressants (including bupropion), a dose reduction in

sertraline is not warranted unless the nausea is very severe or incapacitating.

25. Studies of brain CT scans have demonstrated that pts with schizophrenia have enlarged

ventricles and prominent sulci.

26. The mechanism of action of antipsychotic medications primarily consists of dopamine-D2

receptor blockade. The added serotonin receptor binding of atypical antipsychotics reduces

the likelihood of extrapyramidal side effects.

27. Physicians should be non-committal and non-judgemental when treating psychotic pts. It is

important to acknowledge the pt’s distress without endorsing specific delusions or

hallucinations.

28. Alcohol withdrawl is best treated with treated with long-acting benzodiazepines such as

chlordiazapoxide (Librium). Withdrawal symptoms should be correlated with the time of the

last alcoholic drink. It is important to rule out other medical conditions that could be

responsible (electrolyte abnormalities, infection, or hypoxia) before making a dx. Pts with

alcohol withdrawal develop signs and symptoms of withdrawal between 12-48 hours after the

last drink. During the acute stage they develop sweating, hyperreflexia, seizure, and tremors.

This is followed by alcoholic hallucinosis (auditory and visual) in the absence of autonomic

symptoms. The last stage is that of delirium tremens, which usually occurs 2-4 days after the

last drink. Pts in this stage present with altered sensorium, hallucinations and autonomic

instability (tachycardia and fever).

29. Munchausen syndrome by proxy is the likely dx when one person intentionally produces

syndromes in another person, with the motive being to assume the role of concerned onlooker.

30. Alzheimer’s disease is the likely dx in an elderly pt with gradually progressive memory loss

and one or more of the following apraxia, aphasia, agnosia, or disturbed executive

functioning. Pick’s disease has a clinical presentation similar to that of Alzheimer’s, though

160

behavioral abnormalities tend to precede the cognitive decline. Pts may present in their 50s

and are more likely to be female. It is causes atrophy of the frontal lobes and temporal lobes.

31. Disorganized speech is common in schizophrenics. Pts with a circumstantial thought process

deviate from the original subject but eventually return to it, while those a tangential process

drift away without ever returning to the subject.

32. Adjustment disorders is characterized by emotional or behavioral symptoms that develop

within 3 months of exposure to an identifiable stressor and that rarely last more than 6 months

after the stressor has ended.

33. When dealing with an angry pt, the best response is to encourage a discussion about the

source of these feelings. Start the discussion with an open-ended question and let the pt talk.

34. A physician is authorized to provided emergent life-saving tx to the unconscious pt. This

remains true even the pt’s spouse requests that the tx not be given b/c it contradicts a belief

system. The religious beliefs of pts should be honored whenever possible. However, in the

event of a life-threatening situation, the physician should act in the pt’s best interests unless he

is specifically told not to by the the pt himself.

35. Fantasy is an immature defense mechanism that substitutes a less disturbing view of the world

in place of reality as a means of resolving conflict.

36. Anorexia nervosa is most common is adolescent girls from affluent families. The DSM-IV

criteria for the dx of anorexia nervosa include: 1) body weight at least 15% below normal

weight accompanied by a refusal to maintain body weight at normal levels; 2) amenorrhea for

3 months; 3) distortion of body image in which the individual views herself as obese when she

is in fact thin, and 4) fear of gaining weight or becoming fat despite being underweight. To

continue to lose weight, individuals suffering from anorexia nervosa will either: 1) fast and/or

excessively (the restricting subtype), or 2) binge eat followed yb laxative usage or induced

vomiting (the binge and purging subtype). Hospitalization is highly recommended for pts with

anorexia nervosa when there is evidence of dehydration, starvation, electrolyte disturbances,

(ie. Hyponatremia, hypokalemia, or hypophosphatemia). Cardiac arrhythmias, physiologic

instablility, or severe malnutrition (ie. Weight < 75% of average body weight for age, sex and

height). The goals of hospitalization include weight agin as well as prevention and mx of the

medical complications caused by anorexia nervosa.

37. Vaginismus is an involuntary spasm of the perineal musculature that interferes with sexual

intercourse. In general, dx of female sexual disorders requires taking a detailed history from

both sexual partners. Multiple sexual disorders may coexist in the same pt.

Hypoactive sexual desire is persistent deficiency in both sexual fantasy and the desire for sexual

activity activity.

Sexual aversion disorder is a recurrent and persistent aversion to and avoidance of genital sexual

contact with a partner.

Female sexual arousal disorder is defined as an inability to attain or maintain the lubrication and

swelling response associated with sexual arousal.

Female orgasmic disorder is a persistent delay in or absence of orgasm following a normal sexual

excitement phase.

38. Minors normally cannot consent to their own medical tx. Parents or legal guardians must

provide consent on the girls’s behalf before the physician can proceed, although exceptions

are made for emergent situations.

161

39. Acute stress disorder and PTSD present with identical symptoms (recurrent nightmares and

flashbacks, potential memory loss, and exaggerated startle response). Acute stress disorder

can last no more than four weeks, however, while PTSD lasts longer than 4 weeks.

40. Cocaine and amphetamine intoxication present in a similar manner, but psychosis is more

commonly associated wht amphetamine use. Common symptoms of stimulant intoxication

include dilated pupils, hypertension, and tachycardia.

A manic episode (DIGFAST): 1) Distractibility; 2) Insomnia; 3) Grandiosity; 4) Flight of ideas; 5)

Activity increase (goal-oriented); 6) Speech (extreme talkativeness); 7) Thoughtlessness

(excessive involvement in high-risk behavior, such as gambling).

41. Dysthymia is depressed mood lasting most of the days for the majority of days for at least 2

years. The dx also requires at least 2 of the following symptoms: decreased or increased

appetite, poor energy, insomnia or hypersomnia, impaired concentration, low-self esteem, and

feelings of worthlessness. Pts with dysthemia will often say that they have the felt depressed

their entire life.

42. When breaking bad news, physicians should begin with exploratory general statement such as

“how are you feeling today?” to help the pt feel at ease.

43. Atypical antipsychotic medications such as risperidone are considered first-line tx for pts with

psychotic disorders b/c of their more favorable side effect profile.

44. Schizophrenia is divided into 4 subtypes based on the predominant symptoms that the pt

presents with during the active phase of the illness. Disorganized subtype is characterizd by

disorganized behavior, disorganized speech, and flat or inappropriate affect.

Catatonic subtype is characterized by a predominance of physical symptoms, including immobility

or excessive motor activity and the assumption of bizarre postures.

Undifferented subtype presents with mixed symptoms that do not meet the criteria for paranoid,

disorganized, or catatonic subtype.

Paranoid subtype presents with preoccupation with delusions or auditory hallucinations without

prominent disorganized speech or inappropriate affect. These pts are usually less severely disabled

and are most responsive to pharmacotherapy.

Residual subtype occurs in pts with previous dx of schizophrenia who no longer have prominent

psychotic symptoms. The persistent symptoms may include eccentric behavior, emotional

blunting, illogical thinking, or social withdrawal.

1/17/10

1. Displacement is an immature defense mechanism in which the individual displaces negative

feelings associated with an unacceptable object or situation onto a “safer” object or situation.

Acting out is the expression of an unconscious impulse through a physical action. An example

would be this engineer screaming and kicking the floor when he discovered he was fired.

Conversion is a defense mechanism in which emotional conflicts are transformed into

physical (often neurological) symptoms. An example would be this engineer being struck

mute after learning the news of his firing.

2. Physical abuse should be suspected in a woman with multiple bruises and frequent injuries. In

these cases, the following steps should be carried: 1) Confront the pt gently, in a non-

judgemental way. 2) Assure her about confidentiality. 3) Emphasize that she should not allow

abuse to happen to her. 4) Suggest informing the police. 5) Ensure safety of the pt and

162

children, if any. 6) Ask her if she had a plan to escape. 7) Suggest talking to a women’s group

dealing with these problems. 8) Assure her of your continuing support.

3. Passive-aggressive behavior is a psychological defense mechanism in which an individual

expresses his aggression toward another person with repeated, passive failure to meet the

other perosn’s need.

4. When caring for a minor, informed consent from one parent or guardian is considered legally

sufficient to justify proceeding with therapy.

5. Enuresis is defined as the recurrent, involuntary voiding of urine into clothing or bed linens

after age five years. Dx criteria for this condition include the occurrence of symptoms at least

twice per week for 3 consecutive months or the development of marked functional

impairment. The child’s history should be reviewed for medical conditions or medications that

couldn be responsible for the enuresis. When behavior modification is unsuccessful, Low

doses of tricyclic antidepressants such as imipramine or desmopressin can be used.

6. The physician has the moral responsibility to always act in the pt’s best interests. When a

mistake made by another physician is discovered, the facts should be clarified and the truth

told to the pt.

7. Older individuals may frequently awaken from sleep and spend less time sleeping overall.

These changes are considered a normal part of aging.

8. Malingering is always associated with secondary gain, whereas the motivating factor in

factitious disorder is to assume the sick role.

9. Severe symptoms of Tourette symdrome are best treated with typical antipsychotics such as

haloperidol or pimozide 哌迷清,双氟苯丁哌啶苯并咪唑酮.

10. Kleptomania is characterized by an inability to resist the impulse to steal objects that either

are of low monetary value or are not needed for personal use. The condition is more prevalent

in females and is occasionally associated with bulimia nervosa.

11. Individuals with schizoid personality disorder are socially detached and aloof避开 but do not

have bizarre cognition. Those with schizotypal personality disorder are also socially detached

but typically demonstrate “magical thinking” and a more eccentric thought process.

Individuals with schizophreniform disorder have full-blown schizophrenic symptoms (eg.

hallucination, delusions) that have been present for 1-6 months.

12. Benzodiazepines are used for the acute tx of panic attacks, and an SSRI should be used for

long-term symptom relief.

13. Manic episodes are characterized by the presence of elevated mood for at least one week,

accompanied by three or more DIGFAST symptomes.

14. Pts who are extremely agitated, psychotic, or manic should be initially managed with an

antipsychotic medication such as haloperidol.

15. The following guidelines apply to lithium therapy in pts with bipolar disorder: 1) single manic

episodes require long-term maintainance for a period of at least one year, 2) three or more

relapses require tx with lifelong maintainance therapy. If there are no relapse and pt has

attained good symptomatic control, then the lithium can be gradually tapered off and ultimatel

discontinued. Because abrupt cessation of lithium increases the risk of suicide and relapse,

tapering the medications is always recommended.

16. Neuroleptic malignant syndrome (NMS) is an unusual but potentially lethal side effect from

the use of antipsychotics (neuroleptics). It is treated primarily with dantrolene sodium and

163

supportive care.

17. Pyromania is characterized by intentional, repeated fire setting with no obvious motive.

Although a history of arson纵火罪may be documented on individuals with conduct disorder,

other features will be present as well (eg. lying, theft, cruelty).

18. If parents refuse to give consent for the tx for their child in a life-threatening situation, the

physician should proceed with the tx despite the parental wishes.

19. The tx of choice for social phobia is assertiveness 自信 training, which is a component of

cognitive behavioral psychotherapy (CBT). SSRI drugs are the first-line drugs in the mx of

these pts, either alone or in combination with CBT.

20. Imaginary friends are a normal component of development for many young children, and are

usually outgrown by the early elementary school years.

21. Alcoholic hallucinations typically develop within 12-24 hours of last drink and resolve within

24-48 hours. Unlike delirium tremens sensorium is intact and vital signs are usually normal.

22. Lithium, valproic acid, and carbarmazepine are first-line treatments for pts with bipolar

disorder. (Ref. 14)

23. Avoidant personality disorder is characterized by shyness, feelings of inferiority, and a desire

to make friends that is overridden by an intense fear of embarrassment or rejection.

24. For the general population, the lifetime risk of developing bipolar disorders is 1%. However,

an individual with a first-degree relative (eg. parent, sibling, or dizygotic twin) who suffers

from bipolar disorder has a 5-10% risk of developing the condition in his lifetime.

25. Do not share pt information with a third party without first obtaining written consent form

from the pt. A breach of confidentiality is justified only if a pt’s dx could prove a risk to others

and he refuses to divulge泄露 this information to those in jeopardy.

26. Body dysmorphic disorder is defined by an excessive preoccupation with animagined bodily

defect. Hypochondriasis results in recurrent thoughts of having a serious, potential fatal

“medical condition”. Somatization disorder presents with multiple medical complaints that

persist despite repeated negative workups.

27. Children who suffer from selective mutism are verbal and talkative at howm but refuse to

speak in select settings (eg. school, public events). To establish the dx, symptoms must be

present for at least one month, cause significant functional impairment, and not be caused by

another communication or learning disorder.

Separation anxiety disorder is characterized by extreme anxiety in children when they are

separated from home or their loved ones.

Stranger anxiety is a normal finding in children until 3 years of age. After that, a persistent fear of

strangers is generally attributable to other causes.

28. Suspect heroin withdrawal in pts with papillary dilatation, rhinorrhea, muscle and joint aches,

abdominal cramping, nausea, and diarrhea. The symptoms are severe and out of proportion ot

physical findings.

29. Borderline personality disorder is characterized by “splitting”, unstable relationships, and

impulsivity. Angry outbursts and suicidal gestures are common.

30. NMS is a potentially life-threatening condition that can occur after administration of

antipsychotic medications. Symptoms include fever, rigidity, altered mental status, and

autonomic instability. It is most likely the result of repeated doses of haloperidol.

31. The symptoms of schizophrenia can be classified as positive and negative. The positive

164

symptoms include hallucinations, delusions, disorganized speech, and disorganized behavior.

Negative symptoms include the “five A’s”: affective flattening (diminished emotional

responsiveness), alogia (poverty of speech), apathy (impaired grooming and hygiene,

unwillingness to perform activities), asociality (few recreational interests, social detachment,

impaired relationship); and attention (inattentiveness and impaired concentration when

interviewed). Atypical antipsychotics such risperidone are particularly effective in the tx of

negative symptoms of schizophrenia.

32. If parents refuse to consent to tx of their child for a non-emergent but potentially fatal medical

condition, the physician should seek a court order mandating tx.

33. Specific or simple phobias can be treated with benzodiazepines. Alprazolam is a short-acting

benzodiazepine, which makes it more suitable than long-acting benzodiazepines such as

clonazepam for decreasing anxiety during a short plane ride. Buspirone is used for the tx of

generalized anxiety disorder.

34. Bereavement is the normal reaction to the loss of a loved one. Symptoms are similar to those

seen with major depression but are less intense and usually significantly taper within 2

months.

35. Most antidepressants must be taken for 4-6 weeks before they provide symptomatic relief.

36. Autism is the likely dx in those children who perform stereotyped behavior, demonstrate an

inability to form emotional bonds with others, and participate in restricted activities before the

age of 3 years.

37. Agranulocytosis is a serious side effect associated with clozapine usage. Pts should have their

white blood cell counts monitored frequently for this complication.

38. Reversible acetylcholinesterase inhibitors such as donepezil, rivastigmine, galantamine, and

tacrine are of benefit in slowing the cognitive decline associated with Alzheimer’s disease.

39. Differentiation of Delirium an DementiaDelirium Dementia

Onset Acute GradualConsciousness impaired IntactCourse Fluctuating symptoms Progressive declinePrognosis Reversible symptoms Irreversible symptomsMemory impairment Global Remote memory spared

40. Abrupt cessation of alprozolam, a short-acting benzodiazepine, is associated with significant

withdrawal symptoms such as generalized seizures and confusion.

41. Generalized anxiety disorder is characterized by excessive anxiety about multiple events, in

conjunction with 3 or more of the following symptoms for a period of at least 6 months:

impaired sleep, poor concentration, easy fatigability, irritability, muscle tension, or

restlessness.

42. Physicians should respond to inappropriate pt requests politely but firmly. If the pt persists, it

is acceptable to be more direct and assertive in response. Professionalism should be

maintained at all times.

43. Dysthymia is characterized by depressive symptoms not meeting full criteria for a major

depressive disorder, lasting two years or more. Tx with antidepressants and/or therapy can

improve symptoms and quality of life.

44. Somatization disorder is characterized by multiple recurrent somatic complaints that have

165

persisted for several years and that have been evaluated by healthcare providers to no avail效

用. The condition presents before the age of 30 and most frequently occurs in females.

1/18/10

1. When dealing with pt (including antagonistic, seductive, or familiar types), the physician must

remain polite and professional while addressing the pt’s medical and psychological needs.

Avoiding the pt, behaving rudely, or calling attention to any inappropriate behavior is not

recommended.

2. The mother and daughter are likely suffering from a rare psychotic disorder known as folie á

deux. In this disorder, a delusion or set of delusions is shared simultaneously by individuals

who share a close relationship. Usually, the dominant individual in the pair becomes

delusional and transfer the delusion onto the second person. Tx includes separating the pair to

break the chain of reinforcing each other’s beliefs. The individual who first had the delusion,

in this case the mother, always requires psychiatric tx, whereas the other individual only

requires tx in some case.

3. The tx of choice for adjustment disorder is cognitive or psychodynamic psychotherapy.

4. Tourette syndrome, occurring more frequently in males, is characterized by multiple motor

and one or more vocal tics that present before the age of 18. Pts with Tourette syndrome have

a significantly increased risk of developing ADHD (60%) or OCD (27%).

5. Pts have the legal right to obtain copies of their medical records . The third party requests for a

pt’s medical records should be refused unless the pt has provided written legal consent

beforehand. Pts don not need to justify requests for medical records. Were the pts to insist

upon obtaining the original chart, however, the physician would be justified in asking why

such an unusual transation is necessary.

6. OCD is best treated with an SSRI. Clomipramine (TCA) is a second-line tx. Alprazolam is a

short-acting benzodiazepine used in the tx of anxiety and panic disorder.

7. Although they are considered first line treatments for depression, SSRI are not without side

effects. One common and troubling potential side effect of the SSRIs is sexual dysfunction

(impotence).

8. Circadian rhythm sleep disorder is a likely diagnosis in apt with insomnia The who often

travels between different time zones.

9. The diagnosis of catatonic schizophrenia include the presence of two or more of the

following: 1) catalepsy, waxy flexibility, or stupor; 2) catatonic excitement (ie. Hyperactivity

or violence); 3) extreme negativism or mutism; 4) posturing, grimacing, prominent

mannerisms; and 5) echopraxia模仿动作 or echolalia模仿言语. This pt’s unresponsiveness,

motionless, sitting, and muscle rigidity are all suggestive of catatonia. Catatonic pts benefic

most from benzodiazepines (eg. lorazepam) or electroconvulsive therapy.

10. The features of bulimia nervosa: 1) recurrent episodes of uncontrolled binge eating followed

by feelings of extreme disgust or guilt; 2) repeated compensatory behavior to prevent weight

gain after binging (eg. induced vomiting, laxative abuse, diuretic abuse, fasting, or excessive

exercise); 3) binging episodes that occur at least twice per week over a 3 month period; 4)

normal or slightly above normal BMI; 5) dissatisfaction with body weight and shape.

Precipitating factors for a binge-purge episode in bulimic pts includes high levels of anxiety,

emotional tension, boredom, exhaustion, poor self-esteem, environmental cues about food and

166

eating, alcohol use, substance abuse, and mood disorders. Unlike pts with anorexia nervosa,

pts with bulimia nervosa maintain a normal body weight and are not amenorrheic.

11. pts have a right to confidentiality unless their condition poses an imminent danger to other

individuals or society. However, pts should be strongly encouraged to discuss their health and

medical conditions with loved ones.

12. Anorexia nervosa is always associated with amenorrhea and a body weight that is below

normal. (ref. 1/17 NO.36)

13. A history of seizure disorder (epilepsy) is an absolute C/I to the use of bupropion.

14. Trichotillomania is an impulse-control disorder, characterized by compulsive hair pulling,

leading to hair loss. 1) recurrent pulling out of one’s hair resulting in hair loss; 2) inability to

resist the behavior; 3) relief on pulling out of hair; 4) significant functional impairment as a

result; 5) features inconsistent with any other medical or dermatological condition causing

hair loss. Other associated features of this disorder are nail biting and cratching.

15. Because symptoms of hypochondriasis usually develop during periods of stress, pts suffering

from the condition should be asked about their current emotional stressors and then referred

for brief psychotherapy.

16. Pts have the right to refused tx except when doing so would pose a threat to the health and

welfare of others.

17. Schizoaffective disorder is defined as the presence of symptoms of schizophrenia along with

mood symptoms (major depression, bipolar disorder, or a mixed episode). To make the

diagnosis, there should be at least two weeks when psychotic symptoms are present without

any mood symptoms. DD: Major depression with psychotic features—the pt reports depressed

mood, poor sleep, and diminished appetite in the four months since her dog’s death. However,

because her social withdrawal and eccentric behavior presented several months before her

depression started, schizoaffective disorder is the most likely dx.

18. Antisocal personality disorder is diagnosed in pts aged 18 years older who engage in illegal

activities (eg. theft or assault) and disregard the rights of others. These individuals ofter

display evidence of conduct disorder as minors.

19. Marijuana abuse causes behavioral changes and two or more of the following symptoms: dry

mouth, tachycardia, increased appetite, or conjunctival injection.

20. Conversion disorder is likely in a pt with sudden onset neurological symptoms preceded by an

obvious stressor. Pts with conversion disorder may be hysterical or strangely indifferent (“la

belle indifference”) to their symptoms. These pts may show some improvement with the

administration of sodium amytal 异戊巴比妥 . Tx options include hypnosis and relaxation

techniques in the acute setting, while psychotherapy offers the best long-term results.

21. Antipsychotic medications can cause hyperprolactinemia secondary to their dopamine

blockade effect. Prolactinomas tend to produce very high levels of prolactin (>200 ng/ml).

22. The extrapyramidal symptoms (EPS) frequently seen with typical antipsychotics include

dystonia, Parkinsonism, tardive dyskinesia, akathisia, and neuroleptic malignant syndrome.

The atypical antipsychotic medication most likely to cause EPS is risperidone. Tardive

dyskinesia is characterized by involuntary periodal movements such as biting, chewing,

grimacing, and tongue protrusions. It is best managed by immediately discontinuing the

offending antipsychotic and replacing it with clozapine. Because clozapine is associated with

agranulocytosis, it is typically considered to be a medication of last resort.

167

Pediatrics

1/18/10

1. Jejunal atresia presents with bilious vomiting, abdominal distention, and abdominal

radiography that shows a “triple bubble” as well as gasless lower abdomen. Intestine atresia

can occur anywhere from the duodenum the colon. In general, the more proximal the

obstruction, the earlier the child will develop symptoms. Radiography can usually confirm the

diagnosis by showing air-fluid levels with a gasless lower abdomen. Duodenal atresia appears

as a “double bubble” on radiograph, and 30-40% of affected infants will have other congenital

anomalies (most common Down syndrome). There is only a 10% incidence of associated

anomalies in pts with jejunal atresia. Tx should initially be focused on resuscitation and

stabilization of the pt, followed by surgical correction. The prognosis following surgery

depends on the length of affected bowel as well as the birth age and weight of the infant.

Pyloric stenosis most commonly presents between 2-4 weeks of life with nonbilious, projectile

vomiting as well as visible gastric peristaltic waves and a palpable mass in the epigastric area.

Hirschsprung disease can present with abdominal distention, but is usually suspected by failure to

pass meconium by 48 hours of age.

Necrotizing enterocolitis usually presents with feeding intolerance and abdominal distention.

Abdominal radiographs show dilated bowel loops and pneumatosis intestinalis (air within the

bowel wall).

Gastroesophageal reflux typically presents with nonbilious vomiting. In addition, abdominal

distention is usually not seen.

2. Tricuspid atresia is a cyanotic congenital heart disease characterized clinically by cyanosis

that appears early in life and left axis devation (left ventricle hypertrophy, since the left

ventricle handles both pulmonary and systemic venous returns). This condition is

characterized by an absent connection between the right heart vavities, and a hypoplastic or

absent right ventricle. Ventrical septal defects (holosystolic murmur on auscultation) occur in

90% of cases. Interestingly, the associated heart defects, such as ASD, VSD or PDA are

necessary for survival. The tx generally includes PGE1 (to keep the dutus arteriosus open) and

balloon arterial septostomy (if the ASD is not large enough to allow an adequate flow from

the right to left atriums).

Both tertralogy of Fallot and tricuspid atresia can present with a normal heart size and decreased

vascular marking on CXR; however, the characteristic EKG findings of the former are right atrium

dilation and right ventricle hypertrophy.

In truncus arteriosus, the CXR demonstrates cardiomegaly, increased vascular markings, and right

aortic arch. The typical EKG finding is biventricular hypertrophy.

Common atrioventricular canal is characterized by an endocardial cushion defect involving both

atrial and ventricular septa. The typical EKG finding is are right and left atrial dilation with left

axis deviation.

Ebstein’s anomaly is a rare congenital heart disease that is characterized by a portion of the right

ventricle that has been incorporated into the right atrium, a tricuspid valve that has been displace

into the right ventricular cavity, extreme cardiomegaly, and marked right atrial enlargement.

3. Upper airway compression by a vascular ring most often due to a double aortic arch, right-

sided aorta, pulmonary sling 悬带 , or anomalous innominate 无名的 or left carotid artery,

168

vascular rings can cause tracheal compression. Pts present with signs of chronic upper airway

obstruction including stridor, wheezing, coughing, and shortness of breath. Symptoms are

often worse while supine and relieved with nech extension. Pts are commonly misdiagnosed

as having reactive airway disease but will not respond to inhaled bronchodilators and

corticosteroids. The tx for severe disease is surgery.

4. Polycythemia is defined as a central venous hematocrit level greater than 65%. The MCC of

polycythemia in term infants is delayed clamping of the umbilical cord resulting in excess

transfer of placental blood. As the hematocrit rises, the viscosity of the blood increases as well

as the arterial oxygen content. This change in blood flow may affect various organs. In

newborns, the hematocrit level peaks within 12 hours of birth and then decreases over the

subsequent 12 hours. Not all infants with polycythemia become symptomatic, however,

symptomatic infants may present with manifestations to almost any organ system. The most

common findings are plethora 多血症 and central nervous system disturbances like lethargy,

irritablility, jitteriness, and seizures. Other common symptoms are respiratory distress,

tachypnea, and cyanosis, caused by the decrease in pulmonary blood flow due to the

hyperviscosity of the blood. Poor feeding, hypoglycemia, and hypocalcemia are also

commonly seen in polycethemic infants. Tx includes hydration and, if symptomatic, partial

exchange transfusion.

5. Duodenal atresia presents with bilious vomiting, and its typical radiographic finding is the

‘double bubble sign’. The physical findings of prominent tongue, flat occiput, slanting eyes,

short hands, wide gaps between the first and second digits, and systolic ejection murmur at the

left sternal border with a widely split and fixed S2 (suggestive of atrial septal defect) point

towards Down’s syndrome, which have a high predilection for duodenal atresia.

6. The characteristic description of a Mongolian spot is a well-demarcated, flat, blue/gray, lesion

over the sacral/presacral area that disappears in the first few years of life. This lesion is seen

more commonly in dark skinned children. It is caused by entrapment of melanin-containing

melanocytes in the dermis.

Cutis Marnorata appears as a lace-like pattern on the skin in response to cold or stress. It can

persist in some disease such as Down’s syndrome and trisomy 18.

A salmon patch is a flat salmon-colored lesion commonly seen over the glabella, eyelids, and

neck. It is a vascular lesion that usually disappeara in early childhood.

7. Hyaline membrane disease should be suspected in preterm infants with respiratory distress

and hypoxia not responding to oxygen therapy. It presents as tachypnea, prominent grunting,

intercostals and subcostal retractions, nasal flaring, and duskiness within a few minutes after

birth. The characteristic CXR finding of HMD demonstrates fine reticular granularity of the

lung parenchyma, and tx includes early mechanical ventilation and surfactant administration.

Transient tachypnea usually follows an uneventful normal term vaginal delivery or cesarean

delivery. It is characterized by the early onset of tachypnea, sometimes with retractions or

expiratory grunting, and typically, cyanosis that is relieved by minimal oxygen. The lungs are

usually clear without rales or rhochi, and the CXR shows prominent pulmonary vascular

markings, fluid lines in the fissures, overaeration, a flat diaphragm, and occasionally, pleural fluid.

Hypoxemia, hypercapnia, and acidosis are uncommon.

Persistent pulmonary hypertension of the newborn (PPHN) should be suspected in all term and

post-term infants with cyanosis with or without fetal distress. Persistence of the fetal circulatory

169

pattern of right-to-left shunting through the PDA and foramen ovale after birth is due to very high

pulmonary vascular resistance. The associated hypoxia is universal and unresponsive to 100%

oxygen. The CXR may be normal or may show parenchymal opacification in the chest, depending

on the etiology. Although the clinical presentation of PPH is indistinguishable form HMD, the x-

ray findings of this case are very typical and therefore very suggestive of the latter.

Meconium aspiration syndrome usually occurs in term or post-term infants. Either in utero or

more often with the first breath, thick and particulate meconium is aspirated into the lungs,

resulting in small airway obstruction and consequent respiratory distress that presents within the

first hour of birth. Partial obstruction of some airways may lead to pneumothorax or

pneumomediatinum. Pathy infiltrates, coarse streaking of both lung fields, increased

anteroposterior diameter, and flattening of the diaphragm characterize the typical CXR.

Intracranial hemorrhage in neonates classically presents as periods of apnea, pallor or cyanosis,

poor suckling, abnormal eye signs, high-pitched, shrill cry, muscular twitching, convulsions,

decreased muscle tone or paralysis, metabolic acidosis, shock and a decreased hematocrit. The

fontanel may be tense and bulging. The condition is rarely present at birth and is diagnosed on the

basis of the history, clinical manifestations, and trans-fontanel cranial ultrasonography or CT.

8. ADHD is diagnosed when inattentive or hyperactivity-impulsive symptoms that cause

impairment in two different settings are present for more than 6 months in a child less than 7

years old.

9. Spondylolisthesis 脊椎前移 is a development disorder characterized by a forward slip of

ventebrae (usually L5 over S1) that usually manifests in preadolescent children. In the typical

clinical scenario, back pain, neurologic dysfunction (eg. urinary incontinence), and a palpable

“step-off” at the lumbosacral area are present if the disease is severe.

Other cause of back pain should be considered in this pt. Mechanical causes like lumbosacral stain

and herniated disk, also very common in adults, are uncommon in children. Ankylosing

spondylitis has slow onset, bu neurologic dysfunction argues against this condition. Epidural

abscess is usually accompanied by fever, and the source of infection is frequently present.

Metastatic or pulmonary tumor is a possibility in this pt, but slow development of the pain and a

palpable “step-off” favors spondylolisthesis as the most probable diagnosis.

10. Vitamin D deficiency rickets presents with craniotabes (ping-pong ball sensation over the the

occiput or posterior parietal bones), rachitic rosary (enlargement of the costochondral

jinctions), Harrison groove (horizontal depression on lower border of chest), large anterior

fontanelle, and thickening of the lower end of the long bones. The typical pts are low-birth

weight infants, unsupplemented dark-skinned infants, infants with inadequate sun exposure,

and breastfed infants. The typical pathology is –defecive mineralization of growing bone or

osteoid tissue. The dx can be confirmed by obtaining the serum calcifediol level (decreased),

alkaline phosphatase level (increased), and observing the characteristic radiologic changes

(cupping and fraying of the distal ends of long bones), and double contour along the lateral

outline of the radius. Tx includes oral vit D administration and adequate sunlight (or artificial

light) exposure. The current recommendation to prevent rickets is oral vitD supplementation,

beginning during the first 2 months of life and coutinuing until early adolescence.

Scurvy is commonly seen in 6 to 24 month-old infants with a characteristic history of vitC

deficiency. Clinical manifestations are general tenderness, pseudoparalysis (frog position), bluish-

purple and spongy/swollen gums over the upper incisors, and depression of the sternum. A rosary

170

similar to rickets may be observed, but the angulation of the beads in pts with scurvy is usually

sharper than in those with rickets. The dx is confirmed radiologically (pencil-point thinners of the

long bones’ cortex), and sharply outlined epiphyseal ends.

Nonrachitic craniotabes is unlikely in this pt because this condition usually presents in the

immediate postnatal period, and disappears before rachitic softening of the skull occurs during the

2nd-4th months of life.

Pt with chondrodystrophy may also manifest with “rosary”, however, radiologic findings will

show irregular, concave outlines of the distal ends of the bones. Chondrodystropy is therefore a

less likely diagnosis in this pt.

11. Arthrocentesis followed by empiric tx with iv nafcillin are the most appropriate measures for

the mx of suspected septic arthritis (occur following skin or upper respiratory tract infections)

in a child.

12. In childhood, cystic fibrosis has a wide range of manifestations, such as anemia, heat

intolerance, steatorrhea, wheezing, recurrent infections, hemoptysis, clubbing and rectal

prolapse. A positive sweat chloride test confirms the dx. It must be repeated twice to avoid

any false positives or negatives. Genetic studies should be done when the sweat test is

equivocal. Prenatal screening should be offered to high-risk families. The tx is a high-calorie

diet, pancreatic enzyme replacements and fat-soluble vitamins. Complications of prolonged

CF include gallstones, cirrhosis with portal hypertension and pancreatic fibrosis.

13. HIV test is standardly offered to all pregnant women in the first trimester to prevent possible

transmission to the infant. Persistent oral thrush, lymphadenopathy, and hepatpsplenomagely

may be the presenting symptoms of AIDS in infants.

14. Friedreich Ataxia (FA) MRI of the brain and spinal cord shows marked atrophy of the cervical

spinal cord and minimal cerebellar atrophy. Never conduction velocity results are within

normal limits. EKG shows T wave inversions in the inferior and lateral chest leads. It is an

autosomal recessive condition characterized by an excessive number of trinucleotide repeat

sequences, resulting in an abnormality of a tocopherol transfer protein. The disorder is

progressive with poor prognosis. Most pts are wheelchair bound by the age of 25, with death

occurring by 30-35 years of age. FA is associated with necrosis and degeneration of cardiac

muscle fibers leading to myocarditis, myocardial fibrosis and cardiomyopathy. Cardiac

arrhythmia and congestive heart failure contritbute to a significant number of deaths. T-wave

inversion in this case is most likely due to myocarditis, and not myocardial infarction.

[remember the differential for T-wave inversion: MI, myocarditis, old pericarditis, myocardial

contusion and digoxin toxicity]. Genetic counseling is recommended for prenatal diagnosis

for pts with one affected child.

15. Pubertal gynecomastia is seen in approximately one-half of adolescent boys, at an average age

of 14 years (increase adrenal androgen at adrenarche, androgen synthesis in Leydig cell↑ -

elevating plasma estradiol). It is often asymmetric or transiently unilateral, and frequently

tender. In prepubertal males the testicular size is normally 2 cm in length and 3 ml in volume.

The initial mx involves reassurance and watchful waiting/observation.Gynecomastia can be

physiological (neonatal, adolescent/pubertal and gynecomastia of aging) or pathological

(secondary to drugs, endocrine disorders, tumor, etc) * Extremely high yield!!

16. Rotavirus is the MCC of acute diarrhea in children, esp in those between 6 months and 2 years

of age. Norwalk virus is the second MCC of non-bacterial, acute diarrhea in children. E.coli is

171

the MCC of traveler’s diarrhea. Campylobacter infection is the MCC of diarrhea in adults in

the US. Shigella is the second most common, documented, food-borne disease in US.

17. The healthy appearance of the neonate, the evanescent nature of the rash, and the distinctive

red halo surrounding the lesions support the dx of erythema toxicum. The presence of

numerous eosinophils in the pustules 小疙疸 is diagnostic . Erythema toxicum is a benign,

self-limited condition usually found in newborns after the first 2 days of birth. Tx is not

necessary.

Milia粟粒疹 are small pearly white cysts, and are distinct from the rash of erythema toxicum.

Sebaceous hyperplasia presents as little yellowish papules and are commonly found on the face.

18. Infants with Edwards’ syndrome (trisomy 18) commonly present with microcephaly,

prominent occiput, micrognathia, closed fists with index finger overlapping the 3rd digit and

the 5th digit overlapping the 4th, and rocker bottom feet.

Wolf-Hischhorn’s syndrome is very low-yield for USMLE. You do not need to remember this

disease. Nevertheless, the cause of this condition is a chromosome 4p deletion. Characteristic

findings include a “Greek helmet” facies with ocular hypertelorism, prominent glabella and frontal

bossing, ptosis, and strabismus. Other accompanying findings are microcephaly, dolichocephaly,

hypoplasia of the eye sockets, bilateral epicanthic folds, cleft lip and palate, beaked nose with

prominent bridge, cardiac malformations and mental retardation.

Patau’s syndrome (trisomy 13) can present with cleft lip, flexed fingers with polydactyl, ocular

hypotelorism 间距缩短 , bulbous nose, low-set malformed ears, small abnormal skull, cerebral

malformation, microphthalmia, cardiac malformations, scalp defects, hypoplastic or absent ribs,

visceral and genital anomalies.

Cri-du-chat syndrome (5p deletion) may also present with microcephaly; however, there is a

characteristic protruding metopic suture in such pts. Furthermore, the pt’s cat-like cry will usually

be the tell-tale sign of this condition. Other accompanying manifestations are hypotonia, short

stature, moonlike face, hypertelorism, bilateral epicanthic folds, high arched palate, wide and flat

nasal bridge, and mental retardation.

19. Kawasaki disease is a vasculitis (medium-sized arteries) characterized by prolonged feve

(>39C, for 5 days), conjunctivitis (bilateral and nonpurulent), extremity changes (erythema,

edema, or desquamation of the hands and feet), cervical lymphadenopathy (must be >1.5cm,

usually unilateral), oral changes (erythema, fissured lips, or a strawberry tongue), and a rash.

Supporting lab evidence include sterile pyuria, a CRP >3, an erythrocyte sedimentation rate

>40, and albumin level <3, and elevated ALT, thrombocytosis, and leukocytosis (>12,000).

Other clinical findings may include urethritis, orchitis, arthritis, or hepatitis.

Scarlet fever has a similar presentation to Kawasaki disease. Symptoms may include fever, a

strawberry tongue, a red sandpaper-like rash over the trunk and extremities, and cervical

lymphadenopathy. However, scarlet fever has a shorter duration than Kawasaki disease. Its fever

usually remits by day 3-5, and the rash begins to fade by day 5-6.

Rocky Mountain spotted fever presents with fever and constitutional symptoms followed by a

centripetal rash (starts on the extremities and spreads inward). Arthralgias, myalgias, and headache

may also be present.

20. Coronary artery aneurysms are the most serious complication of Kawasaki disease.

21. Jaudice within the first 24 hours of life requires immediate attention and may be due to

erythroblastosis fetalis, concealed hemorrhage, sepsis or congenital infections. Jaundice that

172

first appears on the 2nd or 3rd day of life is usually “physiologic”, but sometimes may represent

a more severe disorder. Jaundice appearing after the 3 rd day and within the 1 st week of life

suggests bacterial sepsis or UTI, and requires prompt and aggressive evaluation. Sepsis in the

neonate rarely causes classic findings such as neck stiffness, buldging fontanel or shock. The

susal clues are history of poor feeding, lethargy, vomiting, or alteration of usual activity. All

pts with a suspected dx of sepsis should be investigated with blood cultures and lumbar

puncture.

22. Suspect Beckwith-Wiedemann syndrome in an infant with macrosomia, macroglossia,

visceromegaly, omphalocele, hypoglycemia and hyperinsulinemia. Additional features include

prominent eyes, prominent occiput, ear creases and hyperplasia of the pancreas. It is usually

sporadic, but there are occasional cases involving familiar inheritance. The exact cause is

unknown, although it is sometimes associated with duplication of chromosome 11p. This

region contains the gene encoding for IGF-2, which may explain the macrosomia, Sometimes,

the hypoglycemia may be severe and intractable, and subtotal pancreatectomy may be needed.

Pts have an increased risk of neoplasms such as Wilm’s tumor, hepatoblastoma, and

gonagoblastoma.

Congenital hypothyroidism also presents with hypotonia and an enlarged tongue, and may easily

be confused with Beckwith-Wiedemann syndrome; however, an umbilical hernia (instead of

omphalocele) is another typical finding of hypothyroidism. Furthermore, the head circumference

of pts with congenital hypothyroidism may be increased, whereas pts with Beckwith-Wiedemann

syndrome have microcephaly. There is no hypoglycemia and hyperinsulinemia in pts with

congenital hypothyroidism.

Macrosomia secondary to maternal diabetes is an important differential diagnosis of Beckwith-

Wiedemann syndrome; however, infants with this condition do not present with dysmorphic

features such as omphalocele, prominent occiput and macroglossia. Moreover, the prenatal and

birth histories of the pt in the case were unremarkable. The common congenital problems that you

should remember in an ‘infant of diabetic mother’ are: 1) caudal regression syndrome; 2)

transposition of great vessels; 3) duodenal atredia and small left colon; 4) anencephaly and neural

tube defects.

WAGR syndrome is characterized by Wilm’s tumor, aniridia, genitourinary anomaly and mental

retardation. It is related to a deletion in chromosome 11 involving the gene WT1 and aniridia gene

PAX6.

Denys-Drash syndrome is also associated with an increased rish of Wilm’s tumor, but its clinical

features are totally different from those of Bechwith syndrome. It includes male

pseudohermaphrodism and early onset renal failure characterized by mesangial sclerosis.

The symptoms of galactosemia manifest a few days or weeks after the infant starts taking formula

or breast milk. These include liver failure (hepatomegaly, direct hyperbilirubinemia, disorders of

coagulation), abnormal renal function, emesis, anorexia, acidosis and glycosuria.

Von-Gierke disease is secondary to a deficiency of glucose-6-phosphatase, It also affects the

kidney and liver, and cause severe hypoglycemia; however, the hypoglycemia characteristically

occurs with fasting, since it occurs secondarily to a failure to release glucose from the liver into

the circulation, rather than to hyperinsulinemia.

23. No investigations are needed in the case of migraine headache, unless the neurological exam

is abnormal and/or fever is present. Migraine is a clinical dx, and the initial tx is

173

acetaminophen or NSAIDs.

24. Laryngomalacia or congenital flaccid larynx is the MCC of chronic inspiratory noise in

infants. Larygoscopy shows flaccidity of the larynx, and collapses during inspiration

(epiglottis rolling in from side to side). It is a self-limiting condition in most cases, and

generally subsides by 18 months of age. The mother should be instructed to hold the child in

an upright position for half an hour after feeding, and to never feed the child when he is lying

down.

25. Neuroblastoma is the third most common cancer in the pediatric population (after leukemia

and CNS tumors). The tumor arise form neural crest cells, which are also the precursor cells

of the sympathetic chains and adrenal medulla. The most common site is the abdomen.

Calcifications and hemorrhages are seen on plain x-ray and CT scan. The levels of serum and

urine catecholamines and their metabolites (ie HVA and VMA) are usually elevated.

26. Risperidone is a dopamine and serotonin antagonist that can cause weight gain or

hyperprolactinemia, the latter of which can lead to amenorrhea and galactorrhea. Lamotrigine

is used in the tx of epilepsy and bipolar disorder. The major side effect of lamotrigine is a

rash, although severe skin reactions can develop including Steven-Johnson syndrome or toxic

epidermal necrolysis.

27. Salicylates are C/I in young children with viral infections. Recognize the clinical presentation

of Reye syndrome.

28. Suspect vit A deficiency in a 2 or 3 year old child with impaied adaption to darkness,

photophobia, dry scaly skin, xerosis conjunctiva, xerosis cornea, keratomalacia, Bitot spots

and follicular hyperkeratosis of the shoulders, buttocks and extensor surfaces.

29. Four clinical criteria have been shown useful in differentiating septic arthritis from transient

synovitis: white blood cell count >12,000/mm3, temperature >39C (102F), ESR>40mm/h,

and refusal to bear weight. If at least 3 of these 4 criteria are met, further work-up is indicated

to rule out septic arthritis. Transient synovitis is treated with rest and NSAIDs.

30. Suspect choanal atresia in an infant who presents cyanosis that is aggravated by feeding and

relieved by crying.

31. Acute, unilateral cervical lymphadenitis in children is usually caused by bacterial infection,

and the most common pathogen is Staph aureus.

32. Vaginal discharge in the newborn is due to the effects of maternal estrogens. In such cases,

reassurance of the mother is all that is required.

33. Early neonatal care in an uncomplicated pregnancy include initial physical assessment,

removal of airway secretions, drying the infant and keeping him/her warm, and early

preventive measures (gonococcal ophthalmia prevention, vit K supplementation).

34. Always suspect pyloric stenosis in a neonate who presents with non-bilious, projectile and

persistent vomiting. In such cases, order an abdominal ultrasound to establish the dx.

35. Medulloblastoma is the second most common (after cerebellar astrocytoma) infratentorial

tumor in children and arised form vermis. Recognize posterior vermis syndrome.

36. A large thymic shadow (sail sign) is a normal finding on chest radiographs in children less

than 2 years old.

37. Painless rectal bleeding in a young child often presents a Meckel’s diverticulum (fecal occult

blood test positive), which is diagnosed by performing a technetium-99m pertechnetate scan.

38. A 2-year-old child can build a tower of 6 blocks, obey two-step commands, and use 2- to 3-

174

words phrases.Language Gross Motor Fine Motor Social

2 months Social smile Recognizes parents

3 months Holds head4 months Rolls back-to-

front and front-to-back

6 months Babbles Sits well upsupported

Raking grasp Recognizes stangers (stranger anxiety)

12 months 2 words; obeys 1-step command

Walks alone Throw objects Imitates action

15 months Build tower of 2 blocks

18 months Plays with other children

24 months 2-3 words; obeys 2-step command

Walks up- and downstairs without help

Build tower of 6 blocks; Turn pages of books

Parallel play

39. Conduct disorder is characterized by disruptive behavior patterns that violate basic social

norms for at least one year in pts less than 18 years old.

40. Chlamydia is the MCC agent of infectious neonatal conjunctivitis. Chlamydial pneumonia can

develop in infected infants. Gonococcal conjunctivitis is extremely purulent and presents

between the 2nd and 5th day after birth.

41. Osteogenesis imperfecta is an autosomal dominant disease characterized by mutations in type

I collagen. Pts with the severe type II form of this disease typically expire in utero due to

multiple ntrauterine and/or perinatal fractures. Clinical findings include limb deformities,

growth retardation, multiple fractures and blue sclerae.

42. Pts with genetic B-cell deficiencies begin to develop recurrent infections after passing 6

months of age. The deficient humoral immune response in these pts impairs the body’s ability

to destroy encapsulated organisms. Hence, recurrent sinopulmonary infections with H.

influenzae and S. pneumoniae are common. Lack of IgA also predisposes to Giardia infection.

Chronic granulomatous disease (CGD) is a condition of impaired oxidative metabolism. In CGD,

a defect in the NADPH-oxidase system of phagocytic cells results in defective intracellular killing.

Pts are therefore prone to abscesses secondary to catalse-producing organisms like Aspergillus and

Staphylococcus.

Gonococcal and meningococcal infections are common in pts with complement deficiency.

S.Pneumoniae and H.influenzae infections do occur in this pt population as well, but infections

with Giadia are not typically seen.

Thymic hypoplasia is one element of DiGeorge syndrome. The resultant T-cell deficiency puts pts

at high risk for viral and fungal infections.

Adenosine deaminase deficiency is commonly the genetic defect underlying severe combined

immunodeficiency (SCID). Pts with SCID have a deficiency of both B- and T- cells. The T-cell

deficiency predisposes to viral and fungal infections and the B-cell deficiency predisposes to

infection by bacteria.

43. A 12-month-old child walks alone, speaks two words, throws objects, and comes when called.

175

44. Apgar score0 1 2

Color Body and extremities are blue/pale

Body is pink and extremities are blue

Body and extremities are pink

Heart rate No activity <100 beats/min >100 beats/minReaction to nasal stimulatoin No response grimace Active coughTone/Activity limp柔软的 Some flexion of

extremitiesactive flexion of extremities

Respirations absent Slow and irregular Good respiratory effort

1/19/10

1. Most small ventricle septal defects close paontaneously and require no tx, as

long as there are no signs of pulmonary vascular disease. Reassurance, surveillance (via EKG

and echocardiograph), and infective endocarditis prophylaxis are all that are needed for mx.

VSD murmur: harsh and holosystolic heard at the left, lower sternal border. Surgical repair si

indicated in large or symptomatic defects.

2. Primary amenorrhea, Tanner stage 2 development, and aortic coarctation in a

teenager strongly suggest Turner syndrome (45, XO). Turner syndrome is the MCC of

primary amenorrhea. Dx is confirmed by karyotype analysis.

17-hydroxyprogesterone measurement is indicated when congenital adrenal hyperplasia is

suspected as a possible cause of primary amenrrhea. Serum 17-OH progesterone is elevated in 21-

and 11 hydroxylase deficiencies and is decreased in 17-hydroxylase deficiency.

3. Cystic fibrosis is an autosomal recessive disorder. If one of the parents has the

disease (is homozygous), you cannot predict whether a child would be likely to have the

disease without knowing the other parent’s carrier status.

4. Mongolian spots are most commonly found in dark-skinned populations,

present at birth, and usually fade or disappear in several years.

5. The most common causative organism of acute otitis media is Streptococcus

pneumoniae (40% of cases), followed by non-typable Haemophilus influenzae (25-30%), and

Moraxella catarrhalis (10-15%). The first-line tx is a ten-day course of amoxicillin.

6. Hand-foot syndrome or dactylitis is the earliest manifestation of vaso-

occlusion in sickle cell anemia, thereby warranting a complete workup for previously

asymptomatic sickle cell pts.

7. Rubella is characterized by low-grade fever, lymphadenopathy (sub-occipital

and posterior auricular) and rash. The rash is erythematous, maculopapular and classically

begins on the face, spreading subsequently down the body.

Erythema infectiosum or Fifth disease is caused by human parvovirus B19. Childre have different

presentations and develop a rash with a “slapped cheeks” appearance. Fever is not present or very

mild in this disease.

The clinical manifestation of chickenpox in healthy children generally develop within fifteen days

after the exposure and typically include a predrome of fever, malaise, or pharyngitis, followed by

the development of a generalized vesicular rash, usually within 24 hours. The lesions are

commonly prutitic and appear as successive crops of vesicles over a three to four day period. Pts

typically have lesions in different stages of development on the dace, trunk and extremities. New

176

lesions formation generally stops within 4 days, and most lesions are fully crusted by the 6th day in

normal hosts.

Roseola infantum is caused by herpes virus 6, and is characterized by the abrupt onset of high-

grade fever, which is then followed by a maculopapular rash, appearing on the trunk and then

spreading peripherally. The pt is no longer febrile when the rash develops. There are no positive

physical signs such as sore throat or lymphadenopathy during the febrile stage.

8. Friedreich ataxia is the most common type of spinocerebellar ataxias.

Remember the combination of neurologic (ataxia, hysarthrial), skeletal (scoliosis, feet

deformities), and cardiac (concentric hypertrophy cardiomyopathy) manifestations of the

disease. The MCC of death are cardiomyopathy and respiratory complications. * Very high

yield!!

9. Reactive pericarditis with a pericardial effusion can present after surgery for

congenital heart disease and is called postpericardiotomy syndrome. Symptoms usually occur

1-6 weeks after surgery. Although the cause is no known, postpericardiotomy syndrome is

thought to be an autoimmune response, possibly to a viral infection. Most children develop

mild symptoms which are self limited. In infants, pericardial effusion can present with

abdominal pain, vomiting, and decreased appetite. Older children may complain of chest pain.

Fever is often present. On exam, findings are consistent with pericardial and/or pleural

inflammation, including tachycardia, poor perfusion, pulses paradoxus, distant heart tones,

and jugular venous distention. If the effusion is large enough to affect the function of the

heart, pericardial tamponade can occur.

10. In an infant with meningococcemia, watch out for Waterhouse-Friderichsen

syndrome, which is characterized by a sudden vasomotor collapse and skin rash due to

adrenal hemorrhage.

11. The differential diagnosis of a solitary, painful, lytic long bone with overlying

swelling and hypercalcemia in a child should include Langerhans cell histiocytosis

(histoobtosis X, eosinophilic granuloma—the least severe form) as well as other neoplastic

processes like Ewing sarcoma.

12. Remember the classic triad of congenital rubella syndrome (CRS)-

sensorineural deafness, cardiac malformation (eg. PDA and ASD), and cataracts. CRS occurs

most commonly when transmission occurs in the first 4 weeks of pregnancy.

Struge-Weber syndrome is a neurocutaneous disease characterized by a port-wine stain on

trigeminal nerve distribution, mental retardation, seizures and glaucoma.

13. Microcytic anemia caused by iron deficiency can often be differentiated from

thalassemia by an elevated RDW, which is typically greater than 20% in iron deficiency. The

reticulocyte count is low in pts with iron deficiency due to decreased erythropoiesis.

14. A typically presentation of Tetralogy of Fallot (TOF) individuals is squatting,

which increases the systemic vascular resistance and forces blood into the lungs, thus

improving cyanosis. Other manifestations include “tet” spells, which are hypoxic episodes

that are characterized by paroxysms of deep, rapid breathing, and are caused by an increased

pulmonary vascular resistance. Crying, infection, and exercise can lead to tet spells. Severe

spells may precipitate seizures and loss of consciousness. The immediate tx is asministration

of oxygen and placing the child in a knee-chest position, following by the administration of

fluids, morphine and propranolol.

177

15. The MCC of viral meninigitis are the non-polio enteroviruses, such as

echoviruses and coxsachievirus.

16. Marfan’s syndrome (MFS) is an autosomal dominant disorder that results

from the mutations of the fibrillin-1 (FBN1) gene. It presents with tall stature, long and

emaciated extremities, arachnodactyly, hypermobility of the joints, upward lens dislocation,

and aortic root dilation.

Homocystinuria is an autosomal recessive disorder that results from cystathinine synthase

deficiency, an enzyme involved in the metabolism of methionine. These pts share many features of

Marfan’s syndrome but they usually have a fair complexion 肤色 , thromboembolic events and

osteoporosis. The other main differentiating feature is the type of lens dislocation. In Marfan’s

syndrome, the lens is typically dislocated upward, whereas in homocystinurea it is dislocated

downward.

Congenital contractural arachnodactyly (CCA) is an autosomal dominant condition that presents

with tall stature, arachnodactyly, and multiple contractures involving large joints. Marfan’s pts do

not have joint contractures. Furthermore, ocular and cardiovascular symptoms are not present in

CCA.

Ehlers-Danlos syndrome is a disorder of collagen structure, which may also have hypermobile

joints; however, it also results in easy bruising, poor wound healing and soft, velvety, and

hyperelastic skin. Severe complications such as organ rupture and severe hemorrhage may occur.

17. Internal carotid artery dissection is a potential cause of stroke in children. The

history of a fall on a pencil or on a stick in a child’s mouth within 24 hours of the onset of

symptoms is typical.

18. In pts with sickle cell disease, acute severe anemia with absent reticulocytes in

the peripheral blood smear is due to aplastic crisis.

19. Recurrent hemarthroses in pts with coagulopathies lead to a joint injury called

‘hemaphilic arthropathy’.

20. Transposition of the great vessels (a normal S1, single and a loud S2, and no

murmur) is the most common cysnotic heart disease which presents with cyanosis in the first

24 hours of life. It is commonly seen in infants of diabetic mothers and in males.

Echocardiography confirms the dx. Maintaining the patency of the ductus arteriosus is

important for survival. Surgical tx is definitive.

21. Deficiency of sphingomyelinasae causes Niemann-Pick’s disease, which is

characterized by cherry red macula, protruding abdomen, hepatosplenomegaly,

lymphagenopathy, and regression of development milestones.

Sphingolipidosis due to a deficiency in hexosaminidase A is known as Tay-Sachs’ disease. It is

characterized by hyperacusis, mental retardation, seizure, cherry red macula, but not

hepatosplenomegaly or cervical lymphadenopathy.

Sphingolipidosis due to a deficiency in glucocerebrosidase is known as Gaucher’s disease. It is

characterized by hepatosplenomegaly, anemia, leucopoenia, and thrombocytopenia, but not cherry

res macule.

Sphingolipidosis due to a deficiency in galactocerebrosidase is known as Krabbe’s disease. It is

characterized by hyperacusis听觉过敏, irritability and seizure.

Mucopolysaccharidoses are characterized by coarse facial features, hydrocephalus and umbilical

hernia.

178

22. Turner syndrome should be considered in newborns with a webbed neck, high

palate, short fourth metacarpal, and nail dysplasia. Lymphedema frequently occurs due to

abnormal development of the lymphatic system.

23. The most common predisposing factor for acute bacterial sinusitis is a viral

upper respiratory infection.

24. Bedwetting is considered normal until the age 4-5 years. If nocturnal enuresis

perisists, DDAVP (drug of choice) or imipramine may be used.

25. Post-exposure rabies prophylaxis can be lifesaving and should be initiated

after exposure to saliva or neural tissue from bats, wild carnivores, or other animals suspected

of being rabid. People bitten by domestic animals suspected of being rabid or not available for

observation, or by wild carnivores like raccoons, skunks, and foxed should also receive

prophylaxis.

26. Aplastic anemia should be suspected in any pt with pancytopenia following

drug use, exposure to toxins or viral infections. A bone marrow biopsy is essential to make the

dx, it typically shows profound hypocellularity with a decrease in all cell lines and fatty

infiltration of the marrow.

Classically, pts with Fanconi’s anemia have pancyotpenia and characteristic congenital anomalies,

such as hyperpigmentation on the trunk, neck and intertriginous areas and/or café-au-lait spots,

short stature, upper limb abnormalities, hypogonadism, skeletal anomalies, eye or eyelid changes,

and renal malformations. Blood counts start to decrease between 4 and 12 years of age, and the

initial manifestation is usually thrombocytopenia, followed by neutropenia, then anemia.

Diamond-Blackfan anemia (DBA), or congenital pure red cell aplasia, presents in the first 3

months of life with pallor and poor feeding. CBC reveals a normocytic or macrocytic anemia with

reticulocytopenia. WBC and platelet counts are normal.

Transient erythroblastopenia of childhood (TEC) is an acquired red cell aplasia which occurs in

healthy children between 6 months and 5 years old. There is a gradual onset of symptoms such as

pallor and decreased activity. The physical exam is unremarkable except for pallor and

tachycardia. The typical lab findings ar normocytic normochromic anemia, with hemoglobin

levels ranging from 3 to 8 g/dL, and extremely low reticulocyte count.

Bone marrow infiltration due to leukemia results in pancytopenia by crowding out the normal

bone marrow element.

27. Suspect necrotizing enterocolitis in a newborn with abdominal distention,

bloody diarrhea, and leukocytosis. These symptoms usually occur after the introduction of

formula feeding. The radiologic finding of pneumatosis intestinalis (ie intramural air) in

infants is diagnostic.

28. Undetected hearing impairment can earsily be confused with certain pervasive

and behavioral disorders of childhood. Therefore, hearing tests should be conducted in all

such children.

29. Compartement syndrome can be a complication of supracondylar humerus

fractures (X-ray picture). It is characterized by severe pain, pallor, poikilothermia,

paresthesias, and the late finding of pulselessness and paralysis. Initial tx incudes removal of

any bandages, measurement of compartment pressures, and emergent orthopedic evaluation

for possible fasciotomy.

30. Intussusception is the most common cause of intestinal obstruction in the first

179

two years of life. In addition to the signs of interstinal obstruction (eg. severe, sudden-onset

abdominal pain and vomiting), pts present with red currant jelly stool containing blood and

mucus.

31. Fetal alcohol syndrome is characterized by irritability, mild to moderate

mental retardation, hypoplastic maxilla, long philtrum, thin upper lip border, and

microcephaly.

Fetal hydantoin syndrome is characterized by hypoplasic nails, cleft palate, and vitK deficiency,

which may result in bleeding.

32. Nursemaid’s elbow occurs when infants or children are lifted or pulled by the

hand or arm. The mechanism of the injury involves radial head subluxation. The child

typically keeps the hand in a pronated position, and refuses (cries out in pain) attempted

forearm supination.

Panner disease is osteochondrosis of the capitellum小头 . The typical pt is an adolescent who is

actively engaged in sport activities that involve throwing, and the common complaints include

pain, crpitation and loss of motion of the arm (particularly pronation and supination).

The most common elbow dislocation is posterior dislocation, with is caused by falling backward

onto the outstretched arm with the elbow extended. An obvious deformity is noted, with the

olecranon process displaced prominently behind the distal humerus.

33. Colic presents by 3 weeks of age with excessive crying for more than 3 hours

a day, more than 3 days a week, and more than 3 weeks a month, usually resolving by 4

months of age.

34. Anemia of prematurity is the most common anemia in premature and low birth

weight infants. The pathology involves a combination of diminished RBC production,

shortened RBC life span, and blood loss. Iron supplementation does not prevent falling

hemoglobin levels, and iron deficiency is not the cause of anemia of prematurity. The tx

involves iron supplementation, periodic hemoglobin checking and blood transfusion, if

needed. Erythropoietin is not routinely used. Lab studies: 1) Peripheral smear shows

normocytic and normochromic anemia. No other abnormal forms are seen. 2) the reticulocyte

count is low, and RBC precursors in the bone marrow are decreased. 3) Normal WBC and

platelet counts. 4) Normal total bilirubin level.

35. ADHD should be considered as the most probable dx in any pts with

symptoms of short attention span, impulsivity, and hyperactivity for more than 6 months in

more than one setting.

36. Vaginal foreign bodies should be suspected in children with purulent, foul-

smelling vaginal discharge and bleeding. If a foreign body is seen, irrigation with warmed

fluid should be performed in an attempt to flush the foreign body. If irrigation is unsuccessful,

exam and foreign body removal should be done with sedation or general anesthesia.

37. The body derives vit K from the diet and from gut flora synthesis. Deficiency

in newborn babies is the result of poor placenta transfer, absent gut flora, and inadequate

levels in breast milk. To prevent hemorrhagic disease of the newborn, it is recommended that

all newborn babies receive a vit K injection.

38. Racemic epinephrine decreases the need for intubation in pts with croup and

should always be tried before any invasive procedure.

39. Painless melena in a 2 to 3-year-old child is most likely due to Mechel’s

180

diverticulum. It results from the failure of the vitelline duct to obliterate during the fetal

development. The dx of a Meckel’s diverticulum is best made with technetium-99m

pertechnetate scanning (uptake by heterotopic gastric mucosa).

40. The tx of Kawasaki’s disease consists of aspirin and IV immunoglobulins.

41. Suspect Duchenne muscular dystrophy in a child with proximal muscle

weakness, positive Gower’s sign, depressed reflexes, and pseudohypertrophy of the calf

muscles. Dystrophin is usually absent. EMG shows myopathic pattern. Serum CK levels are

very high (4000-5000IU). The dx is confirmed by muscle biopsy.

42. Human milk is the ideal form of nutrition for term infants. The major protein

source is whey乳清, which is more easily digested than casein酪蛋白 and helps to improve

gastric emptying.

43. Infectious mononucleosis is an infection caused by the EB virus, and is

sometimes detected only when the pt develops a characteristric polymorphous rash after

taking ampicillin for an apparent upper respiratory tract infection.

44. Severe dehydration in a neonate in the presence of hyponatremia,

hyperkalemia, hypoglycemia, and metabolic acidosis suggests a diagnosis of congenital

hyperplasia or salt wasting 21-hydroxylase deficiency.

There are 2 main abnormalities involving the galactose metabolic pathways. Galactose-1-

phosphate uridyl tranferase deficiency is a sever condition that can lead to shock if untreated.

Clinical features include vomiting, diarrhea, hepatomegaly, catatect, mental retardation, and

hypoglycemia. Galactokinase deficiency is a milder condition which generally presents with

catatact if untreated. Both conditions are inherited as autosomal recessive.

1/20/10

1. Subarachnoid hemorrhage (SAH) can be caused by by an intraventricular hemorrhage, which

is common in premature infants. Accumulation of the blood in the subarachnoid space may

lead to destruction of the arachnoid villi and cisterns, thereby blocking the flow or decreasing

the absorption of CSF, and leading to communicating hydrocephalus (CT scan shows dilation

of the entire ventricular system with distinct enlargement of the subarachnoid space over the

cerebral cotex). SAH is the most common cause of communicating hydrocephalus. Dandy-

Walker anomaly and Chiari malformation will both reveal CT findings consistent with

obstructive or noncommunicating hydrocephalus. Dandy-Walker Anomaly will demonstrate a

cystic expansion of the 4th ventricle, and Chiari malformation will reveal protrusion of the

structures of the posterior fossa through the foramen magnum.

2. Intussusceptin is best diagnosed and treated by an air contrast enema.

3. Suspect meningococcemia in a neonate with signs of meningitis and a petechial rash. 75% of

pts with Meningococcus meningitis present with a petechial rash that is prominent on the

axilla, wrist, flanks and ankles. It appears with 24 hours of the infection, and the pt generally

appears sick.

4. Sturge-Weber syndrome is a neurocutaneous syndrome that is characterized by a congenital

unilateral cavernous hemangioma along the trigeminal nerve distribution and radiographic

evidence of intra-cranial calcifications that resemble a tramline.

5. Increasing head circumference and signs of increased intracranial pressure in children should

181

be evaluated with a CT scan of the brain. Symptoms of hydrocephalus in infants include poor

feeding, irritability, vomiting, and decreased activity. On exam, the fontanelle may be tense

and buldgig, the scalp veins may appear prominent with shiny, tight skin overlying the

vessels, and the cranial sutures may be widely spaced. On review of the growth charts, the

head circumference may be rapidly increasing in size to greater than the 97th percentile. Tx

consists of a shunt that is placed form the ventricle to the peritoneum, pleura, or right atrium,

which allow the excess CSF to drain rather than continue to collect in the ventricles.

6. With suprachondylar fracture, the brachial artery can be compromised, resulting in the loss of

the radial artery pulse; therefore, the radial artery pulse must be assessed when the fracture is

reduced.

7. To prevent SIDS (sudden infant death syndrome), infants should be placed on their backs

(supine position) while sleeping. Home monitors do not decrease the risk of SIDS.

8. A positive Coomb’s test points towards autoimmune hemolytic anemia, and a positive osmotic

fragility test points towards hereditary spherocytosis. Hereditary elliptocytosis is a rare

disorder that is characterized by elongated RBC. Microcytes, spherocytes, and other

poikilocytes may also be seen.

9. Recognize fetal alcohol syndrome and know that it is the most common cause of mental

retardation in children. Remember the midfacial abnormalities (short palpebral fissures,

epicanthal folds, ling philtrum, thin upper lip).

10. Suspect ITP in children who develop isolated thrombocytopenia after a viral infection. ITP is

a relatively benign condition. There is some controversy regarding the tx, but in general,

corticosteroids are the drugs of choice in all age groups for thrombocytopenia less than

30,000/mm3, and/or for sever symptoms. Pts with a platelet count of more than 30,000/mm3

usually have very few symptoms and do not require tx.

11. An immediate anaphylactic reaction, an encephalopathy, or any CNS complication within 7

days of administration of the vaccine is a C/I for further administration of DTaP. In these

instrance, DT should be substitute for DTaP since the adverse reactions are susally attribute to

the pertussis compoment of the vaccine. * Extremely high yield!!

12. RSV infection may increase the risk of asthma later in life.

13. Children with a parental history of elevated total cholesterol levels (>240mg/dL), or risk

factors for coromary artery disease should get a screening test for total cholesterol level. A fast

lipid profile is recommended if the total cholesterol level is greater than 200mg/dl. It is

usually not the first step in evaluating an asymptomatic pt.

14. Henoch-Schonlein purpura (HSP) is characterized by palpable purpura, scrotal swelling,

hematuria and abdominal pain. Children with HSP are susceptible to intussusception.

15. Simple febrile seizures do not typically require an extensive work-up, and these pts can

usually be discharged home from the emergency department.

16. Slipped capital femoral epiphysis is a common hip disorder seen in overnight adolescents. On

exam, affected pts tend to hold the hip in passive exernal rotation and exhibit decrased

internal rotation, abduction, and flexion. Dx is made with plain radiographs of the hip (Ap and

frog leg lateral view), which show the posteriorly and inferiorly displaced femoral head. Tx

consists of emergent orthopedia consultation and surgical fixation at the current degree of

slippage to avoud the risk of avascular necrosis. Avascular necrosis can also occur in Legg-

Calve-Perthes disease, which is most commonly seen in younger children (age 5-7) with

182

idiopathic infarction of the femoral head.

17. Turner’s syndrome with 46 XY karyotype is associated with a higher incidence of

gonadoblastoma, hence, prophylactic bilateral gonadectomy is indicated in the mx of such pts.

18. Renal tubular acidosis (RTA) is a normal anion gap metabolic acidosis caused by a defect in

the ability of the renal tubules to reabsorb bicarbonate or excrete hydrogen. There are 3 types

of RTA:

1) Type 1 RTA or distal RTA occurs due to a defect in hydrogen ion secretion. These pts are

acidotic, hypokalemic, and have an elevated urinary PH. In children, Type 1 RTA is often a

genetic disorder. Pts commonly develop nephrolithiasis.

2) Type 2 RTA is caused by decreased bicarbonate reabsorption in the proximal tubule. Fanconi

syndrome is a common cause in children.

3) Type 4 RTA is caused by a defect in the sodium/potassium exchange in the distal tubule

which results in hyperkalemia, hyperchloremic acidosis. In children, obstructive uropathy,

renal disease, or multicystic dysplastic kidney are common causes.

RTA can present as growth failure and should be considered in the differential dx for failure to

thrive. Screening labs will show a low bicarbonate level with an increase in chloride, producing a

normal anion gap metabolic acidosis.

19. Down syndrome pts are prone to endocardial cushion defects which can rapidly cause

pulmonary hypertension.

20. Mammary gland enlargement and non-purulent vaginal discharge are common findings in

newborn infants. These are transitory physiologic events; therefore, such infants only require

observation and routine care. * Extremely high yield!!

21. Pts with functional asplenia are at risk of infection with capsulated organisms, and the most

common cause of sepsis in such pts is pneumococcus. Pts with sickle cell disease encounter

constant clumping of sickle cells, which leads to repeated microinfarctions in the spleen. By

2-3 years of age, most such children have functional asplenia. Since one of the spleen’s

important immunologic functions is the removal of capsulated organisms such as

pneumococci and H. influenza, pts with functional asplenia are at risk of infection with these

organisms. For this reason, antibiotic prophylaxis and vaccination against pneumococci and

H. influenza are standardly given to pts with functional asplenia.

22. Ventrical septal defects can cause failure to thrive, easy fatigability, and heart failure. On

exam they characteristically cause a pansystolic murmur loudest at the left lower sternal

border, plus a rumbling diastolic flow murmur at the apex.

23. X-linked agammaglobulinemia is an inherited immune deficiency characterized by recurrent

bacterial infections early in life. Suggestive lab data includes a normal amount of T

lymphocytes with a low or absent concentration of B lymphocytes. It is treated with regular

infusions of iv Ig.

24. Painless gross hematuria is the most common presentation of sickle cell trait. The

pathophysiology involves a defect in the renal tubular function, thereby leading to the

inability to properly concentrate urine.

25. The long-term neurologic sequelae associated with bacterial meningitis are: 1) hearing loss, 2)

loss of cognitive functions (due to the neuronal loss in the dentate gyrus of the hippocampus).

3) seizures, 4) mental retardation. 5) spasticity or paresis.

26. 21-hydroxylase defeiciency is the most common form of CAH. It presents with virilism, salt

183

wasting, and increased 17-alpha-hydroxyprogesterone level. Other deficiencies leading to

CAH include (1) 11-alpha-hydroxylase deficiency, which results in androgen and

mineralocorticoid excess, and (2) 3-beta-hydroxysteroid dehydrogenase deficiency, which

results in DHEA-S excess and decreased testosterone and mineralocorticoids. In 21-

hydroxylase defeiciency, all other metabolites are decreased.

27. Chronic pyelonephritis is characterized by focal parenchymal scarring and blunting of calices

on IVP. Hydronephrosis is seen on IVP as dilation of the collecting system including the

calyces, pelvis and ureter, depending on the level of obstruction.

28. All children with recurrent episodes of nocturnal vulvar itching should be examined for

pinworms (Scotch tape test) and treated empirically with mebendazole.

29. Pts with Down’s syndrome are more likely to develop duodenal atresia, Hirschsprung’s

disease, endocardial cushion defects and acute leukemia.

30. Food, esp peanut, allergies are the major cause fo outpatient anaphylaxis in children. Signs

include bronchoconstriction, hypotension and urticaria. Subcutaneous injection of epinephrine

is the first-line tx for anaphylaxis in a pt with a patent airway.

31. In pts with apparent subcutaneous emphysema secondary to severe coughing paroxysms,

CXR must be obtained first ot rule out pneumothorax.

32. Vit A can be of benefit in the tx of measles infection.

33. Pts present with precocious pubarchem, it is very important to differentiate between

precocious puberty that is caused by premature activation of the hypothalamus-pituitary-

gonad (HPG) axis and precocious pseudo-puberty that is casued by a gonagotropin-

independent process, typically an excess of sex steroids. Precocious pubarche with signs of

severe androgen excess is suggestive of precocious pseudo-puberty that is caused by a

gonadotropin-independent process (typically an excess of sex steroids). It can be caused by

late-onset congenital adrenal hyperplasia.

Hypothalamic dysfunction leading to precocious puberty is usually less dramatic in presentation.

Sequential development of testicular enlargement, penis enlargement, pubic hair growth, and then

growth spurt is typically present.

34. Displacement anterior fat pad is a radiographic sign of supracondylar fracture, which may be

complicated by Volkmann’s ischemic contracture.

35. Minimal change disease is a highly steroid-sensitive condition and is the MCC of nephritic

syndrome in children. For these reason, empiric steroid therapy is indicated in any child with

a clinical presentation and findings that are suggestive of nephritic syndrome.

36. Edward’s syndrome is characterized by micrognathia, microcephaly, rocker bottom feet,

overlapping fingers, and absent palmar creases. Ventricular septal defect is common in pt with

this disease.

ASD and endocardial cushion defects –trisomy 21

Supravalvular aortic sternosis—William’s syndrome

Conotruncal abnormalities (Truncus arteriosus, Tetralogy of Fallot, interrupted aortic arch)—

CARCH-22 syndromes, including DiGrorge and velocardiofacial syndromes

Congenital heart block—neonatal lupus

Paten ductus arteriosus—congenital rubella

Coronary artery aneurysm—Kawasaki disease

37. Henoch-Schonlein purpura is an IgA-mediated vasculitis of small vessels, which results in

184

rashes, arthralgias, abdominal pain and renal disease. Immunofluorescence microscopy

reveals IgA deposition in the kidney.

38. Anabolic steroids are used to improve physique and athletic performance but they are

associated with numerous adverse effects, including acne, baldness, gynecomastia, hepatic

dysfunction, altered lipid profiles, virilization, testicular failure, and mood and behavior

changes.

39. Neonatal tetanus is generally seen in infants born to unimmunized mothers, frequently

following umbilical stump infection. Affected infants initially present in the first 2 weeks of

life with poor suckling and fatigue, following by rigidity, spasms and opisthotonus.

40. Rheumatic fever is a complication of streptococcal pharyngitis and is diagnosed clinically

using the Jones criteria. The major criteria include carditis, migratory polyarthritis, Sydenham

chorea, subcutaneous nodules, and erythema marginatum.

Juvenile RA is diagnosed by arthritis that is present for greater than 6 weeks. Systemic symptoms

including a rash can be seen in children with JRA, but the arthritis is usually not migratory as

described in this pt.

41. Stranger anxiety is the normal anxiety experienced by infants when they are exposed to

unfamiliar individuals. It peaks at 12-15 months.

42. Isotonic solutions such as normal saline are the fluid of choice for initial resuscitation in

severe hypovolemic hypernatremia.

43. Although adequate hydration is very important in the mx of diarrhea, replacement of

electrolytes is also essential to prevent complications such as water intoxication and

hypomatremia. Water intoxication, hyponatremia, and seizure may result if diarrhea is treated

with large amounts of hypotonic or low-solute fluids (eg. water).

44. The vaccination schedule for preterm infants should be conformed to the child’s chronologic

age, not the gestational age. The exception is that children should be 2kg prior to receiving the

first HBV vaccine.

1/21/10

1. IVH is most commonly seen in premature and LBW infants. Pts may present with pallor,

cyanosis, hypotension, seizurem focal neuologic signs, bulging or tense fontanel, apnea, and

bradycardia; however, many cases remain asymptomatic, thus mandating transfontanel

ultrasound for all newborns with predisposing risk factors.

2. In the pediatric population, infratentorial tumors are more common than supratentorial

tumors, and benign astrocytomas are the most common histology type in both groups.

Medulloblastoma is the second most common.

3. Bedwetting is a normal phenomenon until the age of 5.

4. Umbilical hernia is due to an imperfect closure or weakness of the umbilical ring. It is

especially common in low birth weight, female, and black infants. It appears as a soft

swelling, covered by skin that protrudes during crying, coughing, or straining, and can be

reduced earsily through the fibrous ring at the umbilicus. The hernia consists of ometum or

portions of the small intestine. Most umbilical hernias disappear spontaneously by 1 year of

age. Surgery is advised if the hernia persists to the age of 3-4 years, exceed 2cm in diameter,

causes symptoms, becomes strangulated, or progressively enlarge after the age of 1-2 years.

An omphalocels is a herniation or protrusion of abdominal contents into the base of the umbilical

185

cord, and it usually diagnosed at birth. In contrast to the more common umbilical hernia, the sac is

covered with peritoneum without overlying skin. Immediated surgical repair before infections take

place is essential for survival.

Gastroschisis is diagnosed at birth by protrusion of bright red intestines to the right side of the

umbilicus. There is no covering of the intestines, and the umbilicus is normal. It is a surgical

emergency.

5. The features of Prader-Willi syndrome (obesity hypogonadism syndrome) include hypotonia,

hypogonadism and obesity. Remember mental retardation, and dysmorphic craniofacial

features (ie., narrow bifrontal diameter, diamond-shaped eyes and a small, down-turned

mouth).

6. Breastfeeding jaundice is an exaggeration of physiologic jaundice commonly seen in

exclusively breastfed infants who are not getting enough breast milk. When effective

breastfeeding is not established in the first few days of life, the inadequate enteral intake

prolongs the intestine transit time, resulting in an increased enterohepatic circulation. In

addition, the infant becomes relatively dehydrated from the limited fluid intake. The increased

enterohepatic circulation and relative dehydration together cause an elevated unconjugated

bilirubin level in the first few days after birth.

7. The initial mx of breastfeeding jaundice is to stimulate milk production by increasing the

nursing frequency. The infant should be monitored closely (repeat bilirubin level 12-24 hours)

to ensure that the feeding problems are resoved and the bilirubin decreases. If the infant’s

bilirubin continues to rise, then interventions such phototherapy (bilirubin levels >17-

25mg/dL), exchange transfusion (bilirubin levels > 25mg/dL), or formula supplementation

may become necessary to prevent the complication known as nernicterus.

Breastfeeding jaundice that occurs in the first few days of life must be distinguished form breast

milk jaundice, which usually starts after the first 3 -5 days of life, peaks 1-2 weeks after birth, and

can lasat several months. An otherwise thriving infant will present with jaundice and elevated

unconjugated hyperbilirubinemia. The etiolgoyc is not completely understood, but it is thought

that a substance in breast milk may inhibit UDPGA or that the increased beta glucuronidase

activity in breast milk may increase the enterohepatic circulation of bilirubin.

ABO incompability is caused by maternal antibodies against fetal erythrocytes. Most commonly,

ABO incompatibility is seen when the mother’s blood type is O and the infant’s blood type is A ,

B, or AB. The jaundice of ABO incompatibility tends to manifest in the first 24 hours of life.

8. Urinalysis is a preliminary inverstigation that should be performed first in all pts with

suspected renal disease.

9. Wiskott-Aldrich syndrome is an X-linked disorder characterized by the triad of

thrombocytopenia, eczema (there are dry, scaly patches on his cheeks and lower extremities),

and recurrent bacterial infections. The thrombocytopenia is caused by decreased platelet

production, and the few platelets that exist are typically quite small.

10. Coarctation of the aorta (COA) is characterized by hypertension in the upper part of the body

and relative hypoperfusion in the lower part of the body. A mild, continuous murmur heard all

over the chest is due to the development of collaterals between the hypertensive and

hypoperfused vessels. Rb notching, caused by the dialatation of the collateral chest wall

vessels, is specific for coarctation.

11. The most common esophageal anomaly is esophageal atresia with a tracheoesophageal fistula.

186

This is charaterzied by an atretic esphoageal pouch that communicates diatally with the

trachea just above the carina. This anatomic arrangement allows air to enter the stomach, and

this leads to gastric distention. The discontinuous esophagus prevents the infant from

completely swallowing, thereby resulting in drooling or regurgitation during feeding. In

addition, gastric fluid ascends into the distal esophagus through the fistula, into the trachea

and lungs, thereby producing aspiration pneumonia. Pneumonitis and atelectasis occur

frequently, and rattles are heard during breathing. The inability to pass a feeding tube into the

stomach is suggestive of esophageal atresia with or without tracheoesophageal fistula.

12. Congenital syphilis presents early on with hepatomegaly, cutaneous lesions, jaundice, anemia,

and rhinorrhea. Metaphyeal dystrophy and periostitis may be seen on radiography.

13. The classic triad of Kartegener’s syndrome is recurrent sinusitis, bronchiectasis, and

dextrocardia. The cause is primary ciliary dyskinesia.

14. Pneumococcal vaccination plus penicillin prophylaxis can prevent almost all cases of

pneumococcal sepsis in pts with sickle cell anemia.

15. Arteriovenous malformation (AVM) is the most common of the subarachnoid hemorrhage in

children. The history of seizure and migraine-like headaches is characteristic.

16. Aseptic necrosis of the femoral head is a common complication of sickle cell disease. It

involves occlusion of end arteries supplying the femoral head, bone necrosis, and eventual

collapse of the periarticular bone and cartilage. * Extremely high yield!!

17. Prostaglandin E1 infusions may be used to keek the ductus arteriosus patent in pts with

ductus-dependent congenital heart disease until definitive therapy can be pursued.

18. Those with sickle cell trait have some protection against malaria.

19. Chronic garnulomatous disease is an inherited immunodeficiency disorder marked by an

inability to oxidize pathogens within phagocytes. Pts experience recurrent infections starting

early in life. Pneumonia and suppurative adenitis are especially common infections in these

pts.

20. Acute bacterial sinusitis for children <6 years of age is a clinical dx, and umcomplicated cases

are treated with oral amoxicillin.

21. Hyper-IgM syndrome (HIM) is characterized by high levels of IgM with deficiency of IgG,

IgA and poor specific antibody response to immunization.

22. Individuals who will be exposed to tick-infested areas should wear light-colored clothing with

long-sleeved shirts and long pants tucked into socks or boot tops.

23. Epiglottitis is a pediatric emergency. Secure the airway as soon as possible with endotracheal

intubation and set-up for possible tracheostomy. Visulization of the epiglottis should not be

attempted. In the pre vaccination era almost all the cases were due to Haemphilus influenzae

type b (Hib). Because of the routine infant vaccination with Hib conjugate vaccines, the

antimicrobial spectrum has changed. H.influenza type A and nontypable strains,

H.parainfluenzea, Streptococcus pneumoniae, Staphylococcus aureus, and beta-hemolytic

streptococci are commonly isolated in Hib vaccinated pts. Nebulized racemic epinephrine is

used in the mx of croup.

24. 24-hour esophageal pH monitoring is the gold standard for the dx of GERD.The position that

the child assumes (ie. Tilted head and arched back) is called “posturing of Sandifer’s

syndrome,” and the mechanism by which the child protects his airways and reacts to the pain

of the acid reflux.

187

25. Breath holding spells are episodes of apnea that are sometimes associated with a loss of

consciousness, and are precipitated by frustration, anger, or pain. These episodes usually

occur in children ages 6-18 months. Classically, the child gets frustrated and holds their

breath, which may lead to loss of ocnsciousness followed by spontaneous resolution. Unlike a

seizure, there is no incontinence or postictal phase associated with breath holding spell. These

spells can be simple (no change in circulation or oxygenation), cyanotic, pallid 苍白的 , or

complicated. Electroencephalograms are normal. Breath holding spells are self-limited and

should be treated with education and reassurance. Anemic pts, however, should be treated

with iron supplementation.

26. Tetralogy of Fallot consists of pulmonary stenosis, an overrding aorta, a vertriculoseptal

defect, and right ventricular hypertrophy. “hypercyanotic” spell or “tet” spells can be

improved with a knee to chest position, which increase systemic vascular resisitance and

therefore impoves pulmonary blood flow. Morphine and an iv fluid bolus can also be given to

increase pulmonary flow. Oxygen is of little benefit because the abnormality is decreased

pulmonary blood flow, not sufficienty oxygenation.

27. Post-exposure prophylaxis for chicken pox can be provided with VZIG or acyclovir. Post-

exposure prophylaxis with VZIG is preferred and indicated in susceptible high-risk persons

exposed to varicella within 96 hours (preferred 72 hours) of exposure.

28. Streptococcus pneumoniae is the MCC of acute sinusitis in childhood, followed by

Haehophilus influenza and Moraxella cararrhalis.

29. Spontaneous hemarthrosis raises the suspicions for hemophilia, for which factor VIII assay is

diagonistic. Prolonged PTT, normal prothrombin time, normal bleeding time, normal

fibrinogen level and low serum factor VIII activity are the typical lab findings. The standard

tx for hemophilia is to replace the factor VIII. However, mild hemophilia may be treated with

DDAVP, which causes release of factor VIII from the endothelial cells.

30. When a child present with signs of increased intracranial pressure and meningitis, CT scan of

the head should be performed before performing a limbar puncture to avoid the possiblitiy of

herniation; however, this shoud not delay antibiotic (3rd generation cephalosporin, such as

cefotaxime; penicillin-resistant pneumococcus requires the addition of vancomycin)

administration.

31. Marfan’s features + mental retardation + thromboembolic events + downward dislocation of

the lens = homocystinuria. It is an autosomal recessive disease caused by cystathionine

synthase deficiency. Tx mainly involves administration of high doses of vit B6. * High Yield!!

32. Neonatal jaundice with conjugated hyperbilirubinemia is suggestive of neonatal cholestasis

and warrants further evaluation. Indications of the evaluation of neonatal jaundice:

1)conjugated hyperbilirubinemia (>2mg/dL); 2) jaundice that appears in the first 24-36 hours

of life. 3) serum bilirubin rising at a rate faster tham 5 mg/dl/24 hours . 4) Serum bilirubin

greater than 12mg/dl in full-term (esp n the absence of risk factors) or 10-14 mg/dl in preterm

infants. 5) Jaundice persists after 10-14 days. 6) The presence of signs or symptoms.

33. Tetralogy of Fallot, the most common cyanotic congenital disease in children less than 4 years

of age, presents with cyanotic spells and pansystolic murmur on exam (a harsh pansystolic

murmur is noted at the left sternal border, along with a single S2).

34. Suspect duodenal atresia when the pt has bile-stained vomitus, no abdominal distension,

dehydration, double bubble sign, and Down syndrome. Vomiting usually begins in the first

188

24-48 hours after birth.

35. Hydroxyurea is indicated in sickle cell pts with frequent, acute, painful episodes, as it tends to

increase the HbF levels.

36. Iron poisoning presents as nausea, vomiting, diarrhea, abdominal pain, gastrointestinal

bleeding and metabolic acidosis. Since iron is radio-opaque, the tablets can be seen in the

stomach on abdominal x-ray. The dx is confirmed by measuring serum iron levels. Iv

deferoxamina, an iron chelator, is used in moderate to severe cases of intoxication.

37. Capillary blood specimens (using a finger stick) are widely used in childhood screening for

lead poisoning. Since false positive results are common, the first step after an abnormal

fingerstick lead level is to confirm the dx with a serum (venous blood) lead level. If the serum

lead level is > 10ug/dl, then intervention is needed.

38. Recurrent self-limiting episodes of vomiting and nausea in children, in the absent of any

apparent casue, suggest the dx of cyclical vomiting. The etiology of this condition is unclear,

however, its incidence is high in children whose parents have a history of migraine headaches.

Complications that may arise from the condition are anemia and dehydration. Tx consists of

anti-emetics and reassuranve of the parents.

39. A minor who is a parent can consent to treatments or procedures for his/her child.

40. A retropharyngeal abscess should be suspected in pts who present with pharyngitis, moderate

fever, dysphagia, neck stiffness, a muffled voice, a posterior pharyngeal bulge.

41. Inflammed, fluctuant cervical LN are typically caused by streptococcal or staphylococcal

infection. The antibiotic of choice is dicloxacillin二氯苯甲异恶唑青霉素钠. Other effective

antibiotics include cephalexin or clindamycin.

42. Night terrors occur during non-REM sleep and are characterized by fear, crying or screaming,

decreased level of consciousness, and amnesia of the event.

43. Bordetella pertussis causes whooping cough, which are severe paroxysms of cough that can

last up to two weeks. The coughing spells are so severe that rectal prolapse and

pneumothoraces can ensue.

44. The TORCH infections (toxoplasmosis, rubella, CMV, HSV and syphilis) cause a syndrome

characterized by microcephaly, hepatosplenomegaly, deafness, chorioretinitis, and

thrombocytopenia.

1/22/10

1. Language development:

12-month: “mama”, “dada”, and one other word. Be able to follow simple commands with a

gesture

15-month: “mama”, “dada”, and 3-5 other word. Understand simple commands even without a

gesture.

18-month: a vocabulary of 5-20 words.

2-year: a vocabulary of 150-300 words and be able to combine words into short sentences. In

addition, half of their speech should be understandable to a stranger.

3-year: vocabulary has increased ti close to 1000 words, and about 75% of the children’s speech

can be understood by a stranger. In addition, they speak in 3-4 word sentences and use pronouns

correctly.

2. Suspect growing bone pain in a child with nocturnal pain and no obvious physical or lab

189

finding. The tx is reassurance. Common in age 2-12 year, is usually poorly localized and

common in the legs (usually below the knees and bilateral). In all cases, it resolves in the

morning. The dx of growing pain is a diagnosis of exclusion.

3. Pts with classic PKU present with fair skin, blue eyes, musty body odor and eczema. Dx

of PKU: 1) plasma phenylalanine level greater than 20mg/dl. 2) normal to low-normal plasma

tyrosine level. 3) increased urinary levels of phenylalanine metabolites (phenylpyruvic and o-

hydroxyphenylacetic acids). 4) normal tetrahydrobiopterin level.

Pts with benign hyperphenylalaninemia also have a deficiency in phenylalanine hydroxylase, but

some residual enzyme activity is preserved; therefore, the serum phenylalanine blood levels are

only slightly elevated (less than 20mg/dl). Pts who undergo PKU screening may be identified

during the neonatal period, but they are otherwise asymptomatic and may develop normally

without tx.

Pts with transient hyperphenylalaninemia are asymptomatic.

Tyrosinemia is caused by a deficiency of fumarylacetoacetate hydrolase. Affected infants may

become symptomatic as early as the 2nd week of life. Common manifestations include fever,

irritability, vomiting, hemorrhage, hepatomegaly, jaundice, elevated levels of serum

transaminases, episodes of acute peripheral neuropathy, Fancon-like syndrome, and

hypoglycemia. Pts may smell like boiled cabbage.

4. Sickle cell anemia can cause childhood atroke.

5. This pt with recurrent bacterial infections and necrotic periodontal infection is most

likely suffering from congenital immunodeficiency cause by a leukocyte adhesion defect.

Delayed separation of the umbilical cord (>3 weeks) is characteristic and presents an

important clue to the correct dx. The leukocytes fail to express some adhesion molecules on

their surface. The leukocyte number is increased, but the interleukocyte communication is

defective; therefore, chemotaxis and cytotoxicity are impaired. Normal lymphocyte count and

gamma globulin concentration help to differentiate this condition from a variety of cell and/or

humoral immune defects.

The classic example of opsonization defect is asplenia. Infections with encapsulated bacteria (like

S. pneumoniae) are typical.

Complement deficiencies manifest as recurrent generalized infections with encapsulated bacteria,

typically Neisseria and autoimmune disorders.

Defective intracellular killing is due to impaired oxidative burst within the phagocytes and a

positive NBT test is characteristic.

6. Positive anti-Smith antibodies and/or anti-double stranded DNA antibodies is specific

and comfirmatory for the diagnosis of systemic lupus erythematosus.

7. The USPSTF recommends screening for atrabismus, amblyopia, and refractive errors in

children aged 0-5 years.

8. Batteries lodged in the esophagus on x-ray should be removed immediately under

endoscopic guidance to prevent mucosal damage and esophageal ulceration. Batteries located

distal to esophagus pass unevenfully in most cases and need only to be observed with stool

examination and/or follow up x-rays to confirm excretion.

9. Scarlet fever is characterized by fever, toxicity, pharyngtis, sandpaper-like rash,

circumoral pallor and strawberry tongue. It is caused by strains of Group A streptococcus that

produce erythrogenic exotoxins. Penicillin V is the drug of choice.

190

10. Vit A has been shown to reduce the morbidity and mortality rates of pts with measles.

11. Thyroid dysgenesis is the MCC of congenital hypothyroidism in US.

12. Infants with congenital hypothyroidism initially appear normal at birth, but gradually

develop apathy, weakness, hypotonia, large tongue, sluggish movement, abdominal bloating,

and an umbilical hernia. For this reason, screening newborns for hypothyroidism, along with

phenylketonuria and galactosemia, is standardly performed in all states.

13. Severe combined immune deficiency is a life-threatening syndrome presenting with

recurrent bacterial, viral and fungal infections. Its diagnostic features are: absent LN and

tonsils, lymphopenia, absent thymic shadow on CXR, and abnormal T, B and natural killer

cell enumeration by flow cytometric analysis.

14. Osteogenesis imperfecta can be associated with blue sclera, hearing loss, joint

hypermobility, and dentinogenesis imperfecta (opalescent teeth).

15. Milk protein intolerance may present with vomiting and bloody diarrhea. Stools may

show RBCs and eosinophils, and there may be a family history of an atopic disorder. It is a

hypersensitivity reaction to cow’s milk proteins, and therefore usually occurs in babies who

are fed with cow’s milk, however, it may occur in breastfed babies, as the mother’s milk may

contain proteins from ingested cow’s milk.

16. Topical mupirocin or oral erythromycin is the tx of choice for local impetigo.

17. Watch out for the infant or newborn with failure to thrive, bilateral cataracts, jaundice

and hypoglycemia. Such an infant most likely has galactosemia, which is a metabolic disorder

caused by galatose-1-phosphate uridyl transferase deficiency, thereby leading to elevated

blood level of galatose. The pt presented in this case has the typical presentation: vomiting,

poor weight gain, jaundice, hepatomegaly, convulsions and cataracts. Other common

manifestations include aminoaciduria, hepatic cirrhosis, hypoglycemia, and mental

retardation. Such pts are at increased risk for E.coli neonatal sepsis. Early dx and tx by

elimination of galactose from the diet are mandatory.

Pts with galactokinase deficiency present with cataracts only, and are otherwise asymptomatic.

Uridyl diphosphate galactose-4-epimerase deficiency is a rare diseses compared to uridyl-

transferase deficiency. In addition to the manifestations of transfrease deficiency, pts with

epimerase deficiency have hypotonia and deafness.

18. Meconium aspiration syndrome occurs most commonly in low birthweight and post-

mature infants. It is characterized by respiratory distress at birth, rales, rhonchi,

hyperinflation, and coarse streaking and patchy opacities on CXR.

19. Septic joint in a child is a true surgical emergency and needs immediate surgical

drainage. A delay of even 4-6 hours can lead to avascular necrosis of the femoral head.

20. Hypocomplementemia (low C3, CH50) in post-streptococcal glomerulonephritis

resolves in 8-12 weeks. Hematuria may persist for up to 6 months. Persistent microscopic

hematuria is suggestive of IgA nephropathy. Proteinuria resolves much slower than hematuria.

Up to 15% of pts may have proteinuria for a period of 3 years. Antibody titers tend to rise

after 7 days, peak after one month, and return to normal in 3-4 months.

21. Respiratory distress syndrome is caused by surfactant deficiency and is more common in

premature infants and infants of diabetic mothers.

22. Parinaud’s syndrome and Collier’s sign (paralysis of vertical gaze that may be

associated with papillary disturbances and eyelid retraction) usually indicate a lesion in the

191

rostral midbrain, most likely pinealoma or germinoma.

23. The 3 most common organisms that cause pneumonia in pts with cystic firbrosis are

Haemophilus, Pseudomonas, and Staphylococcus.

It is essential to memorize:

G+ diplococci – Streptococcus pneumoniae

G+ cocci in clusters – Staphylococcus

G- cocci – Neisseria

G+ rods – Listeria and Bacillus

G- rodes – Pseudomonas, Haemophilus, Klebsiella, Legionella

24. Pts wiht cystic fibrosis may initially present with meconium ileus, which is

characterized by bilious vomiting, failure to pass meconium at birth, and ground glass

appearance on abdominal x-rays.

25. By the first birthday, a child should triple his birth weight and increase his height by

50%. The rate of growth slows towards the end of the first year of life. When compared to

birth weight, a child’s weight should double by 5 months and triple by 1 year. Height

increases by 50% in the first year, bouble by 4 years, and triple by 13 yeas.

26. Painful crises (vasoocclusion in connective tissue and muscle) are the most common

manifestation of sickle cell anemia. Sickle cells and reticulocytes will be seen on the

peripheral smear.

Burr cell are speculated RBC of similar size and with regularly spaced profections. They are seen

most commonly in uremia or as an artifact of preparation.

27. Osteonecrosis is a common complication of sickle cell anemia due to vaso-occlusion of

the bone. It causes significant joint pain and functional limitation. The humerus and femur are

the most frequently affected bones.

28. Physiological jaundice usually presents after 24 hours of birth. Jaundice in the first 24

hours of birth or with direct hyperbilirubinemia requires a detailed evaluation to identify its

cause.

29. Howell-Jolly bodies are nuclear remnants of the RBC which are generally removed by a

functional spleen; therefore, their presence in a peripheral smear suggests functional asplenia

in sickle cell pts.

Heinz body-G6PD

Helmet cells are fragmented RBC—traumatic hemolytic conditions such as DIC, HUS and TTP

Basophilic stippling are ribosomal precipitates whichi appear as blue granules of various sizes

dispersed throughout the cytoplasm of the red cell—thalassemias, hevey metal poisoning

30. Acquired torticollis is a relatively common condition in children. The most common

causes include upper respiratory infections, minor trauma, cervical lymphadenitis, and

retropharyngeal abscess. Cervical spine radiographs should be obtained in children with

acquired torticollis to ensure there is no cervical spine fracture or dislocation.

31. Pts with Turner syndrome have a higher risk of osteoporosis due to lower estrogen

levels and only having one copy of X chromosome genes involves in bone metabolism.

32. A unilateral flank mass in a child >3 years of age is most likely due to Wilm’s tumor,

which arises form the metanephros.

A unilateral flank mass in a child <3 years of age is highly suspicious for neuroblastoma, which is

a malignancy of the neural crest cells. Neuroblastoma displaces the kidneys inferolaterally,

192

thereby making the kidneys non-palpable, whereas in Wilm’s tumor, the kidneys are still palpable.

33. The typical pt with glucose-6-phosphatase deficiency (type I glycogen storage disease,

Von-Gierkes’ disease) is 3-4 months of age with hypoglycemia, lactic acidosis, hyperuricemia,

and hyperlipidemia. The characteristic appearance is a doll-like face (ie. Fat cheeks), thin

extremities, short stature, and a protuberant abdomen (due to enlarged liver and kidneys).

Hypoglycemia seizure may occur.

Acid maltase deficiency is a.k.a. type II glycogen storage disease and Pompes’s disease. Although

pts with this condition also have hepatomegaly, they usually present in the first few weeks of life

as a ‘floppy baby’ with feeding difficulties, macroglossia, and heart failure (due to progressive

hypertrophic cardiomyopathy).

Type III glycogen storage disease is caused by a deficiency of glycogen debranching enzyme

activity. Similar clinical manifestations (eg. hepatomegaly, hepoglycemia, hyperlipidemia, and

growth retardation) may make it initially difficult to distinguish from type I; however, their lab

findings differ. Pts with type III have elevated levels of liver transaminases, fasting ketosis, and

normal blood lactate and uric acid concentrations. Other pertinent findings are splenomegaly and

normal kidneys.

Deficiency of branching enzyme activity results in type IV glycogen storage disease or

amylopectinosis. The typical pt is in his first 18 months of life with hepatoslpenomegaly and

failure to thrive. The most common pathology is progressive cirrhosis of the liver.

Pts with liver phosphorylase deficiency have a benign course, and typically present with

hepatomegaly and growth retardation early in childhood. Mild hypoglycemia, hyperlipidemia, and

hyperketosis may occur. Lactic acid and uric acid levels are normal.

34. Guillai-Barre syndrome involves mainly the peripheral motor nerves, although sensory

and autonomic nerves many also affected.

35. Diamond-Blackfan syndrome is a macrocytic pure red aplasia associated with several

congenital anomalies such as short stature, webbed neck, cleft lip, shielded chest and

triphalangeal thumbs.

Fanconi’s anemia is an autosomal recessive disorder characterized by progressive pancytopenia

and macrocytosis. The average age at dx is 8 years. Associated deformities include café-au-lait

spots, microcephaly, microphthalmia, short statue, horseshoe kidneys and absent thumbs (not

triphalangeal thumbs).

36. Suspect Wilm’s tumor in a young child of 2-5 years age with a localized abdominal

mass and hamaturia. If the child age is less than 1 year, suspect neroblastoma.

37. Posterior urethral valves are the MCC of congenital urethral obstruction. The classic

presentation is a male infant with a distended palpable bladder and abnormally low urine

output.

An omphalomesenteric duct cyst is a focal failure of vitelline duct obliteration. Such a vitelline

cyst may cause a small midline mass deep to the umbilicus. The cyst could be attached to the

umbilicus and the wall of the ileum by vitelline remnants and may cause small intestine volvulus.

38. Atlantoaxial instability should be suspected in any pt with Down syndrome who

presents with upper motor neuron findings. Symptomes include behavior changes, torticollis,

urinary incontinence, and vertebrobaslilar symptoms such as dizziness, vertigo, and diplopia.

On exam, upper motor neurom symptoms such as leg spasticity, hyperreflexia, a positive

Babinski sign, and clonus are often present.

193

39. Small for gestational age infants have a weight under the 10th percentile for gestational

age at birth and may have complications such hypocia, polycythemia (increased

erythropoietin secretion in response to fetal hypoxia), hypoglycemia, hypothermia, and

hypocalcemia.

40. Neonatal necrotizing enterocolitis should be suspected in any preterm or low birth

weight infant with fever, vomiting, abdominal distension, and pneumatosis intestinalis.

41. The best drug available for infantile spasms in the US is intramuscular ACTH, because

it is safer and more effective.

42. Gaucher’s disease is due to the deficient activity of the lysosomal enzyme, acid beta-

glucosidase. The typical pt is an Ashkenazi Jewish adolescent with chronic fatigue, easy

bruisability, bone pain, and pathological fractures. The dx is confirmed with radiologic

(Erlenmeyer flask deformity of the distal femur) and bone marrow studies (Gaucher cells with

wrinkled paper appearance).

Niemann-Pick disease types A and B result from the deficienct activity of sphigomyelinase.This is

a fetal disorder of infancy. Clinical manifestations include failure to thrive, hepatosplenomegaly,

and a rapidly pregrassive neurodegerative course that eventually leads to death by age 2-3 years.

GM1 gangliosidosis results from the deficient activity of beta-galactosidase. It presents at birth or

early in infancy. Clinical features include hepatosplenomegaly and skeletal abnormalities (eg.

anterior beaking of the vertebrae, enlargement of the sella turcica, and thickening of the

calvarium).

The GM2 gangliosidosis include Tay-Sachs’ disease and Sandhoff’s disease. The underlying

pathology involves the deficieny of hexosaminidase activity and the lysosomal accumulation of

GM2 gangliosides, particularly in the central nervous system. Even though hepatospenomegaly is

a common presentation in these disorders, pts usually present at an earlier age with a cherry-red

spot in the retina and neurologic symptoms.

Farber disease results from deficiency of lysosomal enzyme ceramidase, thereby resulting in the

accumulation of ceramide in various tissues, especially the joints. Pts may also present with bone

pain and joint swelling.

43. Fragile X syndrome is characterized by low to normal IQ with learning disabilities,

generalized language disability, short attention span, autism, large head, prominent jaw, large

low set ears and macroorchidism. Increase number of CGG trinucleotide repeats.

44. The initial evaluation of enuresis includes a urinalysis to help rule out treatable causes

such infections, bleeding, or structural defects.

1/23/10

1. Acute UTI in children in most commonly due to vesicoureteral reflux, which is mostly

congenital and is seen in 1% of newborns.

2. Individual with thalassemia trait present with microcytic anemia and a mildly elevated

reticulocyte count. The RDW, MCHC, TIBC (240-450 mcg/dl) and ferritin level (7-140ng/ml)

are typically normal.

3. The clinical findings of hypotonia, hyperactive deep tendon reflexes, learning disabilities,

along with an intrapartum history of prolonged labor and low APGAR scores at birth, point

towards the diagnosis of cerebral palsy. Cerebral palsy is a non-progressive disorder

194

characterized by impaired motor functioning. The MCC is cerebral anoxia.

Friedreich’s ataxia is characterized by gait disturbances, pes cavus, ataxia and absent ankle jerk.

4. Always think of autism first in any pt who is less than 3 years old and presents with

impairment in social interactions and communication, delayed language development, and

stereotypal behaviors. DD Asperger syndrome: although pts may have feature of stereotypical

movements and self-injurious behavior, their language development is normal.

5. Neonatal abstinence syndrome is caused by infant withdrawal to opiates and usually presents

in the first few days of life. It is characterized by irritability, a high-pitched cry, poor sleeping,

tremors, seizures, sweating, sneezing, tachypnea, poor feeding, vomiting, and diarrhea.

Fetal exposure to valporic acid and significantly increases the risk of neural tuble defects, may

result in cardiac anomalies, and can produce dysmorphic facies, including cleft lip, narrow

bifrontal diameter, midface hypoplasia, broad and depressed nasal bridge, and long philtrum.

Prenatal exposure to phenytoin can result in fetal hydantoin syndrome, which is characterized by

nail and digit hypoplasia, dysmorphic facies, and mental retardation. Growth deficiency can also

be seen.

Prenatal exposure to cocaine can result in jitteriness, excessive sucking, and a hyperactive Moro

reflex. Withdrawal syndromes are usually not as severe as with opiates. Long-term effects on

behavior, attention level, and intelligence may be seen.

6. Acute lymphoblastic leukemia is the MCC in children. Diagnosis is mainly based on more

than 25% lymphoblasts in the bone marrow.

7. Most cardiac murmurs in childhood are benign and require only observation. Know the

following features of a benign murmur: 1) asymptomatic pt. 2) murmur intensity grade 2 or

less. 3) Normal S2. 4) No audible clicks. 5) Normal pulses. 6) No other abnormalities.

Pathological murmurs: 1) symptoms such shortness of breath, chest pain, syncope, and dizziness,

etc. 2) Murmur intensity grade 3 or higher. 3) Abnormal S2. 4) pansystolic murmur. 5) Murmur

loudest at upper left sternal border. 6) Absent or diminished femoral pulses. 7)Murmur’s quality is

unchanged with position.

8. The recommended first line tx for pertussis is a macrolide antibiotic (erythromycin,

azithromycin, or clarithromycin), regardless of the age of the immunization status. Tx is given

whenever pertussis is suspected or confirmed, and regardless of the stage of the disease.

9. Myotonic muscular dystrophy (MMD) is an autosomal dominant disease which is a.k.a.

Steinert disease. It is the second most common muscular dystrophy in the US. The pathology

is distinct in that all types of muscles (ie. Smooth, striated, cardiac) are involved. The above

case presentation is typical: the pt initially appears normal at birth, then slowly develops

muscle weakness and progressive muscle wasting, especially in the distal muscles of the

hands, posterior forearm muscles, and anterior compartment of the lower legs. The

characteristic facial appearance if illustrated above temporal wasting, thin cheeks, and an

upper lip in the shape of an inverted V. Pertinent physical findings include emaciated

extremities, atrophy of the thenar and hypothenar eminences, proximal muscle weakness,

positive Gowers sign, winged scapula and myotonia. Myotonia s defined as delayed muscle

relaxation, and the classic example is the inablility to release the hand after a handshake. In

addition, anormalities of the endocrine, immunologic and nervous systems occur. Endocrine

manifestations include DM, testicular atrophy, frontal baldness and hypothyroidism.

10. The MCC of anemia in sickle cell pts is chronic hemolysis. Iron deficiency, folate deficiency,

195

and anemia of chronic disease occur less frequently in such pts.

11. Hydroxyurea benefit pts with sickle cell disease by increasing fetal hemoglobin, which dilutes

the number of sickled cells in the circulation and reduces veso-occlusive episodes. Hydoxyura

has been shown to decrease pain crises, the need for transfusions, and episodes of acute chest

syndrome. Side effects occur because hydroxyurea suppresses the bone marrow. Leukopeniam

anemia and thrombobytopenia may occur. These effects are generally temporary and

reversible but may predispose the pt to infection.

12. Infants with DiGeorge syndrome may have cyanotic heart disease, craniofacial anomalies,

thymic hypoplasia, cognitive impairment and hypoparathyroidism. It is essential to monitor

the calcium levels of these infants. Prolonged QT intervals may be caused by hypocalcemia.

13. The tx for trycyclic antidepressant intoxication includes sodium bicarbonate. This drug not

only helps to correct the acidosis, but also helps to narrow the QRS complex prolongation.

Benzodiazepine (eg. diazepam) is given when the pt presents with seizures that require tx.

Extremely high yield!!

14. Whenever intraveoux access cannot be obtained in emergent pediatric cases, intraosseous

access should be attempt next.

15. Upper GI endoscopy is the diagnostic study of choice when a pt presents with acute alkali

ingestion. Attempting to neutralize the alkali with vinegar or lavage is dangerous since these

may increase the extent of the injury.

16. Congenital abdominal wall defectsGastroschisis OmphaloceleBowel protrudes through a defect on the right side of the umbilical cord

Intra-abdominal contents protrude through the umbilical ring. The herniation usually includes small bowel and may include large bowel and liver

Bowel is not covered by a protective membrane Covered by an amnioperitoneal membraneBowel looks “angry” and mattedNot typically associated with other abnormalities outside the GI tract

Can be associated with other congenital abnormatilies (eg. heart, kidney)

The first step in the mx of gastroschisis is to immediately wrap the exposed bowel in sterile saline

dressing and plastic wrap in order to minimize heat and fluid losses. Caesarean section is

performed only in the presenct of the usual obstetric indications. The mx of omphalocele is similar

to that of gastroschisis. Immediate care of the newborn with omphalocele also involves sterile

wrapping of the bowel, insertion of an orogastric tube (to decompress the stomach). Small defects

(<2cm) can be repaired with primary closure, but most defects will require a staged procedure

with Silastic silo.

17. Pure riboflavin deficiency is unusual in industrialized nations, but has been documented in

regions of the world with severe food shortages. The condition is typically mild and

nonspecific in presentation, but symptoms may include sore throat, heperemic and edematous

oropharyngeal mucous membranes, cheilitis, stomatitis, glossitits, normocytic-normochromic

anemia, seorrheic dermatitis, and photophobia.

18. Cephalohematoma is a subperiosteal hemorrhage, and presents a few hours after birth as scalp

swelling limited to one cranial bone. It is a subperiosteal hemorrhage; hence, it is always

limited to the surface of one cranial bone. Most cases do not require any tx and resorb

spontaneously within 2 weeks to 3 months, depending on the size. Rarely, phototherapy may

be necessary to improve the hyperbilirubinemia.

196

Caput succedaneum is a diffuse, sometimes ecchymotic, swelling of the scalp. It usually involves

the portion of the head presenting during vertex delivery. It may extend across the midline and

across suture lines.

19. The presence of dermatitis herpetiformis and chronic non-bloody diarrhea in a child of 12-15

months it suggestive of celiac disease. It is due to abnormal hypersensitivity to gluten and

presents characteristically at 12-15 months of age, when gluten-containing foods such as

wheat, rye, or barley are introduced to the child’s diet.

20. Klumpke’s paralysis is a brachial palsy that occurs in newborns following excessive traction

on the arm during delivery. It consists of hand paralysis and ipslateral Horner’s syndrome

(ptosis and miosis), and is secondary to injury to 7 th and 8 th cervical nerves and 1 st thoracic

nerve. Tx depends on the severity of the injury and, generally, it consists of partial

immobilization and appropriate positioning to prevent contractures. Gentle massages and

range-of-motion exercises can be started by 7-10 days of age. If by 3-6 months there is no

improvement, neuroplasty, neurolysis, end-to-end anastomosis, and never grafting may be

attempt.

Erb-Duchenne palsy is another type of brachial palsy in which the injury involves the 5 th and 6 th

cervical nerves. Clinically, it presents with absent Moro reflex and intact grasp reflex of the

affected arm. Pts present with a characteristic position, which consist of adduction and internal

rotation of the arm with pronation of the forearm.

Cervical fracture presents with irregularity, crepitus and fullness over the fracture site and

decreased movements of the arm.

In neonates, facial nerve palsy results from pressure over the facial nerve in utero, from efforts

during labor, or from forceps delivery. Classic features of facial paralysis are present, most

apparent when the infant cries.

Phrenic never injury involves the 3rd, 4th, and 5th cervical nerve. It results in diaphragmatic

paralysis and upper brachial palsy. It presents with irregular and difficult respiration, and an

abdomen that does not bulge with breathing.

21. Pylotic stenosis is treated surgically, however, surgery should be undertaken only after the

infant’s hydration status and electrolytes levels have been stabilized.

22. The first step in the mx of an infant with suspected congenital diaphragmatic herniation is the

placement of an orogastric tube.

23. Absence seizure is a generalized seizure of childhood that is characterized by sudden and brief

lapses of consciousness without loss of postural control. The typical presentation includes

multiple ‘daydreaming’ episodes (eye blinking asdn gabbling) and a decline in school

performance.

24. Adolescent onset of hirsutism and virilism, with normal menstruations and eleveated 17-

hyoxyprogesterone, are diagnostic of congenital adrenal hyperplasia.

25. Constitutional growth delay is characterized by a delayed growth spurt, delayed puberty, and

delayed bone age. Testosterone supplementation is not necessary with constitutional growth

delay. Hormone supplementation may speed up puberty, but this will shorten the growth spurt

and may result in a shorter final adult height. Hypothyroidism can cause growth delay and

should be considered in the differential dx. However, the expected pattern would be normal

childhood growth followed by an abrupt “falling of the growth curve” when the

hypothyroidism develops. An affected pt would also expected to have other symptoms like

197

hypertension, fatigue, constipation, cold intolerance, and dry hair and skin.

26. Reflux is a risk factor for UTI. Repeated attacks can lead to progressive renal scarring, which

is the major cause of end stage renal disease and hypertension in children. Dx of VUR is best

made with a voiding cystourethrogram or a radionuclide cytogram (RNC). Renal

ultrasonography is less sensitive in detecting reflux nephropathy. The American Academy of

Pediatrics recomm the ends that all children aged 2-24 months with a first UTI should

undergo a VCUG or RNC to detect the presence VUR.

27. Infant botulism is transmitted by the ingestion of food contaminated with C.Botulinum. The

germ grows in the gut and produces a toxin, which is responsible for the clinical signs.

The infant form of botulism is different from the adult form. In the latter, the toxin infested per se

produces the symptoms.

28. The organ system with the highest morbidity and mortality after HUS is the kidneys.

29. Epilgottitis presents with abrupt onset of fever, sort throat, dysphagia, and drooling. Airway

obstruction is the most concerning potential complication of epiglottitis, and often requires

nasotracheal intubation. In unimmunized children, H influenza typr b is a likely cause.

30. If a child shows reluctance to potty training, the best step is to stop training attempts for

several months and allow the child to become interested on his own.

31. Immune thrombocytopenia purpura typically presents in early childhood after a viral infection

with easy bruisability and petechiae. Labs show thrombocytopenia and normal Hb, WBC, PT

and PTT. Peripheral smear shows normal morphology of all cell lines except the platelets,

which are reduced in number and large in size.

32. Todd’s paralysis represents a postictal condition that usually rapidly improves with restoration

of motor function within 24 hours.

33. The pt has mild metatarsus adductus, which is a congenital food deformity. Interestingly, the

deformity is most frequently in first-born infants, this is attributed to the molding effect os the

primigravid uterus. There is no sexual predominance. Approximately 10% of pts also have an

associated acetabular dysplasia; hence, careful hip exam is required. Matatarsus adductus is

subdivided into 3 types. Type I metatarsus adductus is characterized by feet that overcorrected

both passively and actively into abduction. These cases tend to spontaneously correct by

themselves; therefore, tx is not necessary. Type II metatarsus adductus is characterized by feet

that correct to the neutral position with passive and active movements; this is managed with

orthosis or corrective shoes; and sometimes plaster casts if initial tx gives no results. Type III

metatarsus adductus is characterized by rigid feet and do not correct, these are managed with

serial casts.

34. Croup is caused by Parainfluenza virus and commonly presents in children younger than 3

years with hoarseness; a barking coughm and varying degress of respiratory distress.

35. Iron supplementation in the form of oral ferrous salts or iron-fortified milk formula is

warranted in premature infants to prevent anemia of prematurity. Solid food-4 months. Fruit

juices—5-7 months.

36. Preseptal cellulites is an infection of the soft tissue anterior to the orbital septum that is most

commonly caused by contiguous spread of infection from local facial or eyelid trauma.

Preseptal cellulites often presents with eyelid swelling, tenderness, erythema, or discoloration.

The presence of decreased eye movements, decreased visual acuity, proptosis, or double

vision is more suggestive of orbital cellulites.

198

37. The most common mutation that leads to cystic fibrosis is a deletion of a three pair encoding

for phenylalanine (DF508) in the CFTR gene located on chromosome 7. It is believed that this

mutation prevents CFTR protein from trafficking to the corret cellular location.

38. The most effective and potentially life-saving intervention in a pt with known anaphylaxis is

prescription of self-administrable epinephrine for use upon reexposure to the inciting allergen.

39. Mechanical ventilation is indicated in a pt with severe asthma exacerbation that is

unresponsive to medications (silent chest or absent air entry, and condinues to desaturate

despite prednisone tx).

40. Cri-du-chat syndrome is due to 5p deletion and presents as microcephaly, hypotonia, short

stature, and a cat-like cry.

41. In a young child with recurrent upper respiratory tract infections and bilateral polyse, cystic

fibrosis must be ruled out.

42. Turner syndrome is associated with a higher incidence of coarctation and bicuspid aortic

valve.

43. The typical pt with Bruton’s aggamaglobulinemia is a male infant with recurrent pneumonia

and otitis media after 6-9 months of age. The levels of all the serum immunoglobulins and

circulating B-cells are decreased.

44. A pt with an acute onset of headaches and focal neurologic symptoms after an episode of

acute otitis media or sinusitis most likely has a brain abscess. Ring-enhancing lesions are

usually seen on CT or MRI scan.

1/23/10

1. A young boy with symptoms of increased intracranial pressure (eg. headaches, vomiting),

bitemporal hemianopsia, and a calcificated lesion above the sella has a craniopharyngioma

until proven otherwise. Presence of a cystic calcified parasellar lesion on MRI is almost

diagnostic of craniopharyngioma.

2. In a pt with a chronic hemolytic anemia and compensatory reticulocytosis, increased demand

for folic acid can lead to folic acid deficiency. For this reason, pts with sickle cell anemia

should be on folic acid supplementation.

3. Niacin (B3) deficiency causes pellagra, which is characterized by diarrhea, dermatitis,

dementia, and possibly death.

4. Increased gastric residues in a preterm neonate is highly suspicious for necrotizing

enterocolitis.

5. Separation anxiety is a normal behavior in young children. The initial step in the mx of

children with speech delay is an audiology evaluation (esp with a history of otitis media).

6. Osgood-schlatter disease is a traction apophysitis骨突炎 of the tibial tubercle. Radiographic

findings include anterior soft tissue swelling, lifting of tubercle from the shaft, and irregularity

or fragmentation of the tubercle. On physical exam, there is edema and tenderness over the

tibial tubercle. A firm mass can sometimes be felt due to heterotopic bone formation. Pain can

be reproduced by extending the knee against resistance.

Prepatellar bursitis occurs with chronic irritiation of the anterior knee. Symptoms include pain

with direct pressure and superficial swelling over the patella.

Patellar tendonitis is an overuse syndrome resulting from repetitive jumping or kicking. Pts

present with anterior knee pain after exercise. Unlike Osgood-Schlatter disease, pts have point

199

tenderness at the inferior pole of the patella.

Tibial osteomyelitis is a bone infection, usually bacterial in origin. Symptoms include pain,

swelling, tenderness, and erythema. Pts classically present with refusal to bear weight on the

affected extremity. Systemic symptoms may also be present. The pain from ostermyelitis does not

remit with rest.

Patellofemoral stress syndrome is an overuse injury commonly seen in runners. Pts present with

anterior knee pain that worsen upon descending steps or hills. Pain is localized to the patella and

radiographs do not demonstrate separation at the tibial tubercle.

7. Midgut volvulus usually presents in a child less than one month of age as bilious vomiting,

abdominal distension and passage of bloodstained stools.

8. Suspect clubfoot (talipes equinovarus) in a pt who presents with equines and varus of the

calcaneum and talus, varus of the midfoot, and adduction of the forefoot. Clubfoot is initially

managed with stretching and manipulation of the foot, followed by serial plaster casts,

malleable splints or taping. Surgical treatment is indicated if conservative mx gives

unsatisfactory results, and is preferably performed between 3-6 months of age (should be

started immediately!).

9. By the time a childe starts school, he should have recieced

5 doses of DTaP

4 doses of Heamophilus influenza type b (Hib, complete by 15 months), Pneumococcal conjugate

vaccine (PCV complete by 15 months), inactivated polio vaccine (IPV)

3 doses of HBV, rotavirus

2 doses of MMR, varicella, HAV

At the 4-5 years old visit, children sould receive their DTaP #5, IPV#4, MMR#2 and varicella #2.

Children should receive 3 doses of IPV prior to 18 months and then a 4th booster dose at 4-5 years.

10. Birth injuries such as calvicular fracture are common complications in mothers with GDM

and large babies. Most clavicular fractures are green stick and heal rapidly without

complications.

11. A sudden onset of respiratory distress may be caused by foreign body aspiration, and rigid

bronchoscopy is the procedure of choice for both diagnostic and therapeutic purposes.

12. Although, it is typically a self-limited condition, all pts with suspected Kawasaki disease

should be hospitalized and treated with IVIG and high-does aspirin. If untreated, up to 25% of

these pts may develop coronary artery aneurysms.

13. Suspect rheumatic fever in a child with sore throat, fever, pericarditis, erythema marginatum,

arthritis, chorea, and subcutaneous nodules. This condition is caused by group A streptoccus.

14. A PDA is associated with a continuous flow murmur due to constant movement of blood from

the high-pressure aorta to the low-pressure pulmonary artery. Small PDAs are often

asymptomatic and detect incidentally on routine cardiac auscultation.

15. The tx for neonatal chlamydial conjunctivitis is erythromycin, which has to be administrated

systemically (orally) to decrease the risk of chlamydial pneumonia (not topical).

16. Pyloric stenosis presents with non-bilious vomiting in a 4-8 week old infant, and an

abdominal ultrasound confirms the dx.

17. Premature adrenarche and thelarche (isolated appearance of axillary hair before the age of 6

years) often have no clinical significance. On the other hand, pubarche requires a thorough

evaluation, as it may be due to a CNS disorder in 50% of cases.

200

18. The tx for an acute severe exacerbation of lung disease in a cystic firbrosis pt is iv antibiotics

therapy with coverage against Pseudomonas aeruginosa (usually a combination of two drugs,

such as penicillin/cephalosporin+ aminoglycoside). The addition of an anti-staphylococcal

antibiotic (eg.naficillin or vancomycin) depends on the results of the culture.

19. Guthrie test is a qualitative (coloration) test which can detect the presence of metabolic

products of phenylalanine in the urine.

The screening of amonolevulinic acid and porphobilinogen in the blood is used to diagnose

porphyrias, which is a goup of inherited disorders associated with neurological sings,

photosensitivity and hyperpigmentation.

20. Suspect hemolytic uremic syndrome in a child with sudden-onset abdominal pain, bloody

diarrhea, and triad of anemia, thrombocytopenia, and renal failure.

21. The prodrome of measles (Rubeola) may include Koplik spots, a pathognomonic finding

which appears as bluish white lesions on the erythematous buccal mucosa opposite to the first

and second upper molars, and sometimes on the inner conjunctivae and vaginal mucosa. In the

US, even a single case of suspected measles should be reported to the local health authorities.

Aptical measles occurs in persons who have previously received inactivated measles vaccine (the

measles vaccine available now is a live one). This form is potentially life-threatening, and is

characterized by atypical rash (not maculopapular), the absence of Koplik spots, arthritis, hepatitis

and lung involvement. Edema of the hands and feet may occur.

22. Amoxicillin for 21 days is the recommended tx for erythema migrans or stage 1 Lyme disease

in children <9 years old. Doxycycline (100mg po bid for 21 days) is the recommended tx I

children>9 years old. Alternatives for those who cannot take amoxicillin or doxycycline

include cefuroxime axetil or erythromycin for 21 days. Ceftriaxone or penicillin G for 14-21

days is recommended for Stage 3 Lyme disease, which is a disseminated disease with

neurological and/or cardiac manifestations.

23. The most common cause of amblyopia弱视 is strabismus. Other causes include (1): errors of

refraction and (2) opacity of media along the visual axis. The standard of tx is occlusion of the

normal eye. * High yield!!

24. Infants who should not be breastfed include those with certain inborn errors of metabolism

such as galactosemia, PKU, and urea cycle defects. Other C/I to breastfeeding include

maternal use of certain medications (radioactive isotopes, antimetabolites, chemotherapeutic

agents), active drug abuse, TB, HIV.

25. ECG in pts with Wolff-Parkinson-White syndrome is characterized by a shortened PR

interval, delta waves, and widening of the QRS complex. The underlying cause is an

accessory pathway conducts antegrade chest from the atria to the ventricles faster than the

conduction through the AV node, which allows part of the ventricle to depolarize early.

26. Chronic granulomatous disease should be considered in any pt with recurrent or unusual

lymphadenitis, hepatic abscesses, osteomyelitis at multiple sites, or unusual infections with

catalase-positive organism (eg. S aureus, Aspergillus, Serratia marcescens, and Burkholderia

cepacia).

27. Transmission of HIV by breastfeeding is well documented; therefore, the present of maternal

HIV infection is an absolute C/I to breastfeeding.

28. Myocarditis in children is usually due to viral infection and presents with fever, lethargy, and

signs of myocardial dysfunction after a viral prodrome. Affecte children should be mornitored

201

in the ICU because of the risk of acute decompensation and fetal arrhythmias.

29. Presence of inflamed tympanic membrane and diminished movement of the membrane with

insufflation are the hallmarks of otitis media. Fever is not required to make the dx of acute

otitis media.

30. For pertussus prevention, all close contacts should be given erythromycin for 14 days,

regardless of age immunization, or symptoms.

31. Pts with Down’s syndrome or trisomy 21 present with flat face, upward and slanted palpebral

fissures, epicanthal folds, simian crease, short and broad hands, hypoplasia of the middle

phalanx of the 5th fingers, and high arched palate.

32. Consider IgA deficiency in pts with mild immunodeficiency consisting of recurrent

sinopulmonary and GI infections, and anaphylactic transfusion reaction. Dx may be

confirmed by measurement of a low serum IgA with normal serum IgM and IgG levels.

33. Clavicle fracture presents with irregularity, crepitus, and fullness over the fracture site, and

decreased movements of the arm. Predisposing factors are shoulder dystonia, traumatic

delivery, and large size of the infant. Generally, no specific tx is required.

34. Group B streptococcus is the MCC of neonatal sepsis. Early sepsis presents in the first week

of life and usually involves several organs. Late-onset GBS infection is usually a focal

infection, and is most frequently meningitis (75%). E.Coli meningitis is the second MCC.

Listeria is the 3rd MCC. It also tends to result in multiple abscesses and pneumonia, in

addition to meningitis.

35. 3 P’s of McCune-Albright syndrome: precocious puberty, pigmentation (café au lait spots)

and polycostotic fibrous dysplasia. This pt’s moonlight facise and bruises indicate Cushing’s

syndrome, which occurs in association with McCune-Albright syndrome. Cushing disease

consists of hypercortisolism secondary to an ACTH-secreting pituitary tumor.

36. Osteogenesis imperfecta is caused by mutations in type I collagen. Its typical features are blue

sclera and recurrent fractures. * Extremely high yield!!

37. Cystic fibrosis can present as failure to thrive due to fat and protein malabsorption form a

pancreatic insufficiency.

38. Minimal change disease is the MCC of nephritic syndrome in pts younger tha 16 years old.

Light microscopy and immunofluorescence does not usually reveal any change in kidney

architecture. Electron microscopy show diffuse effacement of foot processes of podocytes.

Steroids are the tx of choice.

39. Childhood obesity is a risk factor fo the development of type 2 diabetes later in life.

1/24/10

OB & GYN

1. In the presence of decreased fetal movement, fetal compromise should be suspected and

the best next step is mx is a nonstress test (NST). NST is usually performed in high risk

pregnancies starting at 32-34 weeks gestation or when there is a loss of perception of fetal

movements in any pregnancy. NST is carried out by recording the FHR while monitoring for

spontaneous perceived fetal movements. A test is considered reactive (normal) if in 20min 2

accelerations of the FHR of at least 15 beats per minutes above the baseline lasting at least 15

seconds each are noted. If less than 2 accelerations are noted in 20min, the test is considered

nonreactive (abnormal) and further assessment is required. The most common cause of a

202

nonreactive NST is a sleeping baby , not a diseased baby, so vibroacoustic stimulation is used

to wake the baby up and allow a timely test.

A biophysical profile (BPP) is a scoring system designed to evaluate fetal well-being. It is

indicated in high risk pregnancies and in cases of maternal or physical concern, decreased fetal

movements or a non-reactive NST.

In a contraction stress test (oxytocin challenge test), the mother is given an infusion of oxytocin

sufficient to result in 3 contractions per 10 minutes, and the effect these contractions have on fetal

heart activity is recorded. If a late deceleration is noted at each contraction, the test is positive and

delivery is usually recommended. Because this is a more invasive test, it is not uses as an initial

examination.

Ultrosonography is not as sensitive as sensitive as NST or BPP for evaluation of decreased fetal

movements and fetal well-being. It is, however, the first step if fetal demise is suspected as it can

document the presence or absence of fetal heart movement.

2. The MCC of an abnormal maternal serum alpha-fetoprotein ( MSAFP ) level is

gestational age error. After detecting increased MSAFP levels, the physician should first

perform an ultrasound to detect the presence of any anomaly that may be seen by ultrasound,

assess fetal size to determine if it agrees with dates and to identify possible multiple gestation.

Amniocentesis may also be performed to identify the presence of neutral tube defects. In the

presence of a neural tube defect, amniotic fluid AFP and acetylcholinesterase will both be

elevated.

MSAFP is decreased in Edward syndrome and in Down syndrome. These conditions are screened

for using the quadruple screen. Down syndrome will cause low MSAFP, low estriol, eleveated

bata-HCG and elevated inhibin A level. A typical profile for Edward syndrome shows low

MSAFP, low estriol, vey low beta-HCG and normal inhibin A level.

3. Risk factors for osteoporosis include advanced age, thin body habitus, cigarette

smoking, alcohol consumption, corticosteroid ues, menopause, malnutrition, family history of

osteoporosis, and Asian or Caucasian ethnicity.

4. Screening for syphilis is strongly recommended in all pregnant women, regardless of

their risk factors for sexually transmitted infections. The USPSTF recommends screening for

Chlamydia infection in all women age 24 and younger and those at increased risk (history of

sexual transmitted infections, new or multiple partners).

5. Acquired hypogonadotropic hypogonadism is a cause of amenorrhea seen most

commonly in association with significant stressors, eating disorders and excessive exercise.

Pulsatile GnRH therapy can induce ovulation in these pts.

6. The genital ulcers seen in chancroid and hepes genitalis differ from the ulcer of primary

syphilis in that both are painful. Chancroid is also characterized by ulcers with a deep,

purulent base and painful lymphadenopathy. Genital herpes presents with multiple vesicles

following a prodrome of burning and pruritis. Within days, these vesicles become painful

ulcers. Like syphilis, granuloma inguinale (Donovanosis) presents with painless genital

ulcers. These ulcers have a red, beefy base and there is no associated adenopathy. Unlike

primary syphilis, the ulcer of granuloma inguinale does not resolve without antibiotic tx.

7. Nontreponemal serologic tests (VDRL, RPR) are used as a screening test for syphilis,

and treponemal serologic tests (FTA-ABS) are used for confirmation. Darkfield microscopy is

also an effective method of diagnosing syphilis, but requires proper equipment and clinical

203

expertise. In primary syphilic, there is a high rate of false-negative results to serologic testing,

and therefore darkfield microscopy is necessary.

8. Most menstrual cycles in the first one to two years following menarche are anovulatory.

These cycles are typically irregular and may be complicated by menorrhagia.

9. Endometriosis is a cause of subfertility and infertility. Women may be asymptomatic,

but typically experience pelvic pain, dyspareunia and pain with defecation.

10. In a pt with primary amenorrhea, 1) FSH measurement should be ordered if there is no

breast development. Increased FSH indicates a peripheral cause and decreased FSH indicates

a central cause. 2) GnRH stimulation test is the next step if FSH is decreaed. 3) Karyotyping

is the next step if FSH is increased.

11. Labor should be induced immediately in pts with intrauterine fetal demise who develop

coagulation abnormalities. A fibrinogen level in the low-normal range can indicate developing

DIC. When fibrinogen levels are normal the mx decision will depend on the pt’s preference:

the options are either watchful expectancy or induction of labor. The logic behind watchful

expectancy is that labor occurs spontaneously in 80% of cases within 2-3 weeks of IUFD.

12. Casarean section is used in the mx of placental abruption when there are obstetrical

indications, or when there is a rapid deterioration of the state of either the mother or the fetus

and labor is in an early stage such that vaginal delivery is not emergently possible.

13. Asymptomatic bacteriuria occurs when the urine culture grows >100,000 CFU per ml

of a single organism in an asymptomatic pt. It is important to promptly treat the infection to

prevent progression to pyelonephritis in the pregnant pt, because it may cause septicemia,

preterm labor or low birth weight babies. Amoxicillin, ampicillin, nitrogurantoin and

cephalexin are commonly used to treat the pts.

14. Fetal growth restriction (FGR) can be symmetric and asymmetric . In symmetric growth

restricition, the insult to the fetus begins before 28-weeks gestation and growth of both the

head and the body is deficient. It is usually caused by fetal factors such as chromosomal

abnormalities, congenital infections and congenital anomalies. Asymmetric FGR is a result of

fetal adaptation to non-ideal maternal factors. Asymmetric FGR results from fetal

redistribution of blood flow to vital organs, such as the brain, heart and placenta, as the

expense of less vital organs, such as the abdominal viscera. Maternal factors such

hypertension, hypoxemia, cigarette smoking, vascular disease and toxic exposures can lead to

asymmetric FGR. Asymmetric FGR has a better prognosis than symmetric FGR.

15. In pregnant pts, asymptomatic bacteriuria increases the risk of cystitis, pyelonephritis,

preterm birth, and perinatal mortality. E.coli accounts for more than 70% of cases. Tx in

pregnancy consists of a 7-day course of nitrofurantoin, amoxicillin or a first-generation

cephalosporin.

16. Failure to lactate is the classical initial presentation of Sheehan’s syndrome, or

postpartum ischemic necrosis of the anterior pituitary due to hemorrhagic shock. Failure to

produce milk in this condition results from prolactin deficiency.

17. Pelvic muscle exercises (Kegel exercises) and urethropexy are the recommended tx

options for stress incontinence. Kegel exercises should be attempted before pursuing surgery,

because urethropexy is invasive and carries risk.

18. Gestational trophoblastic disease should be ruled out in pregnant pts with severe

vomiting. Measurement of quantitative beta HCG should be checked first. If the beta HCG

204

levels is markedly elevated an ultrasonogram is then performed.

19. Hyperemesis gravidarum is a severe form of vomiting during pregnancy. It presents

with severe and persistent vomiting, weight loss, and ketonuria. The DD includes

pyelonephritis, gestational trophoblastic disease, gastroenteritis, and hepatobiliary disease.

Mild increase in ALT, AST, bilirubin, amylase, and lipase are seen in up to 50% of

hospitalized pts. Elevated amylase and lipase are from salivary gland due to vomiting.

20. A missed abortion is a form of spontaneous abortion that is characterized by

intrauterine fetal death before 20 weeks gestational age with complete retained products of

conception and a closed cervix. Pts typically present with loss of pregnancy symptoms and

some brown vaginal discharge, and a transvaginal ultrasound is necessary to confirm the dx.

21. The most appropriate tx for a missed abortions is removal of the POC (product of

conception) from the uterus. This can be accomplished surgically with dilation and curettage,

medically with vaginal misoprostol or expectantly with serial imaging to ensure complete

natural expulsion of the POC.

22. In a normal pt, the tone is typically louder when placed next to the ear as opposed to on

the mastoid process, indicating increased air conduction compaired to bone conduction. The

normal finding of air greater than bone conduction can also be found in sensorineural hearing

loss. Bone conduction that is greater than air conduction on the Rinne test is suggestive of

conductive hearing loss. Otoslcerosis is the MCC of conductive hearing loss in adults, and is

most common in pts in their 20’s and 30’s. There is a slight female predominance. In

otosclerosis, the stapes footplate becomes fixed to the oval window, resulting in loss of its

piston action. This disorder is sometimes also referred to as otospongiosis b/c CT may show a

lucent (as opposed to sclerotic) focus in the temporal bone near the oval window. Therefore,

CT could be used to confirm the dx in this case.

23. Radiation therapy is highly effective for squamous cell carcinoma of vagina. It is an

excellent alternative of pts who are poor surgical candidates. Tx of vaginal cancer depends on

staging. Stage I and II tumors (no extension to the pelvic wall and no metastases) which are

less than 2cm in size may be removed surgically, while stage I and II which greater than 2cm

in size are treated with radiation therapy.

24. Non-adolescent women with ASC-US (atypical squamous cells of undetermined

significance) on Pap smear should be followed up with reflex HPV testing to exclude the

presence of HPV types that are considered high-risk for the development of cervical cancer

(types 16, 18, 31,33,35,45 and 58). If low-risk HPV stains are found, then the pt can be

followed with an annual Pap smear.

ASC-US

-------------------------------------------------------------

↓ ↓ Adolescents Non-adolescent woman

↓ ↓Repeat Pap in 12 months Reflex HPV resting

↓ If ‘positive’ → Colposcopy

25. Pseudohyphae are characteristically seen on wet mount preparation of vaginal discharge

from pts with Candida vulvovaginitis. Symptomatic pts can be treated with an azole

205

antifungal, such as fluconazole. It is not a sexually transmitted infection, so sexual partners do

not require tx.

26. Thichomonas infection causes inflammation, pruritus, and a thin malodorous vaginal

discharge (PH 5.0-6.0). In contrast, bacterial vaginosis does not cause inflammation

(abnormal elevation of the vaginal PH), and candida viginitis causes a discharge that is thick

and white (PH 4.0-4.5).

27. In cases of mild preeclampsia, if the pregnancy is remote from term of the fetal lungs

are not yet mature, the pt is best managed with bed rest and close observation.

28. Genital warts are caused by the HPV and present as clusters of pink lesions on the

genitalia. Small lesions can be treated in the office with trichloroacetic acid or podophyllin.

29. The MCC for decreased fertility in women in their fourth decade who are still

experiencing menstrual cycles is age-related decreased ovarian reserve.

30. Pts with PCOS are that increased risk of developing type II diabetes. Standard 2-hour

oral glucose tolerance test (OGTT) identifies most pts with impaired glucose tolerance and

early type 2 diabetes better than fasting glucose alone.

31. DUB is the MCC of abnormal uterine bleeding. It is a diagnosis of exclusion and is

most often due to anovulation. High dose estrogen is the mx of choice for uncontrolled

bleeding. In women >35 with DUB, endometrial biopsy is indicated.

32. In a complete abortion, the whole conceptus passes through the cervix. The cervix then

closes, an the associated pain and uterine contractions subside.

33. Symptoms of menopause include irregular or absent menses, heat intolerance, flushing,

insomnia, and night sweats. Hyperthyroidism and menopause can have similar presentations,

and serum TSH and FSH levels should be checked in pts with these symptoms.

34. In the immediate postpartum period, a low-grade fever, leukocytosis and vaginal

discharge are normal findings. The vaginal discharge (lochia) is initially bloody, then serous

(3-4 days) and finally white to yellow in color days following delivery. Culture of the vaginal

discharge would be required if there is foul smelling discharge or if the pt’s fever and

leukocytosis persists or worsens.

35. Levonorgestrel (“Plan B”) is the recommended method of emergency contraception.

This progestin-only method is considered effective up to 120 hours after intercourse, although

effectiveness is greater the earlier the medication is administered. There are no

contraindications to the use of levonorgestrel, and no physical exam or lab testing is required.

It has the lowest incidence of side effects amongst emergency contraceptives, although nausea

(20%) and vomiting (5%) may occur. Individuals 18 years of age or older can obtain

levonorgestrel over-the-counter, whereas individuals under 18 must obtain a prescription in

most states. Levonorgestrel prevents pregnancy in approximately 7 out of 8 women who

would have otherwise become pregnant form intercourse.

Intramuscular medroxyprogesterone (Depo-provera) is a primary method of birth control, with

shots administered every 3 months. Prosraglandin E2 suppositories may be used as a second

trimester abortifacient. Neither is recommended as a form of emergency contraception.

36. Endometrial biopsy is indicated in cases of DUB affecting women>35 years old. It is

also indicated in cases of DUB if hypertension, diabetes, or obesity are present.

37. One of the effects of preeclampsia is damage to the liver that can cause right upper

quadrant pain due to stretching of the hepatic (Glisson’s) capsule.

206

38. Magnesium sulfate is given to women with eclampsia and severe preeclampsia in order

to prevent the further development of seizures while delivery of the fetus is accomplished.

Delivery is the most important overall tx for eclampsia.

39. Metformin is indicated in polycystic ovarian syndrome pts with impaired glucose

tolerance. It helps in preventing type 2 diabetes mellitus as well as improving obesity,

hirsutism, menstrual irregularity, and infertility.

40. Pseudocyesis is a rare psychiatric condition in which a woman presents with nearly all

signs and symptoms of pregnancy, however, ultrasound reveals a normal endometrial stripe

and negative pregnancy test. Pseudocyesisi is usually seen in women who have a strong desire

to become pregnant. It has been suggested that the depression caused by this need is behind

the occurrence of some hormonal changes mimicking those of pregnancy. This is a form os

conversion disorder, and management requires psychiatric evaluation and tx.

41. In false labor, progressive cervical changes are absent, contractions are irregular and

discomfort is readily relieved by sedation. All such pts need reassurance.

42. Systemic and topical corticosteroids can induce an acneiform eruption characterized by

monomorphous, erythematous follicular papules distributed on the face, trunk and extremities.

Comedones are characteristically absent.

43. Precocious puberty has both central and peripheral causes. Central precocious puberty

presents with increased FSH and LH, while peripheral causes present with low FSH and LH.

All pts with central precocious puberty should receive brain imaging with CT and MRI. Tx is

with GnRH analog therapy.

44. Hormone replacement therapy is now only recommended for the short-term use of

controlling menopausal symptoms. Large studies, such as the WHI, have shown that the long-

term use of combined HRT can slightly increase the risk of coronary heart disease and stroke.

Conclusions from the WHI (Women’s health initiative) trial: 1) there is no cardiovascular

benefit with either unopposed estrogen or combined estrogen-progestin therapy. 2) there is an

increased risk of stroke with long-term use of both unopposed estrogen or combined estrogen-

progestin therapy. 3) There is an increased risk of breast cancer with combined estrogen-

progestin therapy, but not with unopposed estrogen. The benefits of HRT are that it helps

relieve the symptoms of vaginal dryness, hot flashes and some of the mental changes seen in

menopause. It also protects against osteoporosis, and to a minor degree against colon cancer.

The risks of HRT include an associated increased incidence of venous thromboembolism and

endometrial hyperplasia/cancer (only with unopposed estrogen). This risk can be largely

reduced by the addition of progestin therapy.

1/24/10

1. A reactive NST is reassuring and it should be repeated weekly.

2. This pt has a female phenotype but lacks a normal vaginal and uterus, which narrows the

etiology of her primary amenorrhea to mullerian agenesis, androgen insensitivity, or 5-

reductase deficiency. The karyotype is the determinig test, with both androgen insensitivity

and 5-alpha-reductase deficiency being seen in pts with a XY genotype. This pt’s genotype is

XX, which leaves mullerian agenesis as the best explanation for her condition. The mullerian

duct normally leads to the development of the proximal vagina and the uterus; therefore pts

with mullerian agenesis normally have a blind ended vagina with little to no uterine tissue.

207

Androgen insensitivity pts have a male XY genotype but there is an abnormality in the androgen

receptor. The external genitalia develop as female, but mullerian inhibiting factor is still secreted

by the testes which prevents the development of internal female organs.

5-alpha-reductase deficiency cannot convert testosterone to the more potent DHT. They hava a

male XY genotype and female external genitalia, but typically show virilization at puberty.

3. Eclampsia is diagnosed when unexplained convulsion occur in the setting of preeclampsia.

4. The most effective agent used for tx and prevention of seizure in eclampsia is magnesium

sulfate.

5. Abdominal circumference is the most effective parameter for estimation of fetal weight in

cases of suspected FGR, b/c it is affected in both symmetric and asymmetric fetal growth

restriction.

6. Prolactin production is stimulated by serotonin and TRH and inhibited by dopamine.

Hypothyroidism may result in amenorrhea and galactorrhea.

7. Serum BUN and creatinine are usually decreased in pregnant pts due to an increase in renal

plasma flow and glomerular filtration rate.

8. Elevated prolactin levels suppress GnRH release thereby suppressing LH and FSH production

and ovulation. This is the reason for anovulation and amenorrhea in lactating mothers.

9. The superficial femoral vein is a deep vein of the thigh and a thrombus of this vein requires

anticoagulation. Pregnancy is a major risk factor for deep venous vein thrombosis, especially

during the peripartum period.

10. There is no role for medications in the suppression of breast milk production. Pts are advised

to wear a tigh-fitting bra, avoid nipple manipulation and use ice packs and analgesics to

relieve associated pain.

11. Weight gain has not been associated with the use of combination oral contraceptive pills in

controlled studies. Pap smear screening should be initiated in all women 3 years after

initiation of sexual intercourse but no later than 21 years of age.

12. Radiation levels used for most diagnostic exams are not associated with teratogenicity.

Exposure to ionizing radiation doses below 5 rad (5cGy) is not associated with known fetal

abnormalities. Therapeutic abortion is advised when the radiation level is between 5-10 cGy.

13. Pts with androgen resistance present with amenorrhea, normally developed breasts, absent

pubic and axillary hair, absent reproductive organs, and a 46 XY karyotype. Serum

testosterone levels are in a range typical for males. The internal reproductive organs do not

develop because the testes are still present and secret mullerian inhibiting factor.

14. Early decelerations are depressions in fetal heart rate that resolve by the end of the uterine

contraction. Early decelerations are not considered a nonreassuring heart rate pattern, and are

the result of fetal head compression.

Fetal sleep presents with decreased long-term variability.

Fetal cord compression presents with variable decelerations.

Uteroplacental insufficiency presents wht late decelerations.

Intrauterine infections may present with fetal tachycardia (HR>160).

15. In adolescent females, ASCUS or LSIS (low grade squamous intraepithelial lesions) are most

likely due to HPV infection. Because frank cervical cancer in this population is uncommon,

the best mx strategy is a follow-up Pap smear in 1 year.

LSIL (mild dysplasia or CIN 1)

208

-------------------------------------------------------------------------------------------------------

↓ ↓ ↓ Adolescents Premenopausal woman Postmenopausal woman

↓ ↓ ↓-------------------------↓Repeat Pap in 12 months Colposcopy Reflex HPV resting Colposcopy

↓ ↑ If ‘positive’-------------------↑

16. Mx of placenta previa depends on the severity of bleeding and the age of the pregnancy.

Complete placenta previa requires delivery by cesarean section as the placenta completely

covers the os cervix and the fetus cannot be delivered vaginally. At 36 weeks gestation,

amniocentesis should be done in order to assess lung maturity. If the fetal lungs are mature,

elective cesarean section can be performed.

17. Congenital aromatase deficiency is a rare genetic disorder marked by either total absence or

poor functioning of the enzyme that converts androgens into estrogens. Its consequences are

numerous. In utero the placenta will not be able to make estrogens, leading to masculinization

of the mother that resolves after delivery. The high levels of gestational androgens result in

virilized XX child with normal internal genitalia but ambiguous external genitalia.

Clitoromegaly is often seen when excessive androgens are present in utero. Later in life pts

will have delayed puberty, osteoporosis, undetectable circulating estrogens, high

concentrations of gonadotropins and polycystic ovaries.

McCune-Albright syndrome (aromatase)↑ is marked by the triad of café-au-lait spots, polyostotic

fibrous dysplasia, and autonomous endocrine hyperfunction. The most common endocrine feature

is gonadotropin independent precocious puberty. Thus, pts have early puberty.

18. Both general and spinal anesthesia as well as sedation may reduce uterine activity if

administered in the latent phase (<2cm) thus prolonging this stage of labor.

19. All healthy pregnant women with uncomplicated pregnancies are encouraged to exercise for

30 minutes daily at a moderate intensity that allows the mother to carry on conversation while

exercising.

20. Renal calculi in pregnancy require special consideration because most of the standard

investigatory modalities will expose the fetus to radiation. The modality of choice in such pts

is abdominal or pelvic ultrasonography. If non-informative, transvaginal ultrasonography

should be performed next to help detect distal ureteral stones. If this examination is also

negative, but the pt continues to have discomfort or signs suggestive of nephrolithiasis, two

options are available for dx: limited intravenous pyelogram (IVP) and ureteroscopy. Limited

or single-short IVP delivers the maximum recommended monthly radiation exposure, and is

positive in most pregnant women with a symptomatic stone.

21. The most appropriate test to confirm the diagnosis of intrauterine fetal demise (IUFD) is real

time ultrasonography to demonstrate an absence of fetal movement and cardiac activity.

22. A search should be undertaken to determine the cause after the first episode of intrauterine

fetal demise. Autopsy of the fetus and placenta should be performed in all cases of stillbirth

with the permission of the parents.

23. Depression of the deep tendon reflexes is the earliest sign of magnesium sulfate toxicity. Tx

requires stopping the magnesium sulfate infusion and administration of calcium gluconate.

Oxytocin is analogous to ADH and may be responsible for hyponatremia and water

209

intoxication.

24. The combination of thrombocytopenia, microangiopathic hemolytic anemia and increased

liver enzymes in pt wit preeclampsia is defined as HELLP syndrome. Right upper quadrant

pain is typical of this condition sue to distention of the hepatic (Glisson’s capsule). Hemolytic

uremic syndrome (HUS) also causes thrombocytopenia and hemolytic anemia. As in HELLP

syndrome, both PT and PTT are normal; however, HUS also usually results in renal failure

and not necessary associated with preeclampsia.

25. Delivery is the definitive treatment of HELLP syndrome in women beyond 34-weeks

gestation.

26. Endometritis is characterized clinically be fever and uterine tenderness in the postpartum

period and is often associated with foul-smelling lochia. Risk factor include prolonged ROM,

prolonged labor, operative vaginal delivery and caesarian section among others.

27. Psotpartum endometritis is the MCC of puerperal fever on the 2nd and 3rd day postpartum.

Endometritis is a polymicrobial infection caused by a combination of gram positive and gram

negative organisms, aerobic and anaerobic organisms and occasionally other organisms such

as Mycoplasma and Chlamydia.

28. The tx of choice for postpartum endometritis, which is a polymicrobial infection, is

intravenous clindamycin and gentamicin.

29. A young woman who presents with a breast lump can be asked to return after her menstrual

period for reexamination (which may reveal regression of the mass) if no obvious signs of

malignancy are present.

30. The major source of estrogen in menopausal women is from the peripheral conversion of

adrenal androgens by the aromatase enzyme present in adipose tissue. This process is increase

is increased in obese women who may have decreaed menopausal symptoms.

31. A child’s blood group frequently differs from the mother’s blood group, but this fact does not

create a potential for alloimmunization-related fetal damage. Anbodies to ABO antigens to the

IgM class and do not cross the placenta. Anti-D antibodies that are responsible for Rh

alloimmunization belong to the IgG class and cross the placenta easily.

32. An intrauterine pregnancy should be seen with transvaginal ultrasonography at beta-HCG

levels of 1,500-2,000 mIU/mL. If the level is <1000 mIU/mL, both beta-HCG and

transvaginal ultrasonography should be repeated in 2-3 days. Beta-HCG should generally

double about every 48 hours in viable pregnancies, but increases at a slower rate in ectopic

and noninviable intrauterine pregnancies.

33. Dibetes screening is performed between 24 and 28-weeks of gestation. The proper initial

screening test is the one hour 50 gram oral glucose tolerance test. After one hour, if the blood

glucose value is less than 140 mg/dl, then gestational diabetes is ruled out. If >140mg/dl, then

a 3 hour OGTT is performed. Gestational diabetes is diagnosed if two or more of the serum

glucose values obtained during the 3 hour test are elevated above the values listed below:

Fasting serum glucose concentration >95 mg/dl, 1-hour >180, 2-hour > 155, 3 hour > 140.

34. Arrest disorder of dilation has not progressed for more than 2 hours. It can also be of descent

when the descent has not progressed for more than 1 hour. It can be caused by hypotonic

contractions, conduction anesthesia, excessive sedation, cephlopelvic disproportion or

malpresentation. In the present case, the arrest is resulting from a midpelvic contraction

indicated by the prominence of the ischial spines.

210

35. In the case of amenorrhea-hypoprolactinemia, first rule out hypothyroidism by measuring

serum TSH.

36. Mild preeclampsia is defined clinically by hypertension greater than 140/90mmHg and

proteinuria greater than 0.3/24h (300mg/24h) after the 20th week of gestation.

37. Methyldopa is a pregnancy category B agent and is the first-line medication for tx for

hypertension in pregnancy.

38. Lichen sclerosus most commonly affects postmenopausal women and manifests with vulvar

prutitus and discomfort. Exam shows porcelain-white atrophy. Biopsy should be considered to

rule out valvar SCC. Lichen sclerosus is considered a premalignant lesion of the vulva as

vulvar SCC occurs with greater frequency in these pts as compared to the general population.

39. High-potency topical steroids are considered first-line therapy for lichen sclerosus.

40. Premature ovarian failure is characterize by amenorrhea, hypoestrogenism, and elevated

serum gonadotropin levels in women younger than 40 years of age. It may be secondary to

accelerated follicle atresia or a low initial number of primodial follicles. It is most commonly

idiopathic but may also be due to mumps, oophoritis, irradiation or chemotherapy. It can be

associated with autoimmune disorders such as Hashimoto thyroiditis, Addison disease, type I

DM, and pernicious anemia, which supports wht hypothesis that at least some cases of

idiopathic premature ovarian failure are of autoimmune origin. Women present with signs and

symptoms that are similar to those seen in menopause. The dx is confirmed by demonstrating

increased serum FSH and LH levels and secreased estrogen levels. Pts with premature ovarian

failure lack viable oocytes, so the only option available ot allow pregnancy in such pts is in-

vitro fertilization.

41. Infertility in women with irregular menstrual cycles is often due to anovulation. Ovulation can

be tested for by measuring a midluteal phase progestone level.

42. Hypothalamic-pituitary-gonadal axis immaturity is a cause of irregular menstrual cycles in

women shortly following menarche.

43. In any woman of childbearing age with secondary amenorrhea, first rule out pregnancy.

44. Physician can refuse to perform elective abortions for personal or professional reasons. If you

refuse to perform any procedure then you should attempt to refer the pt to another physician

who can and will.

1/25/10

1. Granulosa cell tumors produce excessive amounts of estrogen, and can present with

precocious puberty in younger children and postmenopausal bleeding in elderly pts. This has

to be differentiated from heterosexual precocious puberty or virilizing symptoms which are

usully produced by excessive androgens.

2. Epidural anesthesia may cause overflow incontinence as a transient side effect. It is best

treated with intermittent catheterization. Oxybutynin has anticholinergic properties and

inhibits smooth muscle contraction. This drug is used for urge incontinence.

3. This is a case of hypergonadotropic hypogonadism, b/c the pt has a primary amenorrhea with

absent secondary sexual characteristics and increased FSH level. A differential dx of

hypergonadotropic hypogonadism includes conditions that present with primary amenorrhea

and either a male or female karyotype, making karyotype determination the most appropriated

next step in the work up. These conditions include Turner syndrome (45, XO), Savage

211

syndrome, or ovarian resistance syndrome (46, XX), male gonadal agenesis (46, XY), and

defects in testosterone production, such as in deficiency of 17-alpha-hydroxylase and 17, 20-

desmolase (46, XY).

Prolactin and TSH levels are indicated in secondary amenorrhea after ruling out pregnancy. They

usually do not have problems with secondary sexual characteristics.

4. An increase in blood pressure that appears before 20 weeks gestations is due to either chronic

hypertension or a hydatiform mole.

5. Risk factors for placental abruption include prior placental abruption, chronic hypertension,

pregnancy-induced hypertension, cocaine, alcohol or tobacco use and vascular diseases such

as diabetes and SLE. The most significant risk factor is hypertension of any kind. Sustained

maternal blood pressure in excess of 160/110 should be treated pharmacologically.

6. Idiopathic central precocious puberty, which is the most common type in females, results from

the premature activation of the hypothalamic-pituitary-gonadal axis. Pts with central

precocious puberty have pubertal levels of basal LH that increase with GnRH stimulation,

whereas pts with a peripheral source of precocious puberty, such as in certain ovarian

pathologies, have low LH levels with no response to GnRH. All pts with central precocious

puberty should have brain imaging to rule out an underlying CNS lesion. This pt has had a

fairly rapid progression of pubertal maturation, and should be treated with GnRH agonist

therapy in order to prevent premature epiphyseal plate fusion.

7. Kallmann’s syndrome consists of a congenital absence of GnRH secretion (ie.

Hypogonadotropic hypogonadism) associated with anosmia. Pts have a normal XX genotype

and normal female internal reproductive organs. They present with amenorrhea and absent

secondary sexual characteristics such as breast development and pubic hair; the addition of

anosmia to the presentation may suggest the dx. Abnormal development of the olfactory bulbs

and tracts result in hyposmia or anosmia (decreased sense of smell). The FSH and LH levels

are low, in contrast to the levels in primary ovarian failure which are usually elevated.

8. Galactorrhea presents as guaiac negative bilateral nipple discharge. Prolactonoma,

hypothyroidism, overstimulation of the nipple, oral contraceptive pills, and medications which

lower dopamine levels are common cause. Workup includes ruling out pregnancy, measuring

serum prolactin and TSH levels, and possible MRI of the brain to rule out prolactinoma.

9. Penicillin is considered to be the tx of choice for pregnant pts with syphilis. If a pt is allergic

to penicillin, the allergic shoud be confirmed with skin testing and the pt should then undergo

desensitization so that she can safely take the medication. It is typically accomplished using

incremental doses of oral penicillin V.

10. Postterm pregnancies are at an increased risk for oligohydramnios, which itself is associated

with increased fetal morbidity. Postterm pregnancies should be monitored for

oligohydramnios twice weekly. Oligohydramnios in these cases is defined as no vertical

pocket of amniotic fluid greater than 2cm or an amniotic fluid index of 5cm or less.

11. Hypertension in a pregnant female in the setting of massive proteinuria, a malar rash, and a

strongly positive ANA titer is most likely due to SLE. Glomerulonephritis in general will

cause proteinuria, hematuria and RBC casts.

12. The CDC recommends that all pregnant women without contraindications be vaccinated

against influenza.

13. It is strongly recommended that asymptomatic bacteriuria in pregnant women be treated,

212

especially women at 12-16 weeks’ gestation. Recommended antibiotics include amoxicillin,

nitrofurantoin, and oral cephalosporins. Trimethoprim-sulfamethoxazole is a class C

mediation in pregnancy. It may be used during the second trimester but is not recommended

for use during the first trimester because it interferes with folic acid metabolism or during the

3rd trimester b/c it increases the risk of kernicterus in the newborn.

14. The combination of primary amenorrhea, bilateral inguinal masses, and breast development

without pubic or axillary hair is strongly suggestive of androgen insensitivity syndrome(a

mutation of the androgen receptor gene). A blind vaginal pouch and karyocype of 46XY are

other clues. A gonadectomy should be performed to avoid the risk of testicular carcinoma.

15. Sudden onset abdominal pain, fetal heart rate abnormalities, and recession of the presenting

part during active labor indicated uterine rupture. Rish factors include a preexisting uterine

scar or abdominal trauma.

16. The requirement for L-thyroxine in pts receiving estrogen replacement therapy increases. The

potential causes may include induction of liver enzyme, increased level of TBG, and an

increased volume of the distribution of thyroid hormones. In pregnancy, also, thyroid

hormone requirements will be increased, and the pt should be monitored every 4-6 weeks for

dose adjustment.

17. Secondary amenorrhea is relatively common in elite female athelets and results from estrogen

deficiency (due to decrased LH and GnRH). These amenorrheic women are therefore at

increased risk for all conditions associated with estrogen deficiency, including infertility,

vaginal atrophy, vaginal atrophy, breast atrophy, and osteopenia.

18. Lithium is associated with congenital heart disease, classically Ebstein’s anomaly, and should

be weaned in pregnant women with stable bipolar disorder. When a woman is treated with

isotretinoin, she should receive strict contraception. Inhaled steroids can be used during

pregnancy.

19. Laparoscopy is the gold standard for the dx of endometriosis.

20. Pts with endometriosis are at an increased risk of decreased fertility or infertility. Up to 30%

of females being evaluated for infertility are found to have endometriosis.

21. Ovulation can be induced in pts with PCOS with clomiphene citrate tx as well as with

metformin tx.

22. Anabolic steroid use by a male can produce infertility by suppressing the productin of GnRH,

LH and FSH. Klinefelter syndrome (XXY seminoiferous tubule dysgenesis) in an inherited

disorder characterized by resticular fibrosis (primary hypogonadism), azoospermia,

gynecomastia, decreased intelligence and decreased axial skeletal growth. FSH and LH will

be high.

23. If maternal serum AFP levels are abnormal in a pregnant pt, the next step is ultrasonography

to confirm gestational age, detect fetal structural anomalies, detect multiple gestation and

confirm a viable prennancy.

Amniocentesis is indicated in cases where the MSAFP pr triple/quadruple screen is abnormal but

only after ultrasonography has ascertained gestational age accuracy and ruled out nonviable

pregnancy or multiple gestation. It is best performed between 16-20 weeks gestation.

Chorionic villus sampling is indicated for early screening in women with known genetic diseases,

or who have had children affected by a genetic condition. It is performed between 10-12 weeks

gestation, and therefore offers the advantage of a first trimester termination of pregnancy if the

213

fetus is affected.

Cordocentesis is used for rapid karyotype analysis, or when fetal blood dyscrasias恶液质, such as

fetal anemia and Rhesus isoimmunization, are suspected. In the present case, ultrasound should be

performed first to rule out causes of MSAFP elevation other than genetic anomalies.

24. Abdominal pain in a young female in the middle of her cycle with a benign history and

clinical examination is most likely mittleschmerz (midcycle pain). Ovarian torsion is a

medical emergency. Pts present with sudden-onset lower quardrant abdominal pain that

radiates to the groin or back and it accompanied by nausea and vomiting. An adnexal mass is

usually present.

25. Female offsping of women who ingested diethylstilbestrol during their pregnancy are at

increased risk of developing clear cell adenocarcinoma of the vagina and cervix as well as

cervical anomalies and uterine malformations.

26. Lupus anticoagulant can sometimes be seen in pts with SLE and is a potential cause of

recurrent pregnancy loss due to thrombus development within the placenta.

27. Painful third trimester vaginal bleeding with normal ultrasound is most likely due to placental

abruption.

28. Women shoudn be screened for group B strep colonization with vaginal and rectal cultures

obtained between 35-37 weeks gestation. Women colonized with GBS receive prophylactic

antibiotics (penicillin or ampicillin) at the time of delivery. Women who have had GBS

bacteriuria during pregnancy or who have previously delivered a child that developed an

early-onset GBS infection are automatically given prophylactic antibiotics regardless of the

results of a rectovaginal culture.

29. A biophysical profile score of 2 or less is consistent with severe fetal asphyxia. In this setting,

the baby should be delivered immediately.

30. Primary ovarian failure results in decreased estrogen levels and increased FSH and LH levels

(the elevation of FSH is generally greater than that of LH due to slower clearance of FSH

from the circulation). FSH elevation in the setting of >=3 months of amenorrhea in a woman

inder age 40 confirms the diagnosis of premature ovarian failure.

31. OCPs offer both risks and benefits as outlined below. The risks and benefits should be

weighed carefully for each individual pts.Serious side effect Protective againstVenous throboembolismCardiovascular events/stroke

Elevation of triglyceride levels

Cholestasis or cholecystitis

DM

Hypertension

Ovarian cysts and cancerEndometrial cancer

Benign breast disease

Dysmenorrhea (anemia)

32. Fetal distress (repetitive late decelerations) is an indication for emergent cesarean section.

33. Gestational diabetes is most commonly first treated with a low suger diet, but if diet fails to

produce euglycemia, insulin is the tx of choice.

34. Gestational diabetes carries numerous risks for the fetus including macrosomia, hypocalcemia

(due to parathyroid hormone suppression), hypoglycemia (due to high baseline insulin

production), hyperviscosity due to polycythemia (increased erythropoietin), respiratory

214

difficulties, cardiomyopathy and congestive heart failure.

35. The most important complication of PPROM is pulmonary hypoplasia (immaturity). Steroids

are used to enhance fetal lung maturity when premature rupture of membranes occurs at less

than 34-weeks of gestation.

36. Hypotension is a common side effect of epidural anesthesia. The cause of hypotension is

blood redistribution to the lower extremities and venous pooling.

37. Septic abortion is managed with cervical and blood cultures follows by IV antibiotics and

gentle suction curettage. Vigorous suction curettage may cause perforation of the uterus and

should be avoid.

38. Intersitial cystitis (painful bladder syndrome) is a chronic condition associated with pelvic

pain worsened by bladder filling or intercourse accompanied by urinary frequency, urgency,

and nocturia. The negative urinalysis excludes a urinary tract infection. Stress incontinence

refers to involuntary leakage of urine with effort, exertion, sneezing or coughing. Pain is

typically not a symptom of stress incontinence.

39. Transvaginal ultrasonogram is more accurate than transabdominal ultrasonogram in

diagnosing ectopic pregnancy, and should be performed when beta-hCG levels are between

1500-6500 IU/L.

40. Pts with PPROM whose GBS status is unknown should receive antibiotic prophylaxis.

Corticosteroids are indicated if the gestational age is <32 weeks. If a woman is admitted to the

hospital after 34 weeks gestation with PPROM, delivery is recommended.

41. In the primigravida, cervical dilatation in the active phase progresses at a speed of 1.2cm/hr.

Cesarean section is the appropriate tx for active phase arrest.

42. The first step in a pt with secondary amenorrhea is to rule out common situations; that is

pregnancy, then hyperprolactinemia, and hypothyroidism. The subsequent step should be the

determination of the pt’s estrogen status with progestin challenge test.

43. Intraamniotic infection should be suspected in the setting of prolonged or premature rupture

of the membranes when maternal fever, leukocytosis, uterine tenderness, or tachycardia is

detected. Fetal tachycardia is another feature of chorioamnionitis.

44. The most appropriate tx of chorioamnionitis associated with PPROM is systemic, broad-

spectrum antibiotic therapy and expedited delivery of the fetus. Caesarean section should be

reserved for cases where fetal distress is evident. Fetal tachycardia is a sign of maternal

infection.

1/26/10

1. HCG is a hormone is a hormone secreted by the syncytiotrophblast and is mainly responsible

for the preservation of the corpus leteum in early preanancy. The hCG is composed of 2

subunits, alpha and beta. The alpha unit is common to hCG, TSH, LH and FSH. The beta

subunit is specific to hCG, and is used as the basis of virturally all pregnancy tests. Other

biological function of hCG include the promotion of male sexual differentiation and

stimulation of the maternal thyroid gland.

Progesterones is produced in large amounts during pregnancy, and helps to inhibit uterine

contractions; preparing the endometrium for implantation of a fertilized oveum.

Estrogen is responsible for induction of prolactin production during pregnancy.

2. By treating HIV-infected women zidovudine throughout pregnancy and labor, and treating the

215

newborn for the first 6 weeks of life, the risk of HIV transminssion is reduced by 70%.

3. At the first prenatal visit, blood typing and Rh antibody testing (in Rh-negative women)

should be performed. Unsensitized Rh-negative women potentially carrying an Rh-positive

fetus should have their Rh-antibody status retested between 24-28 weeks’ gestation. Pts at risk

of alloimmunization should be given anti-Rh immune globin at 28 weeks and again at the

time of delivery.

4. CIN in a low risk pt with a low-grade lesion on Pap smear can be observed with annual Pap

smear screening.

CIN I

↓----------------------------------------------------------------------↓CIN I preceded by low grade abnormalities CIN I preceded by high grade abnormalities

(ASC-US, ASC-H, LSIL) (HSIL; AGC-NOS)

↓ ↓ Repeat cytology in 12 months Immediate diagnostic excisional procedure

↓ (post child bearing age)

Still abnormal after 24 months (Or)

↓ Close clinical follow-up

Diagnostic excisional procedure (Women planning future pregnancy)

5. The presence of dysmenorrheal, heavy menses, and an enlarged uterus is classic for uterine

fibroids.

6. The classic manifestations of acute abruption placentae include vaginal bleeding, abdominal

pain, uterine contractions, and uterine tenderness. The absence of blood on pelvic exam does

not rule out this condition.

7. Suction curettage is the tx of choice for inevitable abortion. Anti-D immunoglobulin should be

administered to all Rh-negative women following any form of abortion in order to prevent the

formation of Rh foctor antibodies that may complicate subsequent pregnancies.

8. Infant born to pts with Graves disease treated with surgery are at risk for thyrotoxicosis

because of the passage of thyroid stimulating Ig across the placenta.

9. Missed abortion involves a dead fetus that is still retained in the uterine. The dx is suspected

when there is disappearance of the nausea and vomiting of early pregnancy, and an arrest of

uterine growth.

Inevitable abortion manisfests with vaginal bleeding, lower abdominal cramps that may radiated

to the back and perineum and a dilated cervix. Ultrasonography demonstrates a ruptured or

collapsed gestational sac with absence of fetal cardiac motion. Incomplete abortion has the same

presentation except that there is an incomplete evacuation of the conceptus. Ultrasonography

reveals endometrial debris.

10. Progestin-only oral contraceptives are the preferred hormonal contraceptives in lactating

women as they do not affect the volume or composition of milk produced by the mother, they

have no known effects on the infant and they do not carry the risk of venous thrombosis

associated with combination pills.

11. Ectopic pregnancy should be suspected in any pt of childbearing age, presenting with a triad

of amenorrhea, abdominal pain and vaginal bleeding. In all cases of secondary amenorrhea,

administering a pregnancy test should be the first diagnostic step.

12. Pregnancy is associated with an increase in TBG, resulting in increased total T4 and T3, a

216

normal free T4 and T3, and a normal TSH.

13. Breast engorgement is a common problem associated with breast feeding. Charaterized by

bilateral breast tenderness and swelling, it typically presents 24-72 hours post partum, peaks

3-5 days after delivery, and resolves spontaneously. Plugged dusts present similarly to mastitis

but lack fever or systemic symptoms. They are treated by improving the quality of

breastfeeding. Persistently plugged ducts resulting in galactocele may be treated with

aspiration.

14. During pregnancy, the primary goal of colposcopy is the exclusion of invasive cancer. Any

woman with a cytologic specimen suggesting HSIL should undergo colposcopy and directed

biopsy. If the biopsy is negative, a second biopsy is recommended 6-8 weeks after delivery.

LEEP procedure is an excisional therapy and is recommended for all pts except adolescents

and pregnant woman with HSIL on Pap smear but without CINII or greater on biopsy since

the abnormal area may have been missed by the biopsy. Excisions in pregnanct women should

be considered only if a lesion suggestive of invasive cancer is detected at colposcopy.

15. Copious vaginal discharge that is white or yellow in appearance, nonmalodorous, and occurs

in the absence of other symptoms or findings on vaginal exam is referred to as physiologic

leucorrhea.

16. The dx of bacterial vaginosis (BV) is made when 3 of 4 Amsel criteria are met: 1) thin, gray-

white vaginal discharge. 2) Vaginal pH >4.5. 3) A positive “whiff” test upon addition of KOH

to the vaginal discharge. 4) “clue cells” (vaginal epithelial cells with adherent coccobacilli) on

wet mount.

17. External cephalic version (ECV) can be attempt in women with breech pregnancies at greater

than 37 weeks gestational age if there are no contraindications to vaginal delivery and fetal

well-being has been established. EVC has been shown to reduce the rate of cesarean sections,

but the maneuver has the potential to result in fetal distress, so it should only be performed

when arrangements have been made to allow for an emergent caesarian delivery.

18. The most appropriate first steps in the mx of variable decelerations (due to umbilical cord

compression) are oxygen administration and change in maternal position. If the variable

decerlerations persist, the pt should be placed in the Trendelenburg position and the presenting

fetal part should be elevated. Persist variable decelerations may require amnioinfusion, which

consists of the infusion of fluid into the amniotic cavity. Fetal scalp pH testing should be

performed to assess for fetal hypoxia if the abnormal FHR pattern persists after the initial

measures of position change, oxygen administration, and discontinuation of oxytocin have

been tried. Cesarean section is indicated when fetal distress is confirmed.

19. For acute preterm labor bed rest and tocolysis are continued as long as possible with a long-

term goal of reaching 34-36 weeks gestational age. In otherwise normal pregnancies, systemic

corticosteroids are administered when gestational age is between 24-34 weeks. Tocolysis

should then be attempt with the goal being to maintain the pregnancy for at least 48 hours in

order to realize maximum benefit from the steroids.

20. In the ovulatory phase of the menstrual cycle, cervical mucus is profuse, clear and thin (pH

6.5 or greater, more basic than at other pahses). It will stretch to approximately 6 cm and

exhibit ferning on a microscope slide smear preparation. In early follicular phase is thick,

scant and acidic. In the mid- and late-luteal phase, the cervical mucus becomes progressively

thicker and exhibits less stretching ability.

217

21. Suspect atrophic vaginitis in post-menopausal females with symptoms of vaginal dryness and

dysuria, and physical exam findings of pale, dry vaginal mucosa, diminished labial fat pad,

and scarce pubic hair. Moderate and severe cases require local estrogen therapy.

22. Bilateral edema of the lower extremities in pregnancy is most commonly a benign problem.

Preeclampsia and DVT should also be considered, but are unlikely in the absence of other

classic symptoms of these conditions.

23. Raloxifene is a mixed agonist/antagonist of estrogen receptors. In breast and vaginal tissue, it

is an antagonist, whereas in bone tissue, it is an agonist. It is a first-line agent for the

prevention of osteoporosis, and it decreses breast cancer risk. It increases the risk of

thromboembolism.

24. BPP includes the NST in addition to four parameters assessed by ultrasonography: 1/fetal

tone; 2/fetal movements (3/10min); 3/fetal breathing (30/10min); 4/amniotic fluid index, (5-

20). A total score of 8-10 is considered normal, and should only be repeated once or twice

weekly until term for high risk pregnancies.

1) If the presence of oligohydramnios (AFI<5) delivery is to be considered since it can

be result in umbilical cord compression and therefore fetal compromise.

2) If the score is 6 without oligohydramnios, contraction stress test should be ordered. If

this latter gives non-reassuring results, delivery is usually indicated; if it gives

suspicious results, repeat the next day.

3) If the score is 4 without oligohydramnios and fetal lungs are mature, delivery should

be considered. If fetal lungs have not yet reached their maturity, steroid injection

should be administered and BPP assessed within 24 hours.

4) If the score is <4, the fetus should be delivered.

25. An antepartum hemorrhage with fetal heart changes progressing from tachycardia to

bradycardia and finally to a sinusoidal pattern occurring suddenly after rupture of membranes

suggests the dx of vasa previa. The bleeding in this setting is fetal in origin, so maternal vital

signs will remain stable while the fetus exsanguiantes放血.

26. Low doses of anti-D immune globulin postpartum is the most likely cause of anti-D

immunization in this pt. After the events that are associated with excessive feto-maternal

hemorrhage (eg. placental abruption), the failure to correct the dose of anti-D immune

globulin may result in maternal alloimmunization.

27. Primary dysmenorrhea is caused by increased levels of prostagladins and presents with lower

abdominal pain that occurs with menstruation. NSAIDs and oral contraceptive pills can be

used to improve symptoms.

28. PCOS is characterized by an unbalanced estrogen secretion that may result in endometrial

hyperplasia and carcinoma.

29. Premenopausal women with simple or complex hyperplasia without atypia respond to therapy

with cyclic progestins. All pts should undergo repeat biopsy after 3-6 months of tx. The risk of

pregression to endometrial cancer in pts with complex hyperplasia without atypia is low (1-

2%), and therefore even if this pt does not want more children, hysterectomy is not warranted.

The risk of pregression to endometrial cancer in pts with complex hyperplasia with atypia is

high (30%). Therefore, in premenopausal women diagnosed with complex hyperplasia with

atypia who have completed child bearing, total hysterectomy is the tx of choice. In women

who would like to preserve fertility and in those who are considered poor surgical candidates,

218

cyclic progestins with repeat biopsy in 3-6 months is the appropriate initial mx.

30. An arrest disorder of dilation and descent are defined by an arrest in dilation superior to 2

hours in the former, and an arrest in descent superior to 1 hour in the latter. Arrest disorder

resulting form midpelvic contraction (prominence of the ischial spines) is treated with

C.section.

31. Pts with placental abruption in labor must be managed aggressively to insure a rapid vaginal

delivery. Cesarean section is used only when there are obstetric indictions, or when there is a

rapid deterioration of the state of either the mother or the fetus and labor is in an early stage.

32. Aortic coarctaion presents with elevated blood pressure in the upper extremities but low BP in

the lower extremities. It is a common finding in Turner syndrome. Pts with Turner syndrome

have ovarian dysgenesis and poor ovarian function; FSH should be high due to negative

feedback.

33. Labor should be allowed to proceed in pts where the fetus has been diagnosed with a severe

congenital anomaly incompatible with life.

34. Threatened abortion is characterized by any hemorrhage occurring before the 20th week of

gestation with a live fetus and a closed cervix.

35. Reassurance and outpatient follow up is the standard of care for threatened abortion.

36. Tx for vaginismus is effective with success rates of 80% or better. Typically, tx includes

relaxation, Kegel exercises (to relax the vaginal muscles), and insertion of objects of

gradually increasing size to encourage desensitization. Self-stimulation techniques are useful

for primary anorgasmia. Referral ot a sex therapist is usually indicated for pts with hypoactive

sexual desire.

37. Paresthesias or pain in the distribution of the median nerve during pregnancy is likely to be

due to carpal tunnel syndrome (CTS). Initial txs for CTS involve neutral position wrist

splinting and NSAIDs. Local corticosteroid injection is indicated in cases where wrist

splinting is insufficient to relieve symptoms. Surgical decompression of the carpal tunnel is

reserved for cases when conservative mx fails.

38. In the presence antepartum hemorrhage, pelvic examination must not be done before ruling

out placenta previa. Placenta previa presents with painless third trimester vaginal bleeding.

39. The MCC of excess postpartum blood loss is uterine atony. Initial tx includes bimanual

uterine massage, fluid resuscitation, uterotonic agents (oxytocin, methylergonovine,

carboprost), and blood transfusion as needed. Uterine packing for tamponade is performed if

medical therapy fails and in conjunction with preparations for surgery.

40. The single MCC of postpartum hemorrhage is uterine atony. Risk factors for uterine atony

include multiparity, prolonged labor and any condition that results in uterine hyperdistention.

41. Maintenance of airway, breathing and circulation is always the first priority in any pt

presenting for emergent care. Hemodynamic resuscitation must be promptly initiated before

starting any measure ot diagnose the source of blood loss in pts with antepartum hemorrhage

who are hemodynamically unstable.

42. The MCC of mucopurulent cervicitic is Chlamydia trachomatis.

43. PID dx criteria: a fever >38C, leukocytosis, elevated ESR, purulent cervical discharge,

adnecal tenderness, cervical motion tenderness and lower abdominal tenderness.PID is most

commonly caused by N.gonorhoeae, C. trachomatis, and genital mycoplasmas. It is managed

with empirical wide-spectrum antibiotic therapy; in hospitalized pts, regimens include

219

Cefoxitin/Doxycycline, Cefotetan/ Doxycycline and Clindamycin/gentamicin (all iv). The pt’s

partner should also be treated. In non-hospitalized pts, Cefoxitin (IM) is concurrently used

with Probenecid (PO) plus Doxycycline (PO), or Ceftriaxone (IM) and Doxycycline (PO) are

administered.

44. Semen analysis is a simple test that helps to identify male factor as the cause of infertility.

Semen analysis should be performed early in the evaluation of the infertile couple, usually as

the initial screening test.

1/27/10

1. The maternal quadruple screen will return increased levels of beta-hCG and inhibin A,

decreases levels of maternal serum AFP and estriol in cases of anruploidy. These findings

carry an increased risk of Down syndrome but are not diagnostic of Down syndrome.

2. Cervical dysplasia in a high risk (post-adolescent, premenopausal) pt should be investigated

with colposcopy. (Ref figure 1/25 No.15)

3. Chorioamnionitis refers to infection of the amniotic fluid. The infection is typically

polymicrobial. Maternal fever, tachycardia (>100bpm) and leukocytosis, fetal tachycardia

(>160bpm), uterine tenderness and a foul uterine discharge indicate this dx. Subclinial

infection is confirmed by amniotic fluid analysis (amniocentesis for gram staining, culture and

sensitivities and other studies). Fetal complications of intraamniotic infection include preterm

labor and an increased risk of neurodevelopmental delay and cerebral palsy.

4. All pts who take metronidazole shoud abstain from drinking alcohol, as it is associated with a

disulfiram-like reaction.

5. Adenomyosis occurs most frequently in multiparous women above 40 years of age and

typically presents with dysmenorrheal and menorrhagia. The physical exam reveals an

enlarged and generally symmetrically uterus. In contrastm a fibroid uterus is usually

irregularly shaped.

6. Given the high rate of co-infection with Gonorrhea and Chlamydia and the poor sensitivity of

gram stain for Chlamydia, antibiotic mx must be aimed at eradiction of both organisms when

gram stain confirms cervical Gonorrhea infection (Ceftriaxone and azithromycin). Amipicllin

and gentamicin is indicated if there is a suspicion for polymicrobial infection such as post

partum endometritis.

7. Inevitable abortion is characterized by vaginal bleeding and fluid discharge, low abdominal

cramps and a dilated cervix through which the products of conception can occasionally be

visualized.

8. Thichomonas infection usually causes vaginal discharge, prutitus, dysuria, and dyspareunia,

though it can also be asymptomatic. Regardless of the presence of symptoms, both partners

should be treated with oral metronidazole to prevent recurrent infections.

9. The increased incidence of urinary tract infections in women is due to the shorter length of the

female urethra compared to males. Other predisposing factors for UTI’s include sexual

intercourse, recent antibiotic use, the use of spermicidal contraceptives, and a close proximity

of the urethra to the anus.

10. Fetal hydantoin syndrome presents with small body size, microcephaly, digital hypoplasia,

nail hypoplasia, midfacial hypoplasia, hirsutism, cleft palate and rib anomalies.

11. Cesarean delivery should be performed on women with active genital herpetic lesions

220

(primary or secondary) in order to reduce the risk of neonatal HSV, a severely morbid and

potentially fatal illness.

12. Ureine rupture presents with intense abdominal pain associated with vaginal bleeding which

can range from spotting to massive hemorrhage. Pts also typically exhibit vital signs

consistent with hypovolemia, retraction of presenting parts one pelvic exam, and palpability

of fetal extremities on abdominal exam.

13. A breech presentation diagnosed before the 37th week of gestation does not require any

intervention, as breech presentations often convert to vertex before the 37th week. External

cephalic version is indicated if breech presentation is persistent after 37 weeks, and if this

fails, then planned cesarean delivery is indicated.

14. The best mx of intrauterine fetal demise is delivery of the fetus to reduce the risk of infection

and coagulopathy. Tx options should be discussed with any pt prior to embarking on a plan so

the pt can make an informed decision about therapy with knowledge of the risks and benefits

of all options.

15. A hysterosalpingogram is a radiographic test used to identify structural abnormalities in the

uterus or fallopian tubes that may contribute to infertility. The pt has regular menstrual cycles

accompanied by midcycle pelvic pain (mittelschmerz) and breast tenderness. These facts

indicate that the pt is most likely ovulating, so testing for ovulation is not likely to identify the

cause of infertility.

16. Cigarret smoking is the most common preventable cause of fetal growth restriction in the US.

17. Fetal alcohol syndrome is characterized by growth restriction, midfacial hypoplasia, a smooth

philtrum, short palpebral fissure, a thin upper lip and CNS abnormalities.

18. Bilateral solid ovarian masses discovered incidentally in a multiparous African-American

pregnant woman most likely signify a pregnancy luteoma. This is a benign self-limited

condition and requires no tx.

Sudden onset hirsutism or virulization during pregnancy

↓ Physical examination

Pelvic ultrasonogram

↓------------------------------↓------------------------------↓---------------------------↓No ovarian mass Bilateral cystic Bilateral solid Unilateral solid

↓ ↓ ↓ ↓Abdominal CT scan to Theca Lutein cysts Mostly pregnancy Laparptomy or

rule out adrenal mass (Rule out high-β -HCG states) luteoma Lapaoscopic biopsy to

rule out malignancy

Solid ovarian tumors (viewed by ultrasonography) are almost malignant and demand immediate

and aggressive evaluation and treatment in all age groups, except in pregnancy.

19. Hypogonaotropic hypoganadism is a decrease in circulating sec hormones due to decreased

concentrations of LH and FSH. This condition increases a pt’s risk of osteoporosis. Common

causes include steneous exercise, anorexia nervosa, marijuana use, starvation, stress,

depression, and chronic illness.

20. A menstrual diary for at least 3 cycles is useful aid for comfirning the dx in suspected cases of

PMS. Confirmation of the dx must be made before initiating tx. There is no universally

accepted tx. Reduction of caffeine intake may reduce breast symptoms. An exercise program

221

may be effective in improving the general well being of the pt. In women whose symptoms

are more severe and cause socioeconomic dysfunction, SSRIs are the drug of choice. When

SSRI fail to alleviate symptoms in such pts despite therapy over multiple cycyles, low dose

alprazolam is indicated. Relaxation techniques and bright light therapy have some proven

effect in mx of PMS, but cognitive behavior therapy and insight oriented and supportive

psychotherapy do not play a role. Tx should not be initiated until the dx is made.

21. The tx of choice for bacterial vaginosis in both pregnant and non pregnant pts is oral

metronidazole. Vaginal metronidazole and clindamycin are alternatives.

22. RhoGAM is indicated in unsensitized, Rh-negative women at 28 weeks gestation and withn

72 hours of any procedure or incident such as abortion, ectopic pregnancy and delivery.

23. Asymptomatic pts with Chlamydia detected by screening should be treated with azithromycin

or doxycycline, but concurrent tx for Gonorrhea is not needed if screening for Gonorrhea is

negative.

24. ACEIs and ARBs are C/I in pregnancy. The first-line agents for mx of essential hypertension

during pregnancy are labetalol and methyldopa.

25. Low back pain is a very common complaint in the 3rd trimester of pregnancy. It is believed to

be caued by the increase in lumbar lordosis 脊柱前弯症 and the relaxation of the ligaments

supporting the joints of the pelvic gridle.

26. PCOS is characterized by anovulation or oligo-ovulation, signs of androgen excess, such as

male-pattern hair growth and acne, and ovarian cysts.

27. Mx of placenta previa depends on the severity of bleeding and the age the pregnancy.

Regardless of gestational age, pts with active, uncontrolled antepartum hemorrhage, who also

typically exhibit unstable vital signs and unreassuring fetal heart rates, should undergo

emergent caesarian section delivery.

28. Voiding after intercourse has been shown to decrease the risk of urinary tract infection in

sexually-active females.

1/28/10

1. Acute bacterial parotitis presents with painful swelling of the paroid gland that is aggravated

by chewing. High fever and a tender, swollen and erythematous paroid gland are common.

This post-operative complication can prevented with adequate fluid hydration and oral

hygiene. The most common infectious agent is Stapylococcus aureus.

2. The rotator cuff is formed by the tendons of the supraspinatus, infraspinatus, teres minor and

subscapularis muscles. The supraspinatus is most commonly injured, due to repeated bouts of

ischemia near its insertion on the humerus induced by compression between the humerus and

acromion. A common cause of acute rotator cuff tears is a fall on outstretched hands. Pts

usually have severe shoulder pain and edema following the traumatic event and are unable to

abduct the arm past 90 degrees. The drop arm test is a maneuver that can help to dx a rotator

cuff tear. Here, the pt’s arm is abducted passively to greater than 90 degrees, and the pts is

then asked to lower the arm slowly. With a complete rotator cuff tear, the pt will unable to

lower the arm smoothly and it will appear to drop rapidly from near the 90 degree position.

Rupture of the tendon of the long head to the biceps produces a positive “Popeye sign” where the

biceps muscle belly becomes prominent in the mid upper arm. Weakness with supination is

prominent, and forarm flexion is typically preserved.

222

Injury to the long thoracic nerve causes a winged scapula due to paralsis of the serratus anterior

muscle. The most common cause is iatrogenic injury during axillary lymphadenectomy.

The low (inferior) trunk of the brachial plexus originates from the C8 and T1 cervical roots. Injury

to this trunk, which typically results from sudden upward pulling on the arm, produces Klumpke’s

palsy. This palsy primarily affects muscles innervated by the ulnar nerve, which supplies most of

the intrinsic muscles of the hand. Weakness and atrophy of the pothenar and interosseous muscles

characterize this palsy, and a “claw hand” deformity may also result.

A humeral neck fracture would be more likely to present with swelling, acchymosis and crepitus

over the fracture. Axillary nerve injury may present.

3. Atelectasis is a common post-surgical complication that results from shallow breathing and

weak cough due to pain. It is most common on post-op day 2 and 3 following abdominal or

thoracoabdominal surgery. Adequate pain control and incentive spirometry decrease the

incidence of post-operative atelectasis.

Diaphragmatic paralysis may occur as a consequence of phrenic nerve injury from thoracic

surgery, cervical manipulation, or tumor compression but is less likely during abdominal surgery.

Unilateral diaphragmatic paralysis is often asymptomatic, while bilateral diaphragmatic paralysis

causes hypoxemia, repid shallow breathing, orthopnea, and even respiratory failure.

4. Radial nerve is the most commonly injured nerve in association with fracture of midshaft

humerus.

5. When there is an intimal flap of the carotid artery, surgery is recommended to repair the

vessel. Intimal flap injury can lead to vessel occlusion and symptoms and signs of ischemia.

Today, with the availability of stenting, this may be an alternative option to surgery.

6. The femoral nerve innervates the muscles of the anterior compartment of the thigh, and is

therefore responsible for knee extension and hip flexion. The femoral never provides

sensation to the anterior thigh and medial leg via the saphenous branch.

7. Inflammatory breast cancer is an uncommon form of breast cancer that presents with an

erythematous and edematous cutaneous plaque overlying a mass on the breast commonly with

axillary lymphadenopathy. 1/4 of pts with this condition will have metastatic disease at the

time of presentation. Clinically, inflammatory breast cancer can not be differentiated from

infectious process, such as a breast abscess, with 100% certainty. Therefore, a biopsy for

histology should be done first to exclude or confirm that dx.

8. Extremities subjected to at least 4-6 hours of ischemia can suffer from both intracellular and

interstitial edema upon reperfusion (eg. removal of an embolus). When edema causes the

pressure within a muscular fascial compartment to rise above 30mmHg, compartment

syndrome occurs leading to further ischemic injury to the confined tissues. The clinical

findings are 5 “P’s”: Pain, Paresthesias, Pallor, Pulselessness, Paresis/Paralysis.

9. A pt, who is hypotensive with abdominal pain and has a CT scan showing an enlarged aortic

silhouette, has a dx of a ruptured abdominal aortic aneurysm. This pt needs immediate

surgery.

10. Peripheral artery aneurysms manifests as a pulsatile mass that can compress adjacent

structures (nerves, veins), and can result in thrombosis and ischemia. Popliteal and femoral

artery aneurysms are the most common peripheral artery aneurysms. They are frequently

associated with abdominal aortic aneurysms.

11. Pts who present more than five days after the onset of symptoms of appendicitis, and have

223

localized right lower quadrant findings, should be treated with IV hydration, antibiotics and

bowel rest. Antibiotics should cover enteric gram-negative organisms and anaerobes. A

second/third generation cephslosporin or a fluoquinolone plus metronidazole are usually used.

Cefotetan had a good coverage of gram-negative oraganisms and anaerobes.

12. Biliary colic occurs when the gallbladder becomes distended as it contracts against an

obstructed cystic duct. The pain is typically exacerbated by fatty meals and resolves

completely between episodes. Fever and leukocytosis are not present.

13. A breast mass that produces a non-bloody aspirtate and disappears completely on aspiration

does not need any further evalution other than observation for recurrence.

14. Fat necrosis shows clinical and radiographic findings similar to those seen in breast cancer

including skin or nipple retraction and calcification on mammography. Biopsy of the mass in

fat necrosis will reveal fat globules and foamy histocytes. No tx is indicated for this self-

limted condition. The calcification seen in breast malignancy tend, however, to

microcalcification wihile the clacifications seen in benign conditions such as fat necrosis tend

ot be coarse.

15. Pulmonary contusion represents parenchymal bruising of the lung, which may or may not be

associated with rib fractures. The clinical manifestations develop usually in the first 24 hours

(often with in few minutes); tychypnea, tachycardia, and hypoxia are characteristic. Physical

examination typically reveals chest wall bruising and decreased breath sounds on the side of

pulmonary contusion. Chest x-ray reveals patchy irregular alveolar infiltrate and a CT scan

may be employed to make an early dx. ABG typically shows hypoxemia and, itself, is an

indiction to suspect pulmonary contusion in trauma pts.

16. Acute shoulder pain after forceful abduction and external rotation at the glenohumeral joint

suggests an anterior dislocation, which may cause injury to the axillary nerve or artery.

17. Pts with Paget disease of bone are typically asymptomatic; the dx is made when an increased

bone-specific alkaline phosphatase is discovered on rountine laboratory testing. Pts who do

become symptomatic typically present with pain associated with fractures or with arthritis

resulting from bone deformity (hearing loss).

18. Osteogenic sarcoma (osteosarcoma) is the most common primary malignancy of bone. It most

commonly affects males in their second decade preferentially in the metaphyses of long

bones. Pts present with bone pain but do not have systemic symptoms (x-ray film shows an

osteolytic lesion of the distal femur along with periosteal inflammation).

Osteoclastoma (giant cell tumor of bone), is a tumor of adults that often involves the epiphysis of

the distal femur or proximal tibia. Radiology characteristically shows a soap-bubble appearance

due to the osteolytic quality of the tumor.

Ewing’s sarcoma is also most common in the second decade of life. It is s small, round, blue cell

(neuroectodermal) malignancy classically associated with systemic features such as fever, malaise

and weight loss. It typically affects the diaphyses of long bones as well as the spine and pelvis. X-

ray shows an osteolytic lesion with onion-skin appearance.

Chronic osteromyelitis may present with painful swelling, fever and laboratory abnormalities

including an elevated ESR and leukocytosis. Chronic osteomyelitis most commonly results form

contiguous spread of infection from an adjacent site, such as the skin. This is commonly seen in

diabetic foot ulces.

19. Spinal cord ischemia with lower spastic paraplegia is a rare complication of abdominal aortic

224

aneurysm surgery. The artery of Adamkiewicz arises from the aorta and supplies the anterior

circulation of the middle and lower spine cord. Diminished flow through this artery may

results from its thrombosis, ligation, or systemic hypotension. Resultant neurologic

dysfunction is due to ischemia of antero-lateral structures of the spinal cord. It includes lower

apastic paraplegia, pelvic organ dysfunction, and loss of temperature and pain sensation over

the lower extremities. Vibrotory and proprioceptive sensation is preserved because posterior

circulation of the spinal cord is not affected.

20. In cases of suspected child abuse, physicians are required to thoroughly examine the child for

documentation of all potential signs of abuse. This includes a complete skeletal survey. Child

protective services should be notified of all suspected cases immediately, and the pt should be

admitted.

21. Bowel ischemia may complicate up to 7% of procedures on the aortoiliac vessles and most

commonly affects the distal left colon. Pts report dull pain over the ischemic bowel as well as

hematochezia. Colonscopy shows a discrete segment of cyanotic and ulcrated bowel.

22. A retrograde urethrogram should be the first step in mx of a suspected urethral injury. Foley

catheterization in the presence of a urethral injury will predispose the pt to abscess formation

and worsening of the urethral damage. Classic signs of posterior urethral injury include blood

at the urethral meatus, inability to void and a high-riding porstate on digital rectral exam.

23. Pancreatic cancer classically presents insidiously with a combination of constant visceral

epigestric pain radiating to the back, jaundice due to extrahepatic biliary obstruction, and

anorexia with weight loss. A peptic duodenal ulcer typically causes periodic epigastric pain

relieve by meals.

24. The central cord syndrome classically occurs hyperextension injuries in elderly pts with

degenerative changes in the cervical spine. Such a traumatic injury causes selectively damage

to the central portion of the anterior spinal cord, specifically the central portions of the

corticospinal tracts and the decussating fibers of the lateral spinothalamic tract. Central cord

syndrome is characterized by weakness that is more pronounced in the upper extremities than

in the lower extremities. This unique motor deficit occurs because the motor fibers serving the

arms are nearer to the central part of the cortisospinal tract. Rarely, a pt may also have a

selective loss of pain and temperature sensation in the arms due to damage to the

spinalthalamic tract.

The anterior cord syndrome should be suspected when there is bilateral spastic motor paresis distal

to the lesion. It usually occurs secondary to occlusion of the vertebral artery.

Posterior cord syndrome is associated with bilateral loss of vibratory and propioceptive sensation.

25. Tachypnea, tachycardia, jugular venous distension and tracheal deviation following blunt

chest trauma all suggest tension pneumothorax. This life-threatening emergency requires

immediate decompression with needle thoracostomy. Subsequent tube thoracostomy may be

performed to maintain lung expansion. In a pt who remains hemodynamically unstable

following decompression of a pneumothorax, pericardial tamponade shoud be suspected.

26. Gastric outlet obstruction is a clinical syndrome characterized by early satiety, nausea, non-

bilious vomiting, and weight loss, all of which can be the result of many difference disease

processes. In a pt with a history of acid ingestion, pyloric stricture is the most likely cause.

Esophageal stricture/dysmotility tends to present with dysphagia, which is not a presenting

symptoms of this pt. In addition, abdominal succession splash 液波震颤 is not a finding in those

225

with esophageal stricture/dysmotility.

27. In frail chest, multiple contiguous ribs are fractured resulting in an isolated segment of the

thoracic wall that exhibits paradoxical motion during normal respirtation. Pain control and

supplemental oxygen pressure ventilation is required in many pts with this injury. The

institution of positive pressure mechanical ventilation causes the flail segment to move

normally.

28. When a wound fails to heal after a prolonged period, biopsies should be obtained to ensure

that the ulcer has not degenerated into a squamous cell carcinoma. When SCC arises within a

burn wound, these unclers are known as Marjolin ulcers.

29. In a young individual who presents with a fleshy immobile mass on the midline hard palate,

the most likely dx is torus palatinus. No medical or surgical therapy is required unless the

growth becomes symptomatic or interferes with speech or eating. A pt with torus palatinus

will usually give a history that the lesion has been present for some time and will deny

tenderness. The thin epithelium overlying the bony growth will tend to ulcerate and be slow to

heal due to poor vascularity.

30. Aortoilliac occlusion (Leriche syndrome) is characterized by the triad of bilateral hip, thigh

and buttock claudication, impotence and symmetric atrophy of the bilateral lower extremities

due to chronic ischemia. Impotence is almost always present in men with this condition, in the

absence of impotence, an alternate dx should be sought. The pulse is soft or absent bilaterally

from the groin distally in this condition. Men with a predisposition for atherosclerosis, such as

smokers, are at the greatest risk of this condition. Because impotence is not uncommon in this

age group, and the complaints of hip and thigh pain with walking may also be attributed to

osteroarthritis, there is a risk of missing this dx if a thorough vascular exam is not performed.

31. Injury to a number of different structures within the knee, including ligaments and

cartilaginous menisci, can cause a popping or snapping sensation. Most meniscal tears in the

knee jount occur during a distinctly recalled acute knee injury, often associated with a

popping sensation. Subsequent joint swelling develops gradually, and is often not noticed until

the following day. This is in contrast to ligamentous tears, which may also be associated with

a popping sensation, but which cause rapid joint swelling due to hemarthrosis.

An anterior cruciate ligment (ACL) tear wound cause immediate swelling due to rapid

development of hemarthorosis. Moreover, pts typically experience immediated inability to weiht-

bear and then lasting knee instability following such an injury.

Chronic overuse, typically related to strenuous athletic activities, can cause patellar tendonitis, or

“jumper’s knee”. Physical exam typically reveals point tenderness over the proximal patellar

tendon.

The anserine bursa underline the conjoined tendons of the gracilis and semitendinosus muscles

and separtates them from the head of the tibia. Anserine bursitis causes tenderness over the medial

aspect of the knee, and typically affects athletes and obese middle-aged to elderly women. A

popping sensation is not typically reported.

Stress fractures do not result from discrete injuries such as falls or other trauma. Instead, they arise

insidiously secondary to repeat stress. Athletes, especially long-distance runners, are most

commonly affected. The tibia and fibula are most vulnerable.

32. Nasopharyngeal cancer (NPC, undifferentiated carcinoma of squamous cell origin) is most

common in pts of Mediterranean and Far Eastern descent. It is strongly associated with

226

Epstein-Barr virus infection. Pts with NPC often present with recurrent otitis media (resulting

from Eustachian tube obstruction by tumor), recurrent epistaxia, and/or nasal obstruction.

33. The tibia is the most common bone is the body to be affected by stress fractures, and the pts

affected by ths condition are most commonly competitive athelets. A stress fracture can occur

even in conditioned athletes after only four to six weeks of intense training. Pts who participte

in running sports classically obtain stress fractures in the distal third of the tibia on the

posteromedial border while pts involved in jumping sports, such as dancing, classically abtain

stress fractures in the middle third of the tibia on the anterior side of the bone. Pts typically

complain of pain with activity that improves with rest. If activity is continued, the pain may

persist during rest. Point tenderness to palpation over the fracture is present on exam. X-rays

are usually normal, but they may reveal periosteal reaction in the site of the fracture. The

injury is best defined radiographically using CT or bone scan.

Epiphysitis of the tibial tubercle would cause pain in the proximal tibia, near the knee, where the

iliotibila tract inserts.

34. Urinary calculi present as the flank or abdominal pain radiating to the groin, along with

nausea and vomiting. A non-contrast spiral CT of the abdomen and pelvis is the imaging

modality of choice (best!) for diagnosis. IVP was the test of choice in the past for diagnosing

urinary stones but, because of the risk of contrast-related reactions with IVP, non-contrast CT

is now the preferred test.

35. Scaphoid is the most commonly fractured bone among all the carpals. It is commonly seen in

young adults following a fall on the outstretched hand. Pt generally complaints of pain at wrist

joint. Tenderness in anatomical snuffbox is a very sensitive marker of scaphoid fracture.

Fracure is most commonly located across the waist of scaphoid bone. Initial X rays may be

normal or may show fine radiolucent line in non-displaced scaphoid fractures. Scaphoid views

are necessary to avoid missing the scaphoid fractures. Cast immobilization is recommended in

the tx of all non-displaced scaphoid fractures (fractures <2mm displacement and no

angulation). Open fraction and internal fixation is required if initial X ray shows fracture

displacement.

36. Absent bowel sounds with gaseous distention of both the small and large bowels indicates a

paralytic (adynamic) ileus (X-ray film of abdomen). This classically follows abdominal

surgery but may also occur in cases of retroperitoneal hemorrhage associated with vertebral

fracture.

37. Anterior cord symptom is commonly associated with burst fracture of the vertebra and is

characterized by total loss of motor function below the level of lesion with loss of pain and

temperature on both sides below the lesion and with intact proprioception.

38. The immediate mx of splenic trauma caused by blunt abdominal injury depends on the pt’s

hemodynamic status and reponse to IV fluid. If the pt is initially hemodynamically unstable

but improves with fluid administration, the best next step is to obtain an abdominanl CT scan.

If the pt is initially hemodynamically unstable and unresponsive to fluid administration, then

emergent exploratory laparotomy is requires. In the case of CT scan-documentd splenic injury,

the decision for operative intervention is determined by the grade of the injury. If operative

imtervention is required, every effort is made to repair the spleen rather than remove it,

especially in children. If removal is inavoidable, post-operative immunization against

encapsulated bacteria is mandatory.

227

39. In hemodynamically unstable pts who have suffered blunt abdominal trauma and pelvic

fracture, both intraperitoneal and retroperitoneal bleeding must be ruled out. Focused

Assessment with Sonography for Trauma (FAST) and diagnostic peritoneal lavage are the

appropriate tests for establishing intraperitoneal bleeding. If thest rests are negative, then the

next step in mx would be a pelvic angiography to search for retroperitoneal bleeding. A pelvic

angiography is not only useful in identifying the source of the bleeding, but the bleeding can

also be resolved via embolization of the offending vessel.

40. In a pt with blunt thoracic trauma, suspect the dx of flail chest if he remains tachypneic and

hypotensive despite aggressive fluid resuscitation. Classical x-ray findings reveal multiple rib

fractures overlying a coutused lung. (X-ray film!)

41. In stable pts with abdominal trauma, CT scan with contrast is the single best study to evaluate

solid organ damage. Diagnostic peritoneal lavage is indicated only in unstable pts, where time

is limited to get CT of the abdomen.

42. Subluxation of radial head (Nursemaid elbow) is a common condition in preschool children

and needs closed reduction by flexion and supination of forearm. The child is usually not in

distress at presentation but would cry at any attempt to flex the elbow or supinate the forarm.

Dx is made clinically as radiographs are often normal.

43. It is important to rule out a fracture or dislocation of cervical spine as the first priority because

of grave consequences of missing a cervical spine injury. This pt has epistaxis suggestive of

fracture of base of skull. However, a CY scan is indicated only after cervical spine injury is

ruled out and pt is stable. An alternative could be to do a CT scan of head including the neck.

44. Esophageal rupture most commonly occurs following instrumentation of the esophagus. Less

commonly it may occur in pts with protracted vomiting who have been resisting the urge to

vomit. Rupture in these cases typically occurs into the mediastinum resulting in

pneunomediastinum.

1/30/10

1. Oliguria, azotemia and an elevated BUN/creatinine ratio of >20:1 in the post-operative state

most likely indicate acute pre-renal failure from hypovolemia, though urinary catherter

obstruction should be first be ruled out. The next step in the dx/mx of acute renal failure

manifesting as oliguria or anuria is an IV fluid challenge.

2. Suspect cardiac tamponade in an adult pt who has sustained blunt chest trauma if he has

persistent jugular venous distention, tachycardia, and hypotension despite aggressive fluid

resuscitation. CXR findings typically reveal a normal cardiac silhouette without tension

pneumothorax. DD: lung contrusion, aortic rupture, esophageal rupture, bronchial rupture

3. Esophageal perforation due to iatrogenic cause is very frequent and radiography with water-

soluble contrast is the best way to diagnose esophageal perforation.

4. For hemarrhagically unstable pts in whom blunt abdominal trauma is suspected, fluid

resuscitation should be initiated, followed by ultrasound examination. If ultrasound is not

definitive, then diagnostic peritoneal lavage is performed. Those pts with confirmed

hemoperitoneum whould then undergo laparotomy for surgical repair.

5. Gental traction to attempt alignment of the fragments of a fractured long bone is important to

prevent further vascular and neurological damage and it should be attempt immediately.

6. Severe blunt chest trauma can cause direct physical injury to the underlying pulmonary

228

parenchyma and pulmonary contusions. Signs and symptoms include dyspnea, tachynpea,

chest pain, hypoxemia worsened by iv volume expansion, and patchy, irregular alveolar

infiltrates on CXR. Aspiration of gastric contents has a history of having vomited.

7. Penile fracture is a surgical emergency that requires emergent urethral imaging and surgical

repair. The most common cause is a crush injury of the erect penis, most commonly during

intercourse where the female is on top of the male.

8. Femoral neck fractures are intracapsular fractures and are most commonly seen in

postmenopausal females and males over 70 years of age. The pt will classically present

following a fall with inability to ambulate and hip pain. On exam, the involved limb is

shortened and externally rotated. Active and passive range of motion of the affected limb

elicits pain. The dx can be made with AP and lateral X-rays of the hip, and classification of

the fracture using the Garden classification system may then be made based on radiographic

findings as follows: Type 1: Valgus外翻的 impaction of femoral head. Type 2: complete but

non-displaced femoral neck fracture. Type 3: Complete fracture with displacement <50%.

Type 4: Complete fracture with displacement >50%. Type 1 or stress fractures can be

managed nonoperatively with toe-touch weight bearing on crutches until there is radiogical

evidence of complete healing. Garden types 2, 3 and 4 are unstable fractures and require open

reduction and internal fixation or a primary arthroplasty 关节形成术 . Elderly pts and those

with severely displaced femoral neck fractures (type 4), are preferentially treated with

arthroplasty . Arthroplasty is preferred because the risk of avascular necrosis of the femoral

head is high, even following open reduction and intermal fixation, due to the tenuous blood

supply of the femoral head.

9. Mastitis associated with breast-feeding is treated with analgesics, antibiotics and continuation

of breast-feeding from the affected breast. The most commonly isolated organism is S. aureus.

The most commonly used antibiotics are dicloxacillin or cephalosporins.

10. The differential dx of a widened mediastinum as a solitary finding includes processes as

diverse as anthrax, various tumor, madiastinitis, aortic dissection, hemorrhage and large

pericardial effusion among others. Acute madiastinitis may occur following heart surgery.

Signs and symptoms include fever, chest pain, leukocytosis and mediastinal widening on

CXR. Mediastinitis is a serious condition that requires drainage, debridement and antibiotic

therapy.

11. The nature of this pt’s accident afn his scalp lacerations suggest that he has suffered blunt

head trauma, After initial fluid resuscitation, he lapses into a state of decresed consciousness

with right-sided hemiparesis. The mist likely explanation in this setting is a transtentorial

(uncal) herniation secondary to right-sided epidural hematoma. Uncal herniation mauy also

result from a subdural hematoma or an intracerebral mass.

Epidural hematomas result from rupture of the middle meningeal artery. Because they are under

arterial pressure, they expand rapidly. In this case, the fluid resuscitation likely increased the rate

at which the hematoma expanded, thereby precipitating the above neurologic signs. Focal

neurologic sings result form herniation of the parehippocampal uncus through the tentorial

incisure, which causes pressure on the ipsilateral oculomotor nerve and posterior cerebral artery as

well as compression of the contralateral cerebral peduncle against the edge of the tentorium. The

specific lesions and their respective signs are in the table.Transtentorial (uncal) herniation

229

Lesions SignsComperssion of contralateral crus cerebri against the tentorial edge

Ipsilateral herniparesis

Compression of the ipsilateral oculomotor nerve (CN III) by the herniated uncus

Loss of parasympathetic innervation causes mydriasis (occur early)Loss of motor innervation causes ptosis and

“down and out” gaze of the ipsilateral pupil due

to upopposed trachlear (CN IV) and abducent

(CV VI) action (occur later)Compression of the ipsilateral posterior cerebral artery

Contralateral homonymous hemianopia

Compression of the reticular formation Altered level of consciousness, coma

12. Acute ruptured appendicitis may be complicated by pelvic abscess. Pelvic abscess typically

presents with lower abdominal pain, malaise, low-grade fever and a tender pelvic mass on

rectal examination.

13. Isolated duodenal hematoma most commonly occurs in children following blunt abdominal

trauma. It is treated conservatively with nasogastric suction and parenteral nutrition. Most

hematomas will resolve spontaneously in 1-2 weeks. Surgery may be considered to evacuate

the hematoma if this more conservative method fails.

14. The tx of pancreatitis is primarily supportive. Antibiotics may be used, but the evidence does

not presently support the use of antacids, antichoinergics, glucagan or octreoties. Identifying

the underlying cause may facilitate prevention of recurrent pancreatitis.

15. Legg-Calve-Perthes disease is characterized by avascular necrosis of femoral head. It

typically presents in children four to ten years of age with insidious-onset hip or knee pain and

an antalgic 镇痛的 gait. Physical exam shows marked limitation of internal rotation and

abduction at the hip joint. Proximal thigh atrophy may also be present. X-rays may be

negative initially, but serial studies will show changes in the femoral head consistent with

necrosis and recalcification. MRI and bone scans show findings suggestive of femoral head

necrosis much earlier than standard x-rays and may be used to make the diagnosis months

earlier. Tx is aimed at maintaining placement of the femoral head within the acetabulum so

that it may heal in the proper shape and position. This may be accomplished with splints or

surgery.

16. Unilateral suacute hip pain in a male child coupled with a progressive antalgic gait, thigh

muscle atrophy, decreased hip range of motion, and collapse of the ipsilateral femoral head on

plain pelvic x-rays are findings suggestive of idiopathic avascular necrosis of the femoral

capital epiphysis (Legg-Calve-Perthes disease.) (X-ray film!)

17. High-energy, blunt, rapid deceleration trauma to the chest commonly causes aortic injury. In

most cases of aortic rupture, death is the immediate result. In pts with a contained rupture, the

diagnosis must be made quickly. Widened mediastium and left-sided hemothorax are classic

CXR abnormalities pointing to this dx. In these cases, the diagnosis can be confirmed via CT

scanning. Mx of pts wht established aortic injury includes antihypertensive therapy where

appropriated and immediated operative repair.

18. Oxalate absorption is increased in Crohn disease and all other intestinal diseases causing fat

malabsorption. Increased absorption is the MCC of symptomatic hyperoxaluria and oxalate

230

stone formation. Under normal circumstances, calcium binds oxalate in the gut and prevents

its absorption. In pts with fat malabsorption, calcium is preferentially bound by fat leaving

oxalate unbound and free to be absorbed into the bloodstream. Failure to adequately absorb

bile slats in states of fat malabsorption also cause decreased bile salt reabsorption in the small

intestine. Excess bile salts may damage the colonic mucosa and contribute to increased

oxalate absorption.

19. Pts suffering esophageal perforation most commonly present with acute-onset substernal pain.

The diagnostic study of choice when esophageal perforation is suspected is a gastrografin

contrast esophagogram.

20. Pts with mild to moderate head injury can be discharged under the care of an adult if they

have a normal CT scan. The caretaker is given printed instructions detailing the signs and

symptoms that warrant immediate return to the hospital.

21. This pt presents with symptoms and signs suggestive of meniscal injury. The episode of injury

occurred one month ago, when acute pain and swelling were present. The current symptoms

suggest ‘locking’, a phenomenon that is due to impaired extension of the knee. In addition of

the pt’s history, a positive McMurry sign is present (popping sound on passive

flexion/extension of the joint), which is specific for meniscal injury. Persistent symptoms (eg.

mechanical symptoms) in pts with probable meniscal injury should be further evaluated by

MRI or arthroscopy. Surgery (arthroscopic or open) is often necessary to correct the problem.

22. Persistent pneumothorax and significant air leak following chest tube placement in a pt who

has sustained blunt chest trauma suggests traheobronchial rupture. Other findings include

pneumomediastinum and subautaneous emphysema. The dx can be comfirmed with high-

resolution CT scanning, bronchoscopy, or surgical exploration. Operative repair is indicated.

23. Bowel ischemia and infarction ar possible early complications of operation on the abdominal

aorta, such as abdominal aortic aneurysm reqair. This adverse effect can be minimized by

cheching sigmoid colon perfusion following placement of the aortic graft.

24. All pts with acute carbon monoxide poisoning should be treated with 100% oxygen via a

nonrebreather facemask. Early symptoms of carbon monoxide poisioning are typically

neurologic and include agitation, confusion and somnolence.

25. Positive pressure mechanical ventilation increases intrathoracic pressure, which decreases

venous return to the heart and thereby decreases the ventricular preload. In pts with

hypovolemic shock, this effect may cause circulatory collapse if the pt’s intravascular volume

is not replaced before mechanical ventilation is attempted.

26. Acute appendicitis is a clinical dx. Pts with a classic presentation should be operated on

immediately to remove the inflamed appendix and prevent appendiceal rupture.

27. The medial meniscus is typically injured during forceful torsion of the knee with the foot

planted. Pts complain of a popping sound and severe pain at eh time of injury; McMurray’s

sign is commonly positive on physical exam. A bucket handle tear of the medial meniscus

leads to locking of the knee joint during terminal extension. Because the meniscus is not

directly perfused, effusion following injury typically is not clinically apparent for many hours

following the injury.

In cases of anterior cruciate ligment tear, the pt gives a history of a forceful hyperectension injury

to knee or a noncontact torsional injury of the knee during deceleration. Effusion is seen rapidly

following injury. Lachman’s test, anterior drawer test and pivot shift test are used for clinical dx.

231

Posterior cruciate ligament injury is classically seen in the “dashboard injury”, which refers to

forceful posterior-directed force on the tibia with the knee flexed at 90 degrees. The posterior

drawer, reverse pivot shift and posterior sag下垂 tests will help in clinical dx.

Medial collateral ligament injury is associated with abduction injury to the knee. The valgus stress

test will help in clinical dx.

Lateral collateral ligament injury is very rare and would be seen in adduction injury to the knee.

The varus stress test will help in clinical dx.

28. For hemodynamically unstable pts in whom blunt abdominal trauma is suspected, fluid

resuscitation shoud be initiated, followed by ultrasound examination. If ultrasound reveals

intraperitoneal blood, the pt should then undergo urgent laparotomy for surgical repair.

29. When a pt presents with a pulsatile abdominal mass and hypotension, a presumptive dx of

ruptured abdominal aortic aneurysm must be entertained and the pt should be taken

immediately to the operating room.

30. Postoperative pulmonary complications are common, particularly in pts with known risk

factors. These risk factors include smoking, preexisting pulmonary diasease, age>50, thoracic

or abdominal surgery, surgery lasting > 3 hours, and poor general health. Postoperative

measures such as incentive spirometry and deep breathing exercises are used to prevent such

complications and improve outcomes by promoting lung expansion.

31. A hydrocele is a fluid collection within the processus or tunica vaginalis—the peritoneal

projection that accompanies the testis during its descent into the scrotum. Hydrocele can be

differentiated from other testicular masses by transillumination; a hydrocele will

transilluminate while other masses will not. Most cases of hydrocele will disappear

spontaneously by the age of 12 months and can be safely observed during that period.

Hydroceles that do not resolve spontaneously should be removed surgically due to the risk of

inguinal hernia.

32. In a steady resting state the RQ (Respiratory quotient) depends upon the proportions of

metabolic fuels being oxidized for ATP production. A steady state RQ close to 1.0 indicates

predominant oxidation of carbohydrates and net lipogenesis. The RQ for protein and lipid as

sole sources of energy are 0.8 and 0.7, respectively. Assessment of the RQ is important when

attempting to wean pts from mechanical ventilation, as overfeeding, especially carbohydrates,

can cause excessive CO2 production and make weaning more challenging. This factor is

especially important in pts with preexisting lung disease.

33. A varicocele is a tortuous dilation of the pampiniform蔓形的 plexus of veins surrounding the

spermatic cord and testis in the scrotum. A varicele results form incompetence of the valves of

the testicular vein and occurs most frequently on the left side, possibly because the left

testicular vein enters the left renal vein infeiriorly at a right angle thereby predisposing to

impaired drainage. Pts are commonly asymptomatic. Those who do complain of symptoms

may endorse a “dull” or “dragging” discomfort of the scrotum that is worse when standing.

On exam, the affected side of the scrotum will feel similar to a “bag of worms” and valsalva

maneuver will typically cause the mass to enlarge.

Cystic dilations of the efferent ductules (spermatocele) are painless fluid-filled cysts that contain

sperm. They are located on the superior pole of the testis in relation to the epididymis. These

masses are distinct from testix and classically transilluminate.

34. Complete small bowel obstruction usually present with nausea, vomiting, abdominal bloating,

232

and dilated loops of bowel on abdominal x-ray. Adhesions, typically post-operative, are the

most common etiology.

35. Arterial occlusion of an extremity will cause pain, pulselessness, pallor, paresthesias and

paralysis in the affected limb. Embolic occlusion classically causes sudden-onset severe pain

and asymmetric pulselessness.

36. Volkmann’s ischemic contracture is the final sequel of compartment syndrome in which the

dead muscle has been replaced with fibrous tissue.

37. Reflex sympathetic dystrophy is a vague painful condition seen as a sequel of infection or

trauma which may be minor. It is characterized by pain, hyperesthedia and tenderness, which

are out of proportion to the physical findings. It does not have an acute onset like in this pt

and is not associated with compartment syndrome. Sudeck’s atrophy is a radiographic term for

spotty rarefaction seen in pts with reflex sympathetic dystrophy.

38. Following splenectomy, pts are at increased risk for sepsis secondary to encapsulated

organisms like S. pneumoniae, N.meningitidis, and H.influenzae. Vaccinations against each of

these organisms should be administered. Pneumococcal vaccine boosters are required every 5

years.

39. An exploratory laparotomy is required in all pts with blunt abdominal trauma with signs of

peritonitis or hemodynamic instablility.

40. Three to four percent of pts with spinal cord injuries will develop post-trumatic

syringomyelia. Whiplash is often the inciting injury. Symptoms develop months to years later.

The condition involves enlargement of the central canal of the spinal cord due to CSF

retention, resulting in impairment strength and pain/temperature sensation in the upper

extremities or having a cape-like distribution, with preservation of dorsal column function.

MRI is used for definitive dx.

41. Drooping of the contralateral hemipelvis below its normal horizontal level during monopedal

stance constitutes a positive Trendelenburg sign. It is caused by weakness or paralysis of the

gluteus medius and minimus muscles, which are innervated by the superior gluteal nerve.

42. Blunt trauma to the upper abdomen can cause a pancreatic contusion, crush injury, laceration

or transaction. Pancreatic injuries may be missed by CT scan during the first 6 hours

following trauma. An untreated pancreatic injury can later be complicated by a retroperitoneal

abscess or pseudocyst.

43. Fat embolism presents with dyspnea, confusion and petechiae in the upper part of the body

and occurs after multiple fractures of long bones. Serial x-rays shows increasing diffuse

bilateral pulmonary infiltrates within 24-48 hours of onset of clinical findings.

44. Posterior dislocation of the shoulder may follow tonic-clonic seizures. In posterior shoulder

dislocation the arm is classically held adducted and internally rotated and fullness is palpable

on the posterior shoulder. Neurovascular impairment is unusual.

1/30/10

1. Pts treated with high-dose methylprednisolone within eight hours of spinal cord injury have

significant and sustained motor and sensory neurological improvement. Thus, its use is

warranted as the first priority after stablilizing a pt with clinical evidence of spinal cord injury.

All pts with clinical evidence of a spinal cord injury should be imaged with CT scan to assess

the bone and MRI to assess the cord and other soft tissues, such as the intervertebral disks. Tx

233

with iv corticosteroids should not be delayed for these imaging procedures.

2. A cystic lesion involving the pancreas on CT scan in the setting of acut pancreatitis with

systemic toxicity must be considered an abscess and should be treated with drainage. In the

absence of systemic toxicity, a pseudocyst may be presumed and magaged expectantly.

3. The first step in evaluating a pt with suspected peripheral artery disease (PAD) is to obtain an

ankle-brachial index (ABI). The ABI is calculated by dividing the systolic blood pressure

obtained by Doppler in the posterior tibial and dorsalis pedis arteries by that in the brachial

artery. Ratio of 1 to 1.3 are considered normal. An ABI less than 0.9 is highly sensitive and

specific for greater than 50% occlusion in a major vessel. ABIs less than 0.4 are consistent wit

limb ischemia. After PAD is diagnosed by ABI, a number of different imaging studies may be

performed to more accurately identify the occlude vessel.

4. TIG tetanus immunoglobulin, TT tetanus toxoidHistory of tetanus toxoid immunization

Clean minor wounds All other woulds

< 3 doses of tetanus toxoid in past

TT: YesTIG: No

TT: YesTIG: Yes

>= 3 doses TT: Yes if last does >10 years agoTIG: No

TT: Yes if last does >5 years agoTIG: No

5. Diverticulosis is the MCC of hematochezia in an elderly pt. Chronic constipation resulting

from a low fiber diet is the most common predisposing factor for developing diverticulosis.

6. Any gunshot wound of the abdomen requires exploratory laparotomy. Any gunshot wound

belowt the 4 th intercostal space (the level of nipple) is considered to involve the abdomen .

7. MRI is the investigation of choice for defining soft tissue injuries of the knee. Surgery is

rarely necessary for MCL (medial collateral ligament) tears; bracing and early ambulation is

the preferred tx. Injury to MCL leads to increased angulation (abduction) of the effected knee

on valgus stress.

8. Hypotension not responsive to fluid administration following trauma is suggestive of ongoing

occult blood loss. Pts in this scenario must be treated emergently with surgical intervention to

stop further hemorrhage.

9. Pregnancy test should be checked before ordering X-ray in any pt of childbearing age.

10. In a middle-aged adult, superficial unilateral hip pain that is exacerbated by external pressue

to the upper lateral thigh (as when lying on the affected side in bed) suggests trochanteric

bursitis.

11. In burn victims, clinical indicators of thermal inhalation injury to the upper airway and/or

smoke inhalation injury to the lungs include burns on the face, singing of the eyebrows,

oropharyngeal inflammation/blistering, oropharyngeal carbon deposits, carbonaceous sputum,

stridor, carboxyhemoglobin level > 10%, or history of confinement in a burning building. The

presence of one or more of these indicators warrants early intubation to prevent upper airway

obstruction by edema.

12. Pain relief should be the prime objective in the mx of rib fracture as it will allow proper

ventilation and prevent atelectasis and pneumonia. Oral agents, such as opiates and/or

NSAIDs are most commonly utilized, but an intercostals nerve block with a long-acting local

anesthetic can be used if oral or systemic analgesics are not sufficiently effective. Intercostal

nerve blocks provide pain relief without affecting respiratory function, as opiate analgesics

234

may, but it does carry some risk of pneumothorax.

13. All hemodynamically unstable pts with penetrating abdominal trauma must undergo

immediate exploratory laparotomy to dx and treat the source of bleeding, as well as to

diagnose and treat perforation of any abdominal viscus in an effort to prevent sepsis.

14. After blunt trauma to the chest, if an x-ray shows a deviated mediastinum with a mass in the

left lower chest, one should suspect a diaphragmatic perforation with herniation of abdominal

viscera. Typical symptoms include chest pain radiating to the shoulder, shortness of breath and

abdominal pai. Respiratory distress may follow. A barium swallow or CT scan with oral

contrast will be diagnostic. All diaphragmatic ruptures require operative repair.

15. Intraabdominal pathology causing pain in one or both shoulders suggests subdiaphragmatic

peritonitis. Among the possible blunt traumatic bladder injuries, only an intraperitoneal

rupture of the bladder dome could, by itself, cause a chemical peritonitis. The other lower

urinary tract structures are all extraperitoneal.

16. For any pt who suffers blunt deceleration trauma (motor vehivel accident or fall from >10

feet), blunt aortic trauma must be ruled out. CXR is the initial screening test, and widening of

the mediastium is the most sensitive finding.

17. Acalculous 非结石性 cholecystitis occurs in critically ill pts. The clinical presentation is

vague and similar to calculous cholecystitis if the pt is able to interact. Imaging studies show

gall bladder distention, thickening of the gall bladder wall and pericholecystic fluid. The

emergent tx of choice is percutaneous cholecystostomy.

18. In pts with significant total body surface area burns, the major cause of morbidity and

mortality is hypovolemic shock. In the setting of adequate initial fluid resuscitation, bacterial

infection (usually bronchopheumonia or burn wound infection) leading to sepsis and septic

shock is the leading complication. Criteria for the systemic inflammatory response syndrome

(SIRS): 1. Fever to hypothermia 2. Tachypnea 3. Tachycardia 4. Leukocytosis, leucopenia or

bandemia.

19. If a pt develops a whistling noise during respiration following rhinoplasty, one should suspect

nasal septal perforation likely resulting from a septal hematoma.

20. The most commonly injured viscera in blunt abdominal trauma is the spleen, followed by the

liver and kidney. For hemodynamically unstable pts in whom blunt abdominal trauma is

suspected, fluid resuscitation should be initiated, followed by ultrasound examination (FAST-

focused Assessment with Sonography for Trauma).

21. The next investigational step in the work-up of a solitary pulmonary nodule detected on CXR

is usually high-resolution CT scan. The data obtained, when coupled with demographic data,

can be used to determine a pt’s percent risk of malignancy, which is necessary for determinig

further therapy.

22. All pts who have clinical signs of a scaphoid fracture following an injury likely to cause such

a fracture should be presumed to have a fracture (even if radipgraphs are negative) and treated

with thumb immobilization in a spica cast and reevaluation by x-ray in 2-3 weeks.

23. Hyperventilation helps to prevent and treat intracranial hypertension by causing cerebral

vasoconstriction and thus decreasing cerebral blood flow.

24. Tension pneumothorax is treated with immediate needle or tube thoracostomy.

25. Moving from supine to sitting can increase the functional residual capacity (FRC) by 20% to

35%. In normal adults, this can amount to several hundred cubic centimers of lung volume.

235

Increasing the FRC can help prevent postoperative atelectasis.

26. Following blunt trauma, cardiogenic shock with a high-normal initial pulmonary capillary

wedge pressure (PCWP) reading may be the result of a cardiac contusion. Tx is first with IV

fluid to bring the PCWP to between 15-20 mmHg. Following that, positive inotropic agents

should be used to increase cardiac output.Major forms of shock in thoracic trauma pts MAP HR PCWP PCWP after fluid

challengeHypovolemic/Hemorrhagic ↓ ↑ ↓ U or ↑Cardiogenic (LV dysfunction): contusion, MI, traumatic valve or septal rupture, arrhythmia

↓ V ↑ ↑↑

Cardiogenic (RV dysfunction): contusion, MI ↓ V ↓, N U or ↑Extracardiac obstructive: tension, pneumothorax, massive hemothorax

↓ ↑ ↓, N U or ↑

Extracardiac obstructive: pericardial tamponade ↓ ↑ ↑ ↑ or ↑↑Neurogenic ↓ ↑, N ↓, N ↑Septic (Hyperdynamic) ↓ ↑ ↓, N ↑MAP = mean arterial pressure; V=variable

HR = heart rate; N = normal; U = unchanged

PCWP = pulmonary capillary wedge pressure

27. Non bleeding varices are managed with nonselective beta-adrenergic antagonists, such as

propranolol. Such therapy can reduce the risk of bleeding by up to half. Pt has cirrhosis and

PUD—conservative mx of his encephalopathy (oral lactulose), ascites (diuretics), portal

hypertension and PUD (PPIs). Sclerotherapy, endoscopic band ligation and surgery are

indicated after a pt has experienced a first episode of variceal bleeding, but these procedures

are not recommended for prophylaxis. A portosystemic shunt or TIPS procedure are

considered a last resort in variceal bleeding unresponsive to medical and endoscopic

interventions and may worsen the encephalopathy in this pt.

28. Deep vein thromboses occur when Virchow’s triad of stasis, endothelial injury, and

hypercoagulability are present. Major surgery is a significant risk factor for DVTs. Pts with

DVT should be treated with a heparin product acutely and warfarin for several months. The

goal of therapy is to prevent extension of the clot and development of future clots, not lysis of

the clot already present. Streptokinase and TPA are clot-lysing enzyme used to treat acute ST-

elevation MI. They are not generally recommended for the vast majority of DVT pts.

29. All pts with a clavicular fracture should have a careful neurovascular examination to rule out

injury to the underlying branchial plexus and subclavian artery.

30. Acute GI perforation requires emergent laparotomy. If an affected pt is on warfarin, then

reversal of anticoagulation must be rapidly achived pre-operatively by infusion of fresh frozen

plasma. VitK is not appropriate for emergency. In general, tissue oxygen delivery does not

become deficient until the hemoglobin drops below 7 g/dl. Transfusion (Packed RBC) is often

not required in pts with chronic anemia. Platelet counts greater than 50,000/mm3 provide

adequate hemostasis for most invasive procedures. Desmopressin (DDAVP) is given pre-

operatively to pts with mild hemophilia A in order to prevent excessive bleeding. It indirectly

increases factor VIII levels by causing VWF release form endothelial cells.

31. Orotracheal intubation and surgical cricothyroidectomy are preferred way to establish an

airway in apneic pt with head injury (obvious head and neck injuries). Needle

cricothyroidectomy is an excellent field procedure to establish an airway especially in

children. However, it is not suitable in adults due to risk of carbon dioxide retention especially

in pts with head injury where hyperventilation can be required to prevent or treat intracranial

236

hypertension.

32. The initial tx of both complete and partial small bowel obstruction is IV fluid resuscitation, IV

electrolyte replacement, NPO and NG tube gastric decompression. Surgical is typically

required ultimately in pts with complete obstruction in order to rule out and prevent

strangulation.

33. Venous insufficiency (valvular incompetence) is the MCC of lower extremity edema. It

classically worsens throughout the day and resolves overnight when the pt is recumbent.

34. Technetium-99 labeled erythrocyte scintigraphy should be used in cases of GI bleeding where

the source can not be identified. This study localizes the source of blood loss so that region

can be further evaluated and treated by colonoscopy or angiography.

35. Rupture of the duodenum is frequently diagnosed by noting retroperitoneal air on abdominal

x-ray. Duodenal injuries may then be better characterized using CT scan with oral contrast.

36. In case of amputation injury, amputated parts should be wrapped in saline-moistened gauze,

sealed in a plastic bag, placed on ice and brought to the emergency department with the pt.

37. When hemorrhage occurs, tachycardia and peripheral vascular constriction are the first

physiological changes. These responses act to maintain the blood pressure within normal

limits until severe blood loss has occurred.

38. Dumping syndrome is a common postgastrectomy complication. This process leads to the

fluid shift from intravascular space to the small intestine, release of intestinal vsoactive

polypeptides, and stimulation of autonomic reflexes. The symptoms usually diminish over

time and dietary changes are helpful to control the symptoms. In resistant cases, octreotide

should be tried. Reconstructive surgery is reserved for intractable cases.

39. A chest x-ray required following central line placement (ideally should lie in the superior vena

cava) to ensure proper positioning and absence of pneumothorax.

40. CT scan is the investigation of choice to dx an intraabdominal abscess. Musculoskeletal

infections, such as osteomyelitis or abscess, frequenly result from hematogenous spread of

organisms from another site, such as the skin. In such cases, S.aureus is the most common

offending pathogen.

AP and lateral lumbar films may be useful in making the dx of ureteral stones (if radioopaque),

bowel obstruction and perforated abdominal viscus. A lumbar x-ray may also disclose the

destruction of the vertebral bodies caused by spinal TB (Pott’s disease). Potts disease is a rare

cause of psoas abscess resulting from contiguous spread.

41. Edema, stasis dermatitis and veneous ulcerations result form lower extremity venous

insufficiency due to valve incompetence. Such disease classically occurs on the medial leg

superior to the medial malleolus.

Arterial thrombosis classically presents with a pt complaining of severe pain in a single ectremity.

The onset of pain is less acute than that seen in arterial occlusion due to embolus. The extremity

typically exhibits coolness to touch, pallor, pulselessness and paralysis on exam.

Raynaud disease/phenomenon is characterized by arterial spasm in response to cold or emotional

stress causing discoloration and discomfort of the distal digits. Raynaud phenomenon may

ultimately result in distal digital gangrene if severe.

Peripheral neuropathy predisposes to chronic ulceration resulting from unknown or neglected

injuries or chromic pressure on the soles of the feet. This most commonly occurs in the setting of

uncontrolled diabetes.

237

42. When circumferential full-thickness burns involving the extremities or chest are present, and

escharotomy may be necessary to prevent vascular compromise and respiratory difficulty,

respectively.

43. Perforated peptic ulcer causes sudden onset severe epigastric pain, which spreads over the

whole abdomen deu to chemical peritonitis. Upright abdominal x-rays classically show free

air under the diaphragm.

44. An acute blood loss in excess of 1500ml (25-30% in 70kg man) usually requires a blood

transfusion. The hemotocrit is a poor indicator of acute blood loss as it generally remains near

the normal level in the time period immediately following hemorrhage.

1/31/10

1. Parotid surgery involving the deep lobe of the parotid gland carries a significant risk of facial

nerve palsy resulting in facial droop.

2. In pts who have had at leasr 3 tetanus vaccinations (a complete childhood series), tetanus Ig is

not indicated following wounds, even if the length of time since the pt’s last vaccination is

greater than 10 years.

3. The possibility of aortic disruption must be considered in every victim of a severe

deceleration injury. Clinical signs can include hypertension of the upper extremities due to

pseudocoarctation (caused by compression of the aortic lumen by the aforementioned

hematoma) and a weak, hoarse voice due to pressure on the left recurrent laryngeal nerve by a

periaortic hematoma.

4. Posterior urethral injury is associated with pelvic fractures. Pts present with blood at the

urethral meatus, a high riding prostate, scrotal hematoma, inability to void despite sensing an

urge to void, and a palpably distended bladder.

5. Hymenoptera 膜翅目昆虫 stings account for more deaths in the US than any other

envenomation毒液螫入 . Anaphylactic shock of any cause should be promptly treated with

subcutaneous epinephrine in order to acutely prevent respiratory distress and circulatory

collapse. Antihistamines are useful in facilitating relief of urticaria, angioedema and

bronchospasm as ancillary agents, but epinephrine (exerts both alpha and beta-adrenergic

effects resulting in vasoconstriction, bronchial smooth muscle relaxation and a decrease in

vascular permeability) should be given first.

6. This ileus is possibly due to a vagal reaction due to urethral colic. Needle shaped crystals on

urinalysis indicate uric acid stones. Uric acid stones, which are radiolucent, have to

beevaluated by either CT of the abdomen or iv pyelography. Ileus will be over when the

uretherolithiasis is treated. Stones less than 0.6 cm may pass spontaneously with hydration

and analgesia, otherwise instrumental intervention is needed.

7. The tx of choice for fracture of the femoral shaft is closed intramedullary fixation of fracture.

Plate and screw fixation is reserved only for pts who can not treated with intramedullary

nailing, such as victim of polytrauma who are unable to be placed on the fracture table.

8. The first step in the mx of atraumatic massive hemoptysis (TB) is bronchoscopy to

simultaneously localize and control the site of intrapulmonary bleeding. Bronchoscopy has the

additional benefit of offering increased control and protection of the pt’s airway. When the

underlying cause of hemoptysis is a vascular lesion, such as an arteriovenous malformation,

then pulmonary angiography and embolization might be necessary subsequent therapeutic

238

steps.

9. Acute adrenal insufficiency is a potentially lethal postoperative complication. Preoperative

steroid (lupus pt) use is the main cause. A high index of suspicion is required. Commonly,

they present with nausea, vomiting, abdominal pain, hypoglycemia and hypotension.

10. IV catherters are one of the MCC of nosocomial infections. Femoral central venous catheters

carry a higher risk of causing becteremia than subclavian catherters. IV catherter infections

are MCC by cutaneous organisms, such as Stapylococci, but femoral catheters may also cuase

G- bacteremia. Causes of postoperative fever—“5 W’s”: wind (po 1-2), water (3-5), walking

(4-6), wound (5-7) and wonder drugs (>7 days).

11. After blunt chest trauma, hemorrhagic shock associated with decreased breath sounds and

dullness to percussion over one hemithorax and contralateral tracheal deviation is most likely

due to large ipsilateral hemothorax.

12. CT scan of a diffuse axonal injury shows numerous minute punctuate hemorrhages with

blurring of grey-white interface. It is frequently due to traumatic deceleration injury. Pt loses

consciousness instantaneously and later develops persistent vegetative state.

13. Synovial fluid analysis is essential in managing monoarticular arthritis because the fluid

characteristics dictate treatment. For example, septic arthritis warrants antibiotic therapy and

surgical washout of the joint whereas other types of arthritis can be managed conservatively

with painkillers alone. Synovial fluid Normal Noninflammatory inflammatory SepticAppearance Clear Clear, yellow Clear ot opaque,

yellowOpaque, yellow to green

WBC/mcl <200 <2000 >2000 >2000, often >50,000

Neutrophils (%) <25 <25 >=50 >=75Glucose (mg/ml) Serum

concentrationSerum concentration

Between 25 and serum level

<25

Gram stain/culture Negative Negative Negative PositiveEtiology No pathology Osteoarthritis,

traumaRheumatic disease, crystal arthritis

Bacterial infection

In line with the table above, the indication for surgical washout are positive Gram stain or culture,

glucose <25 mg/dl, >75% neutrophils, or >50,000 leukocytes per mcl

14. Acute pain and swelling of the midline sacrococcygeal skin and subcutaneous tissues are most

commonly due to pilonidal disease. The acute presentation involves infection of a dermal

sinus tract originating over the coccyx. Tx is by drainage of abscess and excision of sinus

tracts.

15. Early recognition and tx of diaphragmatic hernia is extremely important since the mortality of

undiagnosed injury and subsequent bowel strangulation may be up to 30%. An elevated left

diaphragm may be the only abnormal CXR finding. Pts usually have respiratory distress and

deviation of the mediastinal content to the opposite side.

16. Stress (hairline) fractures of the metatarsals are not uncommon in athletes and military

recruits; the second metatarsal is most commonly injured. Tx is with rest, analgesia and hard-

sole shoe. Point tenderness over the affected metatarsal is present on exam. Fractures of the

2nd, 3rd, 4th metatarsals are managed conservatively because the surrounding metatarsals act as

splints and nonunion is uncommon. Plaster casting is used for pts with persistent pain

239

following more conservative tx. Surgical intervention is reserved for fractures of the 5th

metatarsal, such as Jones fractures, or for displaced fractures not amenable to closed

reduction.

17. Tension pneumothorax is an emergency situation and needs immediate needle thoracostomy

based on clinical diagnosis.

18. Acute mesenteric ischemia classically presents with severe acute-onset midabdominal pain

out of proportion to physical examination findings. The MCC is an embolus from the heart.

Progression to bowel infarction causes peritoneal signs and the passage of bloody stool.

19. The risk factors for Developmental dysplasia of the hip (DDH) are Caucasian race, female

gender, first-born infants, breech position and a family history of DDH. The Barlow and

Ortolani tests (act to dislocate and relocate affected infant’s hips) are used to assess for hip

dislocatibility. Positive physical exam findings should prompt a radiologic dx with ultrasound

(< 4 months old) or radiographs (> 4 months old). Tx may involve use of a hip harness or

spica cast.

20. Pts with peripheral vascular disease typically have risk factor similar to those for heart disease

and present with claudication, impotence and skin changes. Ankle-branchial index (obtain

resting and post-exercise systolic BP in the ankle and arm) is the best tool for determining the

severity of disease.

Segmental volume plethysmography and Doppler ultrasound may be used to aid in determining

the location of a lesion causing peripheral arterial disease, but the dx should first be established

with ABI. Doppler ultrasound is also used in the dx of DVT.

21. CT guided percutaneous drainage is the standard tx approach for pelvic abscess. Surgical

drainage may be attempt if percutaneous drainage fails.

22. Diabetic foot ulcers result from neuropathy, microvascular insufficiency and

immunosuppression. They classically occur on the plantar surface of the foot under points of

greatest pressure, such as under the head of the first metatarsal bone.

23. History of a traumatic event and presence of crunching are compatible with fracture of

metatarsal.

24. A mammogram should be the first step in evaluation of all pts over age 35 with a palpable

breast lump. This is particularly important if there is a family history of breast cancer.

Physical examination alone is insufficient to differentiate benign from malignant lesions. All

suspicious lesions should then be biopsied.

25. Necrotized surgical infection is characterized by intensive pain in the wound, decreased

sensitivity at the edges of the wound, cloudy-gray discharge, and sometimes crepitus. Early

surgical exploration is essential.

26. Intermittent bloody discharge from one nipple is the classic presentation of intraductal

papilloma, a benign breast disease. Masses are generally not appreciated in this condition, as

the abnormality is small, soft, and located directly beneath the nipple. Ultrasound is best at

detecting masses greater than 1cm in diameter.

27. Postoperative ileus follows most surgeries where the peritoneal cavity is entered. Morphine

and other opiates compound this problem by decreasing GI motility. Signs and symptoms of

postoperative ileus include nausea, vomiting, abdominal distention, failure to pass flatus or

stool, and hypoactive or absent bowel sounds on physical exam. (In contrast, mechanical

bowel obstruction causes hyperactive “tinkling” bowel sounds).

240

28. Slipped capital femoral epiphysis (SCFE) typically occurs in obese early adolescent males.

Acute SCFE is an emergency condition and should be promptly corrected with external screw

in order to prevent avascular necrosis of the femoral head and chondrolysis.

29. Colicky or paroxysmal abdominal pain with episodic hyperactive bowel sounds attributable to

peristaltic rushes, nausea, vomiting, abdominal distention, diffuse abdominal tenderness and

obstipation are typical signs of complete small bowel obstruction (SBO). SBO is treated

surgically if the obstruction is complete or if signs of strangulation of a bowel loop are

present.

30. Hypotension, tachycardia, flat neck veins, confusion, and cold extremities despite iv fluid

resuscitation in a trauma pt indicates hypovolemic/hemorrhagic shock.Hemorrhage classification

Class I Class II Class III Class IV

Blood loss (cc) 1000 1000-1500 1500-2000 >2000Blood loss % <20 20-30 30-40 >40Systolic BP Compensated Orthostatic Marked decrease Profound decreaseHR <100 >100 >120 >140Respiratory rate

14-20 20-30 30-40 >35

Urine output (cc/h)

>30 20-30 5-20 Anuria

CNS status Normal/anxious Agitated Confused Confused/obtundedCapillary refill Normal Slight

delay/CoolSignificant delay/cool

Significant delay/cold

31. Air under the diaphragm indicates perforated viscus, which is a surgical emergency. Once

diagnosed, immediately obtain surgical consulation for emergent exploratory laparotomy.

32. Hemodynamic stability and the need for blood transfusion are the determinants that guide

surgical versus non-surgical mx of pts with splenic trauma. If a pt being treated with

observation experiences hemodynamic instability unresponsive to a 2L saline bolus or

otherwise requires a blood transfusion, then laparotomy is indicated for splenorrhaphy or

splenectomy. Following splenectomy, pts are at a greater risk of infections by encapsulated

organisms such as H.influenzae, S. pneumoniae, and N.meningitidis.

33. Acute epidural hematoma has a classic presentation of uncomsciousness followed by a lucid

interval followed by gradual deterioration of consciousness. CT scan is diagnostic and it

shows a biconvex hematoma.

34. Pts with compartment symdrome complain of severe pain, pain with passive range of motion,

paresthesias as well as pallor and paresis of the affected limb. The presence of pulses does not

rule out compartment syndrome. Fasciotomy is the tx and must be done urgently.

35. Acute cholecystitis presents with right upper quadrant pain, fever and leukocytosis. The

majority of pts with acute cholecystitis may be treated with observation and supportive care

initially, followed by elective cholecystectomy within a few days of the same hospitalization.

Emergent cholecystectomy is required for pts with biliary gangrene or perforation.

241